63f49496e4b008bde2490f35 Original

You might also like

Download as pdf or txt
Download as pdf or txt
You are on page 1of 325

1|P a g e

Brain Stimulating TIPPS (BST)


INDEX
Topics Page no
1 Foundations of psychiatric classification, diagnosis and psychopathology 6
2 Delusions 10
3 Lewy body dementia 11
4 Neuropsychiatric aspects of Multiple Sclerosis 12
5 Neuropsychiatric aspects of Thyroid disorders 17
6 MMSE and Neuro psychological assessments 19
7 Anorexia nervosa 23
8 Depot antipsychotics 26
9 Sociology of suicide 29
10 Poor prognostic factors of Schizophrenia 30
11 Metabolic syndrome 31
12 Opioid use disorders 36
13 Female sexual disorders 46
14 Breaking bad news- Kayes model 47
15 Soft bipolar disorder 50
16 Investigations for patients with Neuropsychiatric manifestations 52
17 Forensic psychiatric concepts 54
18 NMHP 57
19 Recovery in Schizophrenia 59
20 Prognosis and psychosocial interventions in Schizophrenia 61
21 PMDD 63
22 Recent advances in Dementia 65
23 Landmark studies in Psychiatry 66
24 DM, Dyslipidemia and Obesity- interface with psychiatry 69
25 History of psychiatry 76
26 Neuropsychiatric aspects of Parkinsons disease 81
27 OCPD versus OCD 85
28 Gender dysphoria 88
29 Homosexuality: Status in India 91
30 Hindu marriage act 94
31 POCSO act 95
32 Varenicline 96
33 Psychotropics in pregnancy and lactation 97
34 Psychoeducation for lithium treatment 113
35 Elderly abuse 115
36 Treatment resistant OCD 117
37 Treatment resistant BPAD 124
38 Treatment resistant Depression 129
39 Treatment resistant Schizophrenia 134
40 Personality disorders 141
41 Psychosocial rehabilitation 146
42 ADHD 152
2|P a g e
43 Conduct disorder 156
44 Autism spectrum disorders 159

Case scenarios
Sl.no Topics Page no
1 Nicotine related disorders 163
2 Endicotts substitution criteria 170
3 Tardive Dyskinesia 171
4 Delirium 177
5 Dementia 178
6 Bush Francis Catatonia Rating Scale 182
7 Admission procedures as per MHCA act 2017 184
8 Bipolar Affective Disorder 185
9 Pregnancy and Breastfeeding 193
10 ADHD 198
11 Treatment resistant OCD 205
12 Treatment resistant Depression 207

Multiple Choice Questions (MCQs)


Sl.no Topics Page no
1 Somatoform disorders 204
2 Landmark studies in psychiatry 217
3 Long-acting injectable antipsychotics 227
4 Medically unexplained symtpoms 232
5 Psychocutaneous disorders 237
6 Abnormal involuntary movements 252
7 Sexual disorders 259
8 Paraphilias 264
9 Anorexia nervosa 273
10 Genetic syndromes 279
11 Recent advances 291
12 Borderline and Antisocial personality disorder 295
13 Transsexualism 298
14 IQ testing 310
15 Cognitive distortions 315
16 DBT 318
17 Family therapy 321
18 Miscellaneous topics 322

3|P a g e
Disclaimer

This document is a compilation of academic discussion by TIPPS Team via TIPPS Telegram group. A
gentle remainder to readers that there is absolutely no replacement for standard Psychiatry
textbooks (fish, Sims, Tasman, Lishman, Levenson, Goodwin and Jamison, Stahl, strub and black,
Djongs, bickerstaff, Vyas Ghimire, OTP, CTP, etc.). They are and will be standard reference books.

TIPPS BST is like a supplement. Consider it as the whey protein which professional bodybuilders use
to perform better in the competition. They still have to work hard in the Gym, be disclipned and
regular with their day-to-day diet which cannot be replaced by Whey protein.

-------- Dr Swarna Buddha

Great care has been taken while preparing this manuscript keeping in mind the exam preparing
postgraduates to maintain the authenticity of the content. The information provided in this book is
designed to provide helpful information on the subjects discussed and presented solely for
educational and guidance purposes. The authors are not offering it as a professional advice. Due to
nature of the discussion and content it is not possible to provide reference for the topics discussed
in this document.

While best efforts have been used in preparing this book, the author make no representations or
warranties of any kind and assume no liabilities of any kind with respect to the accuracy or
completeness of the contents and specifically disclaim any implied warranties or fitness of use for a
particular purpose. The authors shall not be liable for any physical, psychological, emotional,
financial, or commercial damages, including, but not limited to, special, incidental, consequential or
other damages. You are responsible for your own choices, actions, and results. Authors declare that
they have no conflict of interest.

We welcome feedback on the content! -

----TIPPS Team----

Compiled by:

Dr. Raj Kiran


Assistant Professor
KIMS, Amalapuram

Dr Kartik Valipay
Senior Resident
DM Addiction Psychiatry
AIIMS, New Delhi

4|P a g e
Administrators & Founding members

Dr. Rishikesh V. Behere, Consultant Psychiatrist, Manoshanti Pune

Dr. Naren P. Rao, Additional Professor, Psychiatry, NIMHANS, Bengaluru

Dr. Girish Babu, Associate Professor, SDM College of Medicine, Dharwad

Academic Coordinators (ACs)

Dr. Amit Chougule, Senior Clinical Fellow, NHS Wales, UK (Chief AC)

Dr. Bhargavi Nagendra, Consultant Psychiatrist, Bengaluru

Dr. Darshan, (DM Addiction Psychiatry), NIMHANS, Bangalore

Dr. Gaurav Uppal, Consultant, Sankalp Psychiatry Center, Amritsar

Dr. Jayakrishnan Menon, Addiction Psychiatrist, Kerala

Dr. Kartik Valipay, (DM Addiction Psychiatry), AIIMS, New Delhi

Dr. Krishna Patel, Associate Professor, CUSMC, Surendranagar

Dr. Mandakini Bhowmick, Senior Resident, Bharati Vidhyapeeth, Sangli

Dr. Ooha Susmita, Consultant Psychiatrist, Hyderabad

Dr. Pavithra Jayashankar, (PDF in Neuropsychiatry), NIMHANS, Bangalore

Dr. Raj Kiran, Assistant Professor, KIMS, Amalapuram

Dr. Raviteja Innamuri, Assistant Professor, CMC Vellore

Dr. Sachin N, Senior Resident, NIMHANS, Bengaluru

Dr. Sharnita Pugalenthi, Consultant Psychiatrist, Coimbatore

Dr. Sharanya B Shetty, Assistant Professor, KMC Mangalore

Dr. Sravanthi. P, Senior Resident, Kamineni Institute (KIMS), Narketpally

Dr. Suhas Satish, Senior Resident, NIMHANS, Bangalore

Dr. Swarna Buddha Nayok, PhD scholar, NIMHANS, Bangalore

Dr. Sweta Sheth, Senior Resident, NIMHANS, Bengaluru

Dr. Udayan Bhaumik, Senior Resident, Pramukhswami Medical College, GJ

Dr. Vijay Raj Pratheek, Assistant Professor, SVS College, Mahabub Nagar

5|P a g e
Notes
Topic: Fundamentals of Psychiatry classification, diagnosis and briefly
touch upon psychopathology
Must know definitions and equations

Illness = General lay-man term used to denote an ill-health condition


Symptom = Subjective distressing experience complained by patient
Sign = Objective finding in the patient elicited by examiner
Syndrome = Symptoms + Signs [A cluster of specific signs and symptoms that co occur
more frequently than expected by chance]
Disorder = Syndrome + Natural course known
Disease = Disorder + Etio-pathogenesis known
Operationalization implies - Making abstract constructs into quantifiable or qualifiable criteria.
We should operationalize the abnormal thoughts, feelings and behavior because we are in the
profession that deals with signs and symptoms that are mostly reported with words, often observed,
rarely objective in nature.
Also, the DSM 1 & 2 did not give diagnostic criteria under each disorder. The earlier editions of the
DSM were hugely influenced by prevailing psychodynamic theory and were completely atheoretical.
As expected, disorders, including psychotic disorders were thought to stem from underlying psychic
conflicts and maladaptive coping mechanisms They relied heavily on the psychosis-neurosis
dichotomy.
Neurobiology as a term was not known of, or not understood in those ages So rather than
undermining it, it prevented the development or progress of neurobiology.
Koch’s postulates help to establish the causal etiology. Currently, the Bradford Hill criteria,
otherwise known as Hill's criteria for causation is a more accepted criterion.

Psychopathology
Stream is related to speech whereas both stream and form to thought.
Formal thought disorder is now rejigged to include thought, language and communication
disorders.
So, psychopathology may be related to the way a thought starts, ends or the connections
All these psychopathological aspects are not pathognomonic to one condition, but nevertheless
commonly seen in specific conditions.
Possession of thought: obsessions, thought insertion, withdrawal, broadcast
First rank symptoms are not unique to schizophrenia. Frequency of FRS in bipolar disorder about
25-30 %, and 15-20 % in anxiety disorders.
The findings in form and possession are, as usual, not pathognomonic to a condition but need to be
taken into account with the rest of MSE, something that Dr U likes to call 'The Chowmein Effect'!

To summarise:
Mood description in MSE:
• Subjective
• Objective
• Range
• Reactivity
• Appropriateness
6|P a g e
• Congruity
• Lability
• Diurnal variation

Mood
Feeling: A positive or negative reaction to an experience/event; the subjective experience of
emotion
Emotion: A stirred up physiological state as a response to an event and which tends to maintain or
abolish the causative event.
Affect: Patients’ present emotional responsiveness. (Cross-sectional and objective)
Classification of emotional reaction and expression:
• Normal Emotional Reactions
• Abnormal Emotional Reactions
• Abnormal Expressions of Emotion
• Morbid Expressions of Emotion
• Morbid Disorders of Emotion
Ref: Fish clinical psychopathology (4th Ed) and Sims textbook of descriptive
psychopathology (5th ed)

Thought content
The psychopathology of delusions was beautifully explained with a series of MCQs, listed below for
your reference and persual. All the options are mention with the answers in bold.

Some historical background:


• Jaspers found all to be essential for a delusion but: Falsity/impossibility, competency,
incorrigibility, certainty
• Who stressed that delusions need not be always be out of accordance of educational and cultural
background? Bleuler, Jaspers, Hamilton, Spitzer, Stoddart
• Who stressed on the social acceptance of delusion, saying that delusion is a judgement that cannot
be accepted by others? Bleuler, Jaspers, Hamilton, Spitzer, Stoddart
• As per Gruhle, the most significant form of delusion is: Delusional percept, Delusional mood,
sudden delusional idea, delusional intuition, delusional atmosphere?
• As per Kretschmer, “Key experience” changes: a delusional mood to delusional idea, idea to
delusional atmosphere, referential ideas to delusion, over-valued ideas to delusions.
• Primary Delusions are: understood in the context of hallucinations, understood if you really know
a person, ultimately un-understandable, ununderstandable until the patient co-operates

Defintions and stages of formation of delusion:


• I feel something is going on around me. I do not know what it is exactly. But something is wrong:
delusional mood, delusional atmosphere, delusional percept, delusional memory?
• I feel something is going on around me. I am tensed up. I feel something is going to be revealed to
me: delusional mood, delusional atmosphere, delusional percept, delusional memory?
• Five stages of delusions development were proposed by? Conrad, Fish, Bleuler, Schneider,
Andreasen
• Yesterday when I read the newspaper, I noticed the front page headlines consisted all the letters
of my name. It was jumbled. What is the newspaper editor trying to tell me: Which stage of delusion
is this? consolidation, Trema, residuum, apophany, anastrophy
• The world is different. Looks different. Feels different. Feels uneasy. I feel apprehension: Which
stage of delusion is this? consolidation, Trema, residuum, apophany, anastrophy
7|P a g e
• This is a new world now. I have understood it all. The newspaper editors are trying to find out my
secrets. They want my money. The delivery boy is going to kill me, I am sure.: Which stage of
delusion is this? consolidation, Trema, residuum, apophany, anastrophy

Establishing a DELUSION:
• A husband has an established delusion that his wife is cheating on him with her ex-lover. The
psychiatrist finds out that the wife has indeed been in an extra marital relationship, but with a
different person. Is it still a delusion? Yes, No
• A husband has an established delusion that his wife is cheating on him with her ex-lover. The
psychiatrist finds out that the wife has indeed been in an extra marital relationship with her ex. Is it
still a delusion? Yes, No
• A husband has an established delusion that his wife is cheating on him with her ex-lover. This idea
had suddenly come to him while he was driving his car. The psychiatrist finds out that the wife has
indeed been in an extra marital relationship, but with a different person. Is it still a delusion? Yes,
No
• A husband has an established delusion that his wife is cheating on him with her ex-lover. This idea
had suddenly come to him while he was driving his car. The psychiatrist finds out that the wife has
indeed been in an extra marital relationship with him. Is it still a delusion? Yes, No
• A husband has an established delusion that his wife is cheating on him with her ex-lover. He says
that he knows this for sure as his wife drops the knife while cutting vegetables to indicate this. The
psychiatrist finds out that the wife has indeed been in an extra marital relationship with him. Is it
still a delusion? Yes, No

Some varieties of delusion:


• I know he is following me. He has been doing it from morning in different forms. He was the
milkman. He was the taxi driver too. Subjective doubles, Fregolis, reduplicative paramnesia,
intermetamorphosis, Capgras.
• I know he is following me. He has been doing it from morning. He has taken the form of my father.
Subjective doubles, Fregolis, reduplicative paramnesia, intermetamorphosis, Capgras.
• I know he is following me. He has been doing it from morning. He is my duplicate. He looks exactly
like me. But he is evil. Subjective doubles, Fregolis, reduplicative paramnesia, intermetamorphosis,
Capgras.
• I know he is following me. He has been doing it from morning. He is my duplicate. Although he
looks different. And he is evil. Subjective doubles, Fregolis, reduplicative paramnesia,
intermetamorphosis, Capgras.
• I know he is following me. He has been doing it from morning. He has changed my house. Its his
house now. Looks exactly same though. Subjective doubles, Fregolis, reduplicative paramnesia,
intermetamorphosis, Capgras.
• I know they are watching me. They have sent a spy. The spy came to meet me as a customer in my
shop. Later a lady came, she was the same spy, but dressed differently. When I was in the bus, the
driver glanced at me. The driver was also the same spy. Subjective doubles, Fregolis, reduplicative
paramnesia, intermetamorphosis, Capgras.
• I know the barber, he is actually my friend. Buthe looks different. My friend is taller. My friend is
also soft spoken, unlike the barber. But I am sure that they are the same person. Subjective doubles,
Fregolis, reduplicative paramnesia, intermetamorphosis, Capgras.
• He is my own evil form, he looks exactly like me. But he is trying to make me a culprit. He does all
the bad things. Subjective doubles, Fregolis, reduplicative paramnesia, intermetamorphosis,
Capgras.

8|P a g e
• He is not my father. He is someone else who has come as an imposter. I don’t know where my
father is. He looks very similar to my father. Subjective
doubles, Fregolis, reduplicative paramnesia, intermetamorphosis, Capgras.
• Capgras syndrome is a culture bound delusion of East Asia: True, false
Fregoli was a famous magician. His most famous act was that he could change his costumes quickly
and appear in a different costume. So it was the same person, who came in different physical forms.
The same person has taken a disguise or many. So someone unfamiliar becomes familiar. As is=f all
are Fregoli himself, coming in disguises. Intermetamorphosis IS when one sees a person who is
physically and psychologically different. But still thinks that the person is someone he knows. So the
person has now metamorphosized. Like a cocoon becoming a butterfly, with completely different
looks and behaviour, but it is indeed the same insect.
Intermetamorphosis: physically different, psychologically different
Subjective doubles: Physically same, psychologically different

• Delusion is an aberrant inference process characterized by either a failure in sensory attenuation


or an aberrant weighting of prior experience: Bayesian theory: True, false
• Dog barks at thiefs. My dog barked at my friend. So my friend is a thief: Bayesian rule, Cartesian
rule, von Domarus rule, Kant’s rule
• Morbid jealousy is further classified by Mullen. Which is NOT included in it: Disorder of passion,
disorder of possession, disorder of erotomania, disorder of querulant

Vectors for delusion:


• The politicians are behind my money. They are jealous of me. I am sure of this as I see their
interviews in newspapers are related to me. There cannot be any other way. They are plotting
against me. Every day it becomes more and more evident. Which vector of delusion is this referring
to the most? Bizarreness, conviction, extension, disorganization, pressure.
• The politicians are behind my money. They watch me at all times. They are there at my office.
They sneak behind my house. They have recruited spies as neighbours. Even when I go for a walk,
they follow me. Which vector of delusion is this referring to the most? Bizarreness, conviction,
extension, disorganization, pressure.
• The politicians are behind my money. They are jealous of me. I am always careful now. Wherever
I go, I carry a pocket knife with me. Now I have stopped going to office and for evening walks. I am
always vigilant to find them. Which vector of delusion is this referring to the most? Bizarreness,
conviction, extension, disorganization, pressure.
• The politicians are behind my money. One wants me to fight against the other.
Their party leader had tried to send clues to me through the shopkeeper. The clerk in my office is
also involved. He gives out information to the manager.
The manager passes it on to the party workers. Which vector of delusion is this referring to the
most? Bizarreness, conviction, extension, disorganization, pressure.
• The politicians are behind my money. They are jealous of me. Whenever they go for a rally, they
indicate this to me. If they say “Hello hello hello” in the mike before speech, that is to threaten me.
Three hello mean three warnings. Which vector of delusion is this referring to the most?
Bizarreness, conviction, extension, disorganization, pressure.

Further diving into the phenomenology of delusions:


• I was walking down the street. Suddenly two dogs started barking at me. I was afraid and ran. The
dogs must be following me. Someone may be tracking me down. Is there two-memberedness in this
example? Yes, No

9|P a g e
• I have headache from last night. Maybe I have a tumour. When I visited the doctor and asked this,
he smiled. He said symptoms of tumour are different.
But I googled and saw headache is a symptom. Is there two-memberedness in this example? Yes,
No
• I am a sinner. This is god’s punishment for me. Yesterday I called my wife, she did not pick up. She
called today and said she is not well. Whoever is with me, also faces problems. All because of me. Is
there two-memberedness in this example? Yes, No
• There is so much corona infection all around. I have sore throat and I am quite sure that I am
infected to. I heard few people coughing behind me. I think I have gotten infected from them. Is
there two-memberedness in this example?
Yes, No
(The person is anxious of corona and has sore throat. Two memberedness does not deal with other
features of delusion, like firm fixed false belief etc. It hints at the internal morbid process. In this
example, the flow of thoughts may be:
Percept: coughing
Next thought: the coughing person is infected.
Next thought: I am near the person
Next thought: corona in infectious.
Next thought: I may be infected now)
• I was walking down the street. It was windy. A plastic bag flew right in front of me. I understood it
immediately. I was meant to save the world. An apocalypse is coming. Is there two-memberedness
in this example? Yes, No
What is the above an example of? Delusional mood, delusional memory, delusional
percept, sudden delusional idea
• I was walking down the street a week back. It was windy. A plastic bag flew right in front of me.
Yesterday while eating dinner it became clear to me. I was meant to save the world. An apocalypse
is coming. Is there two memberedness in this example? Yes, No
What is the above an example of? Delusional mood, delusional memory, delusional
percept, sudden delusional idea
• I was walking down the street. It was windy. It suddenly occurred to me out of the blue. I was
meant to save the world. An apocalypse is coming. Is there two-memberedness in this example?
Yes, no.
What is the above an example of? Delusional mood, delusional memory, delusional percept, sudden
delusional idea
• I was walking down the street. It was windy. A plastic bag flew right in front of me. I remembered
this yesterday when two people came to me and said that plastic is bad for environment. Now I
understand why they came. I was meant to save the world. An apocalypse is coming. What is the
above an example of? Delusional mood, delusional memory, delusional percept, sudden delusional
idea
Delusional percept has a normal percept and a delusional meaning given to that
percept. These two may not happen immediately. Sometimes the delusional meaning may become
apparent to the normal percept decades later. Of course then it becomes difficult to differentiate it
from memory.

Topic: Delusions
Jaspers emphasized on 3 things necessary for a delusion: false judgements(not just belief), held with
extraordinary conviction and impervious to any other reasoning, content is impossible (not
10 | P a g e
implausible). The stress that we provide on the socio-cultural-education (-spiritual-economical etc
etc) in ONLY to make it easier to differentiate. This is stressed by Hamilton as against Stoddart, who
preferred it to be “not acceptable by the same class, education, race and period of life”. This will
make a deluded “racist” a normal person in the Ku Klux Klan. Hamilton stressed on one more part,
a part that is missed out when we memorize definition of delusion: INTERNAL MORBID PROCESS.
This makes a husband deluded if he beliefs wife to be cheating on him “whenever she coughs”, even
if the wife is involved in infedility (and coughs too). The judgement of the husband is from an
internal (not external, he has not read somewhere that coughing is a sign of infedility) morbid ( no
logic) process. Whether there is infedility in real now is not important/ inconsequential. The morbid
process makes it a delusion even if it is true.

Unless it is a single disorder, differentiating multiple delusions in stages for each is quite
cumbersome. Also the stages are not absolute. One can fluctuate between anastrophy and
apophany for a long period, especially with multiple delusions. Stages of formation of delusion is
reduced to a viva question.

Origin of delusion is better understood in context of origin of normal idea. An idea is often originated
due to a percept. When the percept is itself wrong, it becomes illusion, pseudo hallucination,
imagery, hallucination etc. When the percept is real and normal, then the meaning given to it
becomes important. If this meaning is delusional, and arises de novo, without a previous or ongoing
psychopathological process, then it is a primary delusion. There can be multiple primary delusions.
It is not temporal, meaning it isn’t “the first delusion”. It also need not be immediate. But then it
becomes difficult to differentiate it from a memory. This new meaning should not be understood by
previous psychopathology, this is not delusional misinterpretation.

Capgras: someone (with bad intention) is imposing as my father/mother etc (physically same). So I
see my father, recognise his physical characters, recognize his behaviours, and then think he is not
my father. Someone else as come in his form. Why will someone do so? Must be to harm me (the
usual thought process). Thinking familiar (father) to be unfamiliar (not really father, but an
imposter)

Fregoli: thinking unfamiliar to be familiar. The other way to understand is that the same person
coming in different forms. So I see a postman (unfamiliar), I think he is Mr. XYZ (someone familiar,
or someone I have seen before). I see the driver (unfamiliar), I think he is is Mr. XYZ.

Topic: Lewy body dementia


A 60-year-old male, a retired clerk, is brought with complaints of decreased sleep from 2 years, sad
mood and irritability from 1 year. It all started 2 years back, when he slipped near washroom at night
and broke his right femur. He had 2-3 such falls in past 3 years too, but no fractures or head injury.
After he spent 4 months in bed for treatment of fracture femur, he had decreased sleep, woke up
early mornings with vivid scary dreams of people chasing him and he falling down.

He often remained sad, cried and irritable if asked what he was thinking, at times seemed to be lost
in his thoughts and only responding to questions after repeating them few times. H e moved less,
ate and dressed slowly. No history of seizures.

About a month back, he was found screaming, early in the morning, after being awake for 2 hours.
He claimed that he had seen his own self outside the window, and this meant that his death was

11 | P a g e
nearing. He now claims that he has seen his own self lurking behind him a few times more, and there
was no reason to live anymore.

Revised guidelines for LBD:

Essential for a diagnosis:

• Dementia: Prominent or persistent memory impairment may not necessarily occur in the
early stages but is usually evident with progression. Deficits on tests of attention, executive
function, and visuo-perceptual ability may be especially prominent and occur early.
✓ This case had no memory problems, MMSE showed attention deficits. And poor copying.
MMSE is not a good screening for executive functions. MOCA or Addenbrooke’s can be used.
MOCA showed clock drawing problem, trail making problem. Showing poor attention and
visuo-spatial problems.
✓ Visuo-spatial problems can also be responsible for the frequent falls. Also, pentagon drawing
is sensitive to differentiate between LBD and Alzheimer’s.
✓ Attention &Visuo-spatial problems can also be seen in parietal lobe dysfunctions, but if
dominant lobe is involved, finger agnosia, acalculia, finger agnosia and right left confusion
would also be there.
Core clinical features (The first 3 typically occur early and may persist throughout the course.)

• Fluctuating cognition with pronounced variations in attention and alertness: often missed
by the family, families do not do serial subtraction and orientati on checks. Hint: “at times
seemed to be lost in his thoughts and only responding to questions after repeating them few
times”. Can also happen in complex partial seizure. Can also happen in depression, but this
case has lesser depressive feature. Mood disorders, specially depression can often be
preceding LBD.
• Recurrent visual hallucinations that are typically well formed and detailed: This is where
seeing self comes, which were probable visual hallucinations, as he recognised himself with
enough details, which secondarily have produces delusional misidentification syndrome-
delusion of doubles. DMS is harbinger of organicity, in elderly.
• REM sleep behavior disorder, which may precede cognitive decline: indeed the first
symptoms were sleep disturbances (can also be due to pain at fracture site), early morning
vivid nightmares are REM sleep nightmares.
• One or more spontaneous cardinal features of parkinsonism:He walked, ate and dressed
slowly: bradykinesia
Supportive clinical features

• Severe sensitivity to antipsychotic agents: to olanzapine 2.5 mg, unusual.


• Postural instability; repeated falls; syncope or othertransient episodes of unresponsiveness
• Severe autonomic dysfunction, e.g., constipation,orthostatic hypotension, urinary
incontinence; hypersomnia; hyposmia; hallucinations in othermodalities; systematized
delusions; apathy, anxiety, and depression.
Indicative biomarkers

12 | P a g e
✓ Reduced dopamine transporter uptake in basal ganglia demonstrated by SPECT or PET
Alzheimers/normal DLB

✓ Abnormal (low uptake) 123iodine-MIBG myocardial scintigraphy


✓ Polysomnographic confirmation of REM sleep without atonia
Supportive biomarkers

✓ Relative preservation of medial temporal lobe structures on CT/MRI scan (a,b,c are
alzheimer’s, d is LBD)
✓ Generalized low uptake on SPECT/PET perfusion/metabolism scan with reduced occipital
✓ activity 6 the cingulate island sign on FDG-PET imaging.
✓ Prominent posterior slow-wave activity on EEG with periodic fluctuations in the pre-alpha/
✓ theta range.
Probable DLB can be diagnosed if:
a. Two or more core clinical features of DLB are present, with or without the presence of
indicative biomarkers, or
b. Only one core clinical feature is present, but with one or more indicative biomarkers.
Probable DLB should not be diagnosed on the basis of biomarkers alone.
Possible DLB can be diagnosed if:
a. Only one core clinical feature of DLB is present, with no indicative biomarker evidence, or
b. One or more indicative biomarkers is present but there are no core clinical features.
DLB is less likely:

a. In the presence of any other physical illness or brain disorder including cerebrovascular
disease, sufficient to account in part or in total for the clinical picture, although these do not
exclude a DLB diagnosis and may serve to indicate mixed or multiple pathologies contributing
to the clinical presentation, or
b. If parkinsonian features are the only core clinical feature and appear for the first time at
a stage of severe dementia.

DLB should be diagnosed when dementia occurs before or concurrently with parkinsonism. The
term Parkinson disease dementia (PDD) should be used to describe dementia that occurs in the
context of well-established Parkinson disease. In a practice setting the term that is most appropriate
to the clinical situation should be used and generic terms such as Lewy body disease are often
helpful. In research studies in which distinction needs to be made between DLB and PDD, the
existing 1-year rule between the onset of dementia and parkinsonism continues to be
recommended.
✓ Donepezil may be helpful in LBD.
✓ In case of EPS by antipsychotic, stop the offending agent. Dystonia may require
promethazine. EPS should come down by 2 weeks.
References:

13 | P a g e
• Ala, T., Hughes, L., Kyrouac, G., Ghobrial, M., &Elble, R. (2001). Pentagon copying is more
impaired in dementia with Lewy bodies than in Alzheimer’s disease. Journal of Neurology,
Neurosurgery, and Psychiatry, 70(4), 483–488. https://doi.org/10.1136/jnnp.70.4.483
• Cahn-Weiner, D. A., Williams, K., Grace, J., Tremont, G., Westervelt, H., & Stern, R. A. (2003).
Discrimination of dementia with lewy bodies from Alzheimer disease and Parkinson disease
using the clock drawing test. Cognitive and Behavioral Neurology: Official Journal of the
Society for Behavioral and Cognitive Neurology, 16(2), 85–92.
https://doi.org/10.1097/00146965-200306000-00001
• Mak, E., Su, L., Williams, G. B., & O’Brien, J. T. (2014). Neuroimaging characteristics of
dementia with Lewy bodies. Alzheimer’s Research & Therapy, 6(2), 18.
• https://doi.org/10.1186/alzrt248
• McKeith, I. G., Boeve, B. F., Dickson, D. W., Halliday, G., Taylor, J.-P., Weintraub, D., Aarsland,
D., Galvin, J., Attems, J., Ballard, C. G., Bayston, A., Beach, T. G., Blanc, F., Bohnen, N.,
Bonanni, L., Bras, J., Brundin, P., Burn, D., Chen-Plotkin, A., … Kosaka, K. (2017). Diagnosis
and management of dementia with Lewy bodies: Fourth consensus report of the DL B
Consortium. Neurology, 89(1), 88–100. https://doi.org/10.1212/WNL.0000000000004058
• Stinton, C., McKeith, I., Taylor, J.-P., Lafortune, L., Mioshi, E., Mak, E., Cambridge, V., Mason,
J., Thomas, A., & O’Brien, J. T. (2015). Pharmacological Management of Lewy Body Dementia:
A Systematic Review and Meta-Analysis. The American Journal of Psychiatry, 172(8), 731–
742. https://doi.org/10.1176/appi.ajp.2015.14121582

Topic: NP of Multiple Sclerosis


Introduction

Multiple Sclerosis is the most common inflammatory demyelinating disease of the CNS

Immunologically mediated, HLA-DRB1 gene in MHC II

Inflammation, subsequent loss of oligodendrocytes and demyelination, proliferation of astrocytes


and gliosis, axonal damage in some lesions leading to progressive neurological impairment

More commonly in females, typical age of onset between 20-40 yrs of age, lesser prevalence in the
tropics

There are 4 subtypes based on the clinical course

• Clinically isolated syndrome, representing the first attack of MS; Benign MS refers to
disease in which the patient remains fully functional in all neurologic systems 15 years after
the disease onset; Approximately 15 percent of patients with a single demyelinating event
will never experience a second relapse
• Relapsing-remitting MS – Most common type
• Secondary progressive MS – initial RRMS followed by progression, neurological worsening
occurs regardless of presence of superimposed exacerbations, on an average takes 10-20
years after disease onset, no established criteria for when the progression has occurred,
hence a retrospective diagnosis
• Primary progressive MS – 10% of cases, diagnosis made exclusively on history, no specific
physical exam findings to distinguish from RRMS, occurs equally in both genders, over 40
years of age, most common presentation is with spinal cord symptoms

14 | P a g e
Another term associated with course is Progressive Relapsing MS

Disease severity depends on signs and symptoms, frequency of relapses, rate of worsening, and
residual neurological disability.Stresscan lead to relapses. Fever, heat sensitivity after exercise
/Uhthoff sign can lead to pseudorelapses.
Life-expectancy is around 7-14 years lesser than general population. People with MS had increased
rates of death from infection, respiratory diseases, CVD and suicide. Aggressive MS is when there
is progression of disease severity such that patient requires assistance within 5 years after
symptom onset.
Signs and symptoms

A young adult with a clinically isolated syndrome suggestive of MS such as optic neuritis, long tract
symptoms/signs (eg, numbness, paresthesia, or weakness), a brainstem syndrome (eg,
internuclear ophthalmoplegia), or a spinal cord syndrome (eg, transverse myelitis).

Clinical signs and symptoms are based on where the lesion is and may include sensory signs (most
common at onset esp. in the limbs), pyramidal tract signs, cortical signs including cognitive issues
(70% when neuropsychological evaluation is done and largely associated with executive function
deficits, speed of information processing, memory impairment), problems with vision due to optic
neuritis, disorders of eye movement such as Internuclear Ophthalmoplegia, cranial nerve
involvement leading to commonly trigeminal and glossopharyngeal neuralgia or emotional
incontinence, Cerebellar signs, Transverse myelitis that may lead to paraparesis, Lhermitte’s sign
among many. Symptoms develop over the course of hours to days. Bowel and bladder dysfunction
are eventually present in about 50 and 75% respectively in people with MS eventually. Bladder
dysfunction may be due to detrusor hyperactivity, detrusor – sphincter dyssynergia, decreased
contractility.
McDonald’s criteria is used for the diagnosis of Multiple Sclerosis

Investigations

MRI findings are hyperintense white matter lesions on T2 sequences in characteristic locations
(the residual plaques) (periventricular, juxtacortical in the corpus callosum perpendicular to the
fibres called Dawson’s fingers, infratentorial, and spinal cord). Active lesions enhance with the
addition of gadolinium (usually up to a period of 1 month). Disease activity is determined by
clinical relapses or MRI evidence of contrast-enhancing lesions and/or new or unequivocally
enlarging lesions on T2-weighted images.

Other investigations are oligoclonal bands in CSF, Visual Evoked Potentials, Optical Coherence
Tomography, and autoantibody testing in case of atypical features for Aquaporin-4 in
Neuromyelitis Optica, Myelin Oligodendrocyte Glycoprotein (MOG) in MOG associated
encephalomyelitis.

Prognostic indicators

• Relapsing vs Progressive disease


• Early symptoms – Bladder involvement, shorter time period between first and second
attacks, polysymptomatic onset of MS, early accumulation of disability
• MRI Markers – Lesion load (number and volume of lesions) on T2 FLAIR sequences, brain
atrophy and presence of “black holes” on T1 sequences

15 | P a g e
• Pregnancy - appears to have a protective effect against MS relapses, but there is an
increased risk of disease exacerbation in the early postpartum period
• Demographics – Male sex, earlier onset slower progression, however need for assistance at
similar ages, ? ethnicities
• Others – cigarette smoking, Vit D deficiency, exposure to EBV in childhood

Differential diagnosis
Acute Disseminated Encephalomyelitis, NMO Spectrum Diosrders, MOG-EM, Autoimmune
disorders (incl. SLE, Behcet Syndrome, PAN), Infections, Vit B12 Deficiency, HIV infection,
Paraneoplastic syndromes

Treatment
Steroids for acute exacerbations as Methyl prednisolone pulses
Disease modifying agents approved are: Interferon beta, Glatiramer acetate, Natalizumab,
Fingolimod, Dimethyl fumarate, Teriflunomide, Mitoxantrone, Alemtuzumab

Psychiatric syndromes associated with MS


Affective disturbance in 2/3rds of patients with MS. Depression is common. Can be reactive (due
to the diagnosis, relapses, neurological deficits including difficulty in locomotion, blindness due to
the optic neuritis), endogenous, organic due to the MS lesions, due to medications such as
steroids, interferons. Some studies show BPAD is twice as more than general population in
patients with MS. There is a significant reduction in quality of life.
Pseudobulbar effect leading to emotional incontinence might be present in 10% of the patients.
Rates of suicide are higher.
There is an increased incidence of anxiety disorders, GAD appears to be the most common
Psychosis is also more common in people with MS
Cognitive dysfunction as discussed earlier. Correlates with the degree of grey and white matter
atrophy.
Substance Use Disorders – Studies have shown higher rates of problematic alcohol use in patients
with MS. Recently with emergence of studies of cannabis and its products in alleviating spasticity
there is increased use of cannabis products in people with MS. Studies have shown improvement
in subjective mood after use of cannabis
Fatigue is also often present in patients with MS and maybe exacerbated due to the medications

Sleep disturbances can happen due to the spasms and bladder dysfunction. People with MS can
also have Restless Leg Syndrome, obstructive sleep apnoea, REM Behavioural Disorders.
Depression and anxiety can also worsen sleep related disorders.

Pain can be present due to the spasticity, neuropathic pain, Lhermitte’s sign, headache is common,
cranial nerve neuralgias. Some of it can linger and lead onto chronic pain syndromes.

Sexual dysfunction is common. Around 50% people become completely sexually inactive. There
may be difficulties in all phases of the sexual response cycle. Dysfunction maybe due to primary
neurological issue or a secondary psychiatric syndrome.

Steroids can lead to depression, mania, psychosis and long-term use can also lead to cognitive
deficits. Interferon ß treatment is associated with depression.
References

16 | P a g e
1. Olek MJ, Howard J; Clinical presentation, course, and prognosis of multiple sclerosis in
adults: UpToDate
2. Olek MJ, Narayan RM, Frohman EM, Frohman TC; Manifestations of multiple sclerosis in
adults: UpToDate
3. Olek MJ, Howard J; Evaluation and diagnosis of multiple sclerosis in adults: UpToDate
4. Hauser SL, Goodin DS; Multiple Sclerosis and other Demyelinating Disorders; Chapter 458;
Harrison’s Principles of Internal Medicine, 19 th Edition; Pg 2661-2772
5. Other Disorders of the Nervous System; Multiple Sclerosis; Chapter 14; Lishman’s Organic
Psychiatry; Pg 845-854
6. Chwastiak LA, Ehde DM; Psychiatric Issues in Multiple Sclerosis; Psychiatric Clinics of North
America; Dec 2007
7. Politte LC, Huffman JC, Stern TA; Neuropsychiatric manifestations of Multiple Sclerosis;
Primary Care Companion, Journal of Clinical Psychiatry; 2008
Topic: Neuropsychiatric Aspects of Thyroid Disorders

Introduction

Thyroid disorders may induce virtually any psychiatric symptom or syndrome.

Precursor thyroglobulin is stored in the thyroid follicles. In extrathyroidal tissues, T4 is converted to


T3, the metabolically active hormone. TSH (thyrotropin), produced by the anterior pituitary,
promotes the synthesis and release of these hormones and is, in turn, regulated by TRH, produced
in the hypothalamus. Low serum levels of thyroid hormones stimulate release of TRH and TSH.
Dopamine decreases TSH secretion, as do glucocorticoids. T3 and T4 are tightly protein bound to
thyroxine-binding globulin (TBG), so that only 0.3 percent of T3 and 0.02 percent of T4 are free. Total
serum T4 levels vary with TBG levels; for example, exposure to oestrogens (pregnancy and oral
contraceptive) raise TBG and, therefore, serum T4, whereas androgens have the opposite effect.
FT4, produces a more accurate determination of thyroid function. Measures of TSH levels are also
necessary in the workupof thyroid disorders.

Hyperthyroidism

Hyperthyroidism is commonly associated with fatigue, irritability, insomnia, anxiety, restlessness,


weight loss, and emotional lability; markedimpairment in concentration and memory may also be
evident. Such states can progress into delirium or mania (2 – 5%). On occasion, psychosis develops.
In some cases, psychomotor retardation, apathy, and withdrawal rather than agitation and anxiety
are the presenting features. Depressive symptoms are not only the most common psychiatric
features seen in hyperthyroidism, occurring in up to 30% of patients. Anxiety symptoms presenting
as generalized anxiety are also, common, with a prevalence of between 10 – 20%, but panic and
agoraphobia are relatively uncommon. Symptoms of mania also have been reported after rapid
normalization of thyroid status in hypothyroid individuals. In general, behavioural abnormalities
resolve with a normalization of thyroid function. Cognitive dysfunction is reported between 5 and
10 percent of patients with thyrotoxicosis.

Hyperthyroidism may result from a toxic multinodular goitre, a single functioning adenoma, or from
the presence of a thyroid stimulator, such as a thyroid stimulating antibody in Grave disease.

17 | P a g e
Exogenous thyroid hormone can produce a similar picture as can disorders of thyroid hormone
storage consequent to autoimmune thyroiditis. Other causes include, TSH- producing pituitary
adenoma, thyroid carcinoma, choriocarcinoma, hydatidiform moles, and struma ovarii.

Hypothyroidism

The symptoms of hypothyroidism may include fatigue, lethargy, weight gain, constipation, cold
intolerance, stiffness and cramping of muscles, hair loss, cognitive slowing, and depression, a true
secondary psychotic disorder or dementia-like state also can develop.

Generalized anxiety symptoms are described in up to 30% of patients. Cognitive deficits are the
most common neuropsychiatric features of hypothyroidism, occurring in up to 50% of cases.
Psychomotor speed, memory, and visual–perceptual skills are affected. It may be a risk factor for the
development of bipolar disorder, particularly the rapid cycling form. Bipolar disorder patients not
on lithium have high prevalence rates of hypothyroidism, 25% in women and 12% in men. Signs
include, hypothermia, bradycardia, dry skin, sparse hair, periorbital swelling, thickening of the
tongue, coarsening and deepening of the voice, menstrual irregularities, and a characteristic
prolonged relaxation phase of deep tendon reflexes. This clinical picture is often referred to as
myxoedema.

In children hypothyroidism can lead to intellectual disability. Hypothyroidism can be seen asa part
of other disorders such as mitochondrial disorders, velocardiofacial syndrome and
Wilson’s disease.

Laboratory abnormalities associated with hypothyroidism include elevated cholesterol,triglycerides,


and normochromic, normocytic anaemia.

The most common cause of hypothyroidism in adults is primary autoimmune hypothyroidismrelated


to antithyroid antibodies. Other causes include treatment for hyperthyroidism, drug- related effects,
and iodine deficiency. Postpartum thyroiditis may present with hypothyroidism alone (43%), with
hyperthyroidism alone (32%), or with an initial phase of hyperthyroidism followed by
hypothyroidism (25%). Supra-thyroid causes (hypothalamic or pituitary dysfunction) account for less
than 5 percent of cases. There is a generalised slowing in EEG in hypothyroidism and global
hypometabolism in some cases. Hypothyroidism impairs neurogenesis in the hippocampus. The
origin of depression in hypothyroidism appears to relate to the role of thyroxine in serotonergic
transmission, such that reduced thyroid input reduces serotonergic tone. A blunted response of TSH
to TRH infusion has been found in a significant percentage of patients with eating disorders, panic
disorder, AUD,schizophrenia, and, most commonly MDD. Antithyroid antibodies are found more
frequentlyin women with depression than in control subjects and in patients with bipolar disorder
and may contribute to relative treatment resistance.

Designated subclinical hypothyroidism, these scenarios can be further classified into elevated TSH
without changes in thyroid hormones (Grade II hypothyroidism), abnormal TSH response to
stimulation with TRH (Grade III), and the presence of antithyroid antibodies withno thyroid hormone
system abnormalities (Grade IV). Grade II hypothyroidism has been associated with depressive
disorders.

Many malnourished patients with anorexia nervosa develop theeuthyroid sick syndrome, in which
18 | P a g e
decreased levels of TSH, T4, and T3 may be seen, FT4 and FT3 are usually unchanged, and rT3 is
elevated. Thyroid hormones can be misused for weight loss in patients with eating disorders.

Psychotropics and the thyroid hormone

Lithium increases antithyroid antibodies and inhibits iodine uptake into the thyroid, iodination of
tyrosine, release of T3 and T4 from the thyroid (most important mechanism leading to
hypothyroidism, and peripheral breakdown of thyroid hormones. It also regulates TR gene
expression, blocks the thyroid-stimulating effects of TSH through interference with adenylate
cyclase, and may, in certain circumstances, precipitate a rebound thyrotoxicosis. Approximately 30
percent of patients receiving lithium have an elevated TSH level during treatment, and
approximately one-sixth of these patients go on to develop frank hypothyroidism. Transient mild
abnormalities are common early in the course of treatment but are usually of little or no clinical
consequence.Some patients, however, develop goitre or clinical hypothyroidism sometime during
the course of treatment. Females are five times more likely than men to develop hypothyroidism
with lithium treatment. Older age, iodine deficiency and pre-treatment presence of thyroid
antibodies arealso risk factors for hypothyroidism with lithium use.

Carbamazepine decreases peripheral thyroid hormone concentrations while increasing TSH.


Carbamazepine decreases circulating levels of T4, F T4, and T3 without substantially alteringTSH.
Studies indicate that it does not affect BMR and that thyroid replacement is almost never required.
Valproate also can cause subclinical hypothyroidism.

Quetiapine affects total and free thyroxine T4 levels in a dose-dependent manner. At the high
therapeutic dose range T4 levels may be decreased by as much as 20%. These changes are generally
not clinically significant. Less than 1 percent of patients had changes in TSH. Patients on lithium and
quetiapine are likely at greater risk.

Thyroid hormones in psychiatric disorders

T3 levels have been shown to predict relapse in unipolar depression, administration of T3


accelerates clinical response to TCAs. In STAR D trial liothyronine was found to augment response
to antidepressants. Recent studies have also indicated augmentation of SSRI response. Liothyronine
has also been used Adjunctive T4 contributes to decreasing cycling in patients with rapid cycling
bipolar I disorder. Liothyronine has been preferably used in psychiatric disorders whereas thyroxine
is the standard thyroid hormone used for replacementfor hypothyroidism. Liothyronine has a lower
therapeutic index and a much shorter half-life, making thyroid axis suppression—a goal of
replacementtherapy unreliable.
TRH has been administered IV either as a single dose or as several doses over a period of 3 to 4 days,
and transient antidepressant effects have been demonstrated. Has an unclear therapeutic role. TRH
administered intravenously before the ECT yielded greater arousal and improved cognitive
functioning when compared with placebo. TSH also needs more studies.

Reference
All the information quoted are from multiple chapters in CTP 10 th Edition

19 | P a g e
Topic: MMSE and Neuropsychologicalassessments
MMSE- Mini Mental status examination

Cut-off scores- Total score 30

A 60 year old male is referred to you with h/o RTA and is unable to discriminate sounds, recognise
voices and unable to remenber what he saw in the morning.

1. How will you choose a test. Depending on what test you are well versed with, that test is
choosen.

2. Which lobe is likely affected -Temporal lobe.

3. What are the tests available to test the affected lobe.

Tests for Eliciting Temporal Lobe Dysfunction

1) Deficits of visual integration

– Block design test

– Object assembly test

2) Verbal and Visual learning and memory functions test

– The verbal learning and memory functions test

– Visual learning and memory functions test

3) Benton’s visual retention test

4) Test of comprehension

5) Presence of nominal aphasia

6) Presence of conduction aphasia

– Sentence repetition test

• Different tools available for neuropsychological assessment


1.Luria-Nebraska

2.Halstead-reitan battery

3.PGI battery of brain dysfunction

4.NIMHANS Neuropsychological battery (Shobini L Rao)

5. AIIMS comprehensive neuropsychological battery for elderly in Hindi

Tests for Eliciting Frontal Lobe Dysfunction

1) Attention
20 | P a g e
– Spontaneous arousal of attention

– Distraction

– Excessive broadening/ narrowing of attention

2) Tests of visual search

– Visual scanning of numbers

– Visual scanning of pictures

– Visual exploration test

3) Mental set- Psychomotor perseveration

4) Psychomotor deficits

– Test of Optic-kinaestheticorganisation

– Test of optic-spatial organisation

– Kinetic melody disturbance

5) Deficits in working memory

– Test of mental control

– Delayed response tests

6) Deficits of ideational and design fluency test

7) Deficits in visuospatial planning tasks

– Bender gestalt test

– Alexander passalong test

– Object assembly test

– Maze tests

8) Frontal Amnesia

9) Expressive speech disturbances

10) Changes in voluntary activity, personality and affect

Tests for Eliciting Parietal Lobe Dysfunction

1) Tests for visuospatial perception

– Bender gestalt test

-Kohs block design

21 | P a g e
Presence of

– Apraxia (ideomotor, ideational and constructional)

– Agnosia (Visual object agnosia, prospagnosia, finger agnosia, autotopagnosia, hemisomatagnosia,


simultagnosia, visual inattention, astereognosia and left-right disorientation)

Tests of executive functions:

6) Phonemic fluency-controlled oral word association test (COWA)

7) Category Fluency-Animal names test

8) Design fluency-design fluency test

Working memory:

9) N back test (Verbal working memory and Visual working memory)

10) Self ordered pointing test

Planning

11) Tower of London test

Set shifting

12) Wisconsin card sorting test (WCST)

Response inhibition

13. Stroop test-NIMHANS version

Verbal comprehension

14) Token test

Tests of verbal Learning and memory:

15) Rey’s Auditory verbal learning test

16) Logical memory test

Visuo constructive ability

17) Complex figure test

Rey-Osterrieth test

Give subject a pen, place sheet with figure in front of subject, and say, “Please copy this design as
best you can.” Please do not use colored pencils for each element because this can be disruptive
and will not be consistent with other ADCs. Instead, please write notes to track progress. Have
subject make his or her copy in the lower half of the page below the figure. When design is
completed, leave the figure in front of the subject for 5 seconds and say “Be sure to remember this
design, because I’ll ask you to draw it again later from memory.”
22 | P a g e
Purpose- Visio spatial contruction ability and visual memory

Wisconsin Card sorting test – set shifting

In short, in the WCST, people have to classify cards according to different criteria. There are four
different ways to classify each card, and the only feedback is whether the classification is correct or
not. One can classify cards according to the color of its symbols, the shape of the symbols, or the
number of the shapes on each card. The classification rule changes every 10 cards, and this implies
that once the participant has figured out the rule, the participant will start making one or m ore
mistakes when the rule changes. The task measures how well people can adapt to the changing
rules.

Tower of london

Test for Planning

Topic: Anorexia nervosa


DSM-5 Diagnostic criteria for AN

Criteria No.1:

➢ Restriction of energy intake relative to requirements, leading to a significantly low body


weight in the context of age, sex, developmental trajectory, and physical health
➢ Significantly low weight is defined as a weight that is less than minimally normal or, for
children and adolescents, less than that minimally expected
Criteria No.2:

➢ Intense fear of gaining weight or of becoming fat, or persistent behavior that interferes with
weight gain, even though the patient’s weight is already significantly low
Criteria No. 3:

➢ Disturbance in the way in which one’s body weight or shape is experienced


Or

➢ Undue influence of body weight or shape on self-evaluation


Or

➢ persistent lack of recognition of the seriousness of the current low body weight
Changes from DSM IV to DSM-5:

1. Criterion A there is a focus on behaviors including restricting calorie intake, and the word
“refusal” is no longer included related to weight maintenance because of the incorrect and
possibly stigmatizing implication of intention on the part of the patient and because this
aspect may be not something that can accurately be assessed.

23 | P a g e
2. Criterion D requiring amenorrhea, or the absence of at least three menstrual cycles, has been
deleted as that criterion cannot be applied to:
➢ Males or premenarchal females
➢ Females taking oral contraceptives
➢ Postmenopausal females
➢ In some cases, individuals exhibit all other symptoms and signs of anorexia nervosa but
still report some menstrual activity
3. Removed the qualification of low weight being less than 85% ideal body weight
Specifiers or types of AN:

1. Restricting type

2. Binge-eating/purging type

Cardiovascular complications

➢ Cardiac complications are the most common cause of death


➢ The mortality rate is about 10%
➢ Cardiac effects from anorexia nervosa include profound bradycardia, hypotension,
decreased size of the cardiac silhouette, and decreased left ventricular mass associated with
abnormal systolic function
➢ An increased incidence of mitral valve prolapse without significant mitral regurgitation is also
observed
➢ Low potassium-dependent QT prolongation increases the risk of ventricular arrhythmia
➢ Electrocardiography (ECG) findings:
1. Sinus bradycardia
2. ST-segment elevation
3. T-wave flattening
4. low voltage, and rightward QRS axis
5. QT-interval prolongation: may be an indication for those at risk of cardiac
arrhythmias and sudden death
➢ Cardiac decompensation is greatest during the initial 2 weeks of refeeding, when the
myocardium cannot withstand the stress of an increased metabolic demand
➢ However, if the daily weight gain is 0.2-0.4 kg, then complications are limited
Thyroid Function:

24 | P a g e
➢ Thyroid function is also affected in patients with anorexia nervosa, with laboratory data
revealing a decrease in triiodothyronine (T3) and thyroxine (T4) and an increase in reverse
T3
➢ These changes are characteristic of the euthyroid sick syndrome
➢ hormonal replacement is not necessary
Neurobiology of Anorexia Nervosa:

➢ Neurobiologists hypothesize that disruption of both serotonergic and dopaminergic


pathways in the brain mediate the development of anorexia nervosa and may account for
the frequent coexistence of other psychological disturbances
➢ Anorexia nervosa is often heralded by a patient’s desire to lose an insignificant amount of
weight through dieting
➢ Once the weight loss is in progress, immunologic and hormonal factors, including leptins
(involved with signaling satiety) and alpha-melanocyte–stimulating hormone, may play a
role in the downward spiraling and maintenance of anorexia nervosa
➢ During prolonged food restriction in genetically vulnerable individuals, the ensuing
malnutrition perpetuated by the biochemical changes induced by weight loss (ie, ketosis)
further magnifies the impact of the malnutrition on the brain, owing to it being in a
starvation-illness mode
➢ Persistent states of starvation may result in biochemically based treatment resistance due
to neuroadaptive changes including increases in angiopoetin-like protein 6 (ANGPTL6) that
occur that increase the likelihood that anorexia nervosa will become chronic and persistent
Epidemiology:

➢ Anorexia nervosa is more common in women than in men, with a female-to-male ratio of
10-20:1 in developed countries
➢ Gay and bisexual males are more likely to have an eating disorder than heterosexual males,
but they are also more likely to have bulimia than anorexia
➢ Eighty-five percent of patients have onset of the disorder between the ages of 13 and 18
years
➢ Patients who are older at the time of onset of the disorder have a worse prognosis, as do
patients with an onset before age 11 years
Prognosis:

The prognosis of anorexia nervosa is guarded

Morbidity rates range from 10-20%

Only 50% of patients making a complete recovery

Of the remaining 50%:

➢ 20% remain emaciated


➢ 25% remain thin
25 | P a g e
➢ The remaining 10% become overweight or die of starvation
➢ However, mortality from the complications of starvation is less frequent in patients with
anorexia nervosa than is death from suicide
➢ A history of previous suicide attempts, physical pain, drug use, and laxative use may correlate
with a higher likelihood of suicide attempts
➢ Metacognitionand alexithymia plays a role in predicting adverse outcomes or suicide
➢ Recovery from anorexia nervosa generally is accompanied by resumption in growth
➢ There may be residual loss of height that is linear in nature
Psychiatric comorbidities are common with anorexia nervosa, with the following lifetime
incidence:

1. Depression (15-60%)
2. Anxiety disorders (20-60%)
3. Substance abuse (12-21%)
4. Personality disorders (20-80%)
SCOFF questionnaire

The SCOFF questionnaire, as follows, is a screening tool for eating disorders, with 1 point awarded
for every positive reply and a score greater than 2 indicating likely anorexia nervosa or bulimia:

• Do you make yourself Sick because you feel uncomfortably full?


• Do you worry you have lost Control over how much you eat?
• Have you lost more than One stone* in a 3-month period?
• Do you believe yourself to be Fat when others say you are too thin?
• Would you say that Food dominates your life?
Treatment:

➢ Younger individuals with anorexia nervosa, especially adolescents, may respond best to
family-based treatment (FBT), which appeared superior to individual therapy
➢ The recovery rate for adults appeared higher with CBT
➢ Specialist supportive clinical management (SSCM) and the Maudsley method
(MANTRA) showed an outpatient recovery rate of 15% in maintaining remission from
anorexia nervosa
The MOSAIC study - comparison of the Maudsley Model of Treatment for Adults with Anorexia
Nervosa (MANTRA) with Specialist Supportive Clinical Management (SSCM) in outpatients with
anorexia nervosa or eating disorder not otherwise specified, anorexia nervosa type: study protocol
for a randomized controlled trial.

Topic: Depot antipsychotics

26 | P a g e
1. Test Dose:

➢ For FGAs, a test dose consisting of a small dose of active drug in a small volume of oil serves
a dual purpose:
1. It is a test of the patient’s sensitivity to EPS
2. sensitivity to the base oil
➢ For SGAs, test doses may not be required less propensity to cause EPS and aqueous base not
known to be allergenic

2. Steady state plasma levels with LAI are generally achieved after 6-8 weeks
3. As per Cochrane systematic review, Zuclopenthixol is more effective than other FGA LAI
4. Olanzapine-
➢ Post injection syndrome
➢ Post-injection syndrome is caused due to accidental intravasation, and manifests when
plasma levels reach 600 μg/L and leads to delirium and somnolence
➢ Most of the reactions occur in first hour and observation is mandatory for 3 hours
5. The administration of IM procyclidine routinely with an Antipsychotic Depot/LAI is illogical as
the effects of the anticholinergic drug will wear off before plasma antipsychotic levels rise or
peak
6. Invega Trinza:
➢ The 3-month paliperidone palmitate (PPM–3) formulation can only be used if the patient
has been receiving 1-month paliperidone palmitate injections for at least 4 months
➢ It is administered 4 times a year, providing the longest interval of any approved LAI
7. Plasma half life of LAI:
➢ Risperidone- 4 days
➢ Fluphenazine – 10 days
➢ Haloperidol – 21 days
➢ Olanzapine – 30 days
➢ Aripiprazole 30 – 46 days
8. To reach a maintenance dose, the dose of the LAI should be reduced every 6 months
9. Aripiprazole lauroxil is a pro‐drug formulated to be administered at monthly, 6 weekly or 2‐
monthly intervals by IM injection into the deltoid or gluteal muscle depending on the dose
10. Injection site reaction is minimum with Risperidone LAI
11. Fluphenazine Decanoaterequires minimum time to reach steady state concentration

27 | P a g e
12. Rapid Neuroleptization (Psychotolysis) – Practice of administering hourly IM doses of
antipsychotic medications until marked sedation is achieved
Bedside cognitive assessment

Bed side scales validated for Indian population, which are culture free and more suitable for Indian
population:

1. HMSE
2. PGI memory scale
3. Vellor cognitive battery
4. Kolkata cognitive battery
5. ACER (Addenbrook) Malayalam version
Checklist before choosing the scale:

1. Check education status


2. Comprehension
3. Whether in delirium or not
4. Ethnicity and culture
5. Vision (correction if any)
6. Difficulty in writing or tremors (specially for clock drawing etc)
Cognitive assessment in visually Impaired:

1. Montreal cognitive assessment – Blind / MoCA- Blind


2. COGEVIS (COGnitive Evaluation in VISual impairment), a new scale developed to
accommodate impaired vision.
Neuropsychiatric manifestations of Parkinson’s disease

1. PD has 2.4 times higher risk of depression even before the diagnosis of PD is made
2. Risk is especially high in 3 years prior to diagnosis of PD
(Leentjens et al,2004 )
3. Specific risk factors for depression in PD
o Earlier age of onset
o More severe disability
o Presence of on/off fluctuations
o Higher dose of levodopa
o Cognitive dysfunction
o Right sided motor symptoms
o Family history of PD

Treatment:
➢ Response rate for SSRIs: 41% vs 57% for TCAs(Skapinakis et al, 2010)
28 | P a g e
➢ Pramipexole for anhedonia and depression in PD patients (Barone et al, 2010)
Psychosis in PD:
➢ Prevalence 8-40%
➢ Visual Hallucinations are the most common, seen cross-sectionally in 15% to 40% of the
patients
➢ Delusions are less common and usually occur with hallucinations; prevalence varies from 3%
to 30%
Risk factors
1. Older age

2. Disease severity

3. Sleep disturbance

4. Cognitive impairment

5. Dementia and/or depression

Topic: Sociology of suicide


Durkheim:

Classified different types of suicides on the basis ofdifferent types of relationship between the
individual andsociety, based on integration and regulation in society

(1) Egoistic suicide:

➢ This is the suicide of self-centred person who lacks altruisticfeelings and is usually cut off
from main stream of thesociety when there is too little integration
(2) Altruistic suicide:

➢ This type of suicide occurs when individuals and the groupare too close and intimate, when
there is too muchintegration.
(3) Anomic suicide:

➢ This type of suicide is due to little regulation in society


(4) Fatalistic suicide:

➢ This type of suicide is due to overregulation in society


Douglas:

➢ He argues that sociological analysis should focus onmeaning rather than social structure

29 | P a g e
➢ He argues that to understand suicide sociologically weshould examine the meaning of
suicide for the participantsusing diaries, suicide notes, psychiatrist’s notes andbiographies.
1. Suicide as reunion - release from cares/pressures
2. Suicide as atonement - transforming oneself for others
3. Suicide as revenge - most increased form in 20th century

Jean Beachler:

➢ Developed on the work of Douglas


➢ Hebelieved that suicide could only be explained throughpersonal factors rather than
external factors
➢ He concluded that suicide is both a response to aproblem and a method to try and solve a
problem
➢ Baechler also distinguished four different types of suicide:
1. Escapist – Flight from an intolerable situation.
2. Aggressive- As a way of harming other people, makeanother person feel
guilty.
3. Oblative- Sacrifice; giving their life to save others
4. Ludie – Deliberate risks are taken that might lead to death

Taylor:

Taylor concluded that suicide, and suicide attempts couldbe divided into two categories…

1. Etopic – inner directed, results from what the person thinksabout themselves
2. Symphysic – other directed, results from the person’srelationship with others
This results in four different types of suicide which are theresult of personal situations, not wider
society:

1. Submissive Suicide –certain about life and sees themselvesas already dead.
2. Thanatation Suicide –uncertain about themselves, thesuicide attempt is a gamble
3. Sacrifice Suicide – Certain that others have made their lifeunbearable
4. Appeal Suicide –person feeling uncertainty over theattitudes of others towards them

Topic: Poor Prognostic factors in Schizophrenia


Demographic factors
o Male

30 | P a g e
o Single
o Young age at onset
o Family history of Schizophrenia
o Comorbid substance use
Clinical features
o Poor premorbid adjustment
o Insidious onset
o Long duration of untreated psychosis
o Hebephrenic subtype
o Negative symptoms
o Cognitive impairment
o Absence of affective symptoms
o Poor insight
Other factors
o High expressed emotions in the family
o Poor adherence with treatment

Basic components of Psychoeducation


Assessing the knowledge of the patient and caregivers about etiology, treatment and
prognosis. Give them regular opportunities to ask questions, express their views and raise any
concerns they might have.
Introducing the discussion of schizophrenia into discussion. Discuss about the nature of the
illness, identification of any misapprehensions and explanation of the current meaning of the
term schizophrenia.
Discussing about various symptom dimension
Providing information about the etiology, ensure that misconceptions do not lead to
unnecessary guilt.

Providing information about the treatment in terms of available options, their


efficacy/effectiveness, side effects and duration of use and involving the patient and carers in
decision making.
Discussing about the importance of medication and treatment compliance
Providing information about possible course and long term outcome
Discussing about problems of substance use, marriage and other issues
Discussing about communication patterns, problem solving and disability benefits
Discussing about relapse and early signs of relapse
Discussing about day-to-day stress
Improving insight into the illness
Handling expressed emotions and improving communication
Enhancing adaptive coping to deal with persistent/residual symptoms

When the patient is not responding adequately to the medications:

31 | P a g e
Review diagnosis ( e.g. to look for organicity)
Review comorbidities (Substance use, medical comorbidities, psychiatric comorbidities)
Review treatment history (including drugs, doses, psychological therapies)
Using appropriate assessment scales
Detailed physical examination
Discuss patient views regarding treatment
Check for treatment adherence (including drug levels whenever possible)
Consider investigations including imaging

Topic: Metabolic syndrome


Q1. What is metabolic syndrome?

It is a multisystem disorder Obesity, Hypertension, Dyslipidaemia, Hyperuricaemia, Abnormalities


of glucose homeostasis (Insulin resistance, glucose intolerance, diabetes mellitus)

Q2. What are the criteria used for metabolic syndrome?

Various criteria have been proposed including WHO, NCEP, ATP, NIH etc. Generally we use
consensus criteria

3 or more of the following

• Fasting plasma glucose (FPG) >= 110 mg/dL (100 according to newer guidelines)
• Serum triglycerides of >= 150 mg/dL
• Serum HDL cholesterol M< 40, W<50 mg/dL
• Blood pressure of at least 130/85 mm Hg,
• Waist girth M >102, W >88 cm
• (NIH, NCEP-ATP III panel, 2001)
• BMI > 28.8 kg/m2 instead of waist girth
(NIH, NCEP-ATP III panel,2002)

Q3. What are the reasons for increased prevalence of MeS in SCZ?

• High prevalence of smoking


• Increased risk for Obesity and T2DM
• Sedentary lifestyle
• Poor dietary choices
• Limited utilization of non-psychiatrist specialist care
• Antipsychotic induced
Q4. What is the risk of MeS in Schizophrenia?

32 | P a g e
• Four fold increased risk1
• The overall prevalence of Metabolic Syndrome in SCZ is 32.5% 2

• Contributes to 50% of mortality in SCZ2


• Minimal difference3
• According to the different definitions
• Treatment setting (inpatient vs. outpatient),
• Country of origin
• Gender
[1-The Northern Finland 1966 Birth Cohort Study; 2- Papanastasiou 2013; 3- Mitchell et al. Schizophr
Bull. 2013 ]

Q4. What are the receptors implicated in antipsychotic induced obesity and Diabetes?

Atypical antipsychotic- induced metabolic SE

Receptor activity Side effect


5 HT 2c antagonism Weight gain, diabetes
5 HT 1a Weight gain
H 1 antagonism Weight gain, diabetes, sedation
D2 antagonism Weight gain, endocrine effects, EPS
M 1 antagonism Anticholinergic
M 3 antagonism Diabetes

Source: Nasrallah 2008 Mol Psychiatry

Q5. How frequently do we monitor the patient for antipsychotic adverse effects?

Baseline 1 month 3 month 4-6 months 12 months Annually

Height/ Weight/BMI Weight/BMI Weight/BMI Weight/BMI Weight/BMI


Weight/BMI

FBS/HbA1c FBS/HbA1c FBS/HbA1c FBS/HbA1c FBS/HbA1c

Lipid profile Lipids Lipids Lipids Lipids

LFTs LFTs LFTs

ECG ECG

Q6. What are the advantages and adverse effects of Metformin?


33 | P a g e
• Usual doses - 1000 – 1500 mg/day
• Significant weight loss (Mean 3.17 kg) and reduction in BMI
• Significant reduction in Waist Circumference (SMD 0.35) but not the waist hip ratio
• Improves fasting glucose significantly (SMD 0.65)
• Improves serum cholesterol (SMD 0.51), HDL (SMD 0.45) and Triglycerides (SMD 0.56) but
not LDL (SMD 0.03)
• No significant effects on the Blood Pressure
• Significant adverse effects included – Diarrhea (NNH 6) and Nausea/ Vomiting (NNH 16)
[references: Mizuno et al. 2014; Wu et al 2016]

Q7. What are the other pharmacologic treatment options in MeS?

Aripiprazole:

• 3 RCTs Meta-analysis ; Mean Weight loss of 2.13 Kg1


• Switching to Aripiprazole decreases odds of MeS (OR -1.748) and improves Framingham risk
score2
• FRS - twenty-four weeks of switch to aripiprazole resulted in calculated risk reduction in
predicted CHD events over 10 years from 7.0% to 5.2% (reduction of 25.7%) 2
• Cochrane review – Switch to Aripiprazole from Olanzapine leads to decreased weight,
improved BMI, improved fasting glucose3
• Contrario – No differences between Aripiprazole when compared with Olanzapine in terms
of rate of discontinuation and rates of MeS3
[1 – Mizuno et al. 2014; 2 – Mukundan et al. 2010 3 – Parabiaghi et al. 2016]

Topiramate:

• 100 – 400 mg
• May have additional role in improvement of psychopathology (Although some argue that it
may worsen psychopathology)
• Mean weight reduction of 3.14 kg
• Side effects - Paresthesia
[Correll et al. 2016]

Betahistine:

• 3 RCTs
• Maximum evidence in counteracting the H1 antagonism related obesity and therefore –
Clozapine, Olanzapine.
• High dose upto 144 mg; 37% reduction in mean weight gain (Upto 0.7 kg) barack 2016.
Improves daytime alertness1
34 | P a g e
• 48 mg/ day – attenuates weight gain upto 1.95 kg 2
• Does not decrease weight but aids in decreasing clozapine’s propensity for weight gain (3
kg). Does not work as well with other antipsychotics3
[1 – Barak 2016a; 2 – Barak 2016b; 3 – Smith 2018 ]

Single RCT – positive evidence for the following

• D- Fenfluramine1
• Reboxetine – Betahistine combination1
• Zonisamide1
• Phentermine + Topiramate (Qysmia)2
[1- Mizonu et al. 2014; 2- EQUIP and SEQUEL Trials ]

Other agents tried include:

• Reboxitine: Weight loss – 1.9 kg (2 RCTs)1


• Sibutramine: Weight loss – 2.86 kg (3 RCTs)3
• Melatonin and Ramelteon2
• Orlistat – OTC Medicine3
• Naltrexone-Bupropion
• Liraglutide
[1- Zheng 2020; 2- Kozirog et al. 2007; 3- Padwal et al. 2007]

Negative trials in MeS:

• Amantadine – 2 RCTs
• Fluoxetine – 2 RCTs
• Nizatidine – 4 RCTs
• Rosiglitazone – 2 RCTs
Q8. What are new molecules under investigation for use in MeS in Psychiatry?

• Tesofensine - NA, DA, and 5HT reuptake inhibitor


• Bupropion and zonisamide combination
• Pramlintide (Leptin analogue) and metreleptin (amylin analogue) combination
Q9. What are the indications for Bariatric surgery?

• BMI above 35 kg/m2 without co-morbidity


BMI above 40 kg/m2 with comorbidity

Topic: Weight gain due to psychotropics


Dr Sharanya B Shetty
35 | P a g e
Antidepressants- likely mediated by H1 and 5HT2C receptors
Paroxetine is associated with highest weight gain.
Dietary modification, exercise programs, switching to Bupropion, adding topiramate are few
strategies that can be employed if there is antidepressant induced weight gain.

For mood stabilizer like lithium, valproate induced weight gain.


Monitor weight and BMI before and during treatment.
Screen for diabetes, dyslipidemia, hypertension.
Associated with increased appetite.
Can be managed by exercise and dietary modification.
Rule out lithium induced subclinical hypothyroidism as a contributor to weight gain.
Switching to Lamotrigine ( weight neutral)
Adding topiramate to other mood stabilizer.
Antipsychotics:
Highest weight gain with clozapine, olanzapine.
Moderate weight gain with Risperidone, quetiapine
Low weight gain with Ziprasidone, Aripiprazole, Lurasidone.
Monitoring weight, BMI, waist circumference at baseline and 6monthly.
Management of antipsychotic induced weight gain can be done by,
switching to Aripiprazole, ziprasidone, lurasidone.
Adding Aripiprazole to existing treatment.
Lifestyle modification
Adding metformin
Adding Topiramate.

Topic: Opioid use disorders


Based on Case vignette the diagnosis is: Mental and behavioral disturbances due to opioid use

LOOK FOR FEATURES OF INTOXICATION LIKE :

Intoxication effects
O Initial euphoria
O Apathy &dysphoria follows.
O Psychomotor agitation or retardation
O Pupillary constriction (if pupillary dilation is seen the overdose may be very severe and anoxia has
set in)
O Drowsiness or coma
O Slurred speech
O Impairment in attention or memory
36 | P a g e
Look for withdrawal symptoms like :

Withdrawal symptoms (cold turkey): Opioid withdrawal is rarely fatal in a healthy adult. Morphine
and heroin withdrawal syndrome begins 6 to 8 hours after the last dose, peaks in 2 days and reduces
in a week usually.
O Dysphoric mood
O Nausea or vomiting
O Muscle aches
O Lacrimation or rhinorrhea
O Pupillary dilation, piloerection (gooseflesh), or sweating
O Diarrhea
O Yawning
O Fever
O Insomnia
Note that insomnia, bradycardia, temperature dysregulation, and a craving for opioids can persist
for months after an episode of withdrawal, necessitating maintenance methadone in number of
patients.

Differential diagnosis for opioid use disorders

• Patients with chronic pain or intermittent acute pain syndromes (e.g., sickle cell disease)
may also display opioid drug-seeking behavior. They may request specific opioids by name,
claiming it is the only drug that works, argue with medical staff that they want higher doses
of opioids, and seek medications from multiple doctors.
• It can be a diagnostic challenge to determine whether such patients are addicted to opioids,
selling all or some of their opioids (diversion), or truly in pain and seeking opioids only in
order to relieve undertreated pain. The latter possibility has been called pseudo-addiction,
and it can be differentiated from opioid addiction by careful longitudinal observation of
patient behavior, often with the help of outside informants and urine toxicology testing,
alongside continued appropriate attempts at pain relief. A physician–patient contract
outlining expectations and appropriate medication use is highly recommended throughout
this process. The diagnosis typically becomes clear over time, with either the patient
achieving adequate pain relief or exhibiting behavior indicating diversion, abuse, or
addiction.
• Opioid intoxication must be differentiated from mixed intoxications in which opioids play
only a minor role. In general, a failure to respond to modest doses of naloxone suggests that
intoxication is caused by a nonopioid.
• Some patients, more typically the elderly, may respond to therapeutic doses of a µ agonist
with dysphoria and confusion; such reactions are generally short lived. They are seen more
commonly with mixed agonist-antagonists. They should be considered atypical opioid
intoxications rather than opioid-induced intoxication delirium or opioid-induced psychotic
disorders, which, although listed in DSM-IV-TR, are quite rare.

37 | P a g e
• One possible exception is the state associated with the accumulation of toxic meperidine
metabolites. However, even in that case, the syndrome does not usually outlast the
metabolites and should probably be considered intoxication.

Other Opioid-Related Disorders


Opioid Intoxication Delirium
Opioid intoxication delirium is most likely to occur when opioids are used in high doses, are mixed
with other psychoactive compounds, or are used by a person with pre-existing brain damage.
Certain opioids, such as meperidine, have toxic metabolites that can accumulate, causing delirium
and sometimes causing seizures. Impaired renal function increases the likelihood of accumulation.

Opioid-Induced Psychotic Disorder


Opioid-induced psychotic disorder can begin during opioid intoxication. The DSM-IV-TR diagnostic
criteria are contained in its section on schizophrenia and other psychotic disorders. Clinicians can
specify whether hallucinations or delusions are the predominant symptoms and whether the onset
occurs during intoxication or withdrawal.
Opioid-Induced Mood Disorder
Opioid-induced mood disorder can begin during opioid intoxication or withdrawal and can result
from chronic use. Opioid-induced mood disorder symptoms may be of a manic, depressed, or mixed
nature. A person coming to psychiatric attention with opioid-induced mood disorder usually has
mixed symptoms, combining irritability, expansiveness, and depression. The DSM-IV-TR diagnostic
criteria are contained in its section on clinical features of mood disorders.
Some degree of depressed mood (hypophoria) typically occurs during and for several weeks after
opioid withdrawal. Opioid-induced mood disorder should not be diagnosed after opioid withdrawal
unless the severity of mood disturbance exceeds what is normally encountered or persists for more
than a few weeks and is of sufficient intensity to warrant independent clinical attention.
Opioid-Induced Sleep Disorder and Opioid-Induced Sexual Dysfunction
Opioid-induced sleep disorder and opioid-induced sexual dysfunction are diagnostic categories in
DSM-IV-TR. Hypersomnia is likely to be a more common sleep disorder among those given opioids
therapeutically, but disturbed sleep (insomnia) is a common complaint of patients maintained on
opioid agonists such as methadone.
The most common sexual dysfunction is likely to be impotence, but patients maintained on
methadone may complain of inability to achieve orgasm, rather than impotence.

Opioid-Related Disorder Not Otherwise Specified


DSM-IV-TR includes diagnoses for opioid-related disorders with symptoms of delirium, abnormal
mood, psychosis, abnormal sleep, and sexual dysfunction. Clinical situations that do not fit into
these categories are examples of appropriate cases for the use of the DSM-IV-TR diagnosis of opioid-
related disorder not otherwise specified .

Comorbidities of opioid use:

Medical Comorbidity
Opioid drugs, properly administered, are associated with few serious medical complications. Most
complications associated with opioid abuse are those associated with the route of administration.
Because opioid addicts—even physicians who have access to drugs and sterile materials—tend to
neglect the hygienic aspects of injecting, infections of skin and systemic organs are quite common.

38 | P a g e
Filtering illicit opioids through cigarette filters or wads of cotton and injecting materials intended
for oral use allows starch, talc, and other particulate contaminants to enter into the bloodstream.
These particulates can cause pulmonary emboli, which can eventually result in angiothrombotic
pulmonary hypertension and right ventricular failure.
Staphylococcal pneumonitis may also be related to septic emboli. Endocarditis and septicemia
involving lesions of the tricuspid or the aortic and mitral valves are frequent complications. Less
frequent, but equally serious complications are meningitis and brain abscess.
Other frequently seen infections that can be related to injecting the substance or sharing of needles
include viral hepatitis (particularly B and C), malaria, tetanus, osteomyelitis, and HIV. Syphilis
transmission has also been associated with sharing of needles, although most cases are probably
acquired in the usual fashion. False-positive tests for syphilis are also not uncommon among
injecting drug abusers.
Other complications associated with poor IV injecting technique include chronic edema of
extremities (e.g., puffy hands), probably due to lymphatic obstruction caused by contaminants, and
sclerosis of veins caused by the drugs or their dilutants.
Chronic lymphadenopathy was common among addicts even before the advent of HIV and was
also thought to be related to particulate contaminants.
Subcutaneous or intracutaneous injection (known as skin popping) may cause widespread
ulceration and disfigurement as a result of chemical necrosis or infection .These injecting techniques
may be used by addicts who have sclerosed their major veins.
Some drug users, determined to experience the effects of the drug used IV, switch to the use of
femoral and jugular veins when the surface veins of the arms and legs have become unusable.
Complications associated with injection of particulate contaminated material include pulmonary
hypertension (sometimes leading to corpulmonale), occasionally seen in heroin smokers.
Additional medical complications are likely to be due to contamination of illicit opioids with other
chemical substances.
A number of changes found at autopsy, such as degeneration of the globuspallidus and necrosis of
spinal gray matter, may fall into this category. Occasionally there are clinical manifestations in
those users surviving overdose experiences. Examples are transverse myelitis, amblyopia, plexitis,
peripheral neuropathy, parkinsonian syndromes, intellectual impairment, and personality
changes.
Pathological changes in muscles and degeneration of peripheral nerves have als o been seen. Illicit
laboratories sometimes produce opioid-like agents that are extremely toxic or that are so potent
that even small doses are lethal.. Although not technically a contaminant, the illicit fentanyl analog,
3-methylfentanyl (known as China White), is 1,000 times more potent than morphine and may have
been responsible for several hundred overdose deaths.

Cardiovascular Effects
Some opioids can affect heart rhythm by lengthening the QT interval. This effect is caused by
inhibition of a specific K+ ion channel. Large increases in QT intervals can cause serious arrhythmias,
such as torsade des points. LAAM and methadone appear to have the lowest margin of safety,
producing QT lengthening at plasma levels that are not far above those achieved in some individuals.
Torsade de pointes has been reported with both LAAM and methadone. Similar concerns with
buprenorphine have not been noted.

HIV
The advent of HIV has changed the patterns of mortality among drug users. In some areas of the
United States and Europe, acquired immune deficiency syndrome (AIDS) is now an important factor
in mortality among injecting drug users. HIV seroprevalence among users has a wide geographic
39 | P a g e
variability and is lower in those who have been continuously in drug treatment. Although there are
effective medication regimens to delay the progression of HIV to AIDS, HIV-positive opioid users
have numerous barriers in obtaining medical care and have high rates of noncompliance. The finding
that not all drug users who shared needles were infected with HIV stimulated vigorous prevention
efforts aimed at recruiting patients into treatment and teaching them how to avoid infection by
cleaning injection equipment properly, not sharing equipment, and not participating in high-risk sex.
Needle-exchange programs send outreach community workers into neighborhoods that have
large numbers of IV drug users and offer clean needles coupled with specific instruction in how to
reduce the spread of HIV in exchange for used needles. The available evidence indicates that those
who inject drugs will accept teaching on how to use sterile equipment and participate in needle-
exchange programs to avoid disease if the equipment is available at reasonable cost.

Tuberculosis (TB)
Even before the HIV epidemic, the incidence of TB was higher among heroin addicts than in the
general population. Patients with compromised immune systems are far more vulnerable to
developing active TB once infected, and poor compliance with antitubercular medication has led to
the emergence of drug-resistant strains of the tubercle bacillus. According to the World Health
Organization, approximately one third of those with HIV/AIDS worldwide are infected with TB, and
without proper treatment 90 percent die within a month of contracting TB.

Liver Disease
Many opioid addicts who inject have a low-level chronic hepatitis without jaundice and may have
abnormal liver function tests. In seroprevalence surveys conducted by the Centers for Disease
Control and Prevention (CDC), markers of hepatitis B and C infection have been found in sera from
60 to 80 percent of drug injectors. Sharing straws used to snort opioids is also a risk factor for
hepatitis C transmission. Abnormal liver function tests, which are found in approximately two of
three heroin addicts, may persist for long periods after the cessation of injection. Comorbid
alcoholism may, in some cases, contribute to the liver disease.

Chronic Pain
Chronic pain within opioid-dependent populations is common and not surprising given the high
rates of trauma and other medical morbidity associated with drug addiction. In one study of 248
methadone maintenance patients, 61.3 percent reported chronic pain, the majority from a
nonmalignant source. This group reported higher rates of anxiety, depression, irritability, and
medical problems than their peers without pain

Opioids and the Immune System


µ-Opioid actions at CNS receptors can produce immunosuppressive effects (e.g., decreased natural
killer–cell activity). Opioid receptors are also found on lymphocytes and macrophages, and
naloxone-reversible opioid effects can be demonstrated on white cells in vitro. In heroin addicts,
there are changes in the ratio of helper to suppressor T cells and a suppression of cell-mediated
immunity. The relation to opioid use is still unclear; the effects are probably more related to
unhygienic injection practices.
Natural killer–cell activity and immunoglobulins G and M are within normal limits in former heroin
users maintained for several years on methadone, although abnormalities are typically observed
among heroin addicts. In animals, the effects of buprenorphine on the immune system appear to
be less when compared to full opioid agonists like morphine; however, there are no human data to
date suggesting that these differences are clinically meaningful.

40 | P a g e
Psychiatric Comorbidity
The high prevalence of additional psychiatric disorders among treated opioid-dependent patients
has now been repeatedly confirmed. Currently, no subtypology of opioid-dependent patients based
on psychopathology has been proposed. However, the type and severity of those additional
diagnoses can influence the course of the disorder and the kind of treatment that is most likely to
be effective.
Among opioid addicts seeking treatment at a program in New Haven, Connecticut, in the 1980s, 87
percent met Research Diagnostic Criteria (RDC) for a psychiatric disorder, in addition to opioid
dependence, at some point in their lives.

Other substance-related diagnoses, such as alcohol dependence, were considered additional


disorders. A study of addicts seeking methadone treatment in Baltimore in the early 1990s found
(using DSM-III-R criteria) a 24 percent lifetime prevalence of non–substance-related Axis I disorders,
a 35 percent lifetime prevalence of Axis II disorders, and a 47 percent total prevalence of additional
non–substance-related diagnoses. In both studies, the most common diagnoses were mood
disorders, alcoholism, antisocial personality disorder, and anxiety disorders (Tables 11.10-6, 11.10-
7, and 11.10-8).
In the Baltimore study, cocaine dependence (65 percent) was actually higher than alcohol
dependence (50 percent). Multiple psychiatric diagnoses for a given patient were common.
Researchers in Europe and Australia report similar overall distributions of psychiatric disorders
among opioid users seeking treatment.
Among women, depression, anxiety disorders, and borderline personality d isorder were
significantly more common than among male patients, and alcoholism, cannabis misuse, and
antisocial personality disorder were significantly less common.
The rates of current psychiatric illnesses are obviously lower than lifetime rates and vary with the
treatment setting. Among the methadone patients in the Baltimore study, 5.0 percent of men and
11.2 percent of women met criteria for a current diagnosis of a non–substance-related Axis I
disorder, most commonly a mood or anxiety disorder.
Forty percent of the sample had a current diagnosis of cocaine dependence, and 25 percent
concurrently experienced alcoholism. Other studies have confirmed rates of current major
depressive disorder in the range of 17 to 20 percent and concurrent rates of alc oholism (alcohol
abuse or dependence) of 20 to 30 percent. Although stimulant dependence (other than cocaine
dependence) was not seen among the patients in Baltimore, it occurs frequently in the western
United States, where methamphetamine use is more prevalent.

Methadone Substitution (Subject to availability)

• Used for withdrawal from heroin, fentanyl, or any other opiate. • Treatment of choice for many
populations, including those with many treatment failures.

• Opiate-dependent inpatients being treated for an acute medical illness may be administered
methadone if opiate withdrawal would complicate treatment of their medical condition.

(a) SETTING.

(i) Inpatient drug treatment program licensed for methadone detoxification.

• Starting dose of 30-40 mg q day, taken orally

• 10 mg administered 4 times daily, with observation for 2 hours following each dose. If patient is
sleepy, decrease to 5 mg. If patient shows objective signs of withdrawal, increase dose to 15 mg.
41 | P a g e
• After 24 hours, withdraw methadone 5 mg per day. (Most patients are withdrawn over 8 days.)
(ii) Outpatient methadone detoxification clinic.

• 20 mg, given orally twice daily is usual starting point. • After the second day, tapered by 2.5 mg
per day.

(b) SHORT-TERM DETOXIFICATION.

• Patients may not take their methadone home.

• Counselor to monitor progress toward the goal of short-term detoxification and to provide a drug
treatment referral.

• Patient must wait at least 7 days between conclusion of one such treatment episode before
starting another.

(c) LONG-TERM DETOXIFICATION. • Longer than 30 days, but not in excess of 180 days.

• Conditions:

• Patient must be under observation while ingesting the methadone for at least 6 days a week.

• Physician must document in record that short-term detox is not of sufficiently long enough
duration to provide for rehabilitation.

• Initial drug screen required. Additional random urine screens monthly.

• Initial treatment plan and monthly treatment plan evaluation.

• Patient must wait at least 7 days after concluding a long-term treatment episode before beginning
another.

Methadone Suboxone
Full agonist Partial agonist
Limited access in non urban areas Can be prescribed by a trained physician
Long half life Long half life
No ceiling effect Less dependency
Higher rates of taking illict opioids duringLess severe withdrawal
treatment
Provides more efficient relief form withdrawaLower risk of fatal toxicity
symptoms
Standard care for pregnant/ breast feedingPreferred treatment for patients with higher risk
women of toxicity.

Buprenorphine plus naloxone

• Buprenorphine is a safe and effective medication for the treatment of opioid withdrawal.
The parenteral form is only approved for the treatment of pain and its off-label use is
discouraged since there are now sublingual tablets available that are approved for the
treatment of opioid withdrawal and opioid addiction.

42 | P a g e
• A monotherapy product (Subutex) and a combination therapy product that includes
naloxone (Suboxone). Each formulation is available in two buprenorphine dosage strengths:
A small tablet (2 mg) and large tablet (8 mg).
• The combination formulation also contains naloxone in a buprenorphine to naloxone dose
ratio of four to one (i.e., 2.0 mg naloxone with 8 mg of buprenorphine and .5 mg of naloxone
with 2 mg of buprenorphine).
• Naloxone has little bioavailability by the sublingual route. It was added to the combination
tablet to decrease the risk of diversion and misuse of the medication because if the
medication is misused and injected, naloxone will be 100 percent bioavailable and may
precipitate opioid withdrawal in a person who is physically dependent.
• In an inpatient setting, buprenorphine withdrawal typically occurs over 3 to 5 days. Patients
can initially be stabilized on 8 mg per day of buprenorphine with subsequent 2 mg per day
dose decreases. Buprenorphine has higher patient acceptability than clonidine a nd is
generally well tolerated. However, it is possible that because it has a relatively long duration
of action, evidence of withdrawal distress from rapid inpatient detoxifications may not be
evident until several days after the patient leaves the hospital.
• In the outpatient setting, the goals with buprenorphine withdrawal are the same as with
methadone detoxification. An initial stabilization dose should be achieved that is associated
with objective opioid abstinence, and subsequent dose decreases should then occur
gradually. The stabilization dose will vary between patients, but will likely fall within the
range of 8 to 32 mg per day. The first day's dosing should occur under medical supervision.
• This is because buprenorphine is a partial opioid agonist, and the first dose can precipitate
mild opioid withdrawal symptoms under certain circumstances.
• To minimize the risk of buprenorphine-precipitated withdrawal, it is recommended that the
first dose be given when the patient is in mild spontaneous withdrawal, which is typically 12
hours after the last dose of a short-acting opioid or 24 hours after a long-acting opioid like
methadone. The total dose on the first day is typically not greater than 8 mg.
• Subsequent daily dose increases of 2 to 4 mg are recommended to relieve opioid withdrawal
and initiate opioid abstinence. Subsequently, dose decreases can occur.
• As the smallest incremental dose of buprenorphine is 2 mg, and tablets are not scored and
do not break easily, dose reductions should occur in 2-mg increments over a period of several
weeks. Like methadone, more gradual tapers are recommended.
• There are some reports of using a single high dose (e.g., one dose of 32 mg buprenorphine
solution) as single loading doses without further buprenorphine dosing, but the
effectiveness of this procedure is not well documented and controlled studies of outpatient
buprenorphine detoxification procedures are lacking.
• Finally, it is important to note that concurrent treatment with sedating drugs—and, most
especially, benzodiazepines—is strongly discouraged in patients being treated on an
outpatient basis with buprenorphine (or methadone) because of risk of significant
respiratory depression if the patient takes more than what was intended or misuses these
drugs by parenteral routes.

43 | P a g e
Clonidine-assisted detoxification.

Clonidine is a non-opioid antihypertensive drug that reduces symptoms of opioid withdrawal


(nausea, vomiting, diarrhea, cramps, sweating) by reducing noradrenergic hyperactivity.

It does not alleviate myalgia, insomnia, or drug craving. Some patients experience profound
hypotension even at low doses.

(a) PROCEDURE. • 0.1 to 0.3 mg clonidine in 3 divided doses on day 1


• Dose adjusted until withdrawal symptoms are reduced

• Monitoring for hypotension and sedation necessary.

If BP falls below 90/60 mm Hg, next dose should be withheld.

(b) DURATION. • 4-6 days for short-acting opioids (heroin)


• 0-14 days for longer-acting opioids (methadone)

(c) ADVANTAGES OVER METHADONE.


• Does not produce opioid-like tolerance or physical dependence

• Avoids postmethadone rebound in withdrawal symptoms

• Patients completing a clonidine-assisted withdrawal can be immediately given an opioid


antagonist (Naltrexone) if indicated

• May be administered in patch form (subject to availability)

(d) DISADVANTAGES
• Side effects: insomnia, sedation, hypotension

• Will not ameliorate withdrawal symptoms of insomnia and muscle pain

• Low rate of completion for clonidine-treated outpatients, roughly comparable to that with
methadone.

(e) CONTRAINDICATIONS.
• Acute or chronic cardiac disorders

• Renal or metabolic disease

• Moderate to severe hypotension

• Hypersensitivity to clonidine

(f) SETTING.
• Easier in inpatient setting, but outpatient detoxification with clonidine is a reasonable
approach with experienced staff.

Outpatients should not be given more than a 3-day supply of clonidine for unsupervised use.

44 | P a g e
• The clonidine transdermal patch (subject to availability) comes in 3 sizes and delivers an
amount of drug equivalent to twice daily dosing with 0.1, 0.2, or 0.3 mg of oral clonidine (over
24 hours).

One patch lasts for up to 7 days.

The patch minimizes drug cravings, eliminates disruptions caused by administration of


medication, overcomes the problem of missed doses, and prevents the buildup of withdrawal
symptoms during the night.

(3) Clonidine-Naltrexone ultrarapid withdrawal.

• Safe and effective for rapidly withdrawing patients from heroin or methadone.

• Naltrexone-precipitated withdrawal avoided by pretreating patient with clonidine.

• Most useful for patients in transition to narcotic antagonist treatment.

• Limitations: • Potential severity of Naltrexone-induced withdrawal: need to monitor patients


for 8 hours on day 1

• Need for careful BP monitoring during entire detoxification procedure.

1mg of methadone = 1-2 mg of heroin = 3-4 mg of morphine = 30 mg of codeine = 20 mg of


meperidine.

NACO ( National AIDS Control Organization ) AND OPIOID SUBSTITUTION TREATMENT

• Injecting Drug Users (IDUs) have emerged as an important high risk group with potenti
al of contracting and transmitting HIV. The current seroprevalence of HIV among
IDUs is 6.96%, as per the NACO 2006 sentinel surveillance data. HIV prevalence in IDUs
has crossed the threshold for a ‘concentrated epidemic’ i.e. 5% in many places
where surveillance was carried out. IDUs are at increased risk
of HIV because of both unsafe injecting and sexual practice. The unsafe injecting practi
ces
include sharing of needles, syringes and other paraphernalia. The strategy adopted in
NACP III to prevent transmission of HIV among IDUs is ‘harm reduction’.
• The harm reduction strategy includes Needle Syringe Exchange programme
(NSEP), Behaviour Change Communication (BCC), Outreach, Condom Promotion and
Substitution therapy. Substitution therapy with opioids (Opioid Substitution
Therapy, OST) is a well accepted treatment strategy for harm minimisation in IDUs.
• OST has been shown in several
studies from around the world to reduce the prevalence of HIV and HIV risk behaviours
among IDUs. OST is available in various parts of the world.

45 | P a g e
• The two most commonly used opioid medications for OST are Buprenorphine and Met
hadone, both of which have been found to be effective in HIV prevention. Buprenorphi
ne scores over methadone in that buprenorphine, being a partial agonist, decreases the
risk of respiratory depression with overdose.
• Buprenorphine substitution has also been shown to improve retention in treatment.
Buprenorphine is a scheduled drug listed as a ‘psychotropic substance’ under the Narc
otic Drugs and
Psychotropic Substances (NDPS)Act and hence its manufacturing, distribution, sale and
consumption, like other opioids, are controlled.
• The NDPS act also has provision to establish treatment or deaddiction centres by
the Government and the use of psychotropic substances such as
Buprenorphine for medical and scientific purpose to treat ‘addicts’. Rationale for Opioid
Substitution Therapy
Opioid substitution therapy involves replacing the client’s primary drug of use (opioid)
with a medically safe drug or the same opioid in a safer mode of administration under
medical supervision. In OST, an opioid (such as heroin), which is
unsafe, requiring repeated administration through unsafe/ hazardous route is
substituted with a medication (such as Buprenorphine) which is long acting, and
safer, and administered through oral / sublingual route. The medication used in OST
helps the client not to experience either withdrawal or euphoria (‘high’). As OST
helps in achieving a comfortable level, the client stops injecting drug, thus
preventing the potential harm of contracting HIV and other diseases
transmitted through injecting route (e.g. Hepatitis B, Hepatitis Intro d u ctio n
2 Substitution Therapy with Buprenorphine for Opioid Injectin g Drug U sers
C). While on OST, clients do not require to spend all their time looking for their next ‘fix’
or injection. Thus, they can be engaged in other activities
including counselling and group discussions, which help also in delivering Behaviour Ch
ange Communication. In addition, there is also an improvement in the psychosocial
status of the clients, leading to an overall improvement in quality of life.

Topic: Female sexual disorders


Changes in diagnostic criteria from DSM IV to DSM 5

46 | P a g e
“Associated features” /Specifiers

It is subdivided into five categories:

1) Partner factors (e.g., partner sexual problem; partner health status);

2) Relationship factors (e.g., poor communication, discrepancies in desire for sexual activity);

3) Individual vulnerability factors (e.g., poor body image; history of sexual or emotional abuse),
psychiatric comorbidity (e.g., depression; anxiety), or stressors (e.g., job loss; bereavement);

4) Cultural or religious factors (e.g., inhibitions related to prohibitions against sexual activity or
pleasure; attitudes toward sexuality); and finally

5) Medical factors relevant to prognosis, course, or treatment.

Along with these, the symptoms have to be experienced more than 75% of the time , for more than
6 months and should have caused significant distress.

47 | P a g e
27yr old lady who is going to get married in 2months has come to you with complains of fear of
having sex and fear of pain. She was referred to you by gynecologist. She is as of now happy with
her partner and does not report any major conflict. No h/o medical or surgical problems. How to
proceed?

Patient reports having these complains for the last 8 months, and since last 2 months is more
anxious since marriage is approaching

She reports having this fear since school days, she did not

Experience much pain during examination but was very Phobia

Anxious and understands there is no reason for the pain

Pain is inflicted by her neighbors and she is sure of this because ? Schizophrenia

She can hear them conspiring the same

She does not want to be scared, but keeps getting this thought that she

will not be able to have sex. She has to tell herself the opposite exactly 10 times and ?OCD

her anxiety reduces. A lot of times she has lost count and spent hours together just telling herself
opposite of what she thinks

History of child sexual abuse, abusive parents low self-esteem Socio-cultural factors

Complains of severe pain and spasm around the genitals during gynecological examination. Similar
experiences in past when examination or insertion was tried. Due to these experiences she is
anxious now. Was advised use of dilator

Genito-pelvic pain disorder/ Vaginismus

Topic: Breaking bad news – Kayes model


Breaking bad news comes in various forms and is a regrettable but important duty that must be
done conscientiously. Your performance needs planning and rehearsal in order for it to go well for
both you and the patient.

Don’t forget it may feature in your CSA!

Prepare for the consultation

48 | P a g e
If you get the news, then it is your responsibility to tell the patient. Consider using your receptionist
to call the patient and ask them to make an appointment. If you ring them instead then remember
that bad news must be transmitted in person and not over the telephone, although it can be used
as an opportunity to ‘fire a warning shot’.

• The patient may like to be accompanied by a spouse or someone close to them.


• Ensure you have protected time. This may mean turning off bleeps or mobile phones and
making sure the reception and other team members know that you are not to be disturbed.
• It usually takes longer than a normal appointment – so book a double appointment or make
it the last one in your surgery. If it happens during a surgery in which you are running late,
run later.
• You usually need to get the patient to return in the near future to discuss the matter in more
detail but do not seem rushed or too eager to get on. To you this may be just ‘another day
at the office’ but to the patient and the family it is a pivotal day of their lives.
• Make yourself as fully conversant with the facts as possible. This means the facts about this
case like the exact type of tumour and stage as well as more general issues about the disease.
This applies not just to cancer but to all diseases that fall into the bad news category.
You may feel that this is such an intimate moment that you do not want a registrar or medical
student present but if they can be unobtrusive in the background this could be an important learning
opportunity.

Kayes model for structuring the consultation has stood the test of time

Preparation

Know all the facts.

Ensure privacy.

Find out who the patient wants present.

Introduce yourself.

What Does the Patient Know?

Open ended questions.

Statements may make the best questions.

“How did it all start?

Is More Information Wanted?

Not forced on them.

“Would you like me to explain a bit more?”

Give a Warning Shot

Not straight out with it!

“I’m afraid it looks rather serious”

Allow Denial

49 | P a g e
Allow the patient to control the amount of information they receive.

Explain If Requested

Step by step.

Detail will not be remembered but the way you explain it will be.

Listen to Concerns

“What are your concerns at the moment?”

Allow time and space for answers.

Encourage Ventilation of Feelings

Acknowledge the feelings.

Non-judgmental.

Vital step for patient satisfaction.

Summarise

Concerns.

Plans for treatment.

Foster hope.

? Written information.

Offer Further

Availability and information.

Future needs will change.

Avoiding pitfalls

There are traps for the unwary that must be avoided.

• Do not avoid seeing the patient or leave them anxiously waiting for news. Sometimes
anticipation can be worse than even the worst reality. Treat others as you would wish to
be treated yourself.
• Read the notes. Get the facts before you start.
• You need privacy and no interruptions. In a hospital, hospice or residential home, make sure
you will not be disturbed. If necessary switch off phones or bleeps.
• Be factual but sympathetic. Always be empathetic however you may feel personally. You
may feel that the person's lifestyle makes him responsible for his HIV, cirrhosis or lung
cancer but never let it show. Better still, do not let yourself become judgmental. For as
you judge others so too you will be judged and Let he who is without sin cast the first
stone.
• Give time for the information to sink in and the opportunity to ask questions before moving
on. Do not seem rushed.
50 | P a g e
• If the patient does not seem able to take any more be prepared to end the consultation and
to take it up again later. Look for all the cues, verbal or others. "Do you want to leave it
for now and we can discuss it more when you are feeling ready?" Perhaps they would like
you to speak to someone else or to have someone with them for the next meeting.
• If asked the question, "How long have I got to live?" never be precise. You will always be
wrong. To the patient and family 6 weeks means 6 weeks and not 5 weeks or 7.
• Never say that nothing can be done or the patient will lose all hope.
• If you have written material to give that may be useful.
• Whilst trying to be positive never lose track of the fact that this is a serious and potentially
fatal disease. Be optimistic but do not promise success or anything else that may not be
delivered.
• Be mindful of the recent changes in legislation concerning consent. If the patient 'lacks
capacity' under the terms of the The Mental Capacity Act 2005 (enacted 2007), you must
act in the best interests of the patient. You must take all factors into account, including
the views of carers, persons with lasting powers of attorney, deputies appointed by the
court, and anyone whom the patient has previously indicated should be consulted about
their views.

Topic: Soft Bipolar disorders

Prevalence of ‘Spectrum’/‘Soft’ Bipolarity

The lifetime prevalence of Bipolar disorder - I (BP-I; defined as presence of depression and atleast
one manic episode) is 1% in general population surveys.

However, when we focus on the entire spectrum of bipolar disorders, the prevalence is much higher.
The prevalence for the bipolar disorder II (BP-II; defined as presence of depression and atleast a
hypomania)was found to be 1.67% in a large-scale epidemiological survey in U.S.

The secondary analyses from these landmark studies revealed that if we consider the prevalence of
entire bipolar ‘spectrum’ disorders, it was found to be about 6.4% in the community setting implying
that the sub threshold cases are atleast five times more common than BP-I and BP-II.

The findings were further replicated in U.S National Comorbidity Survey- Replication study with the
lifetime and 12-month prevalence estimates being 1.0% and 0.6% for BP-I, 1.1% and 0.8% for B P-II,
and as high as 2.4% and 1.4% for sub-threshold BPD.15 In terms of clinic prevalence, on applying
the broader criteria for ‘spectrum’ bipolarity, it was seen that upto half of the patients with current
diagnosis of depression may be bipolar spectrum disorders. Timely and accurate diagnosis may
facilitate improved management and outcome for these patients.

These alarming figures thereby highlight the importance of focusing not only on the ‘classical’
bipolar disorder, but also a wide variety of difficult-to-recognize /easy-to-overlook bipolar spectrum
disorders for its diagnostic relevance, and clinical as well as public health importance.

Key Diagnostic Schema of Bipolar Spectrum/Soft Bipolarity

Key diagnostic schema of BSD were given by researchers, notably Klerman,16 Akiskal& Pinto17 and
more recently, by Ghaemi and co-researchers.9 Akiskal and Pinto in their landmark paper on ‘bipolar

51 | P a g e
spectrum disorders’ have described the various subtypes of bipolar disorder from I to IV (Box 1).
Additionally, the types I 1/2, II ½ , III1/2 , V and VI have been proposed as well.

More recently, Ghaemiet al proposed diagnostic criteria for bipolar spectrum disorders as follows:
A. At least one major depressive episode
B. No spontaneous hypomanic or manic episodes
C. Either of the following, plus 2 items from criterion D, or both of the following
Plus 1 item from criterion D should be present
1. A family history of bipolar disorder in a first degree relative
2. Antidepressant-induced mania or hypomania
D. If no items from criterion C are present,6 of the following 9 criteria are needed.
1. Hyperthymic personality (at baseline, non depressed state)
2. Recurrent major depressive episodes (>3)
3. Brief major depressive episodes (on average,< 3 months)
4. Atypical depressive symptoms (DSM-IV criteria)
5. Psychotic major depressive episodes
6. Early age of onset of major depressive episode (<25 years)
7. Post partum depression
8. Antidepressant “wear off” (acute but not prophylactic response
9. Lack of response to 3 or more antidepressant trials

E. The closely related concept of soft bipolarity was first given by Akiskal and Mallya.18 The
chief attributes of soft bipolarity are same as bipolar spectrum (only difference being that
the Bipolar I subtype is excluded from rubric of soft bipolarity while included in bipolar
spectrum). The severity of elated phases never reaches level of manic or manic mixed states,
and remains at clinical or sub- threshold hypomanic level. Patients with soft bipolarity are
often referred to as “pseudo-unipolar depression”, and may go undetected for years.

F. Perugi and Akiskal later on have further expanded soft bipolarity encompassing a variety of
conditions ranging from mood, anxiety, impulse control, and eating disorders with
underlying cyclothymic-anxious-sensitive disposition, mood reactivity and interpersonal
sensitivity, though this concept is more of research significance as of now.

Screening for Soft-Bipolar/Bipolar Spectrum

G. The screening of patients for depression has been discussed in more detail in a previous
review paper in JAPI.21 Asking just a few more questions focusing on any periods (few days
to few weeks, even few hours at times) with elated mood, feeling over-energetic, overactive
and decreased need for sleep etc may help delineate the subgroup of patients with bipolar
spectrum who present to the physicians with current depressive symptoms.

52 | P a g e
H. A two-question screen for mood lability may help identify bipolar II disorder patients if there
is positive response to at least one question indicating mood lability. The questions are as
follows:

I. “Are you a person who frequently experiences ups and downs in mood over life?”

J. “Do these mood swings occur without cause?”

K. Along with these presence of atypical symptoms, reversed vegetative symptoms like
hyperphagia and hypersomnia during depression instead of loss of appetite and sleep, past
history of elevated mood and increased activity, family history of bipolarity or treatment
induced mood symptoms should be enquired about, using questions such as:“Have you had
periods of feeling so happy or energetic that your friends told you were talking too fast or
that you were too ‘hyper’ than your usual self?”

L. Mood Disorder Questionnaire is one of the most commonly used screening tool. It has 17
questions pertaining to hypomanic symptoms, presence of several of these symptoms in the
same time duration and the impact of these symptoms. When structured diagnostic
interviews were applied to patients on antidepressant treatment attending family physicians
30% of the patients were found to be having bipolar disorder.

M. The Bipolar Index is another assessment tool which evaluates across five domains namely
signs and symptoms, age of onset, course of illness, response to treatment, and family
history and researchers have found that a score ≥50 had good sensitivity and specificity for
identifying bipolar disorders.

N. Screening using the relevant clinical questions or instruments should be done in all patients
presenting with major depressive episode in a busy outpatient setting.

Topic: Laboratory investigations for use in patients presenting with


neuropsychiatric manifestations

Test Indication
Hemoglobin Fatigue
Impaired cognition
White Blood Cells Psychotropic medications
Leucocytosis- lithium, NMS, infection
Leucopenia- Clozapine, carbamazepine

Platelets Psychotropic medications


Thrombocytopenia- valproate, carbamazepine
Impaired platelet aggregation and bleeding risk with
SSRIs

MCV Elevated in alcohol use, VitB12 and folate deficiency,


HIV
Coagulation factors (PT, PTT, INR) Assessment of hepatic synthetic function- Elevated PT
in Cirrhosis

53 | P a g e
Liver function tests
ALT General neuropsychiatric work up
AST Evaluation of liver disease severity, impairing
Alkaline Phosphatase metabolism of many psychotropics
GGT Alcohol
Bilirubin Delirium
Hepatotoxic psychotropic medications (valproate
carbamazepine, olanzapine)
Elevated AST- hepatic disease (in alcoholic live
disease AST:ALT ratio is 2:1), heart failure, eclampsia
Elevated ALT- hepatic disease
Decreased AST and ALT in vitB6 deficiency
Elevated Alk Phos- hepatic and bone disease
hyperparathyroidism, phenothiazine use.
Elevated bilirubin- hepatic disease
Elevated GGT- alcohol use, hepatobiliary disease

Albumin and total protein


Prior to use of highly protein bound psychotropics in
patients with liver disease
Low levels result in high fraction of unbound drug
which may result in more active and side effects.
Elevated TP- dehydration, inflammation, chronic
infection, multiple myeloma
Elevated ALB- dehydration
Decreased ALB, TP- cirrhosis/hepatic disease
malnutrition, renal disease

Sodium

General neuropsychiatric work up


Decreased- SIADH, polydipsia, carbamazepine
oxcarbazepine, SSRIs
Hyponatremia associated with delirium

Prolactin

Antipsychotics
Poor libido
Menstrual irregularities
Galactorrhea
Elevated with the use of antipsychotics, post seizure
prolactinoma

Metal assays and environmental toxins


Copper studies Neuopsychiatric symptoms with liver disease
delirium

54 | P a g e
Lead Unexplained depressive symptoms, neuropsychiatric
symptoms and somatic complaints with possible lead
exposure

Mercury Unexplained mood symptoms, cognitive symptoms and


somatic complaints in conjunction with possible
mercury exposure

HIV Neuropsychiatric symptoms in high risk patients

Rapid plasma reagin Neuropsychiatric symptoms in high risk patients

Hepatitis C core antibody Intravenous drug use


high risk sexual practices
unexplained hepatic disease

CSF Patient presenting with new or atypica


neuropsyhchiatric symptoms, particularly when there i
evidence of cognitive impairment, seizures or othe
neurological signs and symptoms.
It can detect infectious, inflammatory and neoplastic
etiologies

MRI brain Patients with acute changes in mental status (including


changes in affect, behavior, or personality) with one o
the following,
Age > 50 years
An abnormal neurological examination (especially
with focal abnormalities)
A history of significant head trauma (i.e., with
extended loss of consciousness, neurological sequelae
or a temporal relationship to the mental status change
in question)
Newonset psychosis
Newonset delirium or dementia of an unknown cause

Topic: Forensic psychiatry concepts


Dual agency/loyalty:

Dual loyalty may be defined as clinical role conflict between professional duties to a patient and
obligations, express or implied, to the interests of a third party such as an employer, an insurer, or
the state.

Sole task of health care professionals working in prisons is the care of physical and mental health of
the prisoners by

55 | P a g e
• Acting as the private caregiver to the prisoners and observing the 7 essential principles of
medical care in prison as quoted in the standards of the European Committee for Prevention
of Torture (free access to medical care, equivalence of prison health care and community
health care, confidentiality, patients’ consent, preventive health care, humanitarian
assistance, complete professional independence and competence)
• Advising the prison director on health affairs in prison, strictly obeying the 7 principles; and
• Acting as a health and hygiene officer by inspecting and reporting on food, hygiene,
sanitation, heating, lighting, ventilation, clothing, bedding, and physical exercise.

The American Academy of Psychiatry and the Law (AAPL) has a set of ethical principles for forensic
psychiatry evaluations and testimony (American Academy of Psychiatry and the Law, 1991) that
specifically address several issues:

Confidentiality,: The ethical forensic psychiatrist must identify for the examinee the parties to whom
the gathered information will be released. This always includes the party retaining the psychiatrist
and any involved court. It may also include other parties. Informing the examinee of the limits of
confidentiality is similar to a Miranda warning.

Consent: The ethical forensic psychiatrist must obtain the consent of the examinee (or the party
who has the authority to give consent) to conduct the examination. Exceptions are, for example,
court-ordered examinations.

Honesty: The ethical forensic psychiatrist must adhere to the principles of honesty and strive for
objectivity. In some situations, objectivity is difficult. An example is a presentencing evaluation of a
rapist. But an ethical forensic psychiatrist must strive for objectivity in any case.

Qualifications: The ethical forensic psychiatrist claims expertise only in the areas of actual
knowledge, skill, training and experience.

The only way to avoid these dual-loyalty conflicts is a clear assignment of different medical roles to
separate persons by
(1) Conceding to health care professionals who care for prisoners complete and undivided loyalty to
their prisoner patients and
(2) Calling in forensic or public health officers who do not have a clinical relationship to patients for
all tasks in which the prison administration or the state needs medical expertise that does not accord
with the interests of prisoners.
As a first step we should strive to meet the guidelines and proposed institutional mechanisms of the
2002 Physicians for Human Rights document Dual Loyalty and Human Rights in Health Professional
Practice, including
• Raise awareness of principles of medical ethics as well as human rights among health care
professionals and nonmedical prison communities and prison administrations.
• Train health care professionals working in prisons in human rights, medical laws and ethics,
and skills to identify dual loyalty.

56 | P a g e
• Increase involvement of international and national professional bodies and boards of health
care professionals in both active support and oversight of health care professionals working
in prisons. (Active support comprises support of individual health care professionals as well
as collective professional actions to uphold undivided loyalty of health care professionals in
prison to their patients. National professional organizations and their licensing bodies should
hold professionals accountable for violations of medical ethics and human rights and should
advocate for developing, implementing, and monitoring national policies that comply with
the principles of medical ethics and human rights in prison health care

Duty to warn/protect:

Confidentiality plays a critical role in patient care; however, there may be special circumstances
where confidentiality must be breached in order to not only ensure the safety of the patient but
also to protect third parties.

• This concept of ‘duty to warn’ stems from California Supreme Court case of Tarasoff v.
Regents of the University of California that took place in the 1970s and comprised of two
rulings known as the Tarasoff I (1974) and Tarasoff II (1976).
Tarasoff I decision required mental health providers to warn potential victims.

Following a rehearing of the case in 1976, Tarasoff II established not only a duty to warn potential
victims by clinicians, but also to take reasonable precautions to protect these third parties of the
significant danger posed by patients.

Some important things to keep in mind that may help assess the risk are to review patient’s past
medical records, obtain collateral information if possible, evaluate for a history of violence, and get
a second opinion.

There are several ways to approach a patient who verbalizes threats such as increasing frequency
of appointments, having patient’s firearms/weapons removed from their home, increasing/starting
a medication, hospitalization or partial hospitalization programs, or involving third parties/family
into the treatment. If options mentioned above do not seem feasible, it is generally recommended
to disclose to the patient, if possible, that the identifiable victim and/or the police will receive
notification of the threat.

In a 2013 presidential address by Donald N. Bersoff of Drexel University (who was then-president of
the American Psychological Association), he argued that the decision is "bad law, bad social science,
and bad social policy."

As Bersoff pointed out, one of the major problems with the Tarasoff decision is that it requires
therapists to decide how the seriousness of the threat being made. Is the risk more or less than 50
percent? And how does a therapist decide that? Breaching confidentiality is a serious matter and
can severely undermine the trust that patients have in their therapists.

Even the kind of violence that the patient might engage in is often hard to judge. Does a Tarasoff
warning only apply when a patient threatens death or serious injury? What if the patient threatened
to destroy someone's house or car? Should a threat to property be treated in the same way as a
death threat? While some jurisdictions have ruled that threats to property are covered under
Tarasoff laws, other jurisdictions have not.

57 | P a g e
Bolam and Bolitho tests

The Bolam test was first recognised in the case of Bolam vs Friern Hospital
Management Committee.

It states that if a doctor has acted according to proper and accepted practice, he is not guilty
of medical negligence. That is to say that if there is a group which is of the opinion that the practice
is wrong, it does not automatically mean that the doctor was acting negligently. It also states that
the standards should be judged by one’s own peers. The standard to which the case should be
compared is that of an ordinary and competent doctor acting in everyday practice.

The Bolitho test, on the other hand, was first decided in the House of Lords.It states that the defence
could not be considered reasonable if the body of doctors or supporting witnesses were not capable
of withstanding logical analysis.

That is to say that simply providing a defence is not quite good enough, but that the defence and its
body of opinion must be reasonable and responsible. A case which is defended based on a practice
which is not reasonable or logical thus cannot be defended.

Other tests and standards taken into account include those standardised by the Gregg vs Scott case,
which was brought before the House of Lords in 2002. This states that negligence can only be proven
in the case of a missed diagnosis if the chance of survival would have been over 50% had the illness
been diagnosed.

For example, if a case of cancer was not found, but the patient would have only had a 35% chance
of survival anyway, negligence would not be found. If the patient would’ve had a 75% chance of
survival had the diagnosis been made and treatment proceeded, then it would be decided that the
doctor had been negligent.

Topic: National Mental Health Program


➢ In Feb 1981 in Lucknow first draft of the NMHP prepared by the expert group
➢ The final draft was submitted to the Central Council of health on 18–20th August, 1982 and
recommended the NMHP for implementation all over country in a phased manner
Objectives of NMHP
➢ Basic mental health care to everybody and everyone particularly the most vulnerable and
underprivileged
➢ Encourage application of mental health knowledge in general health care and social
development
➢ Integration of mental health and general health care services
➢ Develop community health care with community participation

Fundings of NMHP
No adequate funding initially
9th five year plan(1997-2002)-28 crores
10th five year plan(2002-07)-190 crores
11th five year plan(2007-12)-338 crores
12th five year plan(2012-17)-753 crores
Success of NMHP
➢ Enhancing reach to the community
58 | P a g e
➢ Improving service delivery
➢ Getting increased budgetary allocation

Critique on NMHP
➢ Did not achieve the initial goals
➢ Not enough fund allocations
➢ No leadership
➢ No clarity on roles and division of work
➢ Did not reach grass root level
➢ Emphasized more on curative component rather than preventive and promotive.

Barriers in implementing NMHP


➢ Lack of human resources
➢ Fragmentation of responsibilities & poor coordination
➢ Inconsistent fund flows
➢ Lack of integration with primary care in all states
➢ Limited accessibility
➢ Poor NGO and private sector participation

DMHP:
A pilot model (Bellary model) was launched in 1985 jointly by NIMHANS, District Health and Family
Welfare personnel and the District Administration, Bangalore
Provided care for about 2 million people, showed the possibility for up scaling what was done at
one primary health centre to over a dozen primary health centers

BELLARY model components:


➢ Training for all primary care staff
➢ Provision of 6 essential psychotropics and antiepileptics at all PHC’s
➢ System of simple mental health case records
➢ System of monthly reporting
➢ Monitoring and feedback from district team

DMHP was launched in 1996 in 4 districts of the country, one each in Andhra Pradesh, Assam,
Rajasthan & Tamil Nadu with assistance of 22.5 lakhs each
✓ 1 Psychiatrist(program officer)
✓ 1 Clinical psychologist/psychologist
✓ 1 Psychiatric social worker/social worker
✓ 1 Psychiatric nurse/trained general nurse
✓ 1 Community nurse/case manager
✓ 1 Record keeper
✓ 1 Case registry assistant

Services provided in DMHP


➢ Basic mental health care
➢ Follow up of patients
➢ IEC activities
59 | P a g e
➢ School mental health program
➢ College counselling services
➢ Work place stress management
➢ Suicide prevention

Topic: Recovery in Schizophrenia


It encompasses complete improvement of symptoms with societal and occupational re-integration.
In schizophrenia, usually considered to be at a level 1 year after complete resolution of symptoms.

Assertive Community treatment-

• Multidisciplinary, intensive and integrated team approach


• Was started in Mendota MH institute Wisconsin.
• Currently this approach is also followed in many other countries of the UK and the European
union (E.g., Netherlands)
• Main goal to address ‘revolving door‘phenomenon-patients discharged from hospital go to
community, live in poor standards of care and support and soon decompensate and come
back
• Small staff to patient load (1:10-12, a typical team has 10-12 members and handles 100-120
patients)
Services:

• 24x 7 crisis intervention

• Managing ADL (d even shopping, housework)

• Involve family (psychoeducation, crisis mx etc)

• Health promotion

• Medication support

• Financial mx

Team members-Typically Psychiatrist, Psychiatricnurse, Substance use specialist, MH consumer-


peer specialist, programmeassistant. So, there is typically no Psychologist.

Work in a designated geographical area

They have clear admission and discharge criteria for patients (read them up!!)

Lockwood found them to be superior in terms of preventing relapse and reduction of hospital stay
compared to standard community care

Therapeutic Community Approach-

• Coined by Thomas Maine,implemented by Maxwell Jones and Laine.

60 | P a g e
• Well-known models are Clubhouse and Fountain housemodels (in New York)
• Positive aspects are structured approach but patient centric i.e. patient themselves have
autonomy to structure the day as they want
• Most studied in chronic psychosis and in substance use disorders.
• Often uses ‘buddy’ (another patient who has now recovered to help a patient)-buddy system
is however not restricted to therapeutic community approach
• Well known example in India-Richmond fellowship, Bangalore
These communities may often use eclectic psychological approaches to deliver help.

Halfway homes-They are there in several states of India. Origin from concept of remedied convicts
being integrated back to their homes/society

• Meant for seamless integration from a life of dependence to autonomy

• Takes care of occupational skills, finances(limited), food and household activities

• Under a supervisor

Day Care centre-Mainly focusses on vocational/occupational training and skill development. Also
helps to reduce face -to-face contact and reduce EE.

Some techniques to help in recovery:

Token economy

Financial incentive

Shaping-A behavioural technique aimed at achieving successive approximations to a desired


behaviour

E.g.: A person has social anxiety and fears giving speech in public-he is first asked to prepare speech,
then practise it before a mirror, then in deliver it in front of family members, then deliver it before
familiar people on stage and thus make progress.

Chaining-Breaking an activity into constituent parts and teaching it

E.g.: How to start a car-First turn the ignition on-then press clutch-then shift gear-then release clutch
slowly and then press accelerator-each part is broken into steps and practised multiple time

Often confused, so remember the examples!!

How to assess cognitive deficits:

May do any neuropsychological battery (NIMHANS/PGI) or MATRICES-NIMH battery:

Remember the mnemonic (common negative sx related deficits in schizophrenia)

S-speed of processing

A -attention and concentration

M -memory (three types-verbal, visual, working)


61 | P a g e
S-social cognition

R-reasoning and problem solving

Cognitive remediation vs restructuring-Both are same. There is a lot of heated discussion on which
term is technically correct though

Models of cognitive remediation-Improve a skill/restructure environment/disability limitation

A lot of material available on net for reading, no satisfactory material in any book though.

Social Skills Training-Only focusses on social skills. May include general social skills, skills for a job
or an interview or dating skills. Has manuals and sessions. A lot of literature is available for reading

[ Note: This information and key is meant to be used as a guide and does not replace reading from
standard texts and articles. So, use it well.]

Topic: Prognosis and psychoeducation in Schizophrenia


Poor Prognostic factors in Schizophrenia
• Demographic factors: Male, single, young age at onset, family history of Schizophrenia
• Comorbid substance use
• Clinical features: Poor premorbid adjustment, insidious onset, long duration of untreated
psychosis, hebephrenic subtype, simple schizophrenia, negative symptoms, cognitive
impairment, absence of affective symptoms, poor insight
• Other factors: High expressed emotions in the family, poor adherence with treatment

Basic components of Psychoeducation


• Assessing the knowledge of the patient and caregivers about etiology, treatment and
prognosis.
• Introducing the discussion of schizophrenia into discussion.
• Discussing about various symptom dimension
• Providing information about the etiology, ensure that misconceptions do not lead to
unnecessary guilt.
• Give regular opportunities to ask questions, express their views and raise any concerns
they might have.
• Providing information about the treatment in terms of available options, their
efficacy/effectiveness, side effects and duration of use and involving the patient and
carers in decision making.

When the patient is not responding adequately to the medications: Review diagnosis,
comorbidities, treatment history, use scales, do detailed physical examination, investigations
including imaging, understand patient views regarding treatment, check for treatment
adherence.

Topic: World Orientation in Schizophrenia ( doubts asked during discussions )

62 | P a g e
• Self-disorders entail a weakening of the natural attitude associated with the emergence of a
solipsistic perspective, which usually culminates in psychosis as a profound and rigid alteration
of the sense of reality and existence: “[the] patients cannot take things to be the case in the
usual way, as the [very] sense of ′is′ and ′is not′ has changed.”

• Many psychotic patients adopt, what might be called, a double ontological orientation,
designated by Bleuler as “double bookkeeping,” which refers to the predicament (and ability)
of simultaneously living in two different worlds, namely the shared-social world (ie, the natural
ontological attitude) and a private, psychotic world (ie, a solipsistic ontological attitude).

• The patients experience both worlds as relevant and in that sense real. They also generally
seem to experience them as two different, incommensurable, and thus not conflicting realities,
thereby typically allowing them to coexist in an idiosyncratic-personal amalgam and, in the
advanced/consolidated stages of the illness, only occasionally to collide (the beginning or
exacerbation of psychosis may be, however, associated with a sense of perplexity).

• Daniel Paul Schreber describes in his memoirs a remarkable world of “nerves,” “rays,” and
deities, providing us with an unusually lucid, first-person account of double bookkeeping. He
claims that God has “entered into exclusive nerve-contact” with him by which he has “gained
deeper insight than all other humans beings.” Schreber explains that this “nerve-contact” has
made him the centre and constitutor of the world, whose existence now seems to depend
entirely on him. Rather than confusing his psychotic experiences with those of real objects,
Schreber seems for the most part to have been able to differentiate the two “worlds”: “I could
even say with Jesus Christ: ‘My Kingdom is not of this world’; my so-called delusions are
concerned solely with God and the beyond… The certainty of my knowledge of God and divine
matters is so great and unshakeable that it is completely immaterial to me what other people
think of the truth or probability of my ideas.”

• More recently, Professor Elyn Saks, the author of the book, The Center Cannot Hold: My
Journey Through Madness, sketched the rationale behind her long-lasting denial of suffering
from schizophrenia: “I completely recognized that the things I was saying and doing and feeling
would be thought to amount to a diagnosis of schizophrenia; but I thought that it was not true—
I didn’t really have the illness… I looked like I had schizophrenia… but if we knew enough, we
would see that I really did not… All of my so-called symptoms were things I simply chose to think
or do. I was choosing, eg, to hold certain beliefs even though the evidence was not what would
classically constitute ‘good’ evidence—I had a special premium on the truth [italics
added].”Saks seems here to say that she too experienced having a special access to or insight
into the real nature of things, a deeper level of reality, which is not readily accessible to others.
As Schneider concluded on the issue of delusional conviction: “the significance [of experience]
is of a special kind; it always carries a great import, is urgent and personal, a sign or message
from another world.”

• Bleuler offers a vivid example of double bookkeeping: “A catatonic patient was in great fear
of a hallucinated Judas Iscariot who was threatening her with a sword. She cried out that the
Judas be driven away, but in between she begged for a piece of chocolate. Next day she
complained about these hallucinations, apologized for her acts of violence; but in the middle
of her complaints she expressed pleasure in a pretty belt. She managed to weave this belt into
her delusions sufficiently to need reassurance that it was not a ‘Judas kiss’ .”
63 | P a g e
• What is enigmatic in Bleuler’s vignette is that the patient’s behavior is strikingly at odds with
her delusional beliefs. Normally, we would expect someone, who firmly believes that she is
about to be slained, to defend herself or seek cover; we would not expect her to ask for a piece
of chocolate. For other illuminating examples of and reflections on double bookkeeping, see
Sass.33,34 From a clinical perspective, double bookkeeping, although not always as spectacular
as in Bleuler’s vignettes or Schreber’s memoirs, is a quite prevalent phenomenon, perhaps
characterizing the majority of psychotic patients with schizophrenia.

• It is important to reemphasize the difference between ordinarily held beliefs such as “there
is an Italian restaurant around the corner” and delusional beliefs such as “I am the creator of
the universe” or “others are automatons.”

• The ordinarily held beliefs reflect the natural ontological attitude, which is an aspect of our
automatic immersion in a shared-social world. These beliefs concern matters of affairs in the
public world, and if confronted with new or contrary information (eg, “I believe that it is in fact
a Greek restaurant”), these beliefs are for the most part readily correctable. In contrast,
delusional beliefs in schizophrenia do typically not belong to the public sphere but rather to a
solipsistic ontological attitude, facilitated and antedated by self-disorders, and these beliefs are
rarely modifiable by counterarguments.

Topic: Premenstrual dysphoric disorder (PMDD)


It is a severe and disabling form of premenstrual syndrome affecting 1.8–5.8% of menstruating
women.

Diagnosis

• Diagnostic criteria for PMDD are provided by a number of expert medical guides.
• Diagnosis can be supported by having women who are seeking treatment for PMDD
use a daily charting method to record their symptoms
• Daily charting helps to distinguish when mood disturbances are experienced and
allows PMDD to be more easily distinguished from other mood disorders. With PMDD,
mood symptoms are present only during the luteal phase, or last two weeks, of the
menstrual cycle.
• While PMDD mood symptoms are of a cyclical nature, other mood disorders are
variable or constant over time. Although there is a lack of consensus on the most
efficient instrument by which to confirm a PMDD diagnosis, several validated scales for
recording premenstrual symptoms include the Calendar of Premenstrual Experiences
(COPE), Daily Record of Severity of Problems (DRSP), and Prospective Record of the
Severity of Menstruation (PRISM).
• In the context of research, standardized numerical cutoffs are often applied to verify
the diagnosis.The difficulty of diagnosing PMDD is one reason that it can be challenging
for lawyers to cite the disorder as a defence of crime, in the very rare cases where PMDD
is allegedly associated with criminal violence

DSM-5

64 | P a g e
• The DSM-5 which established seven criteria (A through G) for the diagnosis of
PMDD.There is overlap between the criteria for PMDD in the DSM-5 and the criteria
found in the Daily Record of Severity of Problems (DRSP)
• According to the DSM-5, a diagnosis of PMDD requires the presence of at least five of
these symptoms with one of the symptoms being numbers 1-4.[1] These symptoms
should occur during the week before menses and remit after initiation of menses. In
order to meet criteria for the diagnosis, the symptoms should be charted prospectively
for two consecutive ovulation cycles in order to confirm a temporal and cyclical nature
of the symptoms. The symptoms should also be severe enough to affect normal work,
school, social activities, and/or relationships with others.[1]
• The symptoms of Criteria A-C must have been met for most menstrual cycles that
occurred in the preceding year, and have to have affected normal functioning to some
degree (Criterion D).
• Criterion A: During most menstrual cycles throughout the past year, at least 5 of the following
11 symptoms (especially including at least 1 of the first 4 listed) must be present in the final
week before the onset of menses, must start to improve within a few days after the onset of
menses, and become minimal or absent in the week post-menses,
1. Marked lability (e.g., mood swings)
2. Marked irritability or anger
3. Markedly depressed mood
4. Marked anxiety and tension
5. Decreased interest in usual activities
6. Difficulty in concentration
7. Lethargy and marked lack of energy
8. Marked change in appetite (e.g., overeating or specific food cravings)
9. Hypersomnia or insomnia
10. Feeling overwhelmed or out of control
11. Physical symptoms (e.g., breast tenderness or swelling, joint or muscle pain, a
sensation of bloating and weight gain)[1][3]

Criterion B: One (or more) of the following symptoms must be present:[1]


1. Marked affective lability (e.g., mood swings, feeling suddenly sad or tearful, or
increased sensitivity to rejection)
2. Marked irritability or anger or increased interpersonal conflicts
3. Marked depressed mood, feelings of hopelessness, or self-deprecating thoughts
4. Marked anxiety, tension, and/or feelings of being keyed up or on edge

Criterion C: One (or more) of the following symptoms must be present additionally, to reach a
total of 5 symptoms when combined with present symptoms from Criterion B above:
1. Decreased interest in usual activities (e.g., work, school, friends, hobbies).
2. Subjective difficulty in concentration.
3. Lethargy, easy fatigability, or marked lack of energy.
4. Marked change in appetite; overeating; or specific food cravings.
5. Hypersomnia or insomnia.
6. A sense of being overwhelmed or out of control.
7. Physical symptoms such as breast tenderness or swelling, joint or muscle pain, a
sensation of "bloating," or weight gain.
65 | P a g e
Criterion D: The symptoms observed in Criteria A-C are associated with clinically significant
distress or interference with work, school, usual social activities, or relationships with others
(e.g., avoidance of social activities; decreased productivity and efficiency at work, school, or
home).[1]
• Clinically significant distress is not defined explicitly by the DSM-IV, where it has been
critiqued by multiple scholars as being too vague, and potentially detrimental for those
who have symptoms of depression, anxiety, or other mood disorders because they do
not meet the clinical significance requirement.[29][30]

Criterion E: The disturbance is not merely an exacerbation of the symptoms of another disorder,
such as major depressive disorder, panic disorder, persistent depressive disorder (Dysthymia),
or a personality disorder—although it may co-occur with any of these disorders.[1]
Criterion F: Criterion A should be confirmed by prospective daily ratings during at least two
symptomatic cycles. The diagnosis may be made provisionally prior to this confirmation.[1]

Criterion G The symptoms are not attributable to the physiological effects of a substance (e.g.,
drug abuse, a medication, other treatments) or another medical condition (e.g.,
hyperthyroidism).[1]

ICD 11
Diagnostic criteria for PMDD are also provided by the 2016 World Health Organization's
International Classification of Diseases (ICD-11-CM)
GA34.41 Premenstrual dysphoric disorder
Description
During a majority of menstrual cycles within the past year, a pattern of mood symptoms
(depressed mood, irritability), somatic symptoms (lethargy, joint pain, overeating), or cognitive
symptoms (concentration difficulties, forgetfulness) that begin several days before the onset of
menses, start to improve within a few days after the onset of menses, and then become minimal
or absent within approximately 1 week following the onset of menses. The temporal
relationship of the symptoms and luteal and menstrual phases of the cycle may be confirmed
by a prospective symptom diary. The symptoms are severe enough to cause significant distress
or significant impairment in personal, family, social, educational, occupational or other
important areas of functioning and do not represent the exacerbation of a mental disorder.
Early drafts of the ICD did not recognize PMDD as a separate condition. In the World Health
Organisation's classification system, the International Classification of Diseases (ICD-11), PMDD
is listed as a "disease of the genitourinary system"

Topic: Recent Advances In Dementia


Q: Which of the following is NOT a change in DSM5 with regards to classification of Dementia

a) ‘Dementia’ is replaced by ‘Major Neuro-Cognitive Disorder (NCD)’


b) New category of less severe cognitive impairment ‘Mild NCD’
c) Memory impairment is no longer the main focus
d) Use of objective neurocognitive assessment
e) Better specification of behavioral symptoms and syndromes
66 | P a g e
f) Emerging role of biomarkers in future criteria
g) An updated listing of neurocognitive domains

(learning and memory, complex attention, executive function, perceptual-motor abilities, social
cognition, and language)
h) Removal of categories like major or minor NCD due to Frontotemporal NCD,Lewy bodies
,Traumatic brain injury,Parkinson’s disease,HIV infection,Huntington’s disease and Prion disease

Answer :h)

DEMENTIA-ADVANCES IN NOSOLOGY

In DSM-5:
1. ‘Dementia’ is replaced by ‘Major Neuro-Cognitive Disorder (NCD)’
2. New category of less severe cognitive impairment ‘Mild NCD’
3. Memory impairment is no longer the main focus
4. Use of objective neurocognitive assessment
5. Better specification of behavioral symptoms and syndromes
6. Emerging role of biomarkers in future criteria
7. An updated listing of neurocognitive domains
(learning and memory, complex attention, executive function, perceptual-motor abilities, social
cognition, and language)

ADVANCES IN PREVENTION OF DEMENTIA


• Confusion regarding the conceptualization of ‘disease’ in recent diagnostic criteria
• Does AD start with the onset of specific pathological changes in the brain or does it start with the
first appearance of specific clinical symptoms ?
• Many elderly individuals die with intact cognition but have a AD-related pathological signs in their
brain

RECENT DEMENTIA PREVENTION STUDIES AND INITIATIVES


European Dementia Prevention Initiative: (EDPI)
Cooperated from three ongoing prevention RCTs using multidomain vascular and lifestyle-related
interventions:
1. preDIVA (Prevention of dementia by intensive vascular care)
2. FINGER (The Finnish Geriatric Intervention Study to Prevent Cognitive Impairment and Disability)
3. MAPT (MultiDomain Alzheimer Preventive Trial)
The Healthy Aging Through Internet Counselling in the Elderly (HATICE)

Topic: Landmark studies in Psychiatry


Community mental health surveys:

Epidemiology is the study of the distribution and determinants of health related states orevents
in a specified populations and the application of this study to the control of health problems

67 | P a g e
Distribution Determinants Application

Mental health survey helps to plan, develop, implement, monitor, evaluate


andstrengthentheservices.
Epidemiological studies help us know prevalence, prognosis, relapse rate, recoveryrates
inpersonswithdisorders

ImportantepidemiologicalstudiesconductedinIndia:
1. IPSS-InternationalPilotStudyofSchizophrenia
2. DOSMed
3. SOFACOS

DOSMed-DeterminantsofOutcomesofSevereMentalDisorders

10countries,AgraandChandigarhfromIndia
The study design was well designed to take care the data collected
wasquantitativelyandqualitatively leakageproof.
Unlike the IPSS format, it used active case finding method within the geographicalareas
including all services and not just mental health services but
prisons/police,religiousshrines,traditional healers.
86%casesdurationofuntreatedpsychosiswaslessthan1year
Nested cohort studies were done to specifically look into determinants and
factorsinfluencingoutcomeinschizophrenia
WHO disability assessment schedule was used
Outcome
a) The outcome of patients in the developing countries was not uniformly better,
ascompared to the outcome in developed countries. While high rates of completeclinical
remission were significantly more common in developing country areas (37%)than in
developed countries (15.5%), the proportions of continuous unremittingillness
(11.1%and17.4%)didnot differsignificantlyacrossthe2types ofsetting.
b) Patients in developing countries experiencedsignificantlylongerperiods ofunimpaired
functioning in the community, although only 16% of them were oncontinuous antipsychotic
medication (compared with 61% in the developedcountries)
c) Across all centres, the best predictors (P < .001) of outcome were type of onset(insidious
vs acute) and type of setting (developed vs developing country),
followedbymaritalstatus,gender, socialisolation, anddrug abuse.

SoFACOS-StudyoffactorsassociatedwithcourseandoutcomeinScizophrenia

ICMRproject,2yearmulti-centricfollowupstudy
3centres-Lucknow,Madras,Vellore
Outcome- Variables with good overall outcome- Short duration of illness,
regularmedication compliance, absence of economic difficulties, increase in SE levels,
68 | P a g e
lackofself-injuriousbehaviour, ruralbackground
Conclusion
o The outcome of schizophrenia is good in India compared to developedcountries
o Factors were identified which are associated with good outcome
o There was no difference between the centres with in India though there were socio
cultural differences between the centres.

National Mental Health Survey 2015-16.


NMHS aimed to estimate the prevalence and burden of mental health disorders inIndia and
identify current treatment gaps, existing patterns of health-care seeking,service utilization
patterns, along with an understanding of the impact and disabilityduetothesedisorders.
Multi-stage,stratified,randomclustersamplingtechnique
Report by National Institute of Mental Health and Neuro Sciences, Bengaluru, with15
institutionsfromacrossIndia
Covers12Indianstates
Report estimates that nearly 150 million Indians currently require mental
healthcareservices
Treatmentgaprangesfrom28to83percentforvariousmentaldisorders
1in20peoplearedepressed
Mentalhealthdisordersmoreprevalentinmales
1.9 percent of the population have had a severe mental disorder in their lifetime.
Moreprevalentinurbanthanrural

Major studies in Schizophrenia

CATIE and CuTLASS


Efficacy studies are done to evaluate performance of an intervention in the ideal orcontrolled lab
settings. Effectiveness studies deal with real world interventions. Wherethere maybe various other
factors influencing outcomes. Efficacy trials generallyoverestimatethe resultsofthetrial.
Astheyaredone inidealsituations.

CATIE and CuTLASS once and for all.

Q1 : What does CATIE and CuTLASS stand for ?


Q2 : What was the nature of CATIE and CuTLASS trials ?
Q3 : Number of patients participating in both trials ?
Q4 : What were the Primary out comes in both the studies ?
Q5 : What were findings of Phase 1 and Phase2 clinical trials in CATIE study ?
Q 6 : What were the findings of CutLASS trials
Q 7 : Which second generation medications were used in CATIE trials and CuTLASS trials ?
Q 8 : For how long were the studies conducted ?
Q 9 : Limitation of both CATIE and CuTLASS trials .
Q10 : Controversies associated with both CATIE and CuTLASS trials

69 | P a g e
S.no CATIE CuTLASS
1. Clinical Antipsychotic Trials of Cost Utility of the Latest Antipsychotic Drugsin
Intervention Effectiveness. Schizophrenia Study

2. Double--‐‐blind pragmatic RCT. Unblinded randomized controlled trial


comparing first--‐‐generation v. second--‐‐
generation antipsychotics.

3. 1493 patients with chronic 1,227 people with schizophrenia whowere


schizophrenia participated (mean being assessed by their clinicalteam for
duration of illness = 14 years), 57 medication review becauseof poor response or
sites,2001 to 2004. adverse effectswere randomised.
: Phase 1 : 18 months Phase ii and iii 1year
were alldoneinyear2001to2004

4. Primary outcome is a ‘real--‐‐world’ The primary outcome was the quality of life at
measure – 1year
discontinuation for any reason, either
patient--‐‐initiated or physician
initiated..76% power to detect 12%
difference in primary outcome

Medications tried were Olanzapine, The second generation drugs were


5. quetiapine, Risperidone, amisulpride, olanzapine, quetiapine or
ziprasidone(added later in the trial), risperidone
perphenazine

70 | P a g e
Ans 1 :CATIE stands for Clinical Antipsychotic Trials of Intervention Effectiveness. CUtLASS
stands for Cost Utility of the Latest Antipsychotic Drugs in Schizophrenia Study

Ans 2 :
Study design of CATIE was double--‐‐blind pragmatic RCT.
The design of CutLASS study was unblinded randomised controlled trial comparing first--‐
‐generation v. second--‐‐generation antipsychotics.
Ans 3 : In CATIE , 1493 patients with chronic schizophrenia participated (mean duration of
illness = 14 years), 57 sites, 2001 to 2004. In CuTLASS, 1, 227 people with schizophrenia
who were being assessed by their clinical team for medication review because of poor
response or adverse effects were randomised..

Ans 4 : Primary Outcome measures in CATIE Primary outcome is a ‘real--‐‐world’ measure


– discontinuation for any reason, either patient--‐‐initiated or physician initiated.
.76% power to detect 12% difference in primary outcome
Primary Outcome in CuTLASS : The primary outcome was the quality of life at 1 year

Ans 5 : .Irrespective of the prescribed drug – 74% discontinued treatment in 18 months


(surprisingly high despite naturalistic design).
• The median time to stop was 4.6 months.
• Olanzapine had lowest discontinuation rate (still 64%) – but highest side effect burden.
64% discontinued
• olanzapine; 75%, perphenazine; 82%, quetiapine; 74%, risperidone; and 79%,
ziprasidone.
• Olanzapine caused most weight gain while quetiapine caused most anticholinergic
symptoms; perphenazine had highest EPSE related discontinuation.
• Those who did not respond after 18 months (those who discontinued for the
ineffectiveness of therapy) were re--‐‐randomised in phase 2 trial (n=99), and Clozapine
was compared to other atypical agents (efficacy pathway).
• Clozapine had lowest discontinuation rate – median at 10 months. This time--‐‐to--‐‐
discontinuation
• was nearly 3 times longer than time--‐‐to--‐‐discontinuation with the other SGAs.
Quetiapine had comparatively less EPSEs.
• As a part of the phase 2 CATIE study (tolerance pathway) those who terminated phase
1 for ‘‘intolerable side effects’’ (444 volunteers) were tested with olanzapine, risperidone,
quetiapine, or ziprasidone.
• olanzapine and risperidone had equivalent effectiveness, and both were better than
quetiapine orziprasidone by significant but modest margins

Ans 6.
.CutLASS : The rate of follow--‐‐up interview was 81% at 1 year.
• .The results showed no advantage of second--‐‐generation drugs in terms of quality of
life or symptom burden over 1 year with those on first--‐‐generation antipsychotic doing
relatively better.
• Participants reported no clear preference for either class of drug.
• The second phase --‐‐ CUtLASS 2 trial was of similar design and compared clozapine with
other second--‐‐ generation drugs in 136 patients who had not responded well to two or
more previous drugs. Results showed that there was a significant advantage for clozapine
in symptom improvements over 1 year; moreover, patients significantly preferred it.

Ans 7 : CATIE : Medications tried were Olanzapine, quetiapine, Risperidone, ziprasidone


(added later in the trial), perphenazine
CutLASS : The second--‐‐generation drugs were amisulpride, olanzapine, quetiapine or
risperidone.

Ans 8 : CATIE : Phase 1 : 18 months


Phase ii and iii were all done in year 2001 to 2004
CUTLaSS 1 year
Ans 9 and 10 :
.CATIE Controversies and limitations
• Quite complicated study design and many outcomes were analysed from the dataset.
• Decisions to add ziprasidone to the protocol was made after recruitment began
• Perphenazine was used only in one randomized phase (phase 1) of the study generating
controversy
• The decision to use double--‐‐blinded treatments decreased the resemblance of the
study procedures to
• those of routine clinical care
• The mean doses used remain controversial though it is claimed that the study was
designed to be pragmatic and not purely experimental.

CuTLASS :
• In CUtLASS, the concept of including 13 different FGAs and four SGAs in the respective
classes was problematic.
• It is of interest that the most widely prescribed drug was sulpiride--of the FGAs, this is
probably the 'most atypical' drug.
• Aside from the finding that the advantages of the SGAs are not as strong as early trials
and marketing suggested or promised, the trials do not provide much helpful information
regarding everyday practice.
• For tardive dyskinesia, no conclusions at all can be drawn. Similarly, methodological
problems inhibited the detection of the other major advantage of the SGAs, i.e. the
improved subjective well-being/quality of life while receiving these agents.
• It is well known that patients' and doctors' perspectives differ markedly, and the Quality
of Life Scale (QLS), an expert-rated scale used in both trials, might not be sensitive enough
to detect the subjective advantages reported by the majority of patients in other trials.
CATIE and CUtLASS suggest that SGAs do not live up to all the previous expectations.

What were findings of phase 1 and phase 2 clinical trials in CATIE study:
Irrespective of the prescribed drug – 74% discontinued treatment in 18 months
(surprisinglyhighdespitenaturalisticdesign).
• Themediantimetostopwas4.6months.
• Olanzapine hadlowest discontinuationrate(still64%)–
buthighestsideeffectburden. 64% discontinued Olanzapine; 75%, perphenazine;
82%, quetiapine; 74%,risperidone; and79%, ziprasidone.
• Olanzapine caused most weight gain while quetiapine caused most
anticholinergicsymptoms; perphenazine had highest EPSE
relateddiscontinuation.
• Those who did not respond after 18 months (those who discontinued for the
ineffectiveness of therapy) were re‐randomised in phase 2 trial (n=99), and
Clozapine was compared to other atypical agents (efficacy pathway).
• Clozapine had lowest discontinuation rate – median at 10 months. This time
todiscontinuationwasnearly3timeslongerthantimetodiscontinuationwiththeoth
erSGAs.Quetiapinehadcomparatively less EPSEs.
• As a part of the phase 2 CATIE study (tolerance pathway) those who terminated
phase
1 for ‘‘intolerable side effects’’ (444 volunteers) were tested with olanzapine,
risperidone, quetiapine, or ziprasidone.
• Olanzapine and risperidone had equivalent effectiveness, and both were better
thanquetiapineorziprasidoneby significantbut modestmargins.

WhatwerethefindingsofCUTLASStrials?
CutLASS:Therateoffollowupinterviewwas81%at1year.

• The results showed no advantage of second generation drugs in terms of quality


oflifeorsymptomburdenover1yearwiththoseonfirstgenerationantipsychoticdoin
grelativelybetter.
• Participantsreportednoclear preferenceforeither classof drug.
• The second phase CUtLASS 2 trial was of similar design and compared clozapine
with other second generation drugs in 136 patients who had not responded well
to two or more previous drugs. Results showed that there was a significant advantage
forclozapine in symptom improvements over 1 year; moreover, patients
significantlypreferred it.

CATIE controversies and limitations:


• Quite complicated study design and many outcomes were analysed from the
dataset.
• Decisions to add ziprasidone to the protocol was made after recruitment began
• Perphenazine was used only in one randomized phase (phase1) of the study
generating controversy
• The decision to use double blinded treatments decreased the resemblance of
the study procedures to those of routine clinical care
• The mean doses used remain controversial though it is claimed that the study
was designed to be pragmatic and not purely experimental.

CUTLASS:controversies and limitations


• InCUtLASS,the concept of including 13 different FGAs and four SGAs in the
respective classes was problematic.
• It is of interest that the most widely prescribed drug was sulpiride of the FGAs,
this isprobablythe'mostatypical' drug.
• Aside from the finding that the advantages of the SGAs are not as strong as
earlytrials and marketing suggested or promised, the trials do not provide much
helpfulinformation regarding everyday practice.
• For tardive dyskinesia, no conclusions at all can be drawn. Similarly,
methodologicalproblems inhibited the detection of the other major advantage
of the SGAs, i.e. theimprovedsubjectivewell-being/qualityoflife while
receivingthese agents.
• It is well known that patients' and doctors' perspectives differ markedly, and
theQuality of Life Scale (QLS), an expert-rated scale used in both trials, might not
besensitive enough to detect the subjective advantages reported by the majority
ofpatients in other trials. CATIE and CUtLASS suggest that SGAs do not live up to
all thepreviousexpectations.
Majorstudiesindepression

STAR*D-theSequencedTreatmentAlternativestoRelieveDepression

Study evaluated feasible treatment strategies to improve clinical outcomes for real-
worldpatientswithtreatment-resistantdepression.
Collaborative study on depression treatment, funded by the National Institute
ofMentalHealth,US
Itrecruitedpatientswithnonpsychoticmajordepressivedisorder
Multicentricstudy
Itincluded outpatientsinthe agegroupbetween18to75years
Ascoreofatleast14on17-itemHamiltonRatingScalefor depression(HAM-D)

Strategies

Level1 Initialtreatment:Citalopram

Level2 Switch to Bupropion / Venlafaxine / Sertraline / Cognitive


therapyAugment withBupropion/Buspirone/Cognitivetherapy

OnlyforthosereceivingCT,SwitchtoBupropion/Venlafaxine

2A

Level3 Switch to Mirtazapine / NortriptylineAugment withLithium/T3


Level4 SwitchtoTranylcypromine/Mirtazapine+Venlafaxine

Over the course of the four levels of treatment, the theoretical cumulative
remissionratewas67%
Remission was more likely to occur during the first two treatment levels (20%–
30%)than duringlevels3and4(10%–20%)

Major studies in Bipolar disorder

1.Bipolar Affective disorder:Lithium/Anticonvulsant Evaluation(BALANCE)Study

people with bipolarI disorder, for whom long-term therapy isclinicallyindicated, both
combination therapy with lithium plus valproate and lithium monotherapyare
morelikelytoprevent relapsethan isvalproate monotherapy.

2. Efficacy of Monotherapy Seroquel in BipOLarDEpressioN(EMBOLDEN) I


Study

Quetiapine (300 or 600 mg/d) was more effective than placebo for the treatment
ofepisodes of acute depression in bipolar disorder. Lithium did not significantly differ
from place boon the main measures of efficacy.

3. Efficacy of Monotherapy Seroquel in BipOLarDEpressioN(EMBOLDEN) II


STUDY

Quetiapine (300or600mg/d), but not paroxetine, was more effective than place
bo for treating acute depressive episodes in bipolar I and II disorder

4. BipOLarDEpRession(BOLDER)Istudy

Quetiapine monotherapy has significant antidepressant efficacy in a group of


patients with bipolar I orII depression based on theprimary efficacy analysis (mean
change in Montgomery-Åsberg Depression Rating Scale total score from baseline to last
assessment).

5. BipOLarDEpRession(BOLDER)IISTUDY

Results suggest that quetiapine monotherapy is no more effective than place


bofor the treatment of depression in adolescents with bipolardisorder

Topic: DIABETES, DYSLIPIDEMIA AND OBESITY - INTERFACE WITH


PSYCHIATRY

Interface of diabetes and psychiatry has been a source of interest for both
endocrinologists and mental health professionals alike. Diabetes and psychiatric
disorders share a bidirectional association -- both influencing each other in multiple
levels.

• First, the two can be independent conditions with no apparent direct


connection. In such a scenario both are outcome of independent and parallel
pathogenic pathways.
• Second, the course of diabetes can be complicated by emergence of psychiatric
disorders. In such cases diabetes contributes to the pathogenesis of psychiatric
disorders. Various biological and psychological factors mediate the emergence
of psychiatric disorders in such context.
• Third, certain psychiatric disorders like depression and schizophrenia act as
significant independent risk factors for development of diabetes.
• Fourth, there could be an overlap between the clinical presentation of
hypoglycemic and ketoacidosis episodes and conditions such as panic attacks.
• Fifth, impaired glucose tolerance and diabetes could emerge as a side effect of
the medications used for psychiatric disorders.
Risk of developing depression is 50-100% higher among patients with diabetes
compared to that among the general population. Emergence of depression in diabetes
is associated with increased complications, mortality rates, and healthcare costs.

The prevalence of anxiety disorders among patients with diabetes is considerably


higher compared to the general population. Anxiety symptoms have been found to be
significant risk factors for development of diabetes.Negative correlations have been
observed between prevalence of anxiety disorders and levels of HbA1c.

The prevalence rate of generalized anxiety disorder (GAD) has been found to be around
three times higher than that reported in the general population. However, rates of
panic disorder, obsessive compulsive disorder (OCD), post-traumatic stress disorder
(PTSD), and agoraphobia have been found to be within the range of those reported in
community studies.

Overall risk of type 2 diabetes in people with schizophrenia is between two and four
times that in the general population. Family history of type 2 diabetes is significantly
higher even among the first-degree relatives of patients of schizophrenia. Similarly, a
positive family history may increase the risk of developing diabetes in individuals with
schizophrenia up to threefold. It has been shown that people with diabetes and
schizophrenia have higher mortality rates than individuals with diabetes alone.
Additionally, the presence of type 2 diabetes is associated with increased mortality risk
in patients with schizophrenia.

A Danish study found that 40% of women with diet-treated GDM had developed
diabetes 10 years after the index pregnancy. Compared to the 30-60-year-old females
in the background population, the incidence of diabetes was increased 10
fold(Lauenborg et al,2004)

The specific biological link between GDM and T2DM is unclear. Both disorders are
characterized by insulin resistance and/or abnormal insulin secretion. In addition
studies provide evidence that several of the known T2DM risk genes are more frequent
in women with previous GDM, and many of the risk factors are the same, such as a
raised body-mass index, high age, family history of diabetes and Asian and black
ethnicity. It thus appears plausible that the pathogenesis is overlapping, and GDM may
serve to identify women at high risk of future T2DM.

Dyslipidemia and Obesity

In conclusion, the results of this study suggest that statins may be effective for the
treatment of depression independent of medical status and escitalopram use, and they
may potentiate the antidepressant action of serotonergic antidepressants in patients
with ACS. In some patients with ACS and depressive disorder who are not tolerant to
antidepressants, statins may be a novel alternative(Kim et al,2015)

In patients with schizophrenia, negative symptoms may be reduced by adjuvant statin


therapy. Studies on cohorts at risk for dementia have generally shown protective
effects of statins(Kim et al,2019)

Human and animal research indicates that low or lowered cholesterol levels may
reduce central serotonin activity, which in turn is causally linked to violent behaviors.
Many trials support a significant relation between low or lowered cholesterol levels
and violence (P < 0.001).High levels of cholesterol are associated with violence,suicidal
or homicidal behaviours while being associated with a better neurocognitive profile in
schizophrenia

**(the above information is not exhaustive and residents are suggested to read
standard text books and latest articles for more knowledge on these aspects)

Topic: History of Psychiatry


1) I was born in 1632 in Wrington, Somerset in the UK. I studied in the prestigious
Westminster School in London and graduated at the age of 20 and earned a
place in the Christ Church in Oxford. I received my Bachelor of Medicine in
1675.My major contributions are in the field of philosophy and politics. I have
written my books during my lifetime. My link to psychiatry is through one
ground-breaking idea I wrote in the book I am known for. This book sets out to
offer an analysis of the human mind and its acquisition of knowledge. Here I
proposed that the human mind is not naturally predisposed into developing
into what they become at adulthood but that it is completely malleable at the
hands of the environment and their experiences they grow up in(if you will,
tabula rasa in Latin).I earned my name as one of the most famous philosophers
and political theorists of the 17th century and died in 1704. Many of the ideas
are popular topics of discourse even today. Who am I?
John Locke.
He was a British philosopher and medical researcher. he has also contributed to
the political sciences and sociology. ” Nature vs nurture” is an ongoing debate
among behavioural psychologists and geneticists.

Locke was a staunch believer in the’nurture’ aspect.In his Paper II of the essay
Concerning Human Understanding” he propounded the the theory of ‘tabula
rasa’-a blank slate. He believed that whatever people developed into was
wholly due to their environment of upringing. These ideas were popular at that
time till their biggest critic arose in Charles Darwin and his beliefs in
evolutionary psychology.

Over the centuries a tug-of -war has continued over the proponents of the
‘nature and nurture ‘ debate.Prof Steven Pinker,a Canadian cognitive
psychologist is a notable critic of the ‘blank slate’ concept in the recent times
and he has written several books to put forward his ideas.

2) I was born in 1927 in Glasgow, Scotland, and a Psychiatrist by profession. I have


felt that my parents –my mother especially – as being somewhat anti-social and
demanding the maximum achievement from me as a child. Unfortunately, as I
grew up, I had few close friends. I have also dabbled in music and poetry and
became an Associate at the Royal College of Music. One of the notable incidents
during my degree was that I founded a ‘Socratic club’ where Bertrand Russell
himself was the President.
I completed my degree in medicine from the University of Glasgow and served
briefly in the army. I also trained at the famous Tavistock clinic in London for a
while. During my time in the army, I encountered experiences of soldiers trying
to fake having schizophrenia to get a disability certificate with the benefit of
pension and relief of duties. These experiences shaped my views significantly. I
believed schizophrenia and psychosis to be valid expressions of subjective
distress and was disinclined to label them as ‘abnormalities. My character was
recently played in a movie “mad to be Normal’. I died in 1989.Who am I?

Ronald David Laing


Laing’s views on the causes and treatment of psychopathological phenomena
were influenced by his study of existential philosophy. In his early life, Laing’s
father, David, seems often to have come to blows with his own brother, and
himself had a breakdown for three months when Laing was a teenager. His
mother Amelia, according to some speculation and rumour about her
behaviour, has been described as “psychologically peculiar”. Laing was troubled
by his own personal problems, suffering from both
episodic alcoholism and clinical depression, according to his self-diagnosis

He believed that schizophrenia was a theory and not a fact and that psychotic
symptoms were a variant expression of distress stemming from adverse life
experiences. Laing never denied the existence of mental illness. He viewed it in
a radically different light from contemporaries. For Laing, mental illness could
be a transformative episode whereby the process of undergoing mental distress
was compared to a shamanic journey. The traveller could return from the
journey with important insights and may have become (in the views of Laing
and his followers) a wiser and more grounded person as a result.

He started a psychiatric community project at Kingsley Hall, where patients and


therapists lived together. There were no restrictions as patients who were
admitted could come and go as they pleased. He did not use antipsychotics,
rather yoga and expressive therapies like painting and music for the patients.
The Norwegian author Axel Jensen contacted Laing at Kingsley Hall after
reading his book The Divided Self, which had been given to him by Noel Cobb.
Jensen was treated by Laing and subsequently they became close friends.

In 1965 Laing co-founded the UK charity the Philadelphia Association,


concerned with the understanding and relief of mental suffering, which he also
chaired. His work influenced the wider movement of therapeutic communities,
operating in less “confrontational” (Laingian perspective) psychiatric settings.
Notable writings by him include ‘The Divided Self: an Existential Study in Sanity
and Madness’ ,’Sanity, Madness and the Family’ and ‘Mad to be Normal:
Conversations with R D Laing’. A fictionalized account of the Kingsley hall Project
was portrayed in the movie “ Mad to be Normal” in 2017.

3) I was born in Karlsruhe, Germany in 1889 and a contemporary of Sigmund


Freud. I grew up in an orthodox Jewish family. I immigrated to the US during
World War 2 and spent my last life there. During my studies, I was romantically
involved with one of my patients, who also made a name in psychiatric history.
After we grew close, I stopped psychoanalysing him and got married 1 year
later.
We divorced in 1942 and had no biological children, which probably shaped my
interest in studying their development. I believed that communication by
parents is critical to the adequate development of children and used these
principles in treating individuals with schizophrenia. I was the first woman to be
invited to the Macy Conference. The American Psychoanalytic Society gives
away an award in my name each year. Who am I?
Freida Fromm Reichmann
Born in 1889,she was a devout and practisingJew.She was a Major in the German
army during World War I and treated german soldiers using a combination of
therapy,Jewish diet and Sabbath observance.It was mocked among non-Jews who
saw her practices.She fled from Germany after Hitler’s rise to power.She spent a
while in exile in France before moving to the US.She married her patient Erich
Fromm,whom she divorced later in 1942.
‘In many ways, she was the Oliver Sacks of mental health, not merely applying
her robust professional expertise to the healing of her patients but bathing
them in largehearted perseverance of faith in the inextinguishable light of their
humanity’.

Fromm-Reichmann succeeded in using intensive psychotherapy to treat


schizophrenic and manic-depressive patients who had previously been
considered unsuitable for psychoanalysis. She also wrote a book on her ideas.
She fostered their creative talents and developed fresh insights into the
relationship between art and mental illness. A highly gifted clinician and
outstanding teacher, she shared her discoveries with large audiences through
her popular lectures. Her enthusiasm could not be dampened even when she
lost her hearing in he later part of her life.

She is famous for coining the term ‘schizophrenogenic mother’ at a time when
family and social theories for schizophrenia were actively being explored. She
believed that poor communication by mothers who were ‘cold and aloof’ could
lead to schizophrenia. She emphasized the need for clear, detailed and
reassuring communication for her patients.

(Read about social and family theories of schizophrenia and transacti onal
analysis-has come in past exams)

4) Identify the movie and its link with Psychiatry

Black Swan – Movie review

Does an exceptional job relaying elements of psychosis and the psychosocial


implications of untreated mental illness. For example, Nina’s demographics and
social circumstances present actual risk factors for first-break psychosis,
including, an individual in their twenties, the presence of other psychiatric
comorbidities, intense external stress, and a primary family member with
mental illness. Additionally, Nina’s experimentation with drugs, alcohol, and
risky sexual behavior present very clearly her inability to safely care for herself
while untreated. However, the film relays a few details inaccurately, largely for
dramatic effect and viewer experience. Namely, Nina’s preponderance of visual
hallucinations is not frequently encountered in actual patient presentations,
whereas auditory hallucinations are much more frequently reported. Lastly,
Nina’s inclination towards violent behavior as she becomes increasingly
psychiatrically ill is a social stigma and misrepresentation of psychotic illness
that has not been substantiated by the literature. Overall, this film serves as a
great tool for illustrating the debilitating nature of psychosis, and highlights the
need for treatment of mental illness in order to preserve patients’ quality of
life.

(Those who haven’t should watch the movie)

Other movies on ? Psychosis – Shutter island

- Fight club
- A Beautiful mind
- The soloist
5. Identify this painting and its link with Psychiatry

‘Kaleidoscopic Cats’ by Louis Wain.


He is widely believed to have suffered from schizophrenia. His cats were initially
depicted as cats themselves, but later believed to have human attributes.

Writing in 2001, Dr Michael Fitzgerald disputed the claim of schizophrenia,


saying that Wain more likely had Asperger's syndrome. Fitzgerald indicated that
while Wain's art took on a more abstract nature as he grew older, his technique
and skill as a painter did not diminish, as one would expect from a person with
schizophrenia. Moreover, elements of visual agnosia are demonstrated in his
painting. If Wain had visual agnosia, it might have manifested itself merely as
extreme attention to detail.

O'Flynn suggested Maclay saw in the series a proof of his own ideas, partly
based on his 1930s experiments with art and mescaline-induced psychosis.

The cats show bright colours (later paintings), gradually becoming more
abstract and distorted. These may be dysmorphsias owing to mescaline abuse
or schizophrenia(this is what I had asked)

5) Identify what is happening and how it works.


Psychodrama
The term was coined by Jacob Moreno. Moreno is also credited with the concept of
‘sociometry’. While living in Vienna, he started an improvisational theatre company,
and from a lot of ideas he used in psychodrama were borrowed from there.
Psychodrama is a group approach to therapy. It is believed to help understand conflicts
and improve problem solving.Moreno described psychodrama as the "scientific
exploration of truth through dramatic method."

Psychodrama sessions are often performed as weekly group therapy sessions, typically
comprised of 8-12 members. Sessions generally last between 90 minutes and 2 hours.
Each psychodrama focuses on the life situation of one individual, with group members
taking on roles as needed.

A session is typically executed in three phases: the warm-up phase, the action phase,
and the sharing phase. Through role and drama-based play, the protagonist and other
participants develop insight into past issues, present challenges, and future
possibilities.

The goal of the warm-up phase is to help establish trust, group cohesion, and a sense
of safety among members. Without trust, group members may not feel comfortable
performing action methods or exploring raised issues or conflicts. One technique often
used in warm-up is role presentation, where members of the group adopt a certain role
in order to introduce themselves. Because in psychodrama, members of the group
often act out roles in other members' lives, this technique can help provide insight to
those in the group. As the members get to know one another, one member
may volunteer to act as the psychodrama protagonist, or the main focus of the
psychodrama.

In the action phase, the protagonist—with the therapist's help—creates a scene based
on significant events in the protagonist’s current life. The therapist directs the session,
while other group members serve as auxiliary egos, or individuals from the
protagonist's life. The rest of the group members act as an audience.

The following techniques are commonly used as part of the action phase:

• Role reversal: The protagonist steps out of their own role and enacts the role
of a significant person in their life. This action can help the protagonist
understand the other person's role and help the director (therapist) better
understand relationship dynamics. Doing so may also help increase the
protagonist's empathy.
• Mirroring: The protagonist becomes an observer while auxiliary egos take up
the part of the protagonist, acting out an event so the protagonist can watch.
This technique can be helpful when a protagonist is experiencing extremely
negative feelings or is feeling separated or distanced from feelings
or emotions about the scene.
• Doubling: A group member adopts the protagonist's behaviour and
movements, expressing aloud any emotions or thoughts that member believes
to be the protagonist's feelings and thoughts. This technique can be used to
build empathy for the protagonist or to challenge, in a constructive and non-
aggressive way, some aspect of the scene or the protagonist's actions.
• Soliloquy: The protagonist relates inner thoughts and feelings to the audience.
This may be done when speaking to a double, or at the encouragement of the
director (therapist).
During the sharing phase, the director shifts back to a therapist role in order to facilitate
the processing of the scene. Processing the meaning of the feelings and emotions that
have come to light is believed to be essential for transformation to occur. The sharing
phase provides time for a group discussion about the events that took place in the
action phase. The audience might consider, among other topics, how their thoughts or
observations could have an impact on the protagonist’s ways of interacting or relating
with others.

Major limitation: Confidentiality is not preserved

Diamond of opposites: A type of two-dimensional plot used in psychodrama groups.


This tool can illuminate the presence of contradictions in processes that cannot be
detected by any single questionnaire item using a traditional format such as the Likert
scale. The diamond of opposites is a sociometric scaling method that simultaneously
measures positive and negative responses to a statement.

E.g.: On a scale of 1 to 10 what was your level of attraction?

On a scale of 1 to 10 what was your level of repulsion?

Role plays are also used in gestalt therapy(empty chair technique) and was described
by Adler in ‘acting as if’.

(Psychodrama and gestalt therapy,expressive forms of therapy can come in


exams.Read about them)

Topic: Psychiatric aspects of Parkinson’s

It’s a progressive neurodegenerative disorder, due to loss of dopaminergic neurons in


the substantia nigra
Named after James Parkinson, 1817- “Essay on the shaking Palsy”
Neuropsychiatric manifestations
Cause-reactive, common etiopathogenesis, medication induced, multifactorial
2.4 times higher risk of depression even before the diagnosis of PD is made
Risk is especially high in 3 years prior to diagnosis of PD (Leentjens et al,2004 )
Specific risk factors for depression in PD
• Earlier age of onset
• More severe disability
• Presence of on/off fluctuations
• Higher dose of levodopa
• Cognitive dysfunction
• Right sided motor symptoms
• Family history of PD

Treatment:
Response rate for SSRIs: 41% vs 57% for TCAs(Skapinakis et al, 2010)
Pramipexole for anhedonia and depression in PD patients (Barone et al, 2010)
CBT(Farabaugh et al, 2010)
ECT (Moellentine et al, 1998)

Depression- assess its relationship with PD drugs- adjust the dose- depression may
improve
Mild depression- Supportive psychotherapy, CBT
Moderate Depression- SSRIs, TCAs, CBT
Severe Depression- SSRIS, TCAs, Consider ECT
TCAs be used cautiously- anticholinergic effect
Prefer- drugs with a short half life, rapid clearance, little anti cholinergic effect
Desipramine and Nortriptyline- least sedative and anticholinergic-use in PD with
cognitive impairment
Amitryptiline- sedative property, doxepin- PD like reactions- avoided
Benefit- PD patients with sleep disorder & anxiety disorder (Veazey C, 2005)
Anxiety Disorders are quite prevalent
Primary anxiety disorder
Secondary to other psychiatric comorbidities (e.g.: depression, psychosis)
Secondary to the use of antiparkinsonian medications (e.g.: levodopa, pergolide)
Secondary to fluctuation of motor symptoms (on/off periods)
Prodromic symptoms of PD
Secondary to the impairment and limitations caused by PD
(Chagas et al,2009 )
Psychosis in PD: Prevalence 8-40%
Visual Hallucinations are the most common, seen cross-sectionally in 15% to 40% of
the patients
Delusions are less common and usually occur with hallucinations; prevalence varies
from 3% to 30%

The “On” state – manic, hypo-manic symptoms along with mood congruent psychotic
symptoms and inappropriate or risky goal directed behavior
Perseverative behaviors and pathological gambling, shopping (Weintraub & Hurtig,
2007)

Risk factors
• Older age

• Disease severity

• Sleep disturbance

• Cognitive impairment

• Dementia and/or depression

(Zahodne & Fernandez , 2008)

Clinical Description of trauma related psychiatric disorders


This category of psychiatric disorders differs from others in that it includes disorders
identifiable not only on grounds of symptomatology and course but also on the basis
of one or other of two causative influences - an exceptionally stressful life event
producing an acute stress reaction, or a significant life change leading to continued
unpleasant circumstances that result in an adjustment disorder. Less severe
psychosocial stress ("life events") may precipitate the onset or contribute to the
presentation of a very wide range of disorders classified elsewhere in this work, but the
etiological importance of such stress is not always clear and, in each case, will be found
to depend on individual, often idiosyncratic, vulnerability. In other words, the stress is
neither necessary nor enough to explain the occurrence and form of the disorder. In
contrast, the disorders brought together in this category are thought to arise always as
a direct consequence of the acute severe stress or continued trauma. The stressful
event or the continuing unpleasantness of circumstances is the primary and overriding
causal factor, and the disorder would not have occurred without its impact. These
disorders can thus be regarded as maladaptive responses to severe or continued stress,
in that they interfere with successful coping mechanisms and thus lead to problems in
social functioning.

Acute Stress Reaction: A transient disorder of significant severity which develops in an


individual without any other apparent mental disorder in response to exceptional
physical and/or mental stress and which usually subsides within hours or days. The
stressor may be an overwhelming traumatic experience involving serious threat to the
security or physical integrity of the individual or of a loved person(s) (e.g. natural
catastrophe, accident, battle, criminal assault, rape), or an unusually sudden and
threatening change in the social position and/or network of the individual, such as
multiple bereavement or domestic fire. The risk of this disorder developing is increased
if physical exhaustion or organic factors (e.g. in the elderly) are also present. show a
mixed and usually changing picture; in addition to the initial state of "daze", depression,
anxiety, anger, despair, overactivity, and withdrawal may all be seen, but no one type
of symptom predominates for long; and resolve rapidly (within a few hours at the most)
in those cases where removal from the stressful environment is possible; in cases
where the stress continues or cannot by its nature be reversed, the symptoms usually
begin to diminish after 24-48 hours and are usually minimal after about 3 days.

Post-traumatic Stress disorder: This disorder should not generally be diagnosed unless
there is evidence that it arose within 6 months of a traumatic event of exceptional
severity. A "probable" diagnosis might still be possible if the delay between the event
and the onset was longer than 6 months, provided that the clinical manifestations are
typical and no alternative identification of the disorder (e.g. as an anxiety or obsessive-
compulsive disorder or depressive episode) is plausible. In addition to evidence of
trauma, there must be a repetitive, intrusive recollection or re-enactment of the event
in memories, daytime imagery, or dreams. Conspicuous emotional detachment,
numbing of feeling, and avoidance of stimuli that might arouse recollection of the
trauma are often present but are not essential for the diagnosis. The autonomic
disturbances, mood disorder, and behavioural abnormalities all contribute to the
diagnosis but are not of prime importance.

Adjustment Disorder: States of subjective distress and emotional disturbance, usually


interfering with social functioning and performance, and arising in the period of
adaptation to a significant life change or to the consequences of a stressful life event
(including the presence or possibility of serious physical illness). The stressor may have
affected the integrity of an individual's social network (through bereavement or
separation experiences) or the wider system of social supports and values (migration
or refugee status). The stressor may involve only the individual or also his or her group
or community. The onset is usually within 1 month of the occurrence of the stressful
event or life change, and the duration of symptoms does not usually exceed 6 months,
except in the case of prolonged depressive reaction.

Dissociative disorders: The common theme shared by dissociative (or conversion)


disorders is a partial or complete loss of the normal integration between memories of
the past, awareness of identity, immediate sensations, and control of bodily
movements. There is normally a considerable degree of conscious control over the
memories and sensations that can be selected for immediate attention, and the
movements that are to be carried out. Dissociative disorders as described here are
presumed to be "psychogenic" in origin, being associated closely in time with traumatic
events, insoluble and intolerable problems, or disturbed relationships.The onset and
termination of dissociative states are often reported as being sudden. All types of
dissociative state tend to remit after a few weeks or months, particularly if their onset
was associated with a traumatic life event. More chronic states, particularly paralyses
and anaesthesias, may develop (sometimes more slowly) if they are associated with
insoluble problems or interpersonal difficulties. Dissociative states that have endured
for more than 1-2 years before coming to psychiatric attention are often resistant to
therapy.
Includes- Dissociative amnesia, fugue, stupor, trance and possession, motor disorders,
convulsions, sensory loss, mixed disorders. Also includes Ganser’s syndrome.

ComplexPTSD Complex posttraumatic stress disorder (CPTSD) has been included in the
International Classification of Diseases, 11th Edition, consisting of six symptom
clusters: the three PTSD criteria of reexperiencing, avoidance, and hypervigilance, in
addition to three disturbances of self-organization (DSO) symptoms defined as
emotional dysregulation, interpersonal difficulties, and negative self-concept and
dissociative symptoms.

Reactive Attachment Disorder - caused by social neglect during childhood (meaning a


lack of adequate caregiving).

Disinhibited Social Engagement Disorder is similar to Reactive Attachment Disorder


but presents with externalizing behavior and a lack of inhibitions in behavior, rather
the internalizing, withdrawn behavior and depressive symptoms present in Reactive
Attachment Disorder. It is also recognized as an emotional disorder which begins during
childhood

. Epileptic seizures Non epileptic seizures


Not inducible Inducible by suggestion
Aura> 10 s No aura
Duration 2-3 min More than 3 min
Movements are stereotyped, Less stereotyped, asynchronous,
rhythmic, synchronous bizarre character
Consciousness impaired ,eyes Consciousness may be impaired,
may be open active resistance to eye opening
Automatisms If present may be bizarre
Vocalizations, initial screaming Yelling, stuttering
groans
Incontinence, tongue bite Rare
Injury rare
Post ictal confusion, headache, rare
drowsiness
Often single episode May continue, cluster like
Not always witnessed Usually occurs when witness is
present
Ictal and inter ictal EEG altered normal
Pupils dilated n fixed usually not dilated
Prolactin levels elevated within Not elevated
20 min
Plantar reflex extensor flexor

Topic: OCPD versus OCD


Comorbid OCPD and OCD:

• A most linked personality disorder with OCD is OCPD.

• Community prevalence varies 5-15%, prevalence in outpatient clinics ranges


between 15-30%.

• OCD+OCPD patients had a significantly earlier age at onset of initial OC


symptoms, earlier age at onset of OCD and more obsessions and compulsions
than pure obsessions compared to the patients with OCD/OCPD. OCD+OCPD
patients also had a higher rate of comorbidity with avoidant personality
disorder and showed more impairment in global functioning, paradoxically
secondary to poorer insight. They also have a higher predilection for hoarding
as a symptom dimension. (Garyfallos 2010; Lochner 20111.

• There is a paucity of research into the treatment of OCPD. With respect to


pharmacotherapy, there has been only a small (n = 24) randomized placebo-
controlled trial using fluvoxamine for 12 weeks. The fluvoxamine group had a
significant reduction in personality scores compared to placebo suggesting
OCPD traits may also respond to selective serotonin reuptake inhibitor (SSRI)
just like in OCD(Thamby,2019)

• Efficacy literature on psychotherapy is limited. OCPD may interfere treatment


outcomes in OCD with CBT.

Atypical panic attacks

• A subtype of panic disorder.

• Here there a symptom(s) restricted to one system

• They can be present without sense of impending doom but are still worrisome
for the patient.

• Four types appear to be most notable: 1) cardiac, 2) respiratory, 3)


gastrointestinal, and 4) vestibular. Among the four described subtypes of panic
disorder, three (cardiac, gastrointestinal, and vestibular) have corresponding
somatic mimics. somatic symptom that displays this characteristic texture may
be a somatic variant of panic disorder. In conclusion, all clinicians need to be
suspicious of somatic symptoms that have the texture of panic attacks because
panic disorder can, indeed, be a prolific and deceptive somatic mimic (Sansone
2010).

• They are notoriously linked to MDD.

Monosymptomatic OCD

• OCD is highly heterogeneous and difficult to diagnose. Based on anecdotal


evidence, it can be as disabling as schizophrenia, paradoxically to those with
good insight. It also has one of the highest degrees of perceived stigma among
patients. Patients are often hesitant to describe OC sx spontaneously to even
Psychiatrists for fear of ridicule. (Personally, I always explore OCD when
patients give me vague complaints, especially when it fits the typical profile)

• When there is a high index of suspicion its always advisable to check all the
major symptom dimensions as due to the waxing and waning nature of illness,
patients often tend to deny fewer troubling symptoms.

• Skoog and Skoog (1999) did a 40 yr. old follow up of patients with OCD.They
found a monosymptomatic picture in 12-14% patients at admission. They found
most symptoms to mimic hypochondriacal picture. Qualitative changes of
symptoms were also seen in 58% patients throughout the course.
• Presence of somatic obsessions makes it challenging. It is more common in BDD
with OCD where antipsychotic have been found to have a role.

Monosymptomatic hypochondriacal psychosis

• Historically, first used in 1978 by Munro.

• MHP has been divided into 4 main categories: Delusions of infestation(including


parasitosis); delusions of dysmorphophobia, such as of misshapenness,
personal ugliness, or exaggerated size of body parts (this seems closest to that
of body dysmorphic disorder), delusions of foul body odours or halitosis or
delusional bromosis (also known as olfactory reference syndrome) and a
miscellaneous group.

• The term “monosymptomatic” does not imply the absence of symptoms of


psychiatric disability other than the central delusion, rather that such symptoms
occur as a psychological reaction to, or as a co-morbid disorder with, the
primary psychotic or physical illness.

• Secondary non-psychotic phenomena such as anxiety symptoms, minor


depressive features, phobic avoidance behaviour and compulsive actions are
frequent accompaniments to MHP.

• The drug of choice is pimozide, mixed response has been seen with other FGAs
and TCAs.

Hypochondriacal symptoms in OCD (Fallon et al.2000 in Psychiatric Clinics of North


America)

• Many hypochondriacs experience a cycle of intrusive thoughts followed by


compulsive checking. Intrusive thoughts and sensations followed by
compulsions to check also are characteristic of patients with OCD.

• Patients with OCD experience a panoply of symptoms, some of which may


include obsessions about illness, contamination, and somatic sensations.

• Patients with hypochondriasis experience somatic sensations on which the


irrational illness fears are superimposed, whereas patients with OCD who have
a fear of illness may not necessarily have accompanying somatic sensations.

• Patients with OCD more likely view their fears as unrealistic and attempt to
resist them whereas hypochondriacs more likely have a high degree of
conviction that they have or might have a severe disease. However in case of
poor insight I OCD the lines get blurred.

• Hypochondriasis has comorbid OCD in 8-15%, whereas somatic obsessions in


OCD may be found in 20-30% roughly.

• For hypochondriasis per se, Idzorek made a functional classification into 4 types:

• As a warning signal
o In psychosis
o In depression
o Pure hypochondriacal disorder
• Barsky’s report defines transient hypochondriasis as a group of patients who
became transiently hypochondriac when experiencing a major medical illness
or the threat of one (applicable to the pandemic situation). This group had high
levels of hypochondriasis but failed to meet all DSM criteria for
hypochondriasis, most commonly because their health concerns were not
disproportionate to their health status or because their somatic symptoms had
a medical basis.

Topic: OCRDs
• DSM 5 introduced OCRDs: obsessive compulsive disorder (OCD), body
dysmorphic disorder (BDD), trichotillomania (TTM; hair pulling disorder),
excoriation disorder (skin picking; SP), and hoarding disorder (HD). This also
shows a shift from previously thought proximity to predominant anxiety related
to some of these disorders. This is also based on findings that these disorders
share: neuronal circuitry, familial relationship, bidirectional comorbidity and
treatment response, but not with anxiety spectrum disorders as much. In BDD,
the preoccupations about flaw in body parts, regions etc are mostly imaginative,
or out of proportion, and these are excessive, distressing, and time-consuming,
with repetitive rituals performed, leading to significant distress and functional
impairment. Muscle Dysmorphia is specifier in DSM 5, insight is so in both ICD
11 and DSM 5. Increased risk of anabolic steroid use is in those with muscle
dysmorphia.
• Hoarding Disorder: involves persistent difficulty parting with possessions and a
perception that items must be saved, resulting in clutter in one’s active living
space that reduces the space’s usability it has Insight specifier in ICD 11 and DSM
V and Excessive acquisition a specifier in DSM V
• Body focused repetitive behavioural disorders includes trichotillomania and skin
picking disorder. Trichotillomania (hair-pulling disorder) is characterized by
recurrent pulling out of one's hair resulting in hair loss, and repeated attempts
to decrease or stop hair pulling. Excoriation (skin-picking) disorder is
characterized by recurrent picking of one’s skin resulting in skin lesions and
repeated attempts to decrease or stop skin picking. Some individuals displaying
more focused attention on the behavior (with preceding tension and subsequent
relief) and other individuals displaying more automatic behavior (with the
behaviors seeming to occur without full awareness).
• Tics and Tourette syndrome (In DSM V it is classified as a neurodevelopmental
syndrome). A tic is a sudden, rapid, recurrent, nonrhythmic motor movement or
vocalization (e.g., eye blinking, throat clearing). A stereotyped movement is a
repetitive, seemingly driven, non-functional motor behavior (e.g., head banging,
body rocking, self-biting).
• Neurocognitive overlaps: Response inhibition and motor output suppression,
Task switching and reversal, Cognitive inflexibility, Reward processing, Conflict
and error monitoring
• Management: Drugs increasing serotonin (fluoxetine, clomipramine) may be
betterthan TCAs, buspirone may augment (specially for BDD). Medications may
be required at higher dosages and for longer duration. N-acetyl cysteine, low
dose olanzapine ERP with Cognitive restructuring, Perceptual retraining,
Motivational interviewing, DBT, ACT, may help, but for at least 6 months. Habit
reversal therapy developed by Azrin and Nunn for Trichotillomania, tics.
• Hoarding disorder: May be a separate construct altogether, has poor prognosis,
response and overall outcomes across all forms of treatment. Venlafaxine,
paroxetine, CBT model of hoarding enhanced with ACT maybe helpful.

Topic: Gender Dysphoria


Basic terms:

1. Gender - denotes the public (and usually legally recognized) lived role as boy or
girl, man or woman. Biological factors combined with social and psychological
factors contribute to gender development.
2. Assigned gender - refers to a person’s initial assignment as male or female at
birth. It is based on the child’s genitalia and other visible physical sex
characteristics.
3. Gender-atypical - refers to physical features or behaviours that are not typical
of individuals of the same assigned gender in a given society.
4. Gender-nonconforming - refers to behaviours that are not typical of individuals
with the same assigned gender in a given society.
5. Gender reassignment - denotes an official (and usually legal) change of gender.
6. Gender identity - is a category of social identity and refers to an individual’s
identification as male, female or, occasionally, some category other than male
or female. It is one’s deeply held core sense of being male, female, some of both
or neither, and does not always correspond to biological sex.
7. Gender dysphoria - as a general descriptive term refers to an individual’s
discontent with the assigned gender. It is more specifically defined when used
as a diagnosis.
8. Transgender - refers to the broad spectrum of individuals who transiently or
persistently identify with a gender different from their gender at birth. (Note:
the term transgendered is not generally used.)
9. Transsexual - refers to an individual who seeks, or has undergone, a social
transition from male to female or female to male. In many, but not all, cases
this also involves a physical transition through cross-sex hormone treatment
and genital surgery (sex reassignment surgery).
10. Genderqueer - blurring the lines around gender identity and sexual orientation.
Genderqueer individuals typically embrace a fluidity of gender identity and
sometimes sexual orientation.
11. Gender fluidity - having different gender identities at different times.
12. Agendered - ‘without gender,’ individuals identifying as having no gender
identity.
13. Cisgender - describes individuals whose gender identity or expression aligns
with the sex assigned to them at birth.
14. Gender expansiveness - conveys a wider, more flexible range of gender identity
and/or expression than typically associated with the binary gender system.
15. Gender expression - the manner in which a person communicates about gender
to others through external means such as clothing, appearance, or mannerisms.
This communication may be conscious or subconscious and may or may not
reflect their gender identity or sexual orientation.

Medical Transition Vs Social Transition:

1. Some people may cross-dress, some may want to socially transition, others may
want to medically transition with sex-change surgery and/or hormone
treatment
2. Socially transitioning primarily involves transitioning into the affirmed gender’s
pronouns and bathrooms.
Gender Dysphoria vs Gender Non-conformity:

1. Gender dysphoria is not the same as gender nonconformity, which refers to


behaviors not matching the gender norms or stereotypes of the gender
assigned at birth
2. Examples of gender nonconformity (also referred to as gender expansiveness
or gender creativity) include girls behaving and dressing in ways more socially
expected of boys or occasional cross-dressing in adult men
3. Gender nonconformity is not a mental disorder
4. Gender dysphoria is also not the same being gay/lesbian
Is Gender Dysphoria a Mental illness?

1. Not all transgender people experience dysphoria, and some controversy exists
among the medical community regarding the necessity of the psychiatric
diagnosis of gender dysphoria
2. Many transgender advocates believe that inclusion of this diagnosis increases
awareness and helps advocate for health insurance that covers the medically
necessary treatment recommended for transgender people
3. Being transgender is no longer classified as a mental illness by the World Health
Organization. In the agency's 11th International Classification of Diseases (ICD)
catalog, "gender incongruence"—the organization's term for people whose
gender identity is different from the gender they were assigned at birth—has
been moved out of the mental disorders chapter and into the organization's
sexual health chapter
ICD-11:

The Manual of International Statistical Classification ofDiseases and Related Health


Problems (ICD-11) eliminatesthe term “transsexualism” and replaces it with the
term“Gender Incongruence” (GI). This new terminology willno longer be part of the
chapter on mental disorders(chapter 6) but a new chapter is created (chapter 17)called
“conditions related to sexual health”.

Topic: Paraphilia
Paraphilia: Recurrent intense sexually arousing fantasies, urges or behaviours deviating
from the social norm that causes significant distress and associated with socio-
occupational deterioration
Prevalence is unknown
May have higher than average sex drive
Fetishism – sexual urges associated with non-living objects
Exhibitionism - enjoying exposing his genitals to strangers.
Voyeurism - enjoying observing unsuspecting strangers who are naked, disrobing, or
engaged in sexual relations (may use small video cameras and lenses)
Masochism - associating the receipt of pain or humiliation with sexual arousal.
Sadism - enjoying inflicting pain on others to gain sexual arousal. fetishist.
Frotteurism - experiencing arousal by rubbing or touching non-consenting unknown
people
Psychoanalytic theory views the paraphilias as originating in castration fears during the
Oedipal period
Lovemap Theory combines biological, psychological, and sociocultural factors to
explain the development of paraphilias
Therapist uses “Organic Reconditioning” to increase sexual arousal to appropriate
stimuli In Sexual addiction:
the person engages in illicit sexual behaviour that endangers his or her own well-
being.
the person lacks control over his or her sexual impulses.
the person can't resist sexual opportunities.
the person uses sexual behaviour as a means of reducing anxiety.
Toucherism - persistent urge to fondle nonconsenting strangers
Transgender man (transman): A transgender individual who, assigned female at birth,
currently identifies as a man. In this course, the terms transgender man, female-to male
transgender person, and FTM are used interchangeably.

HISTORICAL PERSPECTIVE
The DSM-IV abandoned the term "transsexualism" and instead used the term "gender
identity disorder" . The DSM-5 now uses the term "gender dysphoria." This is defined
as "the distress that may accompany the incongruence between one's experienced or
expressed gender and one's assigned gender". The critical element of this diagnosis is
the presence of clinically significant distress associated with the condition, as the
American Psychiatric Association (APA) points out that gender nonconformity is not a
mental disorder. The intent of this change was to better characterize the experiences
of affected children, adolescents, and adults and to avoid stigma and ensure clinical
care for individuals who see and feel themselves to be a different gender than their
assigned gender.

ETIOLOGY OF GENDER DYSPHORIA


At present, scientific investigation has neither established the true incidence nor
clarified the etiology of non-conforming gender identity formation. Genetic,
physiologic, and psychologic causes have been investigated, and available evidence
seems to support physiologic causes. Gender dysphoria cannot be explained by
variations in chromosomal patterns or identifiable hormonal abnormalities, nor is there
convincing evidence that psychologic factors (e.g., being exposed to certain family
dynamics or being raised as a member of the opposite sex) cause this condition.

PSYCHOLOGIC MANAGEMENT
While obtaining a referral for hormone therapy is the main reason that transgender
individuals seek psychologic therapy, other reasons include understanding the meaning
of their feelings; whether to externally express those feelings; coming out to self, work,
and family; seeking to network following a negative experience; or following up on
information obtained on the Internet. In some cases, an individual may have been
"caught" expressing his or her gender variance by a spouse or significant other or when
there has been a program on transgender issues in the popular media. Coming out
trans is a time of heightened vulnerability . Not everyone transitions. Some learn to live
comfortably in their role consistent with their assigned birth sex, and others may
transition partially or totally. Research on the incidence of suicide in transgender
people is scarce, but transgender people are believed to have similar suicide risks as
other people who experience major life changes, relationship difficulties, chronic
medical conditions, or discrimination on the basis of minority status. The incidence of
suicide ideation is as high as 64% and suicide attempts as high as 38% in t he adult
transgender population. These rates are significantly higher than in the general
population. Predictors of suicide among transgender individuals are similar to those of
the general population, most notably previous suicide attempts or near attempts, past
psychiatric hospitalizations, and past psychiatric treatment. Parental rejection is one of
the highest risk factors for suicide among transgender youth.

Topic: Homosexuality: status in India


Homosexual – person having same sex sexual orientation.
As per IPC 377 sexual activities against the order of nature is illegal in Delhi high court
in July 2009 read down IPC 377, which was reversed by the Supreme Court in 2013
Kamasutra has a chapter on erotic homosexuality.
Consensual Homosexuality was made legal in India by the Supreme Court.
While homosexuality has been decriminalized, gay marriage is still not legal.
Indian psychiatric society actively supports Gay rights
LGBTQIA+ stands for: lesbian, gay, bisexual, transgender, queer, intersex, asexual and
others
Estimated LGBTQIA+ individuals in India constitute of 3.8% of the population In India,
among Men who have Sex with Men, stigma is most for the receptive/predominantly
feminine partner
In India, the average age of “coming out” for gay individuals is around 20 years
Prevalence of Major depression in gay men in India is around 20 – 30 %
In gay, lesbian and bisexual, the lifetime prevalence of depression may be 1.5 times
higher than normal population
“Cheating” while being in relationship is most common in Gay
The hijra community in India predominantly worships Bahuchar mata
Most hijras in India are castrated males dressed as females

A person X, of 20 years likes to wear shirt and trousers, preferably with a tie and oxford
shoes. Spends free time by reading about bikes, sports cars and watches WWE. Gym
workout routine is mainly focussed on bigger biceps and chest. Often likes to go to the
pub with group of exclusively male friends, drinking large amounts of beer and passing
out, after becoming argumentative.

• Can we comment about gender role in this case?


• If yes, what is the probable role?
• Can we comment about gender identity in this case?
• If yes, what is the probable identity?
Yes, we can comment on the gender role and identity of X from the available
information. Both are masculine in this case. Gender identity is also called as the
experienced gender and gender role is called as expressed gender.

X often finds other men attractive, sexually. Fantasizes about a romantic relationship
with a crush, Jake. At times, X likes wearing clothes of elder sister, secretly. It is
erotically arousing for X. Finds a female body unattractive though.

• Can we comment about the sexual orientation in this case?


• If yes, what is it?
• Does this person have transvestitism?
• Is this person a transgender?
Yes, the sexual orientation is homosexual, as X has gender role and identity of a male,
and likes male sexually. Note that the natal gender of X is not mentioned and is not
necessary to derive these conclusions. X may have transvestitism as he wears female
clothes at times for erotica. However, does not express that he is uncomfortable with
his gender and does not wish to be transformed into another gender, So X is not
transgender

As per IPC 377 what is illegal in India?

• Sexual activities which are forced


• Sexual activities against the order of nature
• Sexual activities in public
Which high court in July 2009 read down IPC 377, only to be reversed by the Supreme
Court in 2013? Kolkata, Chennai ,Delhi, Mumbai
Kamasutra has a chapter on erotic homosexuality. True or false

Consensual Homosexuality is legal in India. True or false

Consensual Homosexuality was made legal in India by the Supreme Court in 2017
201820192020

Homosexuality is not legal in Jammu and Kashmir. True or false

Gay marriage is legal in India. True or false. While homosexuality has been
decriminalized, gay marriage is still not legal.

Indian psychiatric society:

• Is neutral on gay rights


• Is opposed against gay rights
• Is passive in support of gay rights
• Is active in support of gay rights
Prevalence of Major depression in gay men in India is around

• 0-10 %
• 20-30%
• 50-60%
• 80-90%
In gay, lesbian and bisexual, the lifetime prevalence of depression may be:

• 1.5 times lower than normal population


• 1.5 times higher than normal population
• 3.5 times higher than normal population
• 3.5 time lower than normal population
Homosexuality is NOT illegal in: Turkey, Yemen, Iran, Egypt

The hijra community in India predominantly worships:

• Bahucharamata
• Kali mata
• Durga mata
• Lakshmi mata
Most hijras in India are

• Males with penis and testis dressed as females


• Females with vagina dressed as males
• Castratedmales dressed as females
• Females with vagina dressed as females
LGBTQIA+ stands for: lesbian, gay, bisexual, transgender, queer, intersex, asexual and
others

Estimated LGBTQIA+ individuals in India constitute of ___% of the population: 0.8, 3.8,
6.8, 10.8

In India, among Men who have Sex with Men, stigma is most for:

• Both the partners


• The receptive / predominantly feminine partner
• The insertive/predominantly masculine partner
In India, the average age of “coming out” for gay individuals is around:

• 10 years of age
• 20 years of age
• 30 years of age
• 40 years of age
In India, the longest period of remaining in a stable consensual relationship is for

• Gay
• Lesbian
• Bisexual
“Cheating” while being in relationship is most common in:

• Gay
• Lesbian
• Bisexual
Acceptance of homosexuality in India is: Increasing, Decreasing, same

References:

• Wandrekar JR, Nigudkar AS. What Do We Know About LGBTQIA+ Mental Health
in India? A Review of Research From 2009 to 2019. J Psychosexual Health. 2020;
2(1) 26–36.
• Niranjan Hebbar YR, Majumder U, Singh RL. A study on homosexuals and their
psychiatric morbidities in a northeastern state of India, Manipur. Indian J Soc
Psychiatry. 2018; 34:245-8.
• Sathyanarayana Rao TS, Rao GP, Raju M, Saha G, Jagiwala M, Jacob KS. Gay
rights, psychiatric fraternity, and India. Indian J Psychiatry. 2016; 58:241-3.
• Sathyanarayana Rao TS, Jacob KS. Homosexuality and India. Indian J Psychiatry.
2012; 54:1-3.
• Mishra G. Decriminalising homosexuality in India. Reproductive Health Matters.
2009;17(34):20–28

Topic: Hindu Marriage Act (1955)

Condition for a Hindu Marriage.-

A marriage may be solemnized between any two Hindus, if the following


conditions are fulfilled, namely:

(i) Neither party has a spouse living at the time of the marriage;

(ii) At the time of the marriage, neither party,-

(a) Is incapable of giving a valid consent of it in consequence of unsoundness of


mind; or

(b) Though capable of giving a valid consent has been suffering from mental
disorder of such a kind or to such an extent as to be unfit for marriage and the
procreation of children; or

(c) Has been subject to recurrent attacks of insanity or epilepsy;

(iii) The bridegroom has completed the age of twenty one years and the bride the
age of eighteen years at the time of the marriage;

(iv) The parties are not within the degrees of prohibited relationship unless the
custom or usage governing each of them permits of a marriage between the two;

(v) The parties are not sapindas of each other, unless the custom or usage
governing each of them permits of a marriage between the two

Topic: Protection from children against Sexual Offences Act-


POCSO (2012)
POCSO Act provides for mandatory reporting of sexual offences against children,
so that any adult, including a doctor or other health care professional, who has
knowledge that a child has been sexually abused is obliged to report the offence
(Sections 19, 20, 21 of the ACT).

You can read about the Act online.

Age Gap between victim and offender


While acquitting a young accused of sexual assault charges under the POCSO Act,
Madras High Court made two significant suggestions that:

1. the age for the definition of a “child” be taken as 16 rather than 18, and
2. the Act account for the difference in age between the offender and the girl involved in
consensual sex

Madras HC Judgement(2018 )

• The definition of ‘Child’ under Section 2(d) of the POCSO Act can be redefined as 16
instead of 18.
• Any consensual sex after the age of 16 can be excluded from the rigorous provisions of
the POCSO Act and such sexual assault, if it is so defined can be tried under more liberal
provision.
• The Act can be amended to the effect that the age of the offender ought not to be more
than five years or so than the consensual victim girl of 16 years or more.
• While legal experts and child rights activists welcomed the redefinition of “child”, some
of them called for further discussions on the suggestion for an amendment that would
factor in the age difference.

Why decriminalizing the consensual act?

• A senior Supreme Court advocate called for decriminalization of consensual sex


between those aged between 16 and 18.
• This provision denies young person falling in this age bracket consensual sexual agency,
and subjects them to the control of families which motivated by casteist, communal or
orthodox and regressive views lodge false criminal complaints.
• Various studies tell us in the age group between 16-18, there is a lot of experimental
consensual sexual acts that take place.
• However in most of the cases, the parents of the girl lodge a complaint against the boy
that it was non-consensual.
• The judgement will help eliminate the unwarranted criminalisation of consensual or
romantic sexual relations.

Enabling more scope for fair trial

• The issue of consent would have to be decided from the circumstances rather than
putting the victim on the stand and asking her if she gave consent.
• In consensual sex both are offenders or both are victims.
• Many activists feel that consensual sex cannot be criminalised at an age when sexual
exploration is common, but argued against singling out the boy.
• Normally the boy is tried under JJ and the girl is sent to CWC. They are equal partners.

Way Forward

• There can always be a discussion on what should be the age gap between alleged
offender and victim.
• It is important to acknowledge that the law at present on this subject is very harsh and
does not leave any scope for details and dynamics of a relationship to be taken into
account by courts.
• It is also suggested that an age gap more than 5 yrs between victim and perpetrator be
ruled as aggravated assault by various courts,but the amendment is due.

Topic: Varenicline
1) Varenicline Tartarate is a partial agonist binding with high affinity to α4β2 nicotinic
acetylcholine receptor

It maintains moderate levels of dopamine to counter withdrawal symptoms and


reduces both the urge to smoke and negative mood.
Smokers using varenicline are 3 times more likely to succeed in short- and long-term
cessation compared with no medication.
• Dosing: First 3 days begin at 0.5 mg OD
Next 4 days - increase to 0.5 mg BD
Day 8 onwards - 1.0 mg BD
• Nearly 1 week is required to achieve steady-state blood levels and quit attempt
should occur during 2nd week of treatment.
• Treatment to be continued for nearly 12 weeks
• Side-effects: Nausea, insomnia, and headache.
neuropsychiatric symptoms - changes in behavior, agitation, depressed mood, suicidal
thoughts orbehavior, as well as worsening of preexisting psychiatric illnesses.
! Not recommended for pregnant women, children or people with a mental illness.
! Dose adjustments are recommended for the elderly and persons with renal insufficiency.
! Should not be used in combination with NRT due to increases in adverse affects.

2) Carbohydrate Deficient Transferrin (CDT) is a biomarker for heavy alchohol


consumption specific in tracking heavy alcohol consumption over an extended period of
time. Unaffected by mild alcohol related liver disease or fatty liver (due to obesity,
diabetes etc) which can falsely increase γ-glutamyl transferase (GGT), the enzyme in blood
most commonly used to indicate excess alcohol consumption.
CDT is also unaffected by chronic disease or B12/folate deficiencies which can increase
the mean corpuscular volume (MCV), another blood test used to detect alcohol misuse.
Transferrin remains in the circulation for 7 to 14 days and CDT, therefore, can give an
indication of alcohol consumption over this period of time, which gives it an advantage
over blood or urine alcohol measurements, which only remain elevated for 24-48 hours
after alcohol consumption.

3) California Rocket Fuel is a medical slang term for combined administration of


Mirtazapine (NaSSA) & Venlafaxine (SNRI)
Was used in STAR D trial at Level 4

4) Median time to recover from a treated episode of mania is 4 – 5 weeks.


5) A clinical test used to differentiate conversion disorder from true neurological deficit is
Hoover Test
Hoover's test is useful to identify patients with conversion disorder, who has paralysis
with one or more limbs or one side of the face or body may be affected. Here the patient
is unable to raise the affected limb from the couch but is able to raise the unaffected limb
against resistance with demonstrable pressing down of the heel on the affected side

Topic: PSYCHOTROPICS IN PREGNANCY AND LACTATION


Q. Do Psychiatric illnesses require treatment? Is pregnancy protective factor against
relapse of psychiatric illness?

Yes, psychiatric illness in pregnancy needs treatment. As it can cause both harmful
consequences to mother and fetus such as lack of obstetric care, suicide, poor
judgement, pre term deliveries, Growth retardation, infanticide etc.

Pregnancy is period of altered physiological state and also associated with significant
psychosocial stress. Pregnancy is associated with risk of spontaneous major
malformation (2 to 3% of all pregnancies) and drugs account for only 5 of every 100
malformations that occur.

Pregnancy is not a protective factor for psychosis and depression. Pregnancy does not
protect against relapse. This may be due to hormonal, social, personal changes and
emotional stress.psychiatric illness during pregnancy is an independent risk factor for
congenital malformations, stillbirths and neonatal deaths. Pregnancy with psychiatric
disorders is considered to be at high-risk pregnancy. Treatment of mental health
problems should be a priority but should be based on an individual risk-benefit
assessment.

Once a decision to offer pharmacotherapy is made, important factors in drug selection for
the mother include efficacy of the drugs available, the anticipated response of the
individual patient, and the overall toxicity profile of the drug for the mother and fetus.
Potential adverse effects for the fetus and the neonate include: 1) structural
malformations, 2) acute neonatal effects including intoxication and neonatal abstinence
syndromes, 3) intrauterine fetal death,
4) altered fetal growth, and 5) neurobehavioral teratogenicity.

Neurobehavioral teratogenicity encompasses long-term central nervous system defects


thatresult in delayed behavioral maturation, impaired problem solving, and impaired
learning.27Physical malformations do not necessarily accompany the functional deficits.
Chronic in uteroexposure to drugs may result in intoxication or tolerance postnatally.28
Neonatal drugwithdrawal symptoms may occur when drug exposure ceases at birth.29

What are FDA categories of drugs in pregnancy?


Which category drugs should be absolutely avoided in pregnancy? What are drug
categories A, B, C, D and X in pregnancy
A - Controlled studies show no risk. Adequate, well controlled studies in pregnant
women have failed to demonstrate risk to the fetus.
B - No evidence of risk in humans. Either animal findings show risk, but human findings
do not, or if no adequate human studies have been done, animal findings are negative.
C - Risk cannot be ruled out. Human studies are lacking, and animal studies are either
positive for fetal risk or lacking as well. However, potential benefits may justify the
potential risk.
D - Positive evidence of risk. Investigational or postmarketing data show risk to the fetus.
Nevertheless, potential benefits may outweigh the potential risk.
X - Contraindicated in pregnancy. Studies in animals or humans, or investigational or
postmarketing reports have shown fetal risk that clearly outweighs any possible benefit
to the patient.

Q. Spontaneous risk of major malformations in the fetus during pregnancy

- 0.5%
- 3% (ANS)
- 10%
- 20%

Q. Risk of drugs causing a malformation out of 100 malformations


-1
- 5 (ANS)
- 20
- 50

Q. Risk of using psychotropic drugs in pregnancy include


- Major structural malformations in first trimester
- Neonatal toxicity in third trimester
- Neonatal toxicity and withdrawal in lactation
- Long term nerobehavioral changes
- IUD or altered fetal growth
- All of the above (ANS)

Q. Category C of FDA drug use in pregnancy means


- No evidence of risk
- Evidence of risk with reports in phase 3 or post marketing data.

Risk cannot be ruled out as human studies are lacking (ANS)


- Absolutely contraindicated and teratogenic
Q. True regarding pregnancy and drug use except
- First trimester is most vulnerable to teratogeniciity
- Most drugs might require increased dose during 3rd trimester
- Safety profile of commonly used Psychotropics is not clearly established
- Avoid sudden stoppage of psychotropics
- Most drugs require dose decrease during third trimester (ANS)

Q. Most vulnerable period of teratogeniciity


- 2-4 weeks
- 6-10 weeks (ANS)
- 10-14 weeks
- 18-20 weeks

What are other general guidelines to be followed before initiating or continuing


psychiatric treatment in pregnancy? How to manage a woman with history of psychiatric
disorder discovering pregnancy currently maintaining well on Psychotropic

Some General guidelines


• If new onset illness, try non drug treatments like CBT if possible and try to avoid
drugsif possible, in first trimester.
• Can consider discontinuing drugs, if maintaining well and has low risk of relapse.
• Abrupt discontinuation is generally avoided in severe mental illness or high
relapse rates
• If not, consider remaining on the same drug at lower and effective doses in first
trimester if possible. Or use the drug with lowest risk.
• Generally, by the time pregnancy is detected, crucial period might be completed
hence advise proper scanning for any anomalies and see the need of supplements
to prevent anomalies if any.
• Minimise number of drugs if possible and try to bring on monotherapy
• Educated the couple and advise for regular monitoring if possible both for side
effectsand dose adjustments
• Involve the couple in decision making and Document whenever possible.
• And always benefits outweigh the risks.
• Individualised case management is necessary.

Psychiatric disorders

• A significant increase in new psychiatric episodes in first 3 months of the


postpartumperiod (Sometimes Can extend upto first 12 months). 80% are mood
disorder (mainly depression)
• Depression during pregnancy around 10%. The relapse rate of depression in those
whohad a past history is around 50% when they are pregnant.
• Postpartum depression: 10% risk, begins before delivery, higher risk in patients
with previous depressive illness & risk highest with bipolar illness.
• The risk of postpartum psychosis is 0.1 to 0.25% in general population, 50% in
bipolardisorder and 50 to 90% in a patient with a history of postpartum psychosis.

Q. Risk of post partum psychosis in general population


- 0.25% (ANS)
- 1%
- 5%
- 10%

Q. Risk of untreated psychiatric illness for the mother


- Suicide
- Substance abuse
- Unhealthy life style
- Lack of self-care and poor judgement
- All of the above (ANS)

Q. What proportion of depression patients relapse during pregnancy


- 10-20%
- 40-50% (ANS)
- 80-90%
- 3-5%

Q. True regarding pharmacokinetics in pregnancy except


- Increased GFR and plasma clearance
- Delayed gastric emptying (Important in ECT)
- Increased volume of distribution
- Increased plasma volume (Hemodilution in 3rd trimester, that’s why drug doses need
to beincreased)
- Increased protein binding
- Decreased CYP2D6 hepatic enzyme activity (ANS) – False as it is Increased
- Reduced CYP1A2 activity

DRUGS
AntiPsychotics

• First‐generation antipsychotics (FGAs) are generally considered to have minimal


risk
• of teratogenicity, although data are not conclusive, as might be expected.
• Neonatal dyskinesia has been reported with FGAs
• Neonatal jaundice has been reported with phenothiazines. Small number of
cases have reported limb defects with haloperidol, although inconclusive.

• SGAs are unlikely to be major teratogens.


• SGA exposure was associated with gestational diabetes, increased birth weight,
increased risk of cardiac septal defects.
• There are most data for olanzapine, which is relatively safe. It is associated with
bothlower birth weight and increased risk of intensive care admission, a large
head circumference and microsomia. Cloazapine associated with neonatal
seizures

• drug‐induced hyperprolactinaemia may prevent pregnancy


• If a pregnant woman is stable on an antipsychotic and likely to relapse without
medication, advise her to continue the antipsychotic. Switching medication is
generally not advised owing to the risk of relapse. Consider using the
antipsychotic that has worked best for the woman after discussion of benefits
and risks
• monitor for excessive weight gain and Gestational diabetes
• Antipsychotic discontinuation symptoms can occur in the neonate (e.g. crying,
agitation, increased suckling).
• Lactation – Clozapine is contraindicated.

Antidepressants

TCA

• Foetal exposure to tricyclic antidepressants (TCAs) via umbilicus and amniotic


fluid is high.
• TCAs have been widely used throughout pregnancy without apparent
detriment tothe foetus.
• TCA use during pregnancy increases the risk of pre‐term delivery.
• Use of TCAs in the third trimester is well known to produce neonatal withdrawal
effects: agitation, irritability, seizures, respiratory distress and endocrine and
metabolic disturbances. These are usually mild and self‐limiting.
• Preferably use nortriptyline & desipramine as they have less hypotensive and
anticholinergic side effects
SSRI
• No increase in major malformation (exception--‐‑ paroxetine); the neonatal
withdrawal is reversible complications
• SSRIs have also been associated with decreased gestational age(usually a few
days), spontaneous abortion, preterm delivery and decreased birth weight
(mean 175 g).
• Paroxetine, particularly high dose first--‐‑trimester exposure, is clearly linked to
cardiac malformation – VSD and ASD. Third--‐‑trimester use can give rise to
neonatal complication due to abrupt withdrawals.
• Fluoxetine seems safer than paroxetine.
• Sertraline appears to result in the least placental exposure
• Exposure to SSRI, when taken in late pregnancy, may increase risk for persistent
pulmonary hypertension of the newborn
• Risk of neonatal withdrawal symptoms is high with paroxetine and venlafaxine
due toshort half life

Approach to depression

• Relavant past history of episodes and course (if newly diagnosed, missed or
subthreshold symptoms)
• Family history and treatment response to mood disorders
• Grading the severity of depression and suicidality.
• Delay the medical management. Exploring alternative treatments like CBT,
REBT, IPT.
• Patients who are already receiving antidepressants and are at high risk of
relapse are best maintained on the same antidepressant during and after
pregnancy.
• If initiating an antidepressant during pregnancy or for a woman considering
pregnancy, previous response to treatment must be taken into account. The
antidepressant that has previously proved to be effective should be considered.
• If moderate to severe, First use recommended drugs would be with lower risk
such as Sertraline, Fluoxetine, Desipramine, Nortriptyline
• Be aware of risks like preterm delivery
• Monitor for serotonergic toxicity and neonatal withdrawal if given late in the
pregnancy
• If severe and Resistant, ECT can be a treatment option

Mood Stabilizers
• For women who have had a long period without relapse, the possibility of
switching to a safer drug (antipsychotic) or withdrawing treatment completely
before conception and for at least the first trimester should be considered.
• The risk of relapse both pre and post partum is very high if medication is
discontinuedabruptly.
• No mood stabiliser is clearly safe NICE recommends the use of mood‐stabilising
antipsychotics as a preferable alternative to continuation with a mood
stabiliser.
• Women with severe illness or who are known to relapse quickly after
discontinuation of a mood stabiliser should be advised to continue their
medication following discussion of the risks. NICE advises against the use of
valproate in pregnancy.
• In acute mania in pregnancy use an antipsychotic and if this is ineffective
consider ECT.
• In bipolar depression during pregnancy use cognitive behavioural therapy (CBT)
for moderate depression and an SSRI for more severe depression. Lamotrigine
is also an option.

Sedatives: Non‐drug measures are preferred Benzodiazepines, zopiclone and zolpidem


areprobably not teratogenic but are best avoided in late pregnancy. Promethazine is
widely used but supporting safety data are scarce

Anticholinergic drugs: Teratogenicity has been reported with anticholinergic


medications (in a combination with antipsychotic) use in pregnancy. They should be
generally avoided.

Q. Which of the following antidepressant is comparatively safer in pregnancy


- Amitryptiline
- Venlafaxine
- Sertraline (Ans)
- Fluoxetine (Second best)
- Paroxetine

Q. High doses of SSRI in late stages of pregnancy can cause persistent pulmonary
hypertensionin Newborn
- True
- False

Q. SSRI with most risk for cardiac defects in pregnancy


- Fluoxetine
- Sertraline
- Escitalopram
- Paroxetine (Ans)

Q. Which of the following drugs might cause reduced fetal growth and preterm delivery
- Amitryptiline
- Imipramine
- Fluoxetine (Ans)
- Sertraline
- Duloxetine

Q. Which TCA is preferable in pregnancy.


- Amitryptiline
- Imipramine
- Clomipramine
- Nortriptyline (ANS)

Q. Rate of depression during pregnancy


- 1%
- 30%
- 10% (ANS)
- 50%

Q. Risk factors for postpartum psychosis include ]


- depression
- past history of post partum psychosis
- bipolar disorder
- schizophrenia
- all of the above (ANS)

Q. All of the following are associated with SSRI during pregnancy except
- Neonatal irritability
- Low birth weight
- Neonatal respiratory distress
- Neural tube defects (ANS)
- Cardiac defects
Q. Mrs. A has a history of bipolar disorder and remained on mood stabilizers during
pregnancy. Following delivery, the breastfed infant suffered from thrombocytopenia
and anaemia which resolved after stopping breastfeeding. The most likely offending
drug is ]
- lithium
- lamotrigene
- valproate (ANS And Carbamazepine also)
- topiramate

Q. Chance of Ebstein anamoly in patients recieving Lithium during first trimester


- 1 in 10 (ANS)
- 1 in 100
- 1 in 1000
- 1 in 10000

Q. Folate supplementation dose for patients on anticonvulsants or mood stabilizers


to beconsidered is about
- 50 ug/day
- 500 mg / day
- 5 g/day (ANS – Very high doses needed)
- 50 mg / day

Q. True regarding treatment of bipolar disorder in pregnancy


- Valproate should be avoided
- In acute mania, olanzapine can be used
- Olanzapine plus Fluoxetine is used for bipolar depression in pregnancy
- ECT can be considered, in acute mania and resistant depression
- SSRI can be given in moderate to severe depression cases
- All of the above
- None of the above

Q. Anticonvulsant which is relatively safer in pregnancy


- Barbiturates
- Carbamzepine
- Levitirecetam (ANS)
- Valproate

Q. Which of the following drugs can be used for anxiety disorder during pregnancy
- Beta blockers
- Benzodiazepenes (Avoided, associated with floppy baby syndrome)
- Z drugs
- SSRI
- Buspirone

Lactation

The infant weight‐adjusted dose when expressed as a proportion of the maternal


weight‐ adjusted dose is known as the relative infant dose (RID). Drugs with an RID
below 10% are usually regarded as safe in breastfeeding. Where measured, infant
plasma levels below 10%of average maternal plasma levels have also been proposed
as safe in breastfeeding.

Factors to be considered in risk/benefit analysis: Severity and frequency of mental


illness, benefits of breastfeeding, impact of untreated maternal illness on infant and
mother, level of family support, compliance with treatment, ability recognise early
warning signs, physical health and maturity of the infant.

• Neonates and infants do not have the same capacity for drug clearance as
adults.
• Premature infants and infants with renal, hepatic, cardiac or neurological
impairment are at a greater risk from exposure to drugs.
• Infants should be monitored for any specific adverse effects of the drugs as well
asfor abnormalities in feeding patterns and growth and development.
• Infant plasma levels should be monitored if toxicity is suspected.
• Women receiving sedating medication should be strongly advised not to
breastfeedin bed as they may fall asleep and roll onto the baby, with a potential
risk of hypoxiato the baby.
• Sedation may affect a woman’s ability to interact with her children. Women
receiving sedating drugs should be monitored for this effect.
• Wherever possible: Use the lowest effective dose. Avoid polypharmacy.
Continuethe regimen prescribed during pregnancy.

Drugs in lactation
Antipsychotics – Olanzapine and quetiapine may be considered. Clozapine avoided
due toneutropenia/agranulocytosis and neonatal seizures
Antidepressants – Sertraline or Paraoxetine can be tried
Sedatives – Avoided. Lorazepam can be used at low doses, if necessary
Lithium is avoided in lactation, if she is continuing on lithium breastfeeding to be
stopped.
** Paroxetine and valproate can be used in lactation, while they are avoided in
pregnancy.

Q. Drug with relatively less RID (relative infant dose)


- Citalopram
- Escitalopram
- Fluoxetine
- Paroxetine (ANS)

Q. Drugs can be used in lactation phase


- Paroxetine
- Valproate
- Lorazepam
- Olanzapine
- All of the above (ANS)
- None of the above

Q. True regarding Postpartum blues are all except


- Most likely to cause postpartum depression or psychosis in subsequent pregnancies
(ANS)
- Has a prevalence of about 50-70%
- Occurs in the first 2 weeks of birth
- Seldom treatment needed
- Can have anxiety and irritability

Q. Postpartum psychiatric disorders requiring treatment significantly occur during


- First 2 weeks
- First 3 months (ANS – Most significant period is 2 weeks to 3 months. Although can
occur upto 12 months)
- First year
- High rate of occurrence even one year after birth

Q. Which of the following has been widely used for sedation in pregnancy
- BZD
- Z drugs
- Promethazine (ANS)

Q. Which of the following is true regarding Lithium during breastfeeding


- Is contraindicated
- Associated with cyanosis and lethargy
- Highly excreted (50%) in breast milk
- Infant serum level Can rise upto 200% of maternal serum levels
- All of the above (ANS)

Q. All of the following drugs are better avoided in breastfeeding except


- Clozapine
- Zopiclone
- Buspirone
- Zolpidem (Ans – Relatively safer Z-drug, not excreted highly in breast milk unlike
Zopiclone)

Some points discussed during the BST


• Lithium yes can be continued but needs monitoring and need for screening of
anomalies. If possible can switch to second generation antipsychotic with mood
stabilizers action at a lower dose also.

• If a patient who is well maintained on sodium valproate 750 for last 1 year , comes
and ask our advise for planned pregnancy how ll we proceed ?? should we stop and
change or reduce?

Valproate should be gradually stopped and avoided if any patient is planning


pregnancy. Advised to switch to other modes of treatment like second generation
antipsychotic or lamotrigene depending on case to case basis.
If the patient is resistant to all other drugs and have been tried with poor outcomes
or repeated relapses.. where valproate absolutely necessary l, it is advised to give
lessthan 1g per day dosage. Advise the patient to take folic acid supplementation
before planning the pregnancy itself.
It's always better to consider other drugs in pregnancy than Valproate in routine
clinical practice.

If valproate is supposedly given and she gets pregnant. advised to get a screening
for malformations as early as possible and consent for understanding the risks or
benefits,

Lithium and pregnancy

Steps of efficient management:

1. Adequate psychoeducation, clarification of doubts.


Explaining chances in simple terms and knowing your facts. There is 1 in 10 chance of having a
cardiac malformation if lithium is continued through the first trimester. Maximum risk is at 2-6
weeks after conception when many pregnancies are still undiscovered. Risk of cardiac
malformations is dose dependent
2. Consent and necessary documentation
3. Discussing risk vs. benefit ratio.
4. If planned to stop Lithium, taper and stop over 4 weeks.
5. If lithium is continued, then lithium levels tend to fall during pregnancy.

So in first and second trimester serum lithium should be checked every 4 weeks. In third
trimester every week. 24 hours before delivery and 24 hours after delivery

6. Necessary steps for prevention and early detection, such as consulting doctor prior to
pregnancy. To screen for Ebstein's anomaly, mothers should undergo level 2 ultrasound and
echocardiography of the foetus at 6 and 18 week's gestation.
7. Helping in planning delivery and liasion with OBGyn.
Mother should be advised to plan delivery in a multi-speciality hospital with good neonatal
services and where there can be liaison services between Psychiatry, Obgy and paediatrics.
9. Possible problems with babies include re term birth, low birth weight, hypothyroidism,
hypoglycaemia, low APGAR score
10. Discuss breastfeeding while on Lithium. Ideally, to be avoided.
References:
Poels et al. Lithium during pregnancy and after delivery: a review. Int J Bipolar Disord (2018)
6:26 https://doi.org/10.1186/s40345-018-0135-7
Patorno et al. Lithium Use in Pregnancy and the Risk of Cardiac Malformations, N Engl J Med.
2017 June 08; 376(23): 2245–2254. doi:10.1056/NEJMoa1612222

Topic: Psychoeducation for Lithium Augmentation


“Today I want to discuss a new treatment option with you for better management of your
depression. It is called Lithium”

As I see, you have been tried on various antidepressants so far of various groups, but you have
not got complete benefit on any of them despite them being tried for good/sufficient dose for
sufficient time.

Have you been experiencing any other problems?

Check:

• Side effects of antidepressants and adherence


• Substance use
• New stressors
• Any other comorbidities
So, the medicine we are talking about is Lithium. Lithium is a metal which is available for
prescription as a salt. It acts in depression by working together with the antidepressant which
you are continuing now. There are multiple mechanisms for its action in depression.

When you are started on lithium, there are a few things you would need to keep in mind:

1) We need to check lithium levels regularly in the body to see whether they are acceptable
for the medicine to be working properly. So we will be taking the first level after 7 days
and then then continue it at regular intervals for the first 1 year(you can mention NICE
guidelines but no need to say to the actor that it is NICE guidelines).
2) Lithium can affect the kidneys and thyroid and so we will be regularly testing for kidney
function and thyroid function along with lithium levels in blood.
3) Some common side effects are:
It can cause a metallic taste in mouth

It can cause some burning in your stomach

It may cause your hands to shake

It may cause some sleepiness

It may cause you to urinate more frequently

These problems can usually be corrected. If you experience any of these, please come
for a visit and we will be able to sort it out.

There is a possibility sometimes for lithium levels in the blood to rise dangerously, having
severe health impact. Common warning signs are more shaking, loose motion, tiredness,
anxiety, jitteriness, and confusion. There is a risk of jerkiness in your legs. If you
experience this, please stop Lithium, and visit us.

To reduce chances of this from happening, you are advised to drink water regularly.
We will be seeing you frequently and adjusting your Lithium dose based on medicine
levels in the blood. We hope that in 1-2 months you will become much better.

(If a young woman)

Lithium can also have had some dangerous effects on your baby. But chances of it are
low. So, if you plan pregnancy, please let us know beforehand

Do you have any other questions???

Management of Lithium induced tremors:

Most common is a rapid, dose related benign postural tremors especially of hands, which is
exacerbated by fine motor control or social stress.
Sudden worsening is suggestive of lithium toxicity.
Management of tremors:

Wait or decrease the dose.


Switch to slow release lithium preparation.
Beta adrenergic antagonist can be prescribed

Management of Sialorrhea with Clozapine:


Speech and swallow evaluation.
Decreasing the dose.
Sugarless gums
Systemic anticholinergics- atropine, benztropine, trihexyphenidyl, glycopyrrolate.
Local anticholinergic application.
In refractory cases even botulinum toxin injections into parotid gland has been tried (helpful
in neurological conditions)

Management of antipsychotic induced orthostatic hypotension:

Switching to higher potency agent.


Instruct the patient to sit with the feet on the floor before standing up slowly and to sit or lie
down when feeling faint.
Non pharmacological interventions: stockings, volume expansion with fluids.
Last resort can be pressors

Topic: Elderly Abuse:

The Maintenance and Welfare of Parents and Senior Citizens (MWPSC) Act, 2007 (Gazette of
India 2007) defines a senior citizen as a person who has attained the age of 60 years or above.
The United Nations also treats persons aged 60 years or more as elderly. Under the law relating
to income tax in India, persons are regarded as senior citizens only after they become 65 years
old.
Elder abuse is a single or repeated act, or lack of appropriate action, occurring within any
relationship where there is an expectation of trust, which causes harm or distress to an older
person. This type of violence constitutes a violation of human rights and includes physical,
sexual, psychological, and emotional abuse; financial and material abuse; abandonment;
neglect; and serious loss of dignity and respect.

Elder abuse is an important public health problem. A 2017 study based on the best available
evidence from 52 studies in 28 countries from diverse regions, including 12 low- and middle-
income countries, estimated that, over the past year, 15.7% of people aged 60 years and older
were subjected to some form of abuse.

Elder abuse in community settings (1) Elder abuse in institutional settings (2)

Type of abuse Reported by older Reported by older Reported by staff


adults adults and their
proxies
Overall Prevalence 15.7% Not enough data 64.2% or 2 in 3 staff
Psychological abuse: 11.6% 33.4% 32.5%
Physical abuse: 2.6% 14.1% 9.3%
Financial abuse: 6.8% 13.8% Not enough data
Neglect: 4.2% 11.6% 12.0%
Sexual abuse: 0.9% 1.9% 0.7%

Physical abuse is when an elder experiences illness, pain, injury, functional impairment, distress,
or death as a result of the intentional use of physical force and includes acts such as hitting,
kicking, pushing, slapping, and burning.
• Sexual abuse involves forced or unwanted sexual interaction of any kind with an older
adult. This may include unwanted sexual contact or penetration or non-contact acts such
as sexual harassment.
• Emotional or Psychological Abuse refers to verbal or nonverbal behaviors that inflict
anguish, mental pain, fear, or distress on an older adult. Examples include humiliation or
disrespect, verbal and non-verbal threats, harassment, and geographic or interpersonal
isolation.
• Neglect is the failure to meet an older adult’s basic needs. These needs include food,
water, shelter, clothing, hygiene, and essential medical care.
• Financial Abuse is the illegal, unauthorized, or improper use of an elder’s money,
benefits, belongings, property, or assets for the benefit of someone other than the older
adult.
• Financial Abuse: The Criminal Procedure Code, 1973 – Section 125 of Criminal Procedure
Code allows maintenance to parents and wife which indirectly covers the interim
maintenance of elder parents ensuring the economic security of the elders.

MAINTANANCE AND WELFARE OF PARENTS AND SENIOR CITIZENS ACT 2007:


Parent means father or mother whether biological, adoptive or step father or st ep mother.
Senior Citizen means citizen of India above 60 years old. Maintenance includes provision of food,
shelter, medical attendance and treatment

References:
(1) Elder abuse prevalence in community settings: a systematic review and meta -analysis. Yon
Y, Mikton CR, Gassoumis ZD, Wilber KH. Lancet Glob Health. 2017 Feb;5(2):e147-e156.
https://www.ncbi.nlm.nih.gov/pubmed/28104184
(2) The prevalence of elder abuse in institutional settings: a systematic review and meta-
analysis. Yon Y, Ramiro-Gonzalez M, Mikton C, Huber M, Sethi D. European Journal of Public
Health 2018. https://www.ncbi.nlm.nih.gov/pubmed/29878101
(3) The mortality of elder mistreatment. Lachs MS, Williams CS, O'Brien S, Pillemer KA, Charlson
ME. JAMA. 1998 Aug 5;280(5):428-32. https://www.ncbi.nlm.nih.gov/pubmed/9701077
(4) https://www.who.int/news-room/fact-sheets/detail/elder-abuse
(5) https://www.cdc.gov/violenceprevention/elderabuse/fastfact.html

Topic: Neurobiology of treatment resistant OCD

NEUROBIOLOGICAL FINDINGS OF TREATMENT RESISTANT OCD

Neuroanatomical (sMRI Significantly Smaller bilateral OFC


& fMRI) Left precentral gyrus and right orbitofrontal cortex (a/w Hoarding) Bilateral
basal ganglia abnormality (a/w sexual & aggressive obsessions)

Neurotransmitters Increased glutamate in prefrontal regions


Increased dopamine levels (a/w symptom severity)
Neuroimaging • DTI - Persistent white matter abnormalities in anterior cingulate
gyrus and the parietal region, right posterior cingulate gyrus, and
left occipital lobe
• MRS - Significantly lower N-AC/creatine ratio in right basal ganglia
& higher choline/creatine ratio in right thalamus (Abnormal
neuronal metabolism in the right basal ganglia and right
thalamus)
Genetic TPH2 gene polymorphism (a/w early onset) Deletion of Sapap3 gene
Under research - Glutamate transporter gene; NrCAM gene

Neurophysiological Circadian rhythm abnormalities


Neuropsychological Predominant Attention deficits , Spatial working memory deficits (fMRI)
Neuroimmunology PANDAS , Sydenhams chorea
Expression of D8/17 (type of B-lymphocyte antigen)

*DTI – Diffusion Tensor Imaging

*MRS –Magnetic Resonance Spectroscopy

A neurodevelopmental subtype of OCD has been proposed that is characterized by


childhood onset, predominantly male sex and poorer response rate to treatment,
neuromotor abnormalities and increased comorbidity with ADHD and Tourette’s disorder.

Articles to refer:

1. https://www.researchgate.net/publication/11245455_Treatment_non-
response_in_OCD_Methodological_issues_and_operational_definitions
2. https://www.scielo.br/scielo.php?script=sci_arttext&pid=S2237-
60892013000100004&lang=en
3. https://www.ncbi.nlm.nih.gov/pmc/articles/PMC7334048/
4. https://pubmed.ncbi.nlm.nih.gov/16401151/
5. https://www.researchgate.net/publication/275344411_Neurobiology_of_Obsessive-
Compulsive_Disorder

ASSESSMENT AND MANAGEMENT OF TREATMENT RESISTANT OCD

1. Assessment
40 – 60% of patients do not respond adequately to treatment with either SRIs or CBT both in
short term and long term follow-up. There is no single treatment that is effective in diverse
group of patients.

Commonly used assessment tools are-

i. Yale- Brown Obsessive Compulsive Symptom Severity Scale (Y-BOCS)


ii. Clinical Global Impression- Severity and Improvement scales (CGI-S & CGI-I)
Definitions:
• Response: >= 35% reduction in Y-BOCS scores.
• Partial non response: > 25% & < 35% reduction in Y-BOCS score.
• Non response: < 25% Y-BOCS score reduction and CGI-I of 4 or more (thase and rush).
• Resistance: poor response to at least 2 adequate trials with SSRIs. < 35% reduction in Y-
BOCS and minimally improved, no change or worse on CGI-I
• Refractory: failure to respond to all available treatments (thase and rush).Resistance:
Evaluation of non response:
• Re evaluation
• Assess for insight
• Evaluate co morbid axis I & II disorders: OC spectrum disorders, anxiety disorders, tic
disorders, bipolar and psychotic disorders, schizotypal, OCPD.
• Assess if adequately treated or not.

1. Management

Adequate trial with adequate dosage:

• Fluoxetine: 60 – 80 mg
• Fluvoxamine: 250 – 300 mg
• Sertraline: 150 – 200 mg
• Paroxetine: 60 – 80 mg
• Clomipramine: 150 – 250 mg Adequate period: 10 -12 weeks
If non response, then next logical steps:
• Switching:
o To another SSRI . Pr i or drug no n r esp onse t o an initi al SSRi tr ial may
predictpoor response to fluvoxetine, venlafaxine and paroxetine.
o Clomipramine
• Augmentation:
o Anti psychotic agent: risperidone- good evidence.
o Other SRIs
o Promising treatments: SSRI + clomipramine, high dose SSRI monotherapy, anti
glutaminergic drugs (riluzole, memantine, n-acetyl cysteine), inositol, opoid
agonists, mirtazapine.
o May be effective: risperidone, quetiapine, olanzapine, haloperidol, pimozide,
IV clomipramine, clonazapem
o Probably ineffective: nicotine, carbamazepine, lamotrigine, gabapentin,
reboxetine, ondansetron, buspirone, trazadone, lithium, anti convulsants.
• Combination:
o SSRI + Clomipramine (optimal dose of both drugs)
o SSRI + CBT
o Others: IV clomipramine, rTMS, DBS, ECT
• Venlafaxine, if Clomipramine is poorly tolerated.
• Intensive residential treatment (IRT)
• Neuro surgery: anterior cingulotomy, capsulotomy, limbic leucotomy, subcaudate
tractotomy.
Intensive residential treatment (IRT)

• Multidisciplinary team
• Highly structured program monitored closely for treatment adherence
• Includes: 2-4 h CBT daily, weekly pharmacological monitoring and group sessions
• Usual length of the program: 1 to 3 months
• Total responders: 59% (stewart et al, 2005)
• Predictors of response to IRT- low initial severity, female gender, better baseline
work and social adjustment.
• It is time consuming and intensiveReferences:
1. YC Janardhan reddy and Shoba srinath. Obsessive compulsive disorder: current
understanding and future directions. NIMHANS Bangalore.

2. S pallanti and L quercioli. Treatment refractory obsessive compulsive disorder:


Methodological issues, operational definitions and therapeutic lines. Progressin neuro
psychopharmacology and biological psychiatry. 2006, 30, 400-12.

Topic: OCD in Children

One of the most prevalent psychiatric disorder affecting children and adolescents. It is usually
under diagnosed and under treated. Children have difficulty in articulating or frequently hide
the symptoms. Studies suggest that childhood onset OCD is a subtype of OCD with unique
correlates.

Prevalence: 1 – 2% in USA. 0.25% point prevalence in UK. 1 -4% in Indian studies. Incidence:
bimodal, pre adolescence and early adult life. Boys > Girls (3:2).
Etiological findings:

Many have academic difficulties, which cannot be explained by the primary disorder.

Involvement of fronto-striatal system, which is involved in visuo spatial integration,


short term memory, attention and executive functions.

Some studies have found some deficits in executive functioning, visuospatial


performance, attention, visual memory, visual organisation and velocity.

Greater genetic loading in paediatric onset OCD.

In children, OCD that is co morbid with ADHD represents a distinct familial subtype.

Pooled analysis of replication studies have showed significant association in themarker


SLC6A4HTTLPR, functional length polymorphism in promoter region of SLC6A4 that
encodes for the serotonin receptor protein.

Glutamenergic transporter gene SLC1A1 has been studied in patho physiology.

Sporadic case has less genetic loading then familial cases. Environmental triggers are
more relevant in sporadic cases.

There are higher perinatal insults in early onset OCD in males; like dystocic delivery,
use of forceps, breech presentation, prolonged hypoxia etc.

Exposure to in utero; coffee, cigarettes and alcohol have also been implicated.
PANDAS:
Neuro behavioral disturbance associated with Sydenham's chorea. As an immune
response to group A beta-hemolytic streptococcus infection, which leads to cross
reactivity with and inflammation of basal ganglia with a distinct neurobehavioral
syndrome that includes OCD and tics.
Evidence supports the belief that a subset of children with OCD and Tourette's
syndrome can have both onset and clinical exacerbations linked to GABHS.
Diagnostic criteria are laid down by Swedo et al: presence of OCD &/ tic disorder, prepubertal
onset (3 – 12 years), episodic course, symptom onset or exacerbations are temporarily related
2 documented GABHS infections and association with neurological abnormalities. Clinical
features:

Frequently show compulsions without well defined obsessions.

Compulsions like blinking and breathing rituals are common than washing orchecking
rituals.

Obsessions are centered around fear of a catastrophe family event.

Symptoms tend to wax and wane but persisted in majority and changed over time.

Religious and sexual obsessions are selectively over represented in adolescents.

Hoarding is seen more often in children.

Parents tend to get involved in their child’s rituals, especially in reassurance seeking.

Parents unknowingly reinforce compulsive behaviour by providing verbal


reassurance or other assistance to children; like handling things which child avoids.

Co morbid disorders (up to 50%): separation anxiety disorder, mood disorder (major
depression), disruptive mood dysregulation disorder (DMDD), tic disorders, enuresis, learning
disabilities. Co morbid disorder is important in identifying the future course and outcome as
well as treatment response.

Course & prognosis: long term prognosis is better than adults. Many will remit entirely/
become clinically sub threshold over time.

Poor prognostic factors:


Very early age of onset.
Concurrent psychiatric diagnosis (like multiple anxiety disorders, mood disturbance,
DMDD)
Poor initial response
Long duration of illness
Positive first degree family history of OCD

Diagnosis: clinical experience suggests that the adult criteria can be applied to child cases, but
there are important caveats. Some children may not be willing or able to verbalise their
obsessive thoughts, long term contact has demonstrated that about 40% do not have
obsessions and only report compulsions accompanied by a vague sense of discomfort if the
rituals are not carried out. DSM 5 includes a subtype called ‘belief’ in the necessity for these
thoughts/ behaviours.

DD:

Eating disorders
Depression
Phobias
Autism spectrum disorders Treatment:
1. Pharmacotherapy:
• Efficacy of clomipramine and SSRIs are well established in many RCTs.
• All are superior to placebo & clomipramine showed superiority over all the
SRIs which do not differ from one another.
• Clomipramine is associated with severe side effects, including potential
arhythmogenic effects.
• There are wide spread concerns of adverse somatic effects and reports of
suicidal ideation in children who are on SSRIs.
• Thumb rule is: low initial dosage with slow upward titration.

2. Psychotherapy:
• Exposure and Response Prevention (ERP): established efficacy.
Ideal for younger children: < 11 years. Particularly useful for contamination
fears and symmetry rituals.
Tailor therapy to the child’s symptom profile.
A hierarchy of increasingly intense anxiety-provoking situations that trigger
obsessional thinking is constructed and children are exposed gradually to
these situations and encouraged to refrain from engaging in compulsive
behaviours.
Usually combined with other behaviour therapies like; anxiety management
training and extinction.
Cognitive components are added such as normalising intrusive thoughts and
re appraising notions of personal responsibility.

• Cognitive approaches are best suited for obsessional moral guilt or pathological
doubt.
• Family involvement is essential part of the treatment. Central role of family in
children affected with OCD should always be kept in mind in devising the
management plan. Disentangling parental psychopathology from disturbed family
psychopathology associated with child’s OCD is critical. Parents control many
contingencies of child’s daily behaviour and are therefore dependent.
• Family based CBT, conducted with either individual families or in a group setting.
• Poor response: diminished insight, family accomodation, co morbidity

3. Combination treatment:
• Paediatric OCD Treatment Study (POTS): Multicenter study
Randomized 112 children to either sertraline, cognitive behavioral therapy
(CBT), the treatments combined, or pill placebo.
12 weeks
All active treatments were superior to placebo.
Combined treatment also proved superior to CBT alone and to sertraline
alone, which did not differ from each other

Family accommodation: the family members can become enmeshed in rituals. It can be
associated with attacks of rage, coercive and disruptive behaviours. This is of particular
importance and is worth measuring, monitoring, and tracking in clinical care.

References:
1. YC Janardhan reddy and Shoba srinath. Obsessive compulsive disorder: current
understanding and future directions. NIMHANS Bangalore.

2. Thapar A and Pine DS. Rutter’s child and adolescent psychiatry. Wiley Blackwell. 6
edition.

OCD in pregnancy and postpartum


Often goes under recognised and untreated.
Possible factors: pressure on mother to suppress negative emotions during what is
expected to be a joyful occasion. Childbirth education programs do not sufficiently address
the inauspicious aspect. Healthcare providers for the child or new mother may fail to
inquire about these problems.

Symptomatology:
Obsessional phenomena may occur at higher than expected rates in post partum than for
puerperal females
There is a relationship between postpartum depression and OCD symptoms, particularly
unwanted intrusive thoughts of hurting the newborn. However, it is unknown whether
these OCD symptoms represent a cause or effect of postpartum depression. Given that
depression involves unwanted and/or self destructive thoughts, it is possible that
obsessional problems (e.g., unwanted aggressive thoughts) are symptoms of postpartum
depression. Alternatively, it is plausible that the presence of unwanted obsessional
thoughts is distressing to the point that they give rise to depressive symptoms.
Obsessional thoughts may include concern fears of contamination, aggression or violence,
religion, sex, symmetry, making mistakes, or serious illnesses, among other themes. Those
developing symptoms during pregnancy tend to report contamination obsessions and
washing or cleaning rituals. Where as in contrast to post partum they tend to report
unwanted intrusive obsessional thoughts of harming the infant along with phobic
avoidance of fear cues.
Compulsive behaviors may involve ritualistic washing, checking, counting, saving, praying,
repeating routine activities, mentally neutralizing of distressing ideas, etc.

Postpartum OCD in males:


Circumstantial factors play a role in postpartum OCD in females, these symptoms to sometimes
occur in partners of childbearing women (e.g., the child’s father) given that the birth of a new
infant also generates a great deal of strain for such individuals.
Topography of these cases was highly similar to that described in studies of females with
postpartum OCD symptoms.

Treatment:
Pharmacotherapy by serotonin reuptake inhibitor medication and cognitive- behavioral
psychotherapy using the procedures of exposure and response prevention.
SRI pharmacotherapy, the most widely used therapy for OCD, typically results in a 20–40%
reduction in symptoms.
Although CBT is associated with excellent short and long-term response in OCD (average
of 60–70% symptom reduction), there are no data on efficacy of this treatment for women
with postpartum-onset OCD. Pregnancy status does not appear to be related to variables
thought to predict poor response to the therapies

References:
1. Abramowitz JS, Schwartz SA, Moore KM and Luenzmann KR. Obsessive compulsive symptoms in
pregnancy and puerperium: A review of the literature. Anxiety disorders, 2003. 17, 461-78.

Topic: Summary Treatment resistance in Bipolar Disorder

Definition of treatment resistance

There is no consensus on definition of BPAD. In case scenarios consider following pointers while
considering treatment resistance

1.Phase of the illness

• Acute Mania -Common


• Acute depression – Criteria similar to resistance in unipolar depression. In addition to
antidepressants there is also failure to respond to mood stabilizers
• Break through episode during maintenance phase
2.Functional outcome of treatments

3. Specify the number of failed adequate trials

• Maximum tolerated dose and duration for each drug


• Rule out Pseudo resistance – Intolerance,non-compliant
4. Consider use of anti-depressants while making a diagnosis treatment resistance in mania and
continued cycling

5.Definition of Response

• For acute mania, 50% decrease in Young Mania Rating Scale (YMRS) vs YMRS score below a
specified threshold
• For maintenance treatment, one breakthrough episode vs>one episode in specified time
frame
• Require specified number of combination treatments
Factors predicting Treatment resistance

Demographic

➢ Male (Except rapid cycling)


➢ Low income group
➢ Single,widowed,seperated,divorced
Disease factors

➢ Early onset
➢ Delay in initiating treatment
➢ Non Compliance
➢ Inadequate treatment (dose or duration )
➢ Depression more than Mania
Comorbidity

➢ Substance use
➢ Medical comorbidity- HTN, DM, Obesity
➢ Hypothyroidism
Family history of bipolar disorder
Algorithm for treatment of Bipolar resistance in mania

Adjunctive psychotherapies

1. Family-focused treatment (FFT)- FFT is an amalgam of psychoeducationa,


communication skills training for dealing with interfamilial stress and problem-solving
skills,89 administered as approximately a 20-session therapy over 9 months.
Improvement in families with high EE. Medication adherence improved mania

2. Cognitive therapy – 12-18 sessions of individual cognitive therapy with pharmacological


treatment improved medication adherence, social functioning and mean depressive
symptoms
3. Group psychoeducation- 8-12 members, 120 sessions
4. Interpersonal and social rhythm therapy (IPSRT)- IPSRT, an individual therapy derived
from interpersonal therapy, focuses on resolution of interpersonal problems,
prevention of future problems in these areas, the importance of maintaining regularity
in daily routines, and the links between mood symptoms and the quality of social
relationships and social roles.

Neurobiology of BPAD:
CINP review found three major correlates with treatment resistance:

• Presence of family history of mood disorders


• Presence of EEG abnormalities
• Involvement of neuroinflammatory mechanisms
In depression associated with bipolarity, a state of dorsal raphe stasis induced by SSRIs has been
linked with resistance. (doi: 10.3389/fnbeh.2014.00189)

For an overall understanding of neurobiology of BPAD, the following article does a comprehensive
review: doi: 10.3389/fpsyt.2014.00098

• Multiple recurrences were associated with:


o Higher lateral ventricular volumes
o Decreased VLPFC volumes
o Decline in thickness of DLPFC
o Progressive dysfunction of resting state cortico-limbic networks – the presence of
early life stressors starts this process sooner
o Higher overall cortisol levels in addition to aberrant reactivity, and even greater
flattening of their cortisol curves
Bipolar depression:
1. Approach according to IJP guidelines
2. Approach to treatment-resistance according to CINP guidelines
Non-pharmacological interventions would be the same as for prevention of recurrence of Manic
episodes, i.e,

• Family focused treatment


• CBT
• IPSRT
• Group therapy and support groups
Topic: Treatment resistant depression

5 R’s of antidepressant treatment

⚫ Response or Treatment response is greater than 50% improvement of the symptoms (better
ona standard scale like HAM - D). Partial response is some improvement but less than 50 %
improvement. Non response: no clinically meaningful response to treatment.
⚫ Remission is full reduction of symptoms or maintenance of response for over weeks to upto 6
months. (Some authors mention it as in first few weeks or 8-12 weeks.)
⚫ Recovery is if reduction of symptoms maintained for more than 6 month (After Remission).

Note - Ham - D score should be less than 7 to call it as remission or recovery.

⚫ Relapse is when depression occurs before full remission that is in first few months (Occurs
during remission).
⚫ Recurrence is when depression occurs after the patient has recovered (Occurs in recovery
phase).

⚫ Approximately 30% to 45% (one-third) of patients do not have an adequate response. Among
these patients 19% to 34% are considered non-responders. Poorly responsive group are
sometimes called as ‘Resistant’ or ‘Refractory’.
⚫ Inadequate response – Failure to achieve remission, which is potential for :
• Greater social dysfunction
• Greater number of somatic complaints
• Increased relapse risk
• Poor outcome

Factors to consider before establishing resistance or failed treatment strategies are

✓ Correct diagnosis - like Unipolar or Bipolar


✓ Comorbid psychiatric conditions like - Substance abuse, Anxiety disorders, OCD, personality
disorders etc
✓ Medical comorbidities like hypothyroidism, diabetes, MI, cushing's disease etc
✓ Residual symptoms
✓ Adequate dose and duration
✓ Side effect profile
✓ Treatment adherence
✓ Severity of illness and appropriate drug therapy

✓ Risk factors
• Female gender
• Older age – medical co-morbidity, multiple medication, stressors, biological changes
associated with ageing
• Young onset –less responsive and frequently correlated with personality disorders
• Psychosocial - Lower socioeconomic status, poor social support, Family conflicts, Chronic
stressors, Multiple losses/events, Lower levels of education
⚫ Nearly one-third of patients abruptly discontinue antidepressants treatment within first month
and 44% within first three month
⚫ Primary determining factors of non adherence
• lack of patient education
• unpleasant adverse effect
• poor quality of doctor-patient relationship
• Negative attitude to the results of treatment
• Alcohol dependence, personality disorders
(Nemeroff CB 2003)

⚫ Residual symptoms of Depression most commonly include - pain symptoms, fatiguability,


Insomnia, reduced concentration, lack of interest, Low mood and psychomotor retardation are
uncommon.. Rather might be indicators of poor response and relapse than residual symptoms.

TREATMENT RESISTANCE

Estimates of 12 month prevalence 2-3%.


⚫ Pseudo resistance - Inadequate response or partial response due to inadequate dose and
duration of drug intake (Sackeim ,2001)
⚫ Definition - Treatment Resistance. Coming to definition of treatment resistance, different
criteria used by authors.
◼ Traditional definition - The treatment resistant depression indicates failure to respond to
adequate trial of antidepressant therapy.
◼ At least 2 trials of different anti depressants have failed i.e Non response to two adequate
trials (250-300 mg of Imipramine for 6-8 weeks) of two different classes of antidepressants
(Souery et al 2001)
◼ At least 3 or more failed trials, one being TCA (Nierenberg et al)
◼ Absolute resistance - Failure to respond to a maximal, nontoxic dose of a given
antidepressant, with confirmed compliance over an extended treatment period (e.g., an 8-
week trial of imipramine at 300 mg per day or an 8-week trial of sertraline at 200 mg per
day)
◼ Refractory Depression - Refractoriness is often used indiscriminately. Refractory
depressionsuggest a higher degree of resistance and may be associated with absolute or
sustained resistance (generally to both pharmacotherapy and psychotherapy)

Thase and Rush staging (1995)


• Stage I - Failure of at least one adequate trial of anti depressant monotherapy (eg SSRI)
• Stage II - Stage I + Failure of different antidepressant class (eg - SNRI and SSRI)
• Stage III - Stage II + Failure of trial of TCA
• Stage IV - STAGE III + Failure of MAO-I Trial
• Stage V - Stage IV + Failure of ECT course

Massachussets General Hospital Staging


• Stage I – Nonresponse to each adequate trail of antidepressant.

• Stage II – Optimization of dose, duration & augmentation\ combination of each trial.

• Stage III – ECT.

European Staging - Individualised and mentions to each class with duration

⚫ A - Nonresponder to TCA/SSRI/SNRI MAOi


ECT
Other antidepressant(s)
No response to one adequate antidepressant trial Duration of trial: 6-8 weeks

B - TRD / Resistance to 2 or more adequate antidepressant trials Duration of trial(s):


TRD 1:12-16 weeks
TRD 2: 18-24 weeks
TRD 3: 24-32 weeks
TRD 4: 30-40 weeks
TRD 5: 36 weeks-1 year

⚫ C -Chronic resistant depression, Resistance to several antidepressant triais, including


augmentation strategy. Duration of trial(s): at least 12 months.

STAR-D Trial

Link - https://www.nimh.nih.gov/funding/clinical-

research/practical/stard/allmedicationlevels.shtml

STAR*D stands for Sequenced Treatment Alternatives to Relieve Depression. The following are its
findings :
• It was NIMH funded, Nationwide public health clinical trial.

• Largest and longest study ever done to evaluate depression treatment.

• Seven-year period, N>4,000 outpatients (Approx),

• Aged 18-75 years


7 Year long multicentric multiphasic RCT The STAR*D trial included remission (the near-absence of
symptoms, rather than simply a reduction in symptoms) as an outcome measure, as there is
evidence that patients with depression who achieve remission function better and are less prone to
relapse than those who achieve only partial improvement in symptoms. Criticised for many
dropouts and subjects in last levels were very less.

◼ Level I - Citalopram alone


◼ Level II - Switch to Bupropion SR, Sertraline , Venlafaxine ER, Cognitive therapy
◆ Or augment with Bupropion, Buspirone, cognitive therapy
◼ Level IIa - Switch to bupropion or Venlafaxine (for those on CBT/cognitive therapy only in
level II)
◼ LEVEL III - Switch to Mirtazapine or Nortriptyline
◆ Or augment Li or T3( only with Bupropion, Sertraline, Venlafaxine)
◼ LEVEL IV - Switch to Tranylcypromine (MAOI)
◆ or Mirtazapine combined with Venlafaxine

Findings of the trial


Overall remission rate with treatments - Approximately 67%
⚫ Level 1: findings were : About 30% of patients achieved the goal of remission of symptoms.
70%of patients, citalopram alone is insufficient to sustain long-term remission (Trivedi & Rush
et al.,Am J Psychiatry 2006;163:28–40.)
⚫ Level 2: findings were : Augmentation phase, approximately 30% of patients treated with
either bupropion-SR or buspirone in addition to their citalopram achieved remission (Trivedi &
Fava etal., N Engl J Med 2006;354:1243–52.)
⚫ Level 3: findings were : Remission rates did not differ significantly for these two medications.

12.3% for mirtazapine and 19.8% for nortriptyline. 15.9% of lithium-treated patients and 24.7%
of T3 treated patients achieved remission (Fava & Rush et al., Am J Psychiatry 2006;163:1161–
72.)

Strategies for Treatment Resistant Depression


⚫ Switching strategies - include shifting to another class of antidepressant. Eg- SSRI to SNRI,
NaSSA, TCA etc. Advantages: Continuing monotherapy, Avoiding drug interactions , Enhancing
adherence , Minimizing the cost. Disadvantages - Possible loss of partial response of the prior
trial, Discontinuation phenomena
⚫ Combination Strategies - Combining two classes of antidepressants for better response. Any
combination with Bupropion or mirtazapine has better evidence compared to other
combinations.
◼ California Rocket Fuel - Combination of Venlafaxine with Mirtazapine, Combination is
based on the theory that there is inhibition of serotonin, nor epinephrine and dopamine
systems plus blockade of α2 adrenoceptors and 5HT2A receptors (Stahll 1997)
◼ Dalhousie serotonin cocktail - Combination of pindolol, tryptophan and nefazodone.
Enhancement of availability and release of serotonin by tryptophan in the presence of
nefazodone and pindolol
“Newcastle cocktail” (phenelzine, L-tryptophan, and lithium)

⚫ Augmentation strategies include addition of any other mode of treatment to antidepressant


other than intended antidepressants.Eg- CBT, Lithium, Thyroid hormone (T3 >T4), atypical
antipsychotics like olanzapine, quetiapine, aripiprazole and anticonvulsants like lamotrigene
have the best evidence

Other Strategies/Methods of Treatment


⚫ ECT - Response is 50% in people with resistance to medications
• Need to consider continuation therapy
• Post ECT failure patients can be tried with some medication combination as there can be
better response due to post synaptic sensitivity
⚫ Vagus nerve stimulation [FDA Approved in 2005 for adjunctive treatment of TRD]. Mainly as a
Augmentation Strategy. Despite the promising results reported mainly in open studies, further
clinical trials are needed to confirm its efficacy in TRD
⚫ RTMS [FDA Approved in 2008 for patients with failed trial of one,but not more than one
antidepressant]. Given mainly over Left DLPFC. Relatively low response and remission rates,
theshort durations of treatment, and the relative lack of systematic follow-up studies suggest
needfor further studies.
⚫ Psychotherapy – CBT, IPT and IPSRT. Better response to combination of AD & Psychotherapy
than either therapy alone (Keller ,2000). Psychotherapy may reduce attrition rates from
pharmacological treatment and strengthen therapeutic alliances during the long-term
management of chronic mood disorders
⚫ Ketamine - Newer, Rapid response. Good response even with other risk factors of treatment
resistance depression. Esketamine nasal spray FDA approved for depression
⚫ Invasive - T-DCS, Psychosurgery like limbic leucotomy

Topic: Treatment resistant schizophrenia (TRS)


Pathophysiology-

It has been proposed that TRS could be regarded as the result of both neurodevelopmental and
neurodegenerative processes that occur through the life span and resistance to antipsychotic
drugs which develop at a certain time point in the evolution of schizophrenia. The investigators
of this theory propose the following stages for the development of TRS: (1) cortical pathology
and deficient neuromodulatory capacity due to genetic/epigenetic etiologicfactors occurring
during childhood, (2) neurochemical sensitization leading to dopamine release and
development of psychotic episodes occurring during adolescence, and (3) neurotoxicity with
consequent development of structural neuronal changes in adulthood.

Structural Brain Abnormalities in TRS-


Systematic reviews of early studies relating brain abnormalities to clinical outcome foundlittle
evidence that ventricular enlargement is related to treatment response.

Zugman and colleagues, found that, when compared with non-TRS, patients with TRS showed
more pronounced decreased thickness in the dorsolateral prefrontal cortex (DLPFC).

Further reading from-Treatment-Resistant Schizophrenia, Elkis & Buckley, Psychiatr Clin N


Am(2016), http://dx.doi.org/10.1016/j.psc.2016.01.006, psych.theclinics.com

A recent review (Vita et al., 2019) suggests that we still do not know conclusively whether the
brain structural abnormalities progress along with schizophrenia throughout the life, a
confounding factor being antipsychotics. Ventricular enlargement though most commonly
studied remains inconclusive, as mentioned above. Gray Matter (GM) volume reduction
particularly in
frontal, temporal, and occipital regions and enlargement of the posterior corpus callosum,
splenium, and widespread disruptions to White Matter (WM) tract integrity are found in TRS.
So, TRS patients show greater GM reduction, especially in frontal regions, and an increase in
WM volume.

Neurotransmitters:

Dopaminergic System was implicated in past, suggesting role of hyperactivity of D2 receptors.


But, in TRS, studies have also found reduced striatal dopamine synthesis capacity and high levels
of glutamate in the ACC with normal presynaptic dopamine synthesis. These point towards a
“non-dopaminergic” subtype of schizophrenia. This not only includes glutamate/GABA system
but also serotonergic system

Vita A, Minelli A, Barlati S, Deste G, Giacopuzzi E, Valsecchi P, Turrina C, Gennarelli M.


Treatment-Resistant Schizophrenia: Genetic and Neuroimaging Correlates. Front Pharmacol.
2019 Apr 16;10:402. doi: 10.3389/fphar.2019.00402. PMID: 31040787; PMCID: PMC6476957.

Diagnosis of treatment resistant schizophrenia -

To understand treatment resistance, we should know what is treatment response andremission


first.

• Treatment Response is schizophrenia -

Defined as a clinically significant improvement of psychopathology as well as functionality.


Different studies take different cut offs to label 'treatment response' ranging from 20-60%
decrease in symptoms from original values.

To make it easy keep in mind the following points-


Treatment Response- A score of 2 or 1 in the Clinical Global Impression ( CGI ) scaleOR
more than or equal to 20 points on Functional Assessment for Comprehensive Treatment of
Schizophrenia ( FACT-SCZ ) scale
OR
more than or equal to 20% 'decrease' on Brief Psychiatric Rating Scale (BPRS) or Positiveand
Negative Syndrome Scale (PANSS) scores

• Partial Response- A score of 3 on CGI scaleOR


10 to 20 points increase on FACT-SCZ or Global Assessment of Functioning (GAF) scaleOR
Upto 10% reduction on the BPRS or PANSS.

Treatment Resistance-

Results from lack of response to adequate exposure to medication with no confounding factors.

The assessment of TRS could be summarized through following points-


Define inadequate response using the psychopathological dimensions of schizophrenia,
particularly persistence of psychotic symptoms
Use scales such as the BPRS or the PANSS to quantify severity of symptoms
Define what is an adequate trial in terms of duration and dose of antipsychotics
Determine how many trials can be documented and types of antipsychotics previously
used
✓ Use patient charts
✓ Use treatment response timelines
✓ Consider the use of LAI antipsychotic if partial or inadequate adherence to
treatment

Some widely accepted criteria’s/algorithms/guidelines used-

• 1988 criteria proposed by Kane et al.


1990 criteria proposed by Brenner et al.
1992 criteria proposed by Meltzer.2
2004, American Psychiatric Association Guidelines (APA guidelines) by Lehman et al.
2006, Texas Medication Algorithm Project.
2006, International Psychopharmacology Algorithm Project (IPAP)
• 2009, Schizophrenia Patient Outcomes Research Team (PORT) Guideline.
2012, World Federation of Societies of Biological Psychiatry (WFSBP) by Hasan et al.
Indian: “Clinical Practice Guidelines for Management of Schizophrenia” by Grover etal,
2017.

Pseudo-resistance-
when a patient appears resistant, but treatment is inadequate rather than ineffectivePseudo-
resistance causes-
Medication non adherence
Insufficient plasma levels of medication
Inadequate dosage or duration of treatment
Misdiagnosis
Adverse events of a treatment masking a response
Presence of confounding psychiatric conditions- like substance abuse, depressive,
anxiety, obsessive - compulsive disorder or symptoms
Poor premorbid functioning
Substance use/abuse triggering psychosis

Remission -
Considered to be in between response to treatment and full recovery. It is defined as the
reduction of symptoms to a level that does not interfere in a patient’s psychosocial functioning.

Some important criteria defining remission in schizophrenia-

Criteria by Lieberman et al.(1993 )


Schedule for Affective Disorders and Schizophrenia, Change Version, psychotic and
disorganization items, positive symptom item score of ≤3 (suspiciousness, delusions,
hallucinations, impaired understandability, bizarre behavior);
Clinical Global Impression (CGI) severity scale score of ≤3;
CGI global impression of change score of 1 or 2 for 8 consecutive weeks.

Criteria by Curtis et al. (2001)


In a single time point evaluation, Brief Psychiatric Rating Scale (BPRS) total scoreof <30;
scores of <3 (moderate) on the affective flattening item and <2 (mild) on the alogia,
anhedonia, avolition, and attention items of the Scale for the Assessment of Negative
Symptoms (SANS);
Global Assessment of Functioning scale score of >60;no psychotic symptoms for more than
1 month;
no hospitalization for 3 months;
no more than one residual symptom; presence of employment; and association with friends.

Clozapine resistant schizophrenia-

There is a general agreement that patients who are clozapine resistant or patients with ultra-
resistant schizophrenia have persistence of psychotic symptoms even after adequate treatment
with adequate doses of clozapine for at least for 6 months.

Mouaffak and colleagues have proposed an operationalized definition, using the following
multidimensional criteria:
1. At least 8 weeks of treatment with clozapine with plasma levels of >350 micro grams/litre
and failure to improve by at least 20% in total BPRS score.
2. Persistent psychotic symptoms, defined as more than or equal to 4 (moderate) on at least 2
to 4 positive symptom items of the BPRS (18 items, graded 1–7)
3. Current presence of at least moderately severe illness on the BPRS (score more than orequal
to 45) and a score of >4 (moderate) on the CGI scale.
4. Persistence of illness as defined as no stable period of good social and/or occupational
functioning within the past 5 years (inability to maintain work and relationships) and GAFscore
less than or equal to 40.

Further reading from-


Treatment-Resistant Schizophrenia, Elkis & Buckley, Psychiatr Clin N Am(2016),
http://dx.doi.org/10.1016/j.psc.2016.01.006, psych.theclinics.com
Remission in Schizophrenia: Proposed Criteria and Rationale for Consensus, Nancy C. Andreasen
et, https://doi.org/10.1176/appi.ajp.162.3.441

Management of treatment resistant schizophrenia -

Clozapine emerged as gold standard drug for TRS in a study by Kane et al in 1988.

Clozapine Pre-treatment assessment —

Complete blood count that includes an absolute neutrophil count (ANC). According to
US FDA the minimum ANC required to initiate clozapine in general population is
1500/micoL.

Weight and height (body mass index), waist circumference, fasting blood sugar (or
HbA1c), and fasting lipids.

Vital signs- Blood pressure- both supine and standing (check for orthostatic
hypotension), pulse rate, check for arrhythmias.

ECG- rule out any cardiac pathology.

An Abnormal Involuntary Movement Scale documenting absence or presence of


abnormal motor movements- check for myoclonus

Pregnancy test in women of childbearing age (clozapine is pregnancy category B drug)

Liver function test (S.G.O.T., S.G.P.T., Sr. Bilirubin), Sr. Electrolytes.


Consider obtaining measures of inflammation and cardiac muscle damage (eg,troponin
levels, C-reactive protein ) in patients of pre existing cardiovascular pathology like
coronary artery disease, HTN, old

Drug levels for patients on anticonvulsant drugs (need to be in the therapeutic range).

Clozapine dosing and titration strategies-

In general, initial target dose is 300mg to 400mg per day by 2-3 weeks.
That being said, especially in Indian population, clinical response and remission is seen with
lesser doses as well.

Titration is started with 12.5mg once or twice daily on day 1.


If well tolerated, daily dose can be increased in increments of 25mg to 50mg daily oron
alternate/after 2 days.
Twice daily dosing is standard.
Agitated inpatients may do better with thrice daily dosing.
Excessive sedation may warrant once daily evening dosing.
Serum concentrations associated with clinically meaningful response range from 200 ng/mL to
450 ng/mL (from 350 ng/mL to 450 ng/mL in some studies).

In most of the studies, plasma levels associated with clinical response corresponded to oral
doses between 250mg and 650mg.

Patients who have not responded after 6-8 weeks with plasma levels of 350 ng/mL shouldhave
doses increased to new goal of plasma level about 450 ng/mL.
A trial of clozapine monotherapy of 6-12 months is reasonable.
Consider augmentation strategies after suboptimal response at adequate plasma level
(450ng/mL) for adequate duration.

Factors influencing clozapine plasma levels-


Sex: Females metabolize clozapine more slowly than males.
Age: Younger patients metabolize clozapine faster.
Ethnicity: Asian patients may show higher levels than other groups
Body weight: Slightly higher doses are needed in heavier people.
GI hypomotility: can result in delayed absorption

Clozapine is metabolized in the liver by CYP 450 -CYP1A2, CYP2D6, and CYP3A4

CYP 1A2 inhibitors- increase clozapine level- strong inhibitors-fluvoxamine, ciprofloxazin,


enoxacin, others- caffeine.
CYP1A2 inducers- decrease clozapine level- Tobacco smoking (aromatic hydrocarbons insmoke
cause CYP induction, nicotine itself doesn’t have any effect on CYP 1A2) Similarly
CYP3A4 inhibitor- potent inhibitors- fluoxetine, Grapefruit juice, clarithromycin,erythromycin,
diltiazem, itraconazole, ketoconazole, ritonavir, verapamil.
CYP 3A4 inducers- Carbamazepine, St. John’s Wort, phenytoin, rifampicin, glucocorticoids.
CYP2D6 inhibitors- paroxetine, sertraline, fluoxetine.

Clozapine monitoring-
Neutrophil count-
Routine neutrophil monitoring is performed at the following intervals: Weekly during the first
six months of clozapine administration
Every other week for the second six months
Every four weeks after one year, for the duration of treatment

If neutropenia develops during treatment, clozapine should either be monitored more


frequently, stopped temporarily, or discontinued, based on the severity of neutropenia-
Mild neutropenia (ANC: 1000 to 1499/microL) – Continue treatment but increase monitoring
frequency to three times per week.
Moderate neutropenia (ANC: 500 to 999/microL) – Interrupt clozapine treatment, increase
monitoring to daily until ANC is 1000/microL at which point clozapine can be reinstituted.Severe
neutropenia/agranulocytosis (ANC: <500/microL) – Discontinue clozapine.Rechallenge should
only occur if the benefits outweigh the risks, in consultationwith hematology.

ANC-based guidelines for continuing clozapine differ in patients with benign ethnic neutropenia.
Cardiovascular-
All patients should be monitored closely for at least the initial four weeks of treatment.
This should include assessment of symptoms concerning for myocarditis (eg, malaise, chest
pain, shortness of breath), vital signs each visit, electrocardiogram at baseline, and laboratory
tests if required-
ECG
Eosinophil count
Sedimentation rate or C-reactive proteinTroponins

Metabolic-
Weight, BMI, waist circumference, BP monitoring every 3 months for the first year, thenyearly.
Fasting blood glucose should be checked at baseline, after one month, then every 4–6 months.
Lipid profile at baseline, 3 monthly in the first year, then yearly.

BP-supine and standing , pulse, temperature- frequently, especially during dose titration.LFTs,
se electrolytes- baseline and then yearly.

Clozapine Adverse Reactions-

CNS- Sedation, Dizziness/vertigo, Headache, Tremor, seizures, myoclonus Metabolic-Diabetes,


hypercholesterolemia, hypertriglyceridemia, obesity, diabetic ketoacidosis
Cardiovascular -Tachycardia, Orthostatic Hypotension, Syncope, cardiomyopathy, carditis,
pericarditis, cardiac failure.
Autonomic nervous system- Hypersalivation, Sweating, Nocturnal Enuresis GIT- Constipation,
Nausea, adynamic ilius.
Haematology- Neutropenia, agranulocytosis, venous thromboembolism.

Others- Fever, Hepatitis, Visual disturbances, Transaminitis

Further reading-
Current perspectives in the treatment of resistant schizophrenia, R.K.Solanki,Paramjeet
Singh,Deepti Munshi, Indian J Psychiatry.2009 Oct-Dec; 51(4): 254–260., doi: 10.4103/0019-
5545.58289, PMCID: PMC2802371, PMID: 20048449
Clozapine Monitoring in Clinical Practice: Beyond the Mandatory Requirement, NilamadhabKar,
Socorro Barreto, Rahul Chandavarkar
Clin Psychopharmacol Neurosci. 2016 Nov; 14(4): 323–329. doi: 10.9758/cpn.2016.14.4.323,
PMCID: PMC5083942
https://www.uptodate.com/contents/guidelines-for-prescribing-clozapine-in-
schizophrenia/print

Alternatives to clozapine:

Clozapine remains as the only effective drug for TRS and when TRS is established, clozapine
should be started. Other antipsychotic trials, given after two failed trials, seldom work and
should be considered only when clozapine cannot be used. In that case, olanzapine has best
results, often 30-60 mg/day. Then other antipsychotics are added to it, such as amisulpride (800
mg/day), sulpiride (600 mg/day), aripiprazole, risperidone. Addition of allopurinol (300- 600
mg/day), donepezil (5-10 mg/day), memantine (20 mg/day), minocycline (200 mg/day),
N-acetylcysteine (2g/day) can be tried. Risperidone has best effects till 8 mg/day, after whichit
does not provide additional symptom reduction. CBT, ECT are viable options as add on.
Any polypharmacy has risk of increased side effects, mainly cardiovascular.

Clozapine augmentation:

Trials show small effect size, and there is no clear preferred drug to augment. Augmenting trials
should be stopped by 6 months if no improvement is seen. Again, each added drug increases
side-effect risk. Aripiprazole (15-30 mg/day) with clozapine may reduce metabolicrisks.

Topic: Personality disorders


A note on classification of personality disorders (PD) in ICD 11

In ICD 11 the changes expected are; first, rating PD based on its severity in terms of levels of
adaptive functioning, and second, describing the specific traits that are prominent in the
person. They are rated as mild, moderate, or severe PD. Prominent personality difficulties can
also be recognized by clinicians but are not considered to be disorders. In DSM-5, the presence
of PD is based on character traits such as Self-Directedness and Cooperativeness, which
provide a description of personalities that are healthy or disordered.
In the ICD-11 criteria, the proposed traits are described by “the four As”:

• Asthenic/anxious (i.e., negative affective features such as high Harm Avoidance),


• Adventurous/antisocial (i.e., dissocial and disinhibited features such as high Novelty
Seeking),
• Asocial (i.e., detached features such as low Reward Dependence), and
• Anankastic (i.e., perseverative and perfectionistic features such as high Persistence).

The ICD-11 proposal is more like the temperament structure of Cloninger’s psychobiological
model rather than the Five-Factor Model followed by DSM 5. Also ICD 11 needs to be
efficiently applied around the world in places with limited resources, so it requires only ratings
of the severity of PD and may not require the descriptors for subtypes.

Borderline personality disorder

Also called as: Ambulatory schizophrenia, As-if personality, Pseudoneurotic schizophrenia,


Psychotic character disorder, Emotionally unstable personality disorder (ICD-10).

Introduction: They stand on the border between neurosis and psychosis.


Prevalence: (no definitive studies) could be about 1 -2 percent of population, 2x common in
women than men.

Genetics: 5 times more common among relatives of probands with this disorder than in
general population. Increases familial risk for ASPD, Substance use disorder and Mood
disorder.

Biological clues:

1. Shortened REM latency.


2. Sleep continuity disturbances.
3. Abnormal DST.
4. Abnormal Thyrotropin releasing hormone.

Predisposing factors: Early traumatic experiences have been indicated by many studies.
Tripartite etiopathologic model has been formulated for BPD, which includes; childhood
trauma, vulnerable temperament and series of triggering events.

Clinical features:

• Hallmark: pervasive and excessive instability of affects, self image and interpersonal
relationships as well as marked impulsivity.
• Almost always appear to be in a state of crisis.
• Mood swings are common.
• Argumentative at one moment, depressed the next and later complain of having no
feelings.
• Short lived psychotic episodes- micro psychotic episodes (rather than full blown
psychotic breakdown). Psychotic symptoms are circumscribed, fleeting or doubtful.
• Behaviour is highly unpredictable.
• Achievements are below the level of their abilities.
• Repetitive self destructive acts will be present, such as slashing of wrists, self
mutilation etc.
• They have tumultuous interpersonal relationships. They distort their relationships by
considereing each person as either all good or all bad.
• Dependent on those with whom they are close and when frustrated can express
enormous anger towards intimate friends.
• They cannot tolerate being alone, frantically search for a companionship, if only for a
brief period they would accept stranger as friend or behave promiscuous.
• Complain about chronic feeling of emptiness and boredom. They lack consistent
sense of identity.
• Defense mechanisms seen are: projection, splitting, identification.
• Perform normally on structured tests like WAIS, but show deviant responses on
unstructured tests like Rorschach test.
• They can be described as panphobic, pananxious, panambivalent and having chaotic
sexuality.

Complications: psychotic like symptoms in response to stress, premature death/ physical


handicap from suicide/ suicidal attempts/ self injuries behaviour/ para-suicide.

Co-morbidity: Major depressive disorder, substance use disorder, eating disorder esp, bulimia
nervosa, PTSD, ADHD and other PDs.

Differential diagnosis:
1. Dysthymia/ Cyclothymia
2. Schizophrenia
3. Schizotypal PD
4. Paranoid PD Course and prognosis:
• Variable. Follows a pattern of chronic instability in early adulthood. The impairment
and the risk of suicide are the greatest at the young adult years and gradually wane
with advancing age. In the fourth and fifth decades, they tend to attain greater
stability.
• No longitudinal progression towards schizophrenia, but higher incidence of major
depressive disorder.

Treatment:

1. Psychotherapy:
• This is an area of intense investigation and has been the TOC.
• Therapy is difficult: regress easily, act out on impulses, show lability or fixed
negative or positive transferences.
• Counter-transference due to projective identification.
• Dialectical behaviour therapy.
• Mentalisation based treatment:
It is based on construct that patient’s have reduced capacity tomentalize.
Mentalization is a social construct that allows a person to be attentive to
the mental states of oneself and others.
It comes from a person’s awareness of mental processes andsubjective
states that arise in IP relationships.
It is believed that gaining mentalization helps patients’ buildrelationship
skills as they learn to regulate their thoughts and feelings.
• Transference focussed psychotherapy (TFP):
It is modified form of psychodynamic psychotherapy.
It is based on Otto Kernberg’s object relations theory.
Two major processes are used: clarification and confrontation.
In clarification, transference is analysed more directly then in a traditional
way, this makes patient aware of the distortions about the therapist.
In confrontation, therapist point out to the patient how the tranferential
distortions interfere with IP relationships.
2. Pharmacotherapy:
• Deal specifically with personality features that interfere with patients’ overall
functioning. Antipsychotics can be used to control anger, hostility, brief psychotic
episodes. Antidepressants to improve depressed mood. For some carbamazapine
can improve global functioning.

Antisocial personality disorder

It is not synonymous with criminality


Epidemiology: 0.2 to 3 percent (DSM-5), more common among poor urban areas. Higher
prevalence is seen among men with alcohol use disorder and in prison population. More
common in males (M: F = 3: 1). Onset is before 15 years. Familial pattern can be seen and 5x
common among 1st degree relatives.

Genetics: More frequent among the 1 st degree relatives. Familial transmission has been found
in studies. Genetic and environmental factors contribute to the disorder, as evident from the
adoption studies. Conduct disorder and ADHD increase the likelihood of developing the ASPD
in adult.

Biological correlates:

1. Abnormal EEG results


2. Soft neurological signs, suggesting minimal brain damage in childhood. Clinical
features:
• Hallmark: pervasive disregard for the violation of rights of others occurring since the
age of 15 years and continuing into childhood. Evidence of conduct disorder before
the age of 15 years.
• Often seem to be normal, but beneath of mask of sanity lurks tension, irritability and
rage.
• Histories would reveal areas of disordered functioning like; lying, truancy, running
away from home, thefts, fights, substance abuse and illegal activities.
• Often impress opposite sex with colourful, seductive aspects of personalities, but the
same sex may regard them as manipulative and demanding.
• They exhibit no anxiety/ depression, which may seem grossly incongruous with the
situation. They may have suicide threats and somatic preoccupation.
• Extremely manipulative, do not tell the truth and cannot be trusted to carry out any
task or adhere to any conventional standard of morality. Commonly involved in
promiscuity, child abuse, spousal abuse and drunk driving.
• Lack of remorse is a notable finding among them.

Complications: low tolerance for boredom, depressed mood and premature violent death.

Co-morbidity: impulse control disorders, major depressive disorder, substance use disorder,
pathological gambling, anxiety disorders and somatisation disorder.

Differential diagnosis:

1. Criminal behaviour not associated with a personality disorder.


2. Bipolar affective disorder.
3. Other PDs- narcissistic, histrionic, borderline and paranoid. Course and prognosis:
• Unremitting course with the height of behaviour seen during the late adolescence.
• Some reports indicate the symptoms decrease as persons grow older. Flagrant
antisocial behaviour and less severe behaviours and substance use tend to decrease
after 30 years.

Treatment:

1. Psychotherapy:
• Setting firm limits is essential before starting the treatment
• They are often amenable for treatment if immobilised.
• When they are among peers, their lack of motivation for change disappears.
• Self help groups have been more useful than jails in alleviating the disorder.
• Therapists must be prepared to deal with patients’ self destructive
behaviourand fear of intimacy.
2. Pharmacotherapy:
• Dealing incapacitating symptoms such as anxiety, rage, and depression.
Medications must be used judiciously, as they often have substance abuse.
• Methylphenidate can be used if there is evidence of ADHD.
• Antiepileptics can be used to control the impulsive behaviour.Psychopath
versus Sociopath: the terminology dilemma
• Often used to describe people who commit heinous crimes.
• The use of one term over other is a matter of preference or often arbitrary rather
than based on concrete scientific differentiations.
• Psychopaths are believed to possess some constituent disposition to ASPD.
• Sociopaths are biologically normal, but develop antisocial characteristics
throughincompetent or hostile socialisation, mainly defective parenting.
• Both are best viewed on a continuum, thus making pure psychopaths and
puresociopaths just abstractions.
• The construct psychopathy was made famous by Hervey Cleckley‘s “The mask
ofsanity”, first published in 1941.

References:

1. Synopsis of psychiatry, 11th edition.


2. Comprehensive textbook of psychiatry, 10th edition.

Topic: Psychosocial rehabilitation


➢ At the outset w.r.t Substance use cases, students when often asked about
psychological interventions, tell they will do Relapse prevention.
• This is inaccurate as RP is more of an umbrella term for a multitude of interventions.
Most of the interventions in RP are based on CBT or even BT.
• So, based on the history and pattern of substance use you must be able to specify couple
of the interventions you will be doing

➢ One another inaccurate response to questions about psychological intervention is that


people say they will do Motivational Interviewing (MI) and Relapse prevention.
• There is also some inaccuracy to this. MI is employed if based on your MSE the
conclusion is that the client is Precontemplation or Contemplation stages of Motivation.
• You may still discuss about some elements of RP if the client is willing to engage.
• So, RP is done for mostly clients in Preparation stage of Motivation

➢ So why do RP?
• Marlatt and Gordon realised that in many clients with substance use lapse/relapse is
the norm and hence designed interventions targeting them

➢ During a long-term intervention for someone with Substance Use Disorder (SUD) what
should be the goal of treatment?
• Should it be strict abstinence? Though it would be wonderful often does not materialise.

➢ So, what should be the goal?


• TO PREVENT A LAPSE FROM BECOMING A RELAPSE
• Lapse refers to a single reuse of the substance and relapse refers to resumption of
earlier pattern of use
• Before we progress further, I also want you to note that there is a movement within the
mental health and user community to do away entirely with the term relapse. You
should also be aware of that. Please read from this link - https://www.rehabs.com/pro-
talk/one-more-time-can-we-please-do-away-with-relapse/
• Preventing a lapse from becoming a relapse takes into consideration the realities of
recovery, targets achievable goals, does not lead to therapist frustration, which
translates to a better rapport with the client, which can possibly lead to better recovery
rates in the long run
➢ Couple of other concepts that you should be aware
1. Motivation is a dynamic phenomenon
2. Relapse is a process and not a cross sectional event

• Now what does the latter mean? Here is an example


X was a client who had complains of relapse on alcohol use for a period of 10 days after
abstinence for around 2 years. When the reason was sought, he said that friends had forced
him and following that he used alcohol and subsequently loss of control set in quickly.
What’s the next question you will ask?

(There are no perfect answers)

He was asked howover the past 2 years, he managed to remain abstinent when he had gone
out with his friends?On further exploration this was a treat being given by him for getting a
new bike, he had prior to the outing considered using alcohol and so at the mere suggestion
by friends he had used it, he also talked about craving being present as he remembered all
the celebrations in which he had used alcohol, and he reported assuming that he will be
able to have control over his alcohol use

• As you can see there were multiple factors that led to the client's use of alcohol and it
was a PROCESS. So as a practical tip never stop asking questions when someone tells
you the reason for relapse are peer pressure or stress. Remember Relapse is a PROCESS

➢ A High-Risk Situation refers to as the name suggests a situation where the individual's
attempt to refrain from behaviour is threatened.
• One of the most important aspects of RP is identifying the high-risk situations and
designing interventions to handle them

➢ Abstinence Violation Effectis another cognition that you need to handle.


• It refers to guilt the person experiences after a having a lapse.
• It may expressed in two ways
1. viewing the lapse as an abject failure.
o This is especially a problem in many of our clients where clients come to the clinic
with their families and are in search of a permanent solution.
o Remaining abstinent is seen as a function of will power.
o Following the lapse, the guilt and the subsequent negative affect triggers further
substance use.
o Many people turn to faith-based deaddiction centres where use of substance is
equated with sin and hence any lapse and for that matter craving is seen as a sin
adding onto the perception of failure
2. “I have failed and now how does it matter whether I use substances subsequently” is
another abstinence violation effect related cognition

➢ Determinants of Relapse.
• We have intrapersonal and interpersonal determinants of relapse

❖ Intrapersonal factors:
1. Craving
2. Motivation
3. Coping
4. Self-efficacy
5. Outcome expectancies
6. Emotional states

Craving is an IRRESISTIBLE urge or desire to use the substance.

✓ Craving may be triggered by internal or external cues, stress or use of the substance
itself leads to craving
Motivation is the process that initiates, guides, and maintains goal-oriented behaviours.

✓ To specifically address motivation, we have MI (to be discussed later)


Coping is defined as the thoughts and behaviours used to handle an internal or external
situation that is perceived stressful.

✓ We often discuss that a person continues to use substances as a method of coping. It’s
essential for us to understand what we mean when we make this statement (discussed
later)
Self-efficacy is the confidence a person has in being able to handle a particular situation.

✓ To some extent use of good adaptive strategies in handling a situation can increase the
self-efficacy.
✓ W.r.t substance some important situations may be how to handle situations when
others with you are using a substance, how confident a person is in handling a lapse etc.
Outcome expectancies are the individual's anticipation of the effects of the substance.
✓ This where the clinician uses statements like "the patient used cannabis because he
wanted to get a high or wanted to relieve stress"
Emotional states such as negative affect.

✓ Another important tigger for substance use is use of substance to handle boredom
• This is in brief about the 6 intrapersonal factors
• There are questionnaires directed at each of these factors, mostly related to alcohol use
• For Craving the commonly used Penn Alcohol Craving Scale
• For Outcome Expectancies Drinking Motives Questionnaire, Inventory of Drinking
Situations
• For Coping - Coping Responses Inventory (can be used in different situations incl.
substance use)
• For Self-efficacy Alcohol Relapse Situation Appraisal Questionnaire, Situational
Competency Test for Alcohol
• For Motivation University of Rhode Island Change Assessment and Readiness to Change
Questionnaire
• However, my advice will be to determine all the factors through a Clinical interview
• Some of the other factors that you should be aware is of impact of life events, grief

❖ Interpersonal factors refer to interpersonal relationships.


• IPR issues are a major reason for worsening and maintenance of substance use

➢ Dynamic Model of Relapse


• Earlier models conceptualised relapse as a stepwise linear process.
• However according to the dynamic model proposed by Marlatt and Gordon, relapse is
a multidimensional and dynamic process with change in one factor leading to spiralling
effects over other factors.
• Relapse factors are divided into
o Distal factors such as temperament, personality, family loading, environment,
adverse childhood experiences
o Proximal factors being the determinants of relapse we have discussed so far.
• Distal and proximal risk factors interact in a complex way leading to relapse

➢ FRAMES is a model based on MI principles to deliver Brief Interventions in the OP.
• F is for Feedback. The feedback the client receives should be personalized, i.e., during
interview identify the differing harms substance has caused the client and then
subsequently provide feedback.
• R is for Responsibility. MI has origins from humanistic psychology and in MI the process
is collaborative, and the therapist stance is non-judgemental. Role of the therapist is as
a facilitator for change, but client must assume responsibility for change.
• A is for Advice. Advise is something Practitioners of MI will "advise you to avoid" But
information and subsequent advice may be given after taking consent of the client.
Unsolicited advice is one of the therapist behaviours that is detrimental and can lead to
increase in resistance
• M is for Menu of options- Various options may be discussed with the client as to how
his SUD may be managed. Expressing empathy and supporting self-efficacy are also
important components. One should try and be empathetic to the client with SUD as
people with substance use are often marginalized and stigmatised and hence don’t
receive care. Many of us may have difficulty understanding the reasons discussed for
substance use, however as part of MI we should be accepting. Also remember accepting
does not equate with the fact that you agree with the client. For supporting self-efficacy
positive feedback should be given for any gains the client makes towards the road to
recovery. It may be that client has reduced alcohol use from 12U to 6U but that needs
to be pointed out and appreciate the effort the client is putting in the journey towards
abstinence

➢ Craving managementis one of the important inventions, can be one of the solutions for
managing high risk situations.
• Delay - Craving comes in waves and the trick is to delay and to handle the peak of the
wave.
• Distraction can be through some cognitive exercises such as counting backwards or
another activity the client enjoys.
• Drink water and deep breath are self-explanatory.
• Discuss: For Discussing with someone, we must train the person on how to respond
back when the client discusses about his/her craving

➢ Assertiveness Training
• Since peer pressure is one of the commonly cited reasons for lapse, it’s important to
discuss drink refusal.
• This comes under the larger umbrella of Assertiveness training.
• Educate the client about the difference between Assertiveness and Aggressiveness,
through role play use of the Broken record technique.
✓ This is where client replies in the negative when they are the offered a substance.
✓ The same reply is repeated on further insistence with a firm voice and maintaining eye
contact.
✓ The client is asked to refrain from giving any explanation for his choice to stop using the
substance.
✓ On escalation reply in a firmer voice and if the insistencecontinues to excuse
oneselffrom the scenario
• This may often be difficult to carry out, and so, we can advise stimulus control, where
client is advised to avoid high risk situations

➢ Coping
• Coping skills training such as discussion of problem solving or relaxation exercises.
• If there is an underlying disorder leading to deficient coping skills, address the same.
• Often with young users managing boredom becomes an important issue. hence
developing a schedule with multiple activities during the day is useful when the case
scenario needs it
• We have briefly discussed managing self-efficacy during the discussion about FRAMES.
• When we look at outcome expectancies in clients who use substance to get a high,
development of positive addictions can be discussed

❖ In the end we need to with the client lay a clear plan to manage the high-risk situation.
❖ The plan needs to be individualised for every client and over a period of time you need
to determine what proximal and distal factors have interacted leading onto the relapse.
❖ Viewing relapse in this manner gives multitudes of interventions you can plan
What has been discussed above is how to prevent a relapse. The other thing which requires
is what to do when the lapse happens.

➢ Lapse Management
• For lapse management ask the client to come and visit you immediately.
• There should have been a prior discussion about Abstinence Violation Effects.
• Clients should be asked to view lapse as not failures but as learning opportunities to
examine what went wrong.
• A thorough ABC analysis of the circumstance of lapse is required and modifying the
current plans with the new learning

Topic: Attention Deficit Hyperactivity Disorder

Clinical features of ADHD

¬The cardinal features of ADHD are excessive and impairing levels of hyperactivity,
inattention, and impulsivity, and they are pervasive over time.

¬These features must be evident in more than one setting, cause serious impairment and
be
excessive in relation to the person’s mental age and development, and must not be due to
other causes such as anxiety, schizophrenia or autism.

¬Hyperactivity---‑impulsivity symptoms include: fidgeting, being ‘always on the go’, talking


excessively; unable to play quietly; continually interrupting.

¬Inattention symptoms include: being easily distracted, being unable to sustain attention,
difficulties completing tasks, difficulties organizing, avoiding tasks requiring mental effort;
appearing not to listen, being forgetful, and losing things.

¬ Hyperactivity is more impairing and more noticeable in pre---‑school children. In school


inattention is more noticeable, and inattention and impulsivity is noticeable in both
adolescent and adult populations, especially in social situations.

¬Connor’s questionnaire is widely used to obtain information from schoolteachers on ADHD


symptoms. There are also parent and adolescent versions

¬Diagnostic criteria: The hyperkinetic disorder is the ADHD equivalent in ICD---‑10. For
ADHD/HKD, thediagnostic criteria are considered to be more ‘relaxed’ in DSM but stricter
in ICD---‑10. According to DSM---‑IV criteria, to meet the diagnosis of ADHD, some
symptoms must be present before the age of 7 years, although ADHD is not diagnosed in
many children until they are older than 7 years when their behaviours cause problems in
school and other places.

¬To confirm a diagnosis of ADHD, impairment from inattention and/or hyperactivity-


--‑impulsivity must be observable in at least 2 settings and interfere with developmentally
appropriate functioning socially, academically, or in extracurricular activities and should
persist for at least six months.

¬ADHD is not diagnosed when symptoms occur in a child, adolescent, or adult with a
pervasivedevelopmental disorder, schizophrenia, or another psychotic disorder.
Prevalence:
.Using DSM---‑IV criteria, proposed general prevalence in school age children is about 5%.
Using ICD-
‑10 criteria , prevalence is approximately 1---‑2%(Shorter Oxford Textbook of Psychiatry,
6thEdition).
.The prevalence in UK children, using DSM IV criteria is 3---‑4% (The British Child and
Adolescent Mental Health Survey 1999: the prevalence of DSM---‑IV disorders)
.ADHD is 3 times more prevalent in boys than in girls, and more common in areas of social
deprivation and amongst children living in institutions

Aetiology:
. Genetics: Siblings have 2---‑3 times increased risk; heritability of approximately 80%.
Greater concordance in monozygotic compared with dizygotic twins [concordance 79% in
monozygotic and32% in dizygotic twins]. Genes 5, 6, and 11 implicated. This condition is
associated with dopamine transporter gene [DAT1] and dopamine D4 receptor gene. SNAP-
--‑25 gene may also have a role.
.Neuroimaging: Areas of brain affected include:prefrontal cortex, striatum and cerebellum.
Studies using positron emission tomography (PET) have found lower cerebral blood flow
and metabolic rates in the frontal lobe areas of children with ADHD than in controls. PET
scans have also shown that adolescent females with the disorder have globally lower
glucose metabolism than both normal control females and males with the disorder.
.Neurotransmitters: DA and NA dysregulation in the prefrontal cortex is implicated. The
most widely studied drugs in the treatment of ADHD, the stimulants, affect both dopamine
and norepinephrine, leading to neurotransmitter hypotheses that include possible
dysfunction in both the adrenergic and the dopaminergic systems. Serotonin may have a
role in modulating dopamine transmission and the expression of ADHD.
.Potentially important environmental factors include a)prenatal and perinatal obstetric
complications

b) low birth weight & prematurity c) prenatal exposure to alcohol, nictine and
benzodiazepines d) poor attachment and severe early deprivation e) institutional rearing
.Idiosyncratic reactions to food, Other food additives, lead exposure to toxic levels are not
supported by research evidence.
.Previous research shows that ADHD occurs in head injury in 25% cases. Retrospective
cohort study published in BMJ found that head injury before the age of 2 years does not
seem to be causal in the development of ADHD. Medically attended injury before 2 years of
age may be a ‘marker’ for subsequent diagnosis of attention deficit hyperactivity disorder.
.The quality of relationships within the family and at school can be considered as protective
or maintaining factors.
.Comorbidity: 50---‑80% of children with ADHD have a comorbid disorder. 50% children may
meet

criteria of 2 comorbid conditions. Common comorbid problems in childhood include


oppositional defiant disorder (40%), anxiety disorder (34%); conduct disorder (14%), tics
(11%) and mood disorder (6%) (Young et al, 2011, BMC Psychiatry)
Outcome:

.Approximately 15% of cases continue to meet diagnostic criteria for ADHD at the age of 25
years. (Young et al, 2011, BMC Psychiatry). A further 50% of individuals will suffer some
impairment from residual symptoms.
Children with hyperkinetic disorders are 5 times greater risk for antisocial behaviour,
substance abuse and other psychiatric disorders. 15---‑20% develop substance misuse
problems
. Many children initially diagnosed with ADHD, combined type, exhibit fewer impulsive---
‑hyperactive
symptoms as they get older and, by the time they are adults, will meet criteria for ADHD,
inattentivetype.
.Poor prognosis depends on early stressful life experiences such as due poverty,
overcrowding, expressed emotions and parental psychopathology. Prognosis is worse when
the symptoms are severe,predominantly hyperactive---‑impulsive in nature, and associated
with conduct, language or learning disorder.

Treatment priorities in ADHD


• First treat most severe disorder, usually affective disorders, substance misuse, then
treat ADHD
• In case of personality disorder: first treat ADHD
• Treatment of milder depressive and anxiety disorders may be delayed and re-
assessed after ADHD treatment (may improve significantly)

Summary of NICE Guidelines


Medication is the first-line treatment in adults Methylphenidate is the first-line drug
If methylphenidate is ineffective or unacceptable then atomoxetine or dexamphetamine
can be tried2nd line n the following situations: 1. Residual symptoms 2. Poor or no response
3. Medication is not an option 4. The patient refuses medication then consider CBT

Classes of Medication Stimulants


• Methylphenidate
• Amphetamine compounds
• Dextroamphetamine
• Lisdexamfetamine
• Non-stimulant
• Atomoxetine
• Antidepressants
• Tricyclics
• Bupropion
• Antihypertensives
• Clonidine
• Guanfacine
• Miscellaneous
• Combined pharmacotherapyModafinil
• Venlafaxine
• Neuroleptics (only in severe cases with monitoring)

Management

.Treatment [long term, multimodal]: Before resorting to pharmacological treatments,


efforts should be made to provide psychological and social interventions. If medication is
being considered, the youngperson/family need to consent, and must be aware of the
possible side effects of treatment.

. General management principles are:


o Educational/remedial interventions
o Parent training programme for child management skills– based on social learning theory
and behavioural interventions
o Individual/family/group therapies
o CBT methods, especially behavioural, are often effective
o Social skills training
.MTA Study: The Multimodal treatment study of children with ADHD was the largest, most
rigorousrandomised controlled trail to date, involving 579 children. Despite methodological
issues raised by few authors, it is a trial that confirmed the effectiveness of medication
management in children and adolescents. The trial also found that intensive behavioural
therapy involving the child; family and teachers added little to well supervised medication
management. However, psychological interventions are important for families who do not
wish to use medication. Also, there is lack of evidence of efficacy over prolonged periods of
treatment with medication.
Stimulants – release noradrenaline, dopamine and also serotonin, increasing extracellular
dopamine and thus ‘inhibit’ impulses, helping persistence in motor and cognitive functions.
They arerapidly absorbed following oral administration.
• . Methylphenidate has most rapid onset 1---‑3hr, and shortest half---‑life 2---‑3
hours so the effects wear off in 4---‑6 hours. Methylphenidate dose range – 5--
-‑60mg/day. Growth retardation may occur during acute treatment, but no
significant retardation seen in long term. Note: Prescription of stimulants does not
increase the rate of substance abuse by the patients.
• Pimoline is no longer licensed in as it causes abnormal liver function tests, and in
somecases liver failure following long term use
• Atomoxetine is a noradrenaline reuptake inhibitor – good evidence for use in
hyperkinetic disorders.It increases noradrenaline in the synaptic cleft. It doesn’t
affect dopamine levels, and hence doesn’tcause more tics. In December 2012,
MHRA issued a note highlighting that Atomoxetine can cause clinically relevant
increases in blood pressure or heart rate, or both, in a small proportion of patients
and suggested monitoring heart rate and BP.
• Monitoring: Height, weight, blood pressure, heart rate monitoring is
recommended for methylphenidate (initially 3 monthly, then 6 monthly).
Atomoxetine rarely causes liver damage; so routine monitoring of LFT is not
warranted. The MHRA also advises that patients on Atomoxetine should be
monitored for signs of depression, suicidal thoughts or suicidal behaviour.
Monitoring of height and weight is also recommended, initially three monthly and
then six monthly.
Topic: Conduct disorder

Disorders of childhood conduct comprise of (a) Conduct Disorder and (b) Oppositional
Defiant Disorder.
Conduct disorder (CD) is characterized by a severe and persistent pattern of antisocial,
aggressive or defiant behaviours that defy age---‑appropriate societal norms. Oppositional
Defiant Disorder (ODD) also involves a persitent pattern of defiant behaviour. However, the
behaviour in the latter does not defy age---‑ appropriate societal norms to the same extent
as in CD. According to ICD10, oppositional defiant disorder is a subtype of conduct disorder.
DSM---‑5 excludes oppositional disorder if a conduct disorder is
present.
Conduct Disorder Diagnostic criteria:

. Children with conduct disorder are likely to demonstrate behaviours in the following four
categories
o Physical aggression or threats of harm to people, cruelty to people and animals
o Destruction of their own property or that of others
o Theft or acts of deceit
o Frequent and serious violation of age---‑appropraite rules (Like truating or running
away)
.ICD---‑10 requires at least one behaviour to be present for at least six months. According
to DSM---
‑5criteria, atleast 3 out of a list of 15 behaviours should begin before the age of 13, for a
period of 12 months. DSM--‐‑5 has added a limited prosocial emotions specifier to the
diagnosis of conduct disorder for children who do not meet the full criteria but present
with limited prosocial emotions, such as limited empathy and guilt. Other specifiers
retained from DSM---‑IV a childhood onset type –symptomspresent before age 10 and an
adolescent onset type – symptoms develop after age 10.
.CD is the cause of great suffering in both the individual and in society; it is one of the major
risk factors for adult antisocial behaviour, posing a major burden on public resources.
Conduct disorder occurs with greater frequency in the children of parents with antisocial
personality disorder and alcohol dependence than in the general population. In the Isle of
White study, CD was found to be the most common psychiatric disorder amongst 10---‑11
year olds.
.A prevalence of 5---‑7% is noted in the UK (Oxford Handbook of Psychiatry, 3rd Edn.). The
disorder is more common among boys than girls, and the male: female ratio is 4:1.

Aetiology:

.Ontario Child Health Survey[1987] : Three most significant risk factors: o Family
dysfunction
o Parental mental illness
o Low income
.Rutter [1978] :
o Low socioeconomic status, [Low family income]
o Criminality of father,
o Overcrowding,
o Maternal neurosis,

Chronic marital discord

.Biological risk factors:


o Genetic: CD clusters in families. Heritable trait of being more susceptible to externalizing
disorders in general (CD, ODD and and ADHD) (Shorter Oxford Textbook of Psychiatry, 6th
Edition).
o Temperament: ‘callous---‑unemotional’
o Brain injury
o Neurochemical: Low CSF serotonin, deficient serotonergic activity is seen in those with
early onset and more aggressive behaviour. Autonomic under---‑arousal e.g., consistent
lower mean resting heart rates, lower electro dermal activity etc. HR indices can predict
later onset of aggression.
Low salivary cortisol levels are also reported.
o Low IQ. Performance IQ may be higher than Verbal IQ
o Neuroimaging: Prefrontal brain regions may have reduced volumes in affected children.

. Psychosocial risk factors:


o Maternal smoking during pregnancy
o Parental criminality and substance abuse
o Harsh and inconsistent parenting
o Lack of a warm parental relationship and cold/rejecting family relationships
o Domestic violence in the family and child abuse
o Large family size
o Low socio---‑economic status / Low family income and social disadvantage
o Early loss and deprivation
o School failure and poor school achievement
o Social isolation;
o Exposure to urban life does not increase the rate of conduct disorders, though some
studies havefound higher prevalence rates in urban locations
Outcome

. Childhood conduct disorders further predict risk for numerous problems in adulthood
that includes (Moffitt et al---‑2002)
o Criminality and antisocial personality disorder. Less than 50% of conduct disorder
children havepersistent and severe antisocial problems as adults (Zoccolillo et al 1992).
o Serious difficulties in education, work and finances
o Homelessness and abuse
o Drug and alcohol dependence
o Poor physical health including injuries, sexually transmitted infections, compromised
immune function
o Variety of mental disorders and suicidal behaviour
o

Treatment:

. As the causes and risk factors are multifactorial, treatment is multimodal. Explore specific
support for
academic and social skills. Psychological therapies form the mainstay of treatment for
conduct problems. Parent Management Training based on the principles of social learning
theory has been very successful in altering the course of conduct disorders. NICE
recommends that group based parent training/education programmes should be the
mainstay of treatment for children of 12 years and under with oppositional defiant disorder
and conduct disorder.

.Cognitive behavioural therapy: CBT for conduct problems in children and adolescents
typicallyincludes social skills training and anger management. The most common targets
are aggressive behaviour social interactions, self---‑evaluation and emotional
dysregulation.
. Functional family therapy: One of the best---‑known interventions for serious antisocial
behaviour is functional family therapy. It is designed to be practicable and relatively
inexpensive. The target agerange is 11---‑18 years. Between eight and twelve 1---‑hour
sessions are given in the family home to overcome attendance problems. For more
intractable cases, 12---‑16 sessions are offered and it usually lasts for three months. There
are four phases of treatment which includes Engagement, Motivation,Behavioural Change
and Generalisation. The aim is first to keep the family in treatment and only then to move
on to finding what precisely they want. The therapist must understand the parents’ goals
before specific techniques are taught. Functional family therapy addresses family
processes, which need to be present, such as improving communication between parent
and young person, reducing interparental inconsistency, tightening up on supervision and
monitoring, and negotiating rules and the sanctions to be applied for breaking them.
Functional family therapy has been shown to reduce
reoffending rates by around 50 per cent.
. Multisystemic therapy: It is one of the best---‑developed treatments of conduct disorder,
which rests on nine treatment principles. The clinicians take on only four to six cases at a
time and the team is available 24 hours a day. Treatment is usually given for three months
and then stopped. In Multisystemic therapy, the young person'′s and family'′s needs are
assessed in their own context at home and in their relationships with other systems such
as school and peers. Following the assessment, proven methods of intervention is used to
address difficulties and promote strengths. In this therapy, assessing and promoting the
strengths in the young person and the system is very important. The therapist is
responsible for ensuring appointments are kept and for making changehappen—families
cannot be blamed for failing to attend or ‘not being ready'′ to change.
Regula rwritten feedback on progress towards goals from multiple sources is gathered by
the therapist and acted upon and the parents and teenagers fill in weekly questionnaires
on whether
they have beenreceiving therapy as planned. There is close attention to ‘quality control’
by offering weeklysupervisions and the supervisor checks adherence weekly. Several
randomised controlled trialattest toeffectiveness, with reoffending rates typically cut by
half and time spent in psychiatric inpatient care
further reduced.

. Other treatment options:

Treat any comorbidity


o Address any child protection concerns
o Anger Management Programmes

Topic: Autism spectrum disorders


Autism spectrum disorder is a construct used to describe individuals with a specific
combination of impairments in social communication and repetitive behaviours, highly
restricted interests and/or sensory behaviours beginning early in life. The worldwide
prevalence of autism is just under 1%, but estimates are higher in high-income countries.

ASD is more than 4 times more common among boys than among girls. The probable
reasons for these are that girls are more resilient to early insults to the body and brain. It
is also possible that ASD in girl children is underdetected.

Males outnumber females by an approximate 6 : 1 ratio in savant syndrome compared with


an approximate 4 : 1 ratio in autistic disorder. In explaining that finding,Norman and
Geschwind in their work on cerebral lateralization pointed out that the left hemisphere
normally completes its development later than the right hemisphere and is thus subjected
to prenatal influences, some of which can be detrimental, for a longer period of time. In
the male foetus particularly, circulating testosterone, which can reach high levels, can slow
growth and impair neuronal function in the more vulnerably exposed left hemisphere, with
actual enlargement and shift of dominance favouring skills associated with the right
hemisphere. A ‘pathology of superiority’ was postulated, with compensatory growth in the
right brain because of impaired development or actual injury to the left brain.
This finding may account as well for the high male : female ratio in other disorders,
including autism itself since left hemisphere dysfunction is often seen in autism . Other
conditions, such as dyslexia, delayed speech and stuttering, also have a male
predominance in incidence, which may be a manifestation of the same left hemisphere
growth interference in the prenatal period described above.
Savant skills generally narrow to five general categories: music, usually performance, most
often piano, with perfect pitch, although composing in the absence of performing has been
reported as has been playing multiple instruments ; art, usually drawing, painting or
sculpting; calendar calculating ; mathematics, including lightning calculating or the ability
to compute prime numbers, for example, in the absence of other simple arithmetic
abilities; and mechanical or spatial skills, including the capacity to measure distances
precisely without benefit of instruments, the ability to construct complex models or
structures with painstaking accuracy or the mastery of map making and direction finding.
Other skills have been reported less often, including: prodigious language ability; unusual
sensory discrimination in smell, touch or vision including synaesthesia; perfect
appreciation of passing time without benefit of a clock; and outstanding knowledge in
specific fields such as neurophysiology, statistics or navigation.

Midline movements in Rett’s Syndrome

For Classification of midline hand stereotypies refer to figure 1 in article by Marisela et al


2017
They point to deficits in dopamine action, which later reduce with age due to onset of
Parkinsonian symptoms.
Breath holding spells are believed to be due to poor connectivity and differentiation of the
breathing centre.The principal neurotransmitter believed to be involved is GABA

Topic: Cognitive behavioural therapy

1. Which one of the following BEST describes how cognitive behavioural theories CBT are
conceptualized?

Ans: Cognitive behavioual theories are conceptualized as a general category of


theories,which have evoled from the theoretical writings,clinical experiences,and
empirical studies of behavioral and cognitivelyoriented psychologists and other mental
health workers.

2. In order for a therapy to be ”cognitive behavioural”,it must be based on which one of


the following ideas?

Ans: That cognitions lead to behavioral change.

There is nosingle definition of CBT because there are so many different cognitive
behavioral theories.All the cognitive behavioral theories value the role that cognitions play
in the development and maintenance of psychological problems.In order for a therapy to
be “cognitive behavioral “,it must be based on the idea that cognitions mediate behavioral
change.Outcomes of treatment

3. All the following (except for one) are proposotions held by all cognitive therapies.Which
one of the following is not a propositions of cognitive behavioural therapies?

Ans: Desired emotional regulation may be affected through behavioral change.


The unifying characteristic of cognitive behavioural counselling and psychotherapy
approaches is the fundamental emphasis on the importance of cognitive workings as
mediator of behaviour change . All cognitive interventions attempt to produce change by
influencing thinking,which is assumed to play a casual role in the development and
maintenenace of psychological problems . the relationship between thoughts and
behaviour is a major aspect of cognitive-behavioural thnerapy and counselling and
psychotherapy.thus,all cognitive-behavioural therapies share these three fundamental
positions : (1) cognitive activity affects behaviour. (2) cognitive activity may be monitored
and altered . (3) desired behaviour change may be affected throuhh cognitive change .

4. Gillian works as a cognitive behavioue therapist.She is always very conscientious about


attending to the client’s needs in the moment.Most of her client would describe her as
warmand supportive.Gillian most likely attends to which one of the following constructs
associated with cognitive behavioural therapies?

Ans: The importance of therapeutic empathy.

5. Which one of the following choices lists all components of CBT case formulation?

Ans: Problem list,diagnosis,working hypothesis,strengths and assests and treatment


plan.

6. Which one of the following theorists developed the concept of self efficacy that has been
used as a cognitive beavioural theoretical explanation for how people change?

Ans: Albert Bandura.

The self efficacy theory of Albert bandura has been used to provide a cognitive –
behavioural theoretical explanation for how people change . it has been [proposed as
common pathway to explain how people change despite what therapeutic technique has
been employed . self – efficacy theory asserts that individuals devlop expectations for their
success in performing specific behaviours and that these expectations influence their
descision to try new behaviours and maintain behavioural changes . self – efficacy may be
thought of as a sense of personal competence,or feelings of mastery. The degree to which
a person feels efficacious influences the amount of effort that he or she will apply in given
situations . Thus, cognitive behavioural therapy may work through increasing self – efficacy
of clients.

7. Derald always seem to focus on negative events in his life,even when something good
happens for him.For example,when Derald got an A+ on his math test,he immediately
assumed the test was too easy,otherwisw he would not have gotten a good grade.Derald
is most likely engaging in which type of cognitive distortion?
Ans: Disqualifying the positive

8. All of the following (except for one)are criticisms or limitations of cognitive behavioural
theories as discussed by the chapter’s author.Which one of the following is not a criticism
or limitation pf CBT?

Ans: The union of cognitive and behavioural counselling and therapy into “cognitive –
behavioural” has not been able to overcome many of the limitations of either type of
therapies alone.

All of the list items are criticism or limitation of CBT except for the one.Actually ,the union
of cognitive and behavioural counselling and therapy into “cognitive-behavioural “ has
been able to overcome many of the limitations of either type of therapy alone .

9. All of the following (except for one)are guidelines useful for keeping the CBT counseling
and psychotherapy process brief.Which one is not a guideline discussed by the chapter’s
author?

Ans: Only focus on one thing at a time,either behavior or cognitions

10. All of the following (except for one) are principles of cognitive therapy.Which one of
the following choices is not a principle of cognitive therapy?

Ans: It is based upon the direction and sole discretion of the counselor or therapist, it
uses behavioural techniques to elicit cognitive dysfunctions , and it uses repe tition and
memorization for skills development.

Case scenarios
Topic: Nicotene related disorders
Case scenario:

Ms. S was a 45-year-old patient with schizophrenia who smoked 35 cigarettes per day. She
began at approximately 20 years of age during the prodromal stages of her 1st psychotic
break. During the first 20 years of treatment, no psychiatrist or physician advised her to
stop smoking. When the patient was 43 years of age, her primary physician recommended
smoking cessation in view of her hypertension. Ms. S attempted to stop on her own but
lasted only 48 hours, partly because her housemates and friends smoked. During a routine
medication check, her psychiatrist recommended that she stop smoking, and Ms. S
described her prior attempts. The psychiatrist and Ms. S discussed ways to avoid smokers
and had the patient announce her intent to quit and request that her friends try not to
smoke around her and to oer encouragement for her attempt to quit. The psychiatrist also
noted that Ms. S became irritable, slightly depressed, and restless, and that she had
insomnia during prior cessation attempts . .

1) Psychoactive Component Of Tobacco Is…… ?


• NICOTINE
• THC
• LSD

2) Nicotine Is Highly Water Soluble ? T/F


FASLSE. HIGHLY LIPID SOLUBLE , THUS PERMEATES TO CEREBRAL TISSUE TO GIVE
ATMOST IMMEDIATE PLEASURE.

3) Nicotine Has Psychotropic Properties? T/F TRUE

4) Nicotine Cause Dopamine Block In Brain ? T/F


• FALSE. INCREASES DOPAMINE RELEASE- EUPHORIA

5) NICOTINE CASES VASODILATION ? T/ F

• FALSE. IT CASES VASOCONSTRICTION

6) NICOTINE CAUSES HYPERTENSION? T /F

• TRUE. THROUGH INTERACTION OF VASOCONSTRICTION AND HYPERTENSION


CAUSES CVD.

7) Half Life Of Tobacco ?


• 3HRS
• 5HRS
• 2 HRS
• 1 HR
8) Smoking Withdrawal Develops Within …….. Hrs Of Smoking Last Cigrette?
• 5 HRS
• 7HRS
• 2 HRS
• 2.5 HRS

9) Withdrawal Symptoms Peaks With In ………… And Can Lasts From Weeks Or Month.
• 12-24 Hrs
• 24-48 Hrs
• 6-12 Hrs
• 36-48 Hrs

10) Following Are Withdrawal Symptom Of Nicotine , Except


• Paradoxial Trouble Sleeping
• Irritability
• Decreased Hr/Bp
• IMPROVED ATTENTION AND PROBLEM SOLVING ABILITY – STIMULATORY
EFFECTS

11) An Average Smoking Of 1 Cigrette Releases …. Mg Of Nicotine


• 0.7 Mg
• 10mg
• 0.5 Mg
• 0.3 Mg
NICOTINE AND SCHIZOPHRENIA

1) NICOTINE ALLEVIATES THE DEFICITS OF AUDITORY GATING? T/F TRUE


2) CHOLINERGIC DRUGS INCREASES NICOTINE DEPENDENCE? T/F TRUE
3) NICOTINE REDUCES WORKING MEMORY IN SCHIZOPHRENIA ?T/F
FALSE. IMPROVES WORKING MEMORY AND ATTENTION.

4) NICOTINE MAY ALLEVIATE THE NEGATIVE SYMPTOMS OF SCHIZOPHRENIA ? T/F


TRUE
5) ……………….how many % PATIENTS WITH SCHIZOPHRENIA SMOKE?

UPTO 90% SMOKE NICOTINE , OTHER: 50% IN DEPRESSION , 25% IN GENERAL


POPULATION

Findings from studies in people with schizophrenia who smoke suggest they take more
frequent puffs over a shorter period of time and, as a result, extract more nicotine from
cigarettes compared with those without a mental health condition.9 It is therefore
plausible that these smokers may require higher doses of nicotine replacement.

Topic: Nicotene related disorders (2)

Case scenario:

Hence she was recommended medications. Ms. S chose to use a nicotine patch plus
nicotine gum as needed. The psychiatrist had Ms. S call 2 days after her attempt to quit
smoking. At this point, Ms. S stated that the patch and gum were helping One week later,
the patient returned after having relapsed back to smoking. The psychiatrist praised Ms.
S for not smoking for 4 days. He suggested that Ms. S contact him again if she wished to
try to stop again. Seven months later, during another medication check, the psychiatrist
again asked Ms. S to consider cessation, but she was reluctant. Two months later, Ms. S
called and said she wished to try again. This time, the psychiatrist and Ms. S listed several
activities that she could do to avoid being around friends who smoked, phoned Ms. S’s
boyfriend to ask him to assist her in stopping, asked the nurses on the inpatient ward to
call Ms. S to encourage her, plus enrolled Ms. S in a support group for the next 4 weeks.
WHAT WILL BE FURTHER MAMAGEMENT IN THIS CASE ??

NICOTINE AND MANAGEMENT :

1) Nice Recommendations For Smoking Cessation ?

ALL THE ABOVE . NRT, VARENICLINE, BUPROPION

2) Smoker makes a commitment to stop smoking on a particular date and medication


is only continued if the user remains abstinent from smoking at follow up ?? What
is this method known as ……………………………

• Abstinent Contingent Treatment


• Positive Reinforcement
• Reward

3) Can You Offer Nrt, Varenicline,Bupropion Together As Per Nice Guidelines?


• NO
• Yes

4) SPOT THE FALSE STATEMENT -NONE

Motivation To Stop , Availability Of Counselling,Previous Experience With Smoking


Cessation Aids,C/I To Use And Potential Adverse Event, Personal Preference Are
Equally Important.

5) All These Belong To NRT ,EXCEPT


• NICOTINE GUM
• LOZENGES
• SPRAY
• BUPROPION

6) Spot the true statement regarding NRT


• Short period of maintance of 2-4 weeks ,followed by gradual reduction
period of another 6-12 weeks
• Short period of maintance of 6-12 weeks , followed by gradual reduction
period of another 6-12 weeks
• Short period of maintance of 6-12 weeks ,followed by gradual reduction
period of another 10-12 weeks

7) Nrt Adoloscent Treatment Can Be Given Till


• 2-4 WEEKS
• 4-6 WEEKS
• < 12 WEEKS
• > 18 WEEKS
ANS: RESTRICTED TO 12 WEEKS.

8) Most Suitable Nrt For Highly Dependent Smokers?


• GUM
• PATCHES
• NASAL SPRAY
• MOUTH STRIPS

9) Nrt And Diabetes ? Can Be Prescribed


▪ YES
▪ NO .

YES – NICOTINE RELEASE CATECHOLAMINES WHICH AFFECT CARBOHYFRATE
METABOLISM ,HENCE MONITOING BLOOD SUGAR OFTEN IS ESSENTIAL.

10) IN PREGNANCY MENTION THE MAXIUM DURATION THAT PATCH CAN BE USED ?
• 16 HOURS (25mg,15 mg.10mg)
• 24 HOURS (21mg,14mg,7mg )
After assessing risk and benefit ratio , aim to discontinue NRT should be
after 2-3 months. INTERMITTENT forms are preferable , considering nausea
and vomiting PATCH can be used. Preferably 16 hours patch – which should
be removed in night.

11) BUPROPION – RATIO OF RISK OF DEVELOPING SEIZURE IN INDIVIDUAL


o 1 IN 30
o 1 IN 1000
o 1 IN 470

12) Bupropion is FDA approved drug for smoking cessastion


• <18 years
• 12-18 years
• >18 years and above

13) Varenicline is a
Alpha 4 beta 2 partial agoinst at nicotinic acetylcholine receptors
Alpha 4 beta 2 antagonist

Alpha 4 beta 2agonist

Alpha 4 beta 2 inverse agonist

14) POST MARKETING SURVEILLANCE OF VARENICLINE TWO MAJOR SIDE EFFECTS WAS
MENTIONED . WHAT ARE THEY ?
• Depression and anxiety
• Depression and suicidality
• Depression and behavioural disturbance
• Rapid cycling mania

15) PATIENT IS ALLOWED TO SMOKE REPEADELY AND RAPIDLY TO THE POINT OF


NAUSEA, WHICH ASSOCIATES WITH SMOKING WITH UNPLEASANT RATHER THAN
PLEASANT.
• NEGATIVE REINFORCEMENT
• PUNISHMENT
• AVERSION THERAPY
Case scenario:

Ms. S was a 45-year-old patient with schizophrenia who smoked 35 cigarettes per day. She
began at approximately 20 years of age during the prodromal stages of her 1st psychotic
break. During the first 20 years of treatment, no psychiatrist or physician advised her to
stop smoking. When the patient was 43 years of age, her primary physician recommended
smoking cessation in view of her hypertension. Ms. S attempted to stop on her own but
lasted only 48 hours, partly because her housemates and friends smoked. During a routine
medication check, her psychiatrist recommended that she stop smoking, and Ms. S
described her prior attempts. The psychiatrist and Ms. S discussed ways to avoid smokers
and had the patient announce her intent to quit and request that her friends try not to
smoke around her and to give encouragement for her attempt to quit. The psychiatrist
also noted that Ms. S became irritable, slightly depressed, and restless, and that she had
insomnia during prior cessation attempts. Hence she was recommended medications.
Ms. S chose to use a nicotine patch plus nicotine gum as needed. The psychiatrist had Ms.
S call 2 days after her attempt to quit smoking. At this point, Ms. S stated that the patch
and gum were helping One week later, the patient returned after having relapsed back to
smoking. The psychiatrist praised Ms. S for not smoking for 4 days. He suggested that Ms.
S contact him again if she wished to try to stop again. Seven months later, during another
medication check, the psychiatrist again asked Ms. S to consider cessation, but she was
reluctant. Two months later, Ms. S called and said she wished to try again. This time, the
psychiatrist and Ms. S listed several activities that she could do to avoid being around
friends who smoked, phoned Ms. S’s boyfriend to ask him to assist her in stopping, asked
the nurses on the inpatient ward to call Ms. S to encourage her, plus enrolled Ms. S in a
support group for the next 4 weeks. WHAT WILL BE FURTHER MAMAGEMENT IN THIS CASE
??

1) Case being discussed here is Nicotine Dependence syndrome.


Reflect on what parts of criteria are met from the case provided below. Is this enough
to make a dignosis. How will you approach this case

2) Characteristics of Nicotine:
-Highly lipid soluble and thus permeates to cerebral tissue to give almost immediate
pleasure.

-Nicotine Has Psychotropic Properties

- Increases Dopamine release-Euphoria

-Causes Vasoconstriction

- Vasoconstriction and hypertension

- Half life is 2hrs

3) Nicotine Withdrawal starts at 2 hrs and peaks at 24-48hr

4) Withdrawal symptoms- Irritability, paradoxical insomnia, decreased BP/HR, difficulty


concentrating, craving, tingling in hands and feet, anxiety

5) Management:
-1st line : NRT- nicotine lozenge, gum, patch, nasal spray (slightly more effective), and
inhaler

Warning signs foe NRT: History of myocardial infarction within the past 6 weeks,
uncontrolled hypertension (or hypertension that emerges during treatment), severe
dysrhythmia, or unstable angina. In most cases risk outweighs the benefit

Bupropion: Drug doubles the chances of cessation.

Side effects- Insomnia (30-40%), Dry mouth (30%), seizures (1in 100)

Bupropion SR Twice daily or Bupropion XL once daily

Varenicline :Partial agonist at α4β2 nAChR subtypes in the ventral tegmnetal area
Progressively increases cessation rates over 12 weeks . Nausea GI side effects and
headache

Blackbox warning – Depression and suicide

-2nd line :Nortryptiline, clonidine

Duration of treatment :

Short period of maintenance of 6-12 weeks , followed by gradual reduction period of


another 6-12 weeks

Topic: Endicotts substitution criteria


Diagnosing and providing treatment for major depressive disorder in patients with
advanced cancer

Case scenario

56 year male diagnosed with pancreatic cancer 8months back,terminal stage , during ward
rounds oncosurgeon notices and acknowledges his worries about his wife and children
future, excessive thinking about illness, tearfulness and depressed appearance, decreased
speech, pessimistic views and lack of reactivity.

1. Refers him to in house psychiatrist.


2. Is it normal response to illness ?
3. Is it depression ?

The incidence of depression in cancer patients ranges from 10 to 25 percent, and increases
with higher levels of disability, advanced illness, and pain (as high as 77%). The diagnosis
of a major depressive syndrome in a terminally ill patient often relies more on the
psychological or cognitive symptoms of major depression (worthlessness, hopelessness,
excessive guilt, and suicidal ideation) than the usual neurovegetative or somatic signs
(terminal illness itself can produce these). Endicott has proposed substitution criteria

Endicott recommended replacing the DSM-IV somatic symptoms of MDD (poor appetite
or weight loss, sleep disturbance, loss of energy/fatigue, diminished concentration) with
four alternative symptoms: tearfulness or depressed appearance in face or body posture;
social withdrawal or decreased talkativeness; brooding, self-pity or pessimism; and ‘cannot
be cheered up, doesn’t smile, no response to good news or funny situations’cing the DSM-
IV somatic symptoms of MDD (poor appetite or weight loss, sleep disturbance, loss of
energy/fatigue, diminished concentration) with four alternative symptoms: tearfulness or
depressed appearance in face or body posture; social withdrawal or decreased
talkativeness; brooding, self-pity or pessimism; and ‘cannot be cheered up, doesn’t smile,
no response to good news or funny situations’

References
Breitbart W et.al, in Oxford Textbook of Palliative Medicine, 2nd ed., Doyle D., ed. 1999,
pp 937-944.
Endicott J. Measurement of depression patients with cancer. Cancer, 1983;53: 2243-8.

Topic: Tardive dyskinesia


A 56-year-old lady, with history of 24 years duration of illness, diagnosed as schizoaffective
disorder presents with her son. She was maintaining well for several years with
Trifluoperazine 10 mg and Lithium 900 mg. Her son has brought him to your OPD with the
following complaints.

1. Her medicines were not supervised for the past few weeks due to the family being busy
with her daughter’s wedding and the son reports that the patient may not have taken
medicines as prescribed.

2. The patient and the son have noticed that for past 3 weeks, the patient used to smack
her teeth and lash her tongue out without reason and used to deny doing the same.

She has also developed psychotic symptoms in absence of mood symptoms for past 10
days.

The family is concerned about these movements.

Learning objective: Tardive dyskinesias and its differential diagnosis

Q and A

Q1. What is Tardive Dyskinesia?

Abnormal involuntary athetoid or choreiform movements of the body that are mostly
induced by neuroleptics. The risk of TD increases with cumulative years of antipsychotic
exposure. Rarely, dyskinesia can occur independent of neuroleptic exposure in patients
with schizophrenia, in which case it may be considered as a manifestation of schizophrenia
itself 1

Q2. What are the common areas of the body affected with tardive dyskinesia?

In 75% of the individuals, the oro-buccal-facial-lingual musculature is involved. The limb


muscles are involved in 50% of affected individuals, and in 25% of affected individuals, the
trunk is involved. One in ten patients with TD may have involvement of all three areas 2.

Q3. What is the risk of TD with First- and Second-generation antipsychotics?

Carbon et al. 3 conducted a meta-analysis in 2017, which revealed that the mean pooled
rate of TD with the use of FGA was around 30%, and with SGA, it was around 20%.
Pragmatically, the risk of developing TD is around 5% of patients per year of antipsychotic
exposure.
Q4. What are the risk factors for TD?

The following are some of the factors that may increase the risk of TD - Old age, female
gender, mood disorder, comorbid substance use, use of lithium and anti-parkinsonian
medications, Diabetes Mellitus, and presence of early extrapyramidal side effects during
antipsychotic therapy 4.

Q5. What is the neurobiological hypothesis behind TD?

The following hypotheses have been proposed in the neurobiology of TD.

a. Dopamine supersensitivity hypothesis – postsynaptic dopamine receptor


supersensitivity after prolonged exposure to antipsychotics was proposed earlier to
explain TD. However, the lack of sustained dyskinesias in animal models, lack of radioligand
imaging, or post-mortem findings suggestive of supersensitivity in the majority of patients
with TD makes this hypothesis less likely 5.

b. Gamma-aminobutyric acid (GABA) hypothesis – neuroleptic-induced damage to the


striatal GABAergic medium-sized spiny neurons. This is supported by animal models and
human studies through the reduction of glutamic acid decarboxylase in subthalamic nuclei
and reduced GABA in the cerebrospinal fluid 6.

c. Oxidative stress and neurodegeneration - Oxidative damage to GABAergic striatal


neurons leading to disinhibition of lateral pallidal neurons and subsequent alteration of
the dopaminergic tone secondary to neuroleptics has been proposed recently to be the
neurobiological basis of TD 2.

Q6. What is the standard scale used to assess TD?

Abnormal involuntary movement scale (AIMS) developed by the National Institute of


Mental Health 7 is the most commonly used scale. Other scales include the Simpson
dyskinesia scale8, Extrapyramidal symptom rating scale 9.

Q7. What are the diagnostic criteria for TD?

Schooler-Kane criteria10

i. At least three months of continuous or discontinuous exposure to antipsychotics


(However, in elderly it may manifest with exposure for a month itself)

ii. absence of other conditions that may cause involuntary movements

iii. At least, moderate (>3 on AIMS) dyskinetic movements in one body area or mild
(>2 on AIMS) dyskinetic movements in 2 or more areas

DSM 5 recommends the diagnosis of TD be made after ruling our withdrawal emergent
dyskinesias (up to 8 weeks of neuroleptic discontinuation or dose reduction) and highlights
that in elderly TD may manifest even with a shorter duration of exposure to neuroleptics
11.
Q8. What are the symptom domains of schizophrenia that are associated with TD?

Negative and cognitive symptoms 2.

Q9. How frequently should we screen for TD?

Once every six months after initiation of FGA and once every 12 months after initiation of
SGA. (More frequently if required in high-risk population) 12.

Q10. What is the first feature of TD?

Often, subtle vermicular tongue movements at rest when observed with the tongue in the
oral cavity is seen as the first feature of TD 2. With subsequent worsening, fly-catching sign
(intermittent thrusting of the tongue out of the mouth) and bon-bon sign (tongue pressing
against cheek) can be noticed.

Q11. What are the differential diagnoses for TD?

Spontaneous dyskinesias (1 to 7 % of all patients with schizophrenia), acute neuroleptic-


induced dyskinesias, withdrawal emergent dyskinesias, dopamine supersensitivity, senile
chorea, chronic motor tic disorders, rabbit syndrome, neurological disorders such as
Wilsons disease and Huntington’s disease 12.

Q12. What are the variants of Tardive Dyskinesia?

Tardive syndromes include – tardive dystonia, tardive akathisia, tardive stereotypy, tardive
myoclonus, tardive tremors, and tics 13.

Q13. What are the situations that worsen or improve TD?

Relaxation and voluntary activity may temporarily ameliorate TD. TD usually does not
manifest during sleep. TD is worsened by emotional arousal.

Q14. What is the natural history of tardive dyskinesia?

TD usually follows a chronic fluctuating course with periods of spontaneous remissions. It


becomes progressively worse only in the minority. In the long term, up to 50% of
improvement can be seen in 50% of the individuals 14.

Q15. What are the other medications/ substances known to cause TD?

Caffeine, antihistaminics, stimulants, phenytoin, anti-depressants, antimalarials,


antiemetics 4.

Q16. What are the treatment strategies to manage TD?

The following sequential treatment strategies can be considered to manage TD12.

i. Reduce the dose of offending antipsychotic

ii. Stop anticholinergic medication

iii. Consider switching to antipsychotics with a lower propensity for TD


iv. Consider the addition of Clonazepam or Diazepam

v. Consider tetrabenazine trial

vi. Valbenazine, deutetrabenazine can be considered (Unavailable in India)

vii. Consider clozapine, or trial of amantadine or Ginkgo Biloba

viii. Alternatives with less evidence

– Branched-chain amino acids, Vitamin E, Melatonin, Vitamin B6, Donepezil,


Baclofen, Thiamine, Acetazolamide, Botulinum toxin - A, alpha methyl-
dopa, Buspirone, reserpine.

Q17. What are the treatment guidelines regarding VMAT inhibitors?

APA recommends (1B) that patients who have moderate to severe or disabling
tardive dyskinesia associated with antipsychotic therapy be treated with a
reversible inhibitor of the vesicular monoamine transporter2 (VMAT2).

Q18. What are the different VMAT2 inhibitors? [Source: APA guidelines]

Generic name Deutetrabenazine Tetrabenazine Valbenazine


Available Tablet: 6, 9, 12 Tablet: 12.5, 25 Capsule: 40, 80
formulations (mg)
Typical dose range 12-48 25-75 40-80
(mg/day)
Time to peak level 3-4 1-2 0.5-1
(hours)
Hepatic impairment Contraindicated Contraindicated Maximum dose of 40mg
daily with moderate to
severe impairment (Child-
Pugh score 7 to 15)
Renal impairment No information No information available Use not recommended in
available severe renal impairment
(CrCl<30 mL/min)

Common adverse Sedation Sedation, depression, Sedation


effects extrapyramidal effects,
insomnia, akathisia,
anxiety, nausea, falls
Effect of food on Food effects maximal Unaffected by food Can be taken with or
bioavailability concentration. without food. High fat
Administer with food. meals decreased the Cmax
Swallow tablets whole and AUC for valbenazine
and do not chew, crush, but values for the active
or break. metabolite (alpha-HTBZ)
are unchanged
Comments38 Give in divided doses, Give in divided doses, Initiate at 40 mg once daily
increase from initial increase from initial 25- and increase to 80 mg
dose of 12 mg once 50 mg dose by 12.5 once daily after 1 week.
daily by 6 mg per week mg/week to maximum Continuation of 40 mg
to maximum dose of 48 of 150-200 mg. Retitrate once daily may be
mg/day. Retitrate dose dose for treatment considered for some
for treatment interruptions of more patients. Use is not
interruptions of more than 5 days. Test for recommended with strong
than 1 week. Follow CYP2D6 metabolizer CYP3A4 inducer. A
product labeling if status before giving reduced dose is
switching from doses > 50 mg/day. Do recommended with
tetrabenazine to not exceed 50 mg/day in concomitant use of strong
deutetrabenazine. Do poor metabolizers or in CYP3A4 or CYP2D6
not exceed total daily patients treated with a inhibitors or in poor
dose of 36 mg/day (18 strong inhibitor of CYP2D6 metabolizers.
mg/dose) in poor CYP2D6. Avoid use in Avoid use in patients with
CYP2D6 metabolizers or patients with congenital congenital long QT
patients taking a strong long QT syndrome, with syndrome, with
CYP2D6 inhibitor. arrhythmias associated arrhythmias associated
Assess ECG before and with a prolonged QT with a prolonged QT
after increasing the interval or other risks for interval or other risks for
daily dose above 24 mg QTc prolongation (e.g., QTc prolongation (e.g.,
in patients at risk for drugs known to prolong drugs known to prolong
QTc prolongation. QTc intervals, reduced QTc intervals, reduced
Avoid use in patients metabolism via CYP metabolism via CYP 2D6 or
with congenital long QT 2D6). CYP3A4).
syndrome, with
arrhythmias associated
with a prolonged QT
interval or other risks
for QTc prolongation
(e.g., drugs known to
prolong QTc intervals,
reduced metabolism via
CYP 2D6).
Risks with VMAT inhibitors - The harms of treatment with VMAT2 inhibitors include
sedation and, with tetrabenazine, extrapyramidal effects, akathisia, insomnia, anxiety,
nausea, and falls. Depression and suicidal ideas have been reported in individuals who
were administered VMAT2 inhibitors for treatment of Huntington's disease.

References:

1. McCreadie RG, Padmavati R, Thara R, Srinivasan TN. Spontaneous dyskinesia and


parkinsonism in never-medicated, chronically ill patients with schizophrenia: 18-month
follow-up. Br J Psychiatry. 2002 Aug;181(2):135–7.
2. Sachdev PS. The Current Status of Tardive Dyskinesia: Aust N Z J Psychiatry
[Internet]. 2016 Jun 26 [cited 2020 Feb 27]; Available from:
https://journals.sagepub.com/doi/10.1080/j.1440-1614.2000.00737.x

3. Carbon M, Kane JM, Leucht S, Correll CU. Tardive dyskinesia risk with first‐ and
second‐generation antipsychotics in comparative randomized controlled trials: a meta‐
analysis. World Psychiatry. 2018 Oct;17(3):330–40.

4. Cornett EM, Novitch M, Kaye AD, Kata V, Kaye AM. Medication-Induced Tardive
Dyskinesia: A Review and Update. Ochsner J. 2017;17(2):162–74.

5. Seeman P. All roads to schizophrenia lead to dopamine supersensitivity and


elevated dopamine D2(high) receptors. CNS NeurosciTher. 2011 Apr;17(2):118–32.

6. C. Fibiger H, G. Lloyd K. Neurobiological substrates of tardive dyskinesia: the GABA


hypothesis. Trends Neurosci. 1984 Dec 1;7(12):462–4.

7. Guy W, National Institute of Mental Health (U.S.). Abnormal involuntary


movement scale [AIMS]. Natl Inst Ment Health Psychopharmacol Res Branch. 1976;ECDEU
assessment manual for psychopharmacology:534–7.

8. Simpson GM, Lee JH, Zoubok B, Gardos G. A rating scale for tardive dyskinesia.
Psychopharmacology (Berl). 1979 Jan 1;64(2):171–9.

9. Chouinard G, Annable L, Ross-Chouinard A, Nestoros JN. Factors related to tardive


dyskinesia. Am J Psychiatry. 1979 Jan;136(1):79–82.

10. Schooler NR, Kane JM. Research Diagnoses for Tardive Dyskinesia. Arch Gen
Psychiatry. 1982 Apr 1;39(4):486–7.

11. Association AP. Diagnostic and Statistical Manual of Mental Disorders (DSM-5®).
American Psychiatric Pub; 2013. 1519 p.

12. Correll CU, Kane JM, Citrome LL. Epidemiology, Prevention, and Assessment of
Tardive Dyskinesia and Advances in Treatment. J Clin Psychiatry. 2017 Oct 10;78(8):1136–
47.

13. Davis RJ, Cummings JL. Clinical Variants of Tardive Dyskinesia. CognBehav Neurol.
1988 Apr;1(1):31–38.

14. Bergen JA, Eyland EA, Campbell JA, Jenkings P, Kellehear K, Richards A, et al. T he
course of tardive dyskinesia in patients on long-term neuroleptics. Br J Psychiatry J Ment
Sci. 1989 Apr;154:523–8.

Topic: Delirium
Case scenario

A 34 year old male has been brought to the emergency with history of alcohol
consumption from past several years, about 720 ml of IMFL from last 6 months, with
morning drink of 180 ml. Last drink was 2 days back, after which he ran out of money and
friends to get a drink. Now he is tremulous, anxious, sweating, complains of headache. He
is not oriented to time and place, but oriented to person.

1. What is the name of the current condition? Alcohol withdrawal, complicated with
delirium
2. How long after the last drink can this condition start (give range in hours)?
Withdrawal symptoms may start within six hours from the last drink. Becomes
more by 48-72 hours. 10% of those in withdrawal experience complicated
withdrawal. 1-3% of those in withdrawal may have GTCS, peaking after 48 (CTP,
Tasman), 12-18 (Maudsley) hrs of last drink. Protracted withdrawal may last till 2-
6 months.
3. What will be the diagnosis if there is severe autonomic instability with visual
hallucinations? Delirium Tremens (DT). About 3% (CTP)-5% (Tasman, Maudsley) of
AUD/ADS patients have DT
4. What is the mortality rate in this condition with and without treatment (give range
in percentages)?10-20% with treatment, 5-10% without
5. How long after the last drink can this condition start (give range in hours)?Mostly
within 36-72 hrs (Tasman), 72-96 hrs (Maudsley 13th Ed.)
6. Name three benzothiazepines with their range of dosages equivalent to 25 mg of
chlordiazepoxide (give range in mg)?
7. Name one vitamin injection to be given, along with dosage and preferred route of
administration. Thiamine 500mg parental for 5 days, then oral (as long as needed).
IM is preferred, IV may have chances of anaphylaxis
8. Which is the benzodiazepine of choice for managing an ADS uncomplicated
withdrawal patient in the community? Chlordiazepoxide
9. In which related condition is confabulation seen? Korsakoff’s
10. Name the triad of Wernicke’s encephalopathy. Global confusion, ataxia,
ophthalmoplegia
Korsakoff’s amnesticdisorder is usually preceded by several episode
ofWernicke’sencephalopathy, characterized by confusion, ataxia, nystagmus,and
gaze palsies. When this condition subsides,

the characteristic memory deficits of Korsakoff’s psychosisbecome prominent.

Cessation of drinking can lead to an improvement inmemory with approximately


20% of cases demonstratingcomplete recovery. (Tasman)

A 26 year old male is brought to emergency with history of fever and irrelevant talking
from 3 days. On examination, he is disoriented to time, place and person and has rigidity
on all four lead pipe limbs. He was recently started on 8 mg haloperidol at a taluk hospital
because of suspiciousness.

1. What is the most probable condition?Neuroleptic malignant syndrome (NMS)


1. Name any 4 blood investigations to be done. These parameters are seen:
Fluctuating blood pressure, tachycardia, Elevated CK, leukocytosis, altered liver
function tests
2. Name one blood investigation to help in confirming the diagnosis if levels are
elevated. Creatinine Phosphokinase is generally high at ranges above 600.
1. Name two medications used to manage severe cases of this condition.Dantrolene
may be effective for treating severeNMS. Treatment begins with IV administration
of 0.8 to 2.5 mg/kg every 6hours, with a maximum of 10 mg/kg daily. When the
symptoms subsideand the patient can swallow, dantrolene is administered orally
in dosagesof 100 to 200 mg daily. Bromocriptine can then be added at doses of 20
to30 mg daily in four divided doses. Amantadine may be helpful if the otheragents
are not sufficient. The course of treatment is commonly 5 to 10 daysunless a long-
acting injectable agent has been used.(CTP)

A 74 year old female is brought with history of 2 days of confusion. She had several such
episodes in last one years, lasting for 2-3 days and remitting on its own. She also has
forgetfulness from last 1 year. There is no fever or burning micturition. There has been no
seizure, head injury or alcohol use.

1. What is the most probable diagnosis? Delirium superimposed on dementia


2. Name 6 investigations which will help in understanding the aetiology. These may
include any modalities, such as imaging, EEG, ECG, echocardiography, urine
routine along with blood tests etc
3. Name two specific blood investigations to point towards aetiologies. This question
is kept to remind that there are so many causes of dementia, many reversible.
Relevant investigations are important. Please see on the next page.
4. What is sundowning phenomenon? Increased delirium in evenings, may be related
to ambiguous or insufficient sensory stimulations.

• Name three psychomotor subtypes of delirium: Hypoactive, hyperactive and


mixed
• Most common EEG finding in delirium isslowing ordropout of the dominant
posterior rhythm, diffuse theta or delta waves (i.e., slowing), poor organization of
the background rhythm, and loss of reactivity of EEG to eye openingand
closingName a common screening scale used in this condition in general hospitals
The following are to draw attention to the various scales used in delirium, for
screening, severity etc. The Confusion Assessment Method (CAM) is probablythe
most widely used screening tool for diagnosis of delirium in general hospitals in its
4-item form (Levenson)

• Name a common scale used in this condition by the nurses: NEECHAM Scale
• Name a common scale used in this condition by ICU doctors: CAM-ICU
• Name a common severity scale used in this condition: The Delirium Rating Scale is
a 10-item scale assessing a breadth of delirium features and can function both to
clarify diagnosis and to assess symptom severity because of its hierarchical nature.
• Name two differential diagnosis of acute onset of confusion with suspiciousness
in males with age range of 20-80 years other than delirium. Can range from
substance intoxication, withdrawal to psychosis and mania, seizures etc

A 60 year old female with a diagnosis of bipolar affective disorder was on lithium 900
mg/day. From a week, she had low mood, poor sleep, crying spells and decreased food
and water intake. Her lithium value currently is 1.6 mEq/L, done after 16 hours after the
last dosage of lithium (900 mg). She is now tremulous with confusion regarding time and
place.

1. What is the most probable diagnosis? Lithium toxicity


2. Which type of tremor is seen in this condition. Coarse (fine tremors are adverse
effects)
3. Comment on the deep tendon reflexes if done immediately. Hyperreflexia
4. Name 4 investigations which will help you in management of this condition:
Lithium levels, RFT, Electrolytes, EEG,TFT, CBC, ECG etc
5. Name two different ways of classifying this condition: Mild, moderate,
severe……..acute, chronic, acute on chronic
6. Name blood lithium levels of mild, moderate and severe lithium toxicity
7. Confusion, agitation, delirium, tachycardia, and hypertonia are seen in which level
of lithium toxicity
Toxic effects reliably occur at levels >1.5 mmol/L and usually consist of
gastrointestinaleffects (increasing anorexia, nausea and diarrhoea) and CNS effects
(muscle weakness,drowsiness, confusion, ataxia, coarse tremor and muscle
twitching).Above 2 mmol/L,increased disorientation and seizures usually occur, which can
progress to coma, andultimately death. In the presence of more severe symptoms,
osmotic or forced alkalinediuresis should be used (neverthiazide or loop diuretics). Above
3 mmol/L, peritonealdialysis or haemodialysis is often used. (Maudsley)
Topic: Dementia
A 78-year-old male has a progressive illness from 06 years, characterised by forgetfulness,
irritability and decreased sleep. He also has lack of initiation of conversation with his old
friends and family members. He also has obstinacy, egocentricity and rigid adherence to
old habits. Son says he has become “slow” in activity and thinking too.

1. What is the most probable diagnosis? Dementia, probably Alzheimers


2. Name 2 differential diagnoses or comorbidities you will consider. Depression,
Parkinsons, CVA (lacunar infarcts)
3. Name 5 daily activities you will ask: Refer to Blessed Dementia Scale
4. Name 2 scales each for screening this condition and assessment of severity:
MMSE, HMSE, MOCA, Addenbrooke’s, Blessed Dementia Scale
5. Name 2 medical comorbidities that can worsen his condition: Almost any medical
comorbidity worsens it, mainly cardiac and renal.
6. Name 4 FDA approved medications which can be used forhim:
donepezil, rivastigmine, galantamine and memantine
7. Which FDA approved drug for this condition is banned now? Tacrine

Drug: Memantine

1. What is its mechanism of action? (only non-cholinesterase inhibitor indicated for


thetreatment of AD).selective uncompetitive inhibitor blockade of theNMDA
receptor class of glutamate receptors
2. How is it eliminated mostly? Renal, unchanged
1. Name 4 most common adverse effects?constipation,dizziness, headache, and
confusion (for ER preparations: headache, diarrhea, and dizziness)
2. For which condition is it FDA approved? Moderate and severe alzheimer’s
dementia
3. What is the most common medication combined with this? Donepezil
A 72 year old male has difficulty in remembering pathways and rooms of his house. He
frequently falls even in the day time, but has not sustained significant head injuries. He
also complained of fearfulness and suspicion that few people are following him to do him
harm. Shown to a doctor, he was found to have visual hallucinations too and was
prescribed risperidone 2 mg/day. After taking the medication, his movements have
slowed down, with tremors of hand. In a similar patient who died due to cardiac arrest:

1. Histopathologically What is seen in brain? Lewy body


2. It is most commonly detected in which part of the brain? Brainstem
3. What is the most likely diagnosis?Lewy body dementia
4. What is the core diagnostic triad for this condition?fluctuating cognitive
impairment, recurrent visual hallucinations and spontaneous motor features of
Parkinsonism
5. Name 4 supportive features of this condition which will aid in diagnosis.falls,
syncope, neuroleptic sensitivity, delusions,non-visual hallucinations, depression,
rapid eyemovement (REM) sleep disturbance andneuroleptic sensitivity.
A 50 year old male has 3 years of progressive apathy and emotional blunting, with no
forgetfulness. He also has incontinence from last 6 months. There were no depressive
symptoms.

1. Which type of dementia might be present in him?Fronto-temporal dementia


2. What are the variants of it?Frontal/behavioural variant FTD, semantic dementia
and progressivenonfluent aphasia
1. Which lobe is not involved significantly, when compared to Alzheimer’s
dementia?Parietal lobe symptomatology in the form of dysphasia,apraxia and
agnosia is said to be much less common in FTDthan AD (Lishman)
2. What is the EEG finding in most of those with this type of dementia? Mostly
remains normal as compared to other dementias
1. Name two neurological condition that are related to this type of
dementia:Progressive supranuclear palsy and cortico-basal degeneration(CBD) are
both now considered to be related to FTD (Lishman)

Topic: Bush Francis Catatonia rating scale


Mainly from Fink Catatonia and Caroff’s catatonia (has more details on BFCRS).

The first 14 items of BFCRS are to find “caseness” in catatonia, are most common and has
good screening qualities. Two or more suggests catatonia.

Waxy flexibility is the initial resistance given by the patient when his or her limb are being
moved. It then gives away, as if manipulating wax in water. This is not same as posturing,
which is the patient maintaining mundane or even uncomfortable postures for long
periods and ability of the examiner to manipulate the patient in various postures. The
resistance offered initially while doing it is waxy flexibility, and the result is also like a wax
statue, moulded in any form.

Negativism is a behaviour (includes motor system) to apparent meaningless resistance to


the commands. When this happens while moving one’s limbs, it is felt that resistance
offered is proportional to the force applied by the examiner. This is gegenhalten. In
negativism, it can be opposition to any command, like eating, sleeping etc.

In mannerism, the fault is in the presence of a motor action which is not right in the
current situation, although is goal directed if used in proper places. Like saluting to your
superiors in army is normal, but saluting during interview is abnormal. It does not matter
how many times this is done, the problems in in the occurrence of the behaviour and not
the repetition. Stereotypy on the other hand involves purposeless movts which are more
in frequency. The act can be rubbing one’s forehead to grimacing to tapping the table with
finger. This is purposeless behaviour, and is common to be done a few times in general
population. But if the same is repeated for several times in a brief period of time, the
behaviour becomes abnormal.

Topic: Catatonia
Case sceanrio

A 37 Year old female with history of lupus is admitted to the hospital with rash, arthralgias
and altered nmental status. O/E the patient displays stupor, mutism, posturing and
echopraxia+. while vital signs , creatine kinase and MRI brain are NORMAL. Lorazepam 2mg
iv q6 improves her symptom.

a) What is the diagnosis??

b) What the Scales you will use in supporting your diagnosis?

c) Mention few principle signs of the diagnosis?

d) What is LORAZEPAM CHALLENGE TEST?

ANSWERS:

A) CATATONIA

20% in primary medical /neurologic disease

B) BUSH-FRANCIS CATATONIA RATING SCALE

MODIFIES ROGERS SCALE.

(To check and quantify catatonic signs in suspected patients, monitoring the progress
and response to the treatment)

Most commonly used BUSH-FRANCIS CATATONIA RATING SCALE - Brief and


sensitive

INTERPETATION:

• Severity rated on a scale of 0-3

• Uncertain items are rated as 0


• ≥2 from 1-14 items are necessary

• Items 1-23 are for severity

C) PRINCIPLE SIGNS

IMMOBILITY & MUTISM – MOST important signs ,each present in over 90% of
patients.

STUPOR- CLASSIC & STRIKING catatonic sign. Its combination of immobility and
mutism.

1. Excitement 9. Mannerisms 17. Mitgehen


2. Immobility/stupor 10. Verbigeration 18. Gegenhalten
3. Mutism 11. Rigidity 19. Ambitendency.
4. Staring 12. Negativism 20. Grasp reflex
5. Posturing/catalepsy 13. Waxy flexibility 21. Perseveration
6. Grimacing 14. Withdrawal 22. Combativeness
7. Echopraxia/echolalia 15. Impulsivity 23. Autonomic
Stereotypy 16. Automatic abnormality
obedience.

e) LORAZEPAM CHALLENGE TEST:

1 mg intravenous lorazepam

If no response after 5 min, administer another 1 mg

If positive - Treat with lorazepam increasing the dose up to 24 mg/day

If negative - Bilateral electroconvulsive therapy

Topic: Admission procedure related to MHCA 2017


Case scenario:

Mr Rajesh, walks into your hospital alone and complains of multiple somatic symptoms,
decreased interest in work of 2 week duration and says he wants to get admitted in your
hospital. He has not been going to work for the past 4 weeks.
Can you admit Mr Ramesh?
YES
Do you need to inform his family members?
NO
Does Mr Ramesh have the right to refuse medications after getting admitted?
YES
As per Sec 86, Page 32 MHCA,2017

Objectives of the Case Vignette


• Clauses related to voluntary admission
• Need to take consent before informing the family about the patient
• Admission is still a clinical decision
• Once admitted patient is required to follow the by laws of the establishment
• Issues related to voluntary discharge

Mrs R, 56yrs married female, was brought to your hospital by her family members with
history of suspiciousness, abusive, assaultive behaviour, damaging property, talking to self,
smiling to self, poor personal care, disrobing and wandering behavior since past ten years.
Never received any treatment for mental illness.
Mrs R feels that she is completely alright and denies of any mental illness. She has no AD
or NR. Today, she is brought to a nearest nursing home, which is not registered.
What will your next plan of action ?
Will you admit her?
Will you treatment her?

Emergency treatment (Sec 94), page 38


• Any medical treatment may be provided subject to o Informed consent of the nominated
representative,
o If it is immediately necessary to prevent - danger to self or to others or the person causing
serious damage to property belonging to self or to others where such behavior is believed
to flow directly from the person’s mental illness.

• The treatment referred shall be limited to 72 hours only.

• It includes transportation of the patient

• AD & ECT is not applicable during the emergency treatment

Mr D needs admission to a treatment facility for muttering to self, suspiciousness,


irritability and poor personal care. He has an valid advance directive. He wishes to be
treated only with homeopathic treatment and not to give a ECT.
Nominated representative wants to treat him in an allopathic centre. You want to give him
ECT since he has responded earlier to ECT. But the patient has mentioned in his AD that he
should not be treated with ECT
Would you treat him?
What is the role of the MHRB here?
Would you give ECT ?
Would the scenario differ had you not been given a copy of the Advanced Directive?

Objectives of the case:


• Valid advances directives
• Precedence of a valid AD over NR
• Role of the board when NR wants the AD to be revoked
Topic: BPAD
Case scenario:

33 year old mom of 2, is Chartered Accountant by profession, remains withdrawn and


doesn’t fulfil her activities of daily living and prefers to stay on her bed most of the time.
Gradually over 4 months there seems to be deterioration in her work output as she is trying
to do majority of her work from home and eventually has stopped working from last 2
weeks. She reports feeling sad and hurt over her husband’s behaviour towards her. But
now since last 3 days she seems to be crying throughout day, and on questioning answered
that she hears derogatory voice during daytime, when she is left alone. Medical Officer
incharge of the hospital puts her on Sertraline 100 mg/day and Risperidone 4mg/d, due to
unavailability of the Consulting Psychiatrist owing to the restricted OPD hours during the
Lockdown.
• Possible differential diagnosis?
• Could there have been better approach?

Diagnosis: Bipolar II Disorder, current episode Depressed with mood congruent psychotic
features – severe
Points in favour:
1. Depressed mood
2. Diminished interest
3. Guilt feelings
4. Decreased libido
5. More atypical features like
a. More withdrawn
b. Psychomotor retardation
c. Increased appetite and weight gain
d. Hypersomnia
e. Preserved mood reactivity
f. Rejection sensitivity

6. Psychotic features in the form of 2nd person auditory hallucination congruent to her
mood
7. Past episode suggestive of hypomania, with onset at younger age
8. Impairment in socio-occupational functioning

Further Clarification needed:


For premorbid personality, for if she seems to be having hyperthymic temperament then
second differential can be :- “Major Depressive Disorder with mood congruent psychotic
features and atypical features – severe”
Ruled Out:
1. Adjustment Disorder a. Symptoms are present since 4 months, before the lockdown
period(which might act as perpetuating stressor for now)
2. Premenstrual Dysphoric Disorder a. Regular menstrual cycle
b. Symptoms are not associated with the cycle

3. Postpartum Disorder a. Youngest child is 4 years old


4. Depression due to other medical conditions a. All routine investigations including thyroid
profile is normal
5. Delusional Disorder
a. Patient seems to have rejection sensitivity rather than suspiciousness towards
husband
b. Tendency to blame herself due to guilt feelings
6. Schizophreniform disorder/Schizophrenia a. No continuous derogatory hallucinations
b. No other psychotic features suggestive of the same
Recommended Pharmacotherapeutic Options:
I. First line drugs as per NICE guidelines: 1) Fluoxetine Combined with Olanzapine, 2)
Quetiapine
II. Second line drug as per NICE guidelines: Lamotrigine
III. As per BAP guidelines Lamotrigine is first line drug (mood stabiliser or antipsychotic
needed to prevent mania
IV. Lurasidone is also first line drug as per BAP guidelines

MCQs
1) All of the following are characteristics of Bipolar Depression except
a. Lesser anxiety
b. Less withdrawal
c. More retardation
d. More atypical symptoms

2) Choose the atypical features of Depression(multiple answers):


a. Increased appetite and significant weight gain
b. Decreased appetite and significant weight loss
c. Insomnia
d. Hypersomnia
e. Mood reactivity diminished
f. Mood reactivity preserved
g. Rejection sensitivity
h. Early morning awakening

3) Risk factors for antidepressant-induced switch includes


a. Previous antidepressant-induced mania
b. Bipolar family history
c. Family history of recurrent depressive disorder
d. Cushing’s syndrome
e. Exposure to multiple antidepressant trials
f. Initial illness beginning in adolescence or young adulthood

4) Which of the following statements is False?


a. Patients with bipolar 2 disorder suffer more depressed days than those with bipolar 1
disorder
b. Both for bipolar 1 and bipolar 2 patients, mood symptoms are more dominated by
depressive vs manic symptoms
c. Increased suicidality rate and poor functional recovery is seen in bipolar 1 patients
d. Bipolar 1 is a more stable diagnosis as compared to bipolar 2

5) All of the following are recommended treatment for bipolar depression except:
a. Fluoxetine combined with olanzapine
b. Quetiapine
c. Lamotrigine
d. Lurasidone
e. Olanzapine

6) Choose the correct statements pertaining to postpartum depression(multiple answers):


a. Around 10% risk
b. Higher risk amongst those who have past history of depressive illness as well as bipolar
illness
c. 80% of postpartum mood disorders are depression
d. 50% depressive episodes begin before delivery
e. Mood and anxiety symptoms during pregnancy as well as baby blues increase the risk
of postpartum depression

7) The risk of postpartum psychosis is _____% in bipolar disorder


a. 20%
b. 30%
c. 50%
d. 70%

On subsequent follow up she comes and complaints the Psychiatrist that now she has
developed problems with her sleep, which was her way of coping by sleeping more number
of hours. It is quiet distressing as she cannot resist the urge to move lower limbs before
going to bed. She also remains restless throughout the day.
• Describe the phenomenon
• Predisposing factors for this particular phenomenon
• Treatment for the same.

Patient seems to have both Akathisia (most commonly encountered) and Restless Leg
Syndrome
Restless Leg Syndrome:
1. Strong, irresistible urge to move legs (differentiating feature from akathisia, where
whole body is involved)
2. Usually accompanied by paresthesia (sometimes patients do not describe that particular
sensations causes them to move the legs)
3. Motor restlessness
4. Symptoms worsen at rest
5. Symptoms are partially or temporarily relieved by activity
6. Symptoms worsen at night

Predisposing factors for Restless Leg Syndrome:


1. Iron deficiency conditions (anemia, renal insufficiency, pregnancy)
2. Peripheral neuropathy
3. Use of sedating antihistamines
4. Centrally acting dopamine-receptor antagonists (metoclopramide, prochlorperazine)
5. Antipsychotics
6. Caffeine
7. Most antidepressants (except bupropion)

Treatment
1. Pharmacotherapy a. Dopaminergic agents (Ropinirole, Rotigotine, Pramipexole) (ergot
derivative Pergolide)
b. Bromocriptine
c. Dopaminergic precursor (levodopa/carbidopa)
d. Anticonvulsants (Gabapentin, Pregabalin, Carbamazapine)
e. Opioids (Oxycodone, Propoxyphene)
f. Clonazepam

2. Other a. Warm bath & massage


b. Warm or Cool packs
c. Sleep hygiene
d. Exercise
e. Avoid caffeine
f. Foot wrap

MCQs
8) Characteristic features of Restless Leg Syndrome:
a. Irresistible need to move legs
b. Accompanied by paresthesia
c. Symptoms worsen at rest
d. A & C
e. All of the above

9) Following drugs lead to Restless Leg Syndrome:


a. Antihistamines
b. Anticholinergic
c. Beta blockers
d. Antipsychotics
e. Antidepressants
f. antianxiety

10) Drug first licensed to treat RLS:


a. Bromocriptine
b. Pramipexole
c. Ropinirole
d. pergolide

11) Antidepressant that does not cause Restless Leg Syndrome:


a. Escitalopram
b. Venlafaxine
c. Bupropion
d. Sertraline

12) SSRI that is least likely to cause Restless Leg Syndrome:


a. Escitalopram
b. Sertraline
c. Paroxetine
d. Fluoxetine

13) Akathisia is seen in ____% of people on antipsychotics and within ___weeks of


treatment a. 23%, 1 week
b. 23%, 3 weeks
c. 17%, 2 weeks
d. 17%, 3 weeks
e. 36%, 2 weeks
f. 36%, 1 week

14) Most common movement disorder encountered with SSRI


a. Tremors
b. Akathisia
c. Dystonia
d. Dyskinesia

15) Select predisposing factors that lead to the development of movement


disorders with SSRI
a. Concomitant treatment with Antipsychotics
b. Concomitant treatment with Lithium
c. Pre-existing brain damage
d. A & C
e. All of the above

16) Select the management strategies for Akathisia


a. Reduce the dose of offending agent
b. Propranolol
c. Atenolol
d. Benzodiazepines
e. Anticholinergics
f. Antihistamines

17) Which antipsychotic has the least propensity to develop akathisia


a. Aripiprazole
b. Risperidone
c. Quetiapine
d. Asenapine

Now the treating psychiatrist decides to stop sertraline and risperidone and start
Lurasidone.
Please help the psychiatrist decide
• Starting dose
• How to administer
• Predict possible side effects
• Overall uses of the molecule

Lurasidone
FDA Starting dose Recommended Maximum dose
Recommendati range
on for:
Diagnosis
Schizophrenia 40 mg 40 – 80 mg 160 mg
Bipolar 20 mg 20 – 60 mg 120 mg
depression
Administered with food for better absorption and hence to increase bioavailability and
preferable night dose to avoid daytime sedation, akathisia and EPS

Most Common side effects


• Nausea
• Akathisia
• EPS
• Somnolence
Used for
• Schizophrenia
• Bipolar depression
• Acute mania/mixed episode
• Other psychotic disorders
• Bipolar maintenance
• Treatment resistant depression
• Behavioural disturbances in dementia
• Behavioural disturbances in Children & Adoloscents
• Disorders associated with problems with impulse control

MCQs
18) Oral bioavailability of Lurasidone
a. 7 – 10%
b. 9 – 19%
c. 5 – 20%
d. 11 – 25%
e. 15 – 30%

19) Peak serum concentrations with lurasidone are reached in:


a. 1 – 2 hours
b. 1 – 3 hours
c. 2 – 4 hours
d. 2 – 6 hours

20) Lurasidone has high affinity for the following receptors: (multiple answers)
a. D2
b. 5HT 1A
c. 5HT 1B/1D
d. 5HT 2A
e. 5HT 2C
f. 5HT7
g. Alpha 2A
h. Alpha 2C

21) Lurasidone is partial agonist at:


a. D2
b. 5HT 1A
c. 5HT 1B/1D
d. 5HT 2A
e. 5HT 2C
f. 5HT7
g. Alpha 2A
h. Alpha 2C

22) Recommended starting dose of Lurasidone for bipolar depression:


a. 10 mg
b. 20 mg
c. 40 mg
d. 60 mg

23) FDA recommended indications for Lurasidone: (multiple answers)


a. Schizophrenia
b. Acute mania
c. Bipolar depression
d. Bipolar maintenance
e. Treatment resistant depression
f. Psychotic symptoms due to general medical conditions

24) Select most common adverse reactions with lurasidone: (multiple answers)
a. Nausea
b. Gastric discomfort
c. Akathisia
d. EPS
e. Weight gain
f. Somnolence
g. insomnia

25) Antidepressant effect of lurasidone is owing to it’s affinity for the following receptors:
(multiple answers)
a. D2
b. 5HT 1A
c. 5HT 1B/1D
d. 5HT 2A
e. 5HT 2C
f. 5HT7
g. Alpha 1
h. Alpha 2

Topic: Pregnancy &Breast Feeding


CaseScenario:

Mrs. S, a 25 year-old mother of 2 children, has been married for 4 years. She has just had
her 2nd child just 3months ago which was an unplanned pregnancy. She conceived just
6months after the 1st child, unaware as she was having lactational amenorrhea. For a
month after the second child, she was well , but then family began to notice to unusual
behaviors. She had started isolating herself, stopped speaking to anyone at home, losing
interest in her daily activities and ceasing to care for her children and being very irritable
which is not her usual self.
On enquiring she reported having sleep disturbances, feeling excessively tired even while
doing just daily chores, feeling easily tearful and wishes to end all this.
She has been in conflict with her husband and mother in law even during the 1st pregnancy
with respect to the gender of the child. The family was inclined to having a male child and
was disappointed in her. They did not help her in bringing up the baby nor with chores
around the baby. She is now fearful and sad that the 2nd baby is also a female child and
husband and in laws are not pleased with the same. On occasions family has been verbally
and physically abusive to her. She has been brought to OPD by her family

Questions:
1. What aspects of history and examination will you clarify?

- HOPI – any death wishes or active suicidal ideas , any hypomanic symptoms ,
- Past History – any other psychiatric illness / any self-harm attempts/ depressive
symptoms after first child birth
- Family H/o – Bipolar disorder in first degree relatives.
2. Likely diagnosis? PPD
3. How will you approach the case?

- Mode : both non-pharmacological and pharmacological


- Antidepressants safe in pregnancy – Sertraline, Citalopram
- Psychological – Psychoeducation to patient and family members about PPD ,
address family member’s concept of male child, which seems to be current
stressor for the depression, cognitive distortions present currently in the
patient. Interpersonal therapy addressing the IPR issues can help
4. What scales and tools of assessment will you use?

- EDINBURGH POSTNATAL DEPRESSION SCALE

10 item scale – 0-3 scoring ( maximum score – 30)

Related MCQs:

1. SSRI found to be relatively safe in lactating women


- Fluoxetine
- Sertraline
- Fluvoxamine
- Escitalopram
2. Evidence suggests which mood stabilizer is safe in pregnancy and lactation?
- Lithium
- Carbamazepine
- Lamotrigine
- Gabapentin
3. Which of the following are risk factors for postpartum depression?
- Low employment
- Poor social support
- Anemia
- History of psychiatric illness
- High number of stressful life events
- All of the above
4. _______ % of women with post-partum blues develop PPD?
- 10-15%
- 20-25%
- 35-40%
- 50-60%
5. Which atypical antipsychotic has lowest RID?
- Risperidone
- Olanzapine
- Clozapine
- Aripiprazole
6. Lamotrigine is generally considered safe in pregnancy. However reports
suggest rare possibility of the following side-effects.
- CNS depression
- Severe Apnoea
- Skin problems
- Both A & B
- All of the above
7. Breast feeding with Lithium is recommended only if which of the following
conditions are met?
▪ Stable maternal mood
▪ Lithium monotherapy
▪ Healthy Infant
▪ All of the above
8. Which of the following is not an early predictive sign for PPD?
- Unsupportive family
- Ambivalence to pregnancy, child birth
- Aggressiveness to newborn
- Sleep disturbance or severe nightmares
- Intense feelings of loss
- None of the above
9. Postpartum psychosis is associated with ___ times increased risk of maternal
suicide?
a. 50
b. 20
c. 70
d. 10
10. Medications to be avoided or contraindicated in Pregnancy :
o Lithium
o TCAs
o BZDs
o Carbamazepine
o All of the above
11. Which is not a differential to be considered in patient with postpartum
psychosis ( first-episode)
- Delirium
- Eclampsia
- Thyroid disorders
- Infection
- None of the above
12. Antidepressants should be used with caution in
a. Major depressive disorder with mixed features

b. Major depressive episode with first onset in the postpartum period

c. Major depressive episode with onset in early postpartum period

d. History of bipolar disorder in a first degree relative

e. Atypical features: hypersomnia, leaden paralysis or increased


appetite

f. ALL of the above situations

Psychotropic Use in Pregnancy

1. When a woman on psychotropic medication discovers that she is pregnant, how do


you proceed to manage?
- In patients with high risk of relapse and severe illness, abrupt discontinuation is
dangerous.
- Consider remaining with current (effective) medication rather than switching,
to minimize the risk of relapse and hence the number of drugs to which the
fetus is exposed
2. How to manage when a psychiatric illness is newly diagnosed in pregnant women?

- Preferably non-pharmacological management


- Try to avoid all drugs in the first trimester
- If non‐drug treatments are not effective/appropriate, use an established drug
at the lowest effective dose
- ECT – safest option in pregnancy

3. A 25-year-old lady presents to you with 1-month history of feeling sad, irritated
when the baby cries, sleep deprived not interested in eating or talking to anyone.
She just delivered a male boy 1 month ago. Husband and family have been
supportive and are taking care of the baby. She feels guilty about it and fights with
mother to let her take care of baby more and is not appreciative of their help. They
are concerned as it’s become a vicious cycle now. As a attending residents what
steps, what advice will u give them
-Diagnosis – Post Partum depression

Detailed history and physical examination

MSE to get more details on thought content

Baseline rating of depression – Edinburgh’s post-partum depression rating scale


Suicide risk assessment

Psychoeducation of the patient and family members

Patient – Educate the patient that its best to take care of her mental health first before
caring for the baby and its ok for her to let the mother care for the baby for sometime.
Educate regarding the consequences of depression, if left untreated. Educate on how
depression affects the healthy bonding and might affect the infant behavior later in life,
also effect of medicines on infant.

Family – Educate about depressive illness, symptoms, causes and how their behavior will
help in patients’ recovery

Pharmacology

Moderate to severe depression - Antidepressants-SSRIs are 1st line in management

1. Sertraline is the safest


2. Fluoxetine – can cause some activation and diarrhea in the
baby
3. Paroxetine – Shorter half life
Mild depression - CBT- Cognitive behavioral therapy

4. A 30-year-old lady is in her 3 rd trimester of pregnancy and is being treated with IM


olanzapine for bipolar manic episode. Formulate a management plan for her during
delivery and immediate post-partum
Assessment (start all management plans with investigation/assessments)

1. Severity of depression /Mania


2. Blood investigations as baseline before starting medications
to look at liver,renal functions
3. Psychoeducation of patient and family
4. Discuss different medications and option available
5. Plan for in patient vs Outpatient care
Continue IM olanzepine, although mood stbilizers are commonly used, data on
antipsychotic use also reports olanzepine can be given during lactation. Please read the
articles below

https://www.longdom.org/open-access/safety-and-efficacy-of-antipsychotics-in-
pregnancy-and-lactation-2329-6488-1000267.pdf

https://ajp.psychiatryonline.org/doi/10.1176/appi.ajp.158.7.1001?url_ver=Z39.88-
2003&rfr_id=ori:rid:crossref.org&rfr_dat=cr_pub%20%200pubmed
Options that can be considered are, Changing on to oral Olanzapine, lithium,
Valproateafter delivery.

Topic: ADHD
Case scenario:

Master ABC is a 10-year-old boy brought to clinic by his mother. ABC presents with 5 years
history of gradually progressing difficulties characterised by overactivity, chronic
irritability, falling grades in school, multiple complaints from school about his behaviour,
day dreaming in school, not able to remember anything in exams. He also suddenly runs
away on road if sees anything interesting, doesn’t wait for his turn and blurts out answers.
He is also very shy when relatives come home. Gets very angry if he is forced to talk with
relatives or to carry out a task in a group. He is very aggressive towards parents. He always
does opposite of what is being told.

Mother is tearful when she reports that he obeys hi school teacher and any relatives but
not his parents. So, 5 days a week he is with his grandparents and over the weekend with
his parents. Though he hates school he attends school regularly. As far as mother knows
he is not cruel towards animals and has not physically assaulted anyone. When you observe
him in the clinic, he is quite a good boy. Though plays with your pen and clinic papers, he
is not very disruptive. As assessment progresses, he insists on leaving as he is bored and
becomes irritable towards mother.

Mother reports that she had perinatal complications in the form of prolonged labour and
ABC has difficulty breathing. Doesn’t know further details. She feels that his milestones like
talking and walking may be delayed by 2 to 3 months but she dint find any problems after
3 years of age with regards to his milestones.

To make a final diagnosis of ADHD what are the criteria required and from how many
settings. In the above case can you make a final diagnosis ?

Methylphenidate Atomoxetine Lisdexamphetamine/Clonidine


DOC in pt with no DOC when not DOC when MPH cant be given
psychiatric or physical responding to MPH and
comorbidity Lisdexamphetamine
DOC in tic disorder DOC in pt with
substance abuse in
parents
DOC in ADHD with
ASD
DOC ADHD with
Anxiety
5 years and above

Case scenario:

• Jack is a 14-year-old boy


• Had global delayed milestones but achieved them by the age of 6 years.
• His medical history is not significant except incidental heart murmur at the age of
7 years.
• By age of 7 years Mom noticed that he is very hyperactive and always on a go, very
impulsive and doesn’t finish his task. He was also noticed to be in his own imaginary
world. He is socially ackward and doesn’t make eye contact with strangers.
• He had speech difficulties which were ok after taking speech therapy.
• He was on Methylphenidate
• After which presented with excessive aggression, non-stop talking, not sleeping,
talking on imaginary phone, says he is receiving secret codes from Japanese and
Chinese.
• He shows a blank page with codes written on that page. He is all over the opd room,
difficult to control. He has assaulted mother as he thinks mother is with the
Japanese agency.
• Mother says this started 2 weeks ago when he was not allowed to go out and play
and meet his “bad friends” due to corona restrictions..
Discussion was focused on reason for current condition and management.

Debate was on the drug methylphenidate and its propensity to cause psychosis Vs ASD
with ADHD

1. In ADHD you are giving methylphenidate which increases dopamine and starting
an Antipsychotics may be indicated . So many guidelines suggest stopping of
methylphenidate till the psychotic episode is controlled.
2. Another fundamental issue is EPS are more in patients with ID. So antipsychotic
with minimal propensity for EPS should be chosen
3. Third is in view of heart murmur in the current , getting a basic baseline blood
evaluation and ECG is indicated before starting antipsychotics.
Differential diagnosis that can be considered:

1. Methylphenidate induced Psychosis- in case he took overdose or was abusing it

2. We will have to clarify with mother and may be get collateral history from school. After
clarification, we get to know that since childhood he is obsessed with chinese and japanese
secret services after watching a war movie. as he is not able to express his distress or
emotions directly, he uses stories or imaginary character when he wants to show his anger
towards mother. As mother was not well, his compliance was poor, so ADHD was worsened
and he was not allowed to out, smoke weed or do his routine he became very angry and
irritable. so is this this presentation just secondary to worsened ADHD and ASD on the
background of ID ?

So in child psych we will have to think of developmental aspects.

As per evidence , ADHD kids has high abuse rate with cannabis as it helps in concentration
and feels better, (increases dopamine )

Along with stimulants, tetrahydrocannabinol also acts as stimulant increases glutamate


resulting in psychosis
Management plan

After admission:

Thorough physical & psychiatric examination, to rule out syndromic causes of IDD

Keeping in mind high possibility of ‘ cannabis induced psychosis ‘I would expect the
psychosis to resolve within 2-3 days after stopping cannabis if it’s induced by it , then my
diagnosis would be cannabis induced psychosis, If not decreased, as you advised I would
have stoped stimulants (as per evidence usually psychotic symptoms resolve within 10
days of stopping stimuants).

Treatment:

Short term: treatment of acute psychosis with antipsychotics as u adviced

Medium term: treating the underlying disorder - adhd , asd, idd

Long term:

1. Prevention of psychosis ( it is better not add stimulant again because once the psychosis
has resulted increases future risk of psychosis ) ,

2. Management of cannabis dependence (MET),

3. As per Multimodal treatment study of children with ADHD - combination of behaviour


therapy and medications

4. Involvement of psychologists and community health care plan

Mother should be provided with support and suggestions regarding how to talk to child,
and what to do if he relapses or refuses to take medication.

For prominent sleep disturbance- low dose quetiapine & / or chlorpromazine used which
helps sleep & prevent psychosis

If psychosis is transient – risperidone is better.

CASE 3:

33 year old mom of 2, is a chartered accountant, remains withdrawn and doesn’t fulfil her
activities of daily living and prefers to stay on her bed most of the time. Gradually over 4
months there seems to be deterioration in her work output as she is trying to do majority
of her work from home and eventually has stopped working since last 2 weeks. She reports
feeling sad and hurt over her husband’s behaviour towards her. But now since last 3 days
she seems to be crying throughout day, and on questioning answered that she hears
derogatory voice during daytime, when she is left alone. Medical officer incharge of the
hospital puts her on Sertraline 100mg/day and Risperidone 4mg/day, due to unavailability
of the Consulting Psychiatrist owing to the restricted OPD hours during Lockdown. No
similar complaints after first child birth,No past history available after marriage and the
patient is not able to recall anything before marriage ,No previous medication history for
these 4 months as she was of the opinion that she is becoming lazy and can overcome on
her own her sad feelings. Husband working in a multinational company is often very busy
and had been on business tour for nearly one and half month in last 4 months, and
obviously was under the impression that even wife must be busy and managing her work
at the year end. It is only because of the Lockdown , he is able to observe her closely
throughout the day now. Voice and not voices coming from near the window of her
bedroom- “you are good for nothing “.... she is unable to recognise the voice but agrees to
it and cries on hearing it.... more so since last 3 days, in last week or so she had heard it
occasionally. she tries to cope with feelings of sadness by sleeping more. She often gives
into craving for chocolates and sweets and has gained around 12 kg weight in last 4 months
. Though she does cheer up when asked about her first child’s birthday celebration, and
recalls her best time in life was the 4 months period during her 5th semester in college,
when she won quiz and also had enrolled in multiple online courses and had maximum
number of friends in her life.... but following which she got busy with her exams and
everything collapsed and just like any other girl got married. Attained menarche at 12 ,
menstrual cycle is regular currently and decreased libido

On subsequent follow up she comes and complaints the Psychiatrist that now she has
developed problems with her sleep, which was her way of coping by sleeping more number
of hours. It is quiet distressing as she cannot resist the urge to move lower limbs before
going to bed. She also remains restless throughout the day.

Diagnosis: Bipolar II Disorder, current episode Depressed with mood congruent psychotic
features – severe

Points in favour:

Depressed mood 2. Diminished interest 3. Guilt feelings 4. Decreased libido 5. More


atypical features like a. More withdrawn b. Psychomotor retardation c. Increased appetite
and weight gain d. Hypersomnia e. Preserved mood reactivity f. Rejection sensitivity 6.
Psychotic features in the form of 2nd person auditory hallucination congruent to her mood
7. Past episode suggestive of hypomania, with onset at younger age 8. Impairment in socio-
occupational functioning

Ruled Out:

1. Adjustment Disorder a. Symptoms are present since 4 months, before the


lockdown period(which might act as perpetuating stressor for now)

2. Premenstrual Dysphoric Disorder a. Regular menstrual cycle b. Symptoms are


not associated with the cycle

3. Postpartum Disorder a. Youngest child is 4 years old

4. Depression due to other medical conditions a. All routine investigations including


thyroid profile is normal

5. Delusional Disorder a. Patient seems to have rejection sensitivity rather than


suspiciousness towards husband b. Tendency to blame herself due to guilt feelings

6. Schizophreniform disorder/Schizophrenia a. No continuous derogatory


hallucinations b. No other psychotic features suggestive of the same
Recommended Pharmacotherapeutic Options:

First line drugs as per NICE guidelines : 1) Fluoxetine Combined with Olanzapine, 2)
Quetiapine II. Second line drug as per NICE guidelines : Lamotrigine III. As per BAP
guidelines Lamotrigine is first line drug (mood stabilizer or antipsychotic needed to
prevent mania IV. Lurasidone is also first line drug as per BAP guidelines

Restless Leg Syndrome:

1. Strong, irresistible urge to move legs (differentiating feature from akathisia,


where whole body is involved)

2. Usually accompanied by paresthesia (sometimes patients do not describe that


particular sensations causes them to move the legs)

3. Motor restlessness

4. Symptoms worsen at rest

5. Symptoms are partially or temporarily relieved by activity

6. Symptoms worsen at night Predisposing factors for Restless Leg Syndrome: 1.


Iron deficiency conditions (anemia, renal insufficiency, pregnancy) 2. Peripheral
neuropathy 3. Use of sedating antihistamines

4. Centrally acting dopamine-receptor antagonists (metoclopramide,


prochlorperazine)

5. Antipsychotics

6. Caffeine

7. Most antidepressants (except bupropion)

Treatment

1. Pharmacotherapy

a. Dopaminergic agents (Ropinirole, Rotigotine, Pramipexole) (ergot derivative


Pergolide)

b. Bromocriptine

c. Dopaminergic precursor (levodopa/carbidopa)

d. Anticonvulsants (Gabapentin, Pregabalin, Carbamazapine)

e. Opioids (Oxycodone, Propoxyphene)


f. Clonazepam

2. Other

a. Warm bath & massage b. Warm or Cool packs c. Sleep hygiene d. Exercise e.
Avoid caffeine f. Foot wrap

Lurasidone

FDA Recommendation for:

Diagnosis Starting dose Recommended Maximum


Scizophrenia 40mg 40-80mg 160mg
Bipolar depression 20mg 20-60mg 120mg

Administered with food for better absorption and hence to increase bioavailability and
preferable night dose to avoid daytime sedation, akathisia and EPS

Most Common side effects • Nausea • Akathisia • EPS • Somnolence Used for •
Schizophrenia • Bipolar depression • Acute mania/mixed episode • Other psychotic
disorders • Bipolar maintenance • Treatment resistant depression • Behavioural
disturbances in dementia • Behavioural disturbances in Children &Adoloscents • Disorders
associated with problems with impulse control

CASE 4:

A 32 year old, MrGovind, a manual laborer from Bihar finds himself stuck in Kerala due to
the sudden declaration of a national lock-down. His friends describe him as an impulsive
man, attached to his family, drinks 6U alcohol once a week and is novelty-seeking. They
also notice that increasingly for the last 10 days, he is becoming increasingly irritable, with
disturbed sleep and mild decrease in appetite. His last alcohol use was 7 days ago (3U) and
last to last was 14 days ago. At their last drink, they noticed Govind crying excessively,
remembering his family in Bihar. So, they reported that they were glad that they ran out
of alcohol. His friends tried to console him by offering him their stack of cannabis sativa
which according to them help him get his mind off things. This was the first time MrGovind
used cannabis and his friends liked him stoned, they said. Since the last 7 days, he has been
smoking 3-4 joints (cannabis+tobacco)/day and since the last 3 days, they notice occasional
abnormal behavior such as talking and smiling to self, shouting at his friends, telling them
to stop planning to kill him. They called you on the psychiatry helpline. What more
questions would you ask them and how would you help them through the telemedicine
option made available to you?

No past history of withdrawal related delirium or seizures. No past attempts to quit alcohol
but, He says he didn't touch a drop during the mala period for his Sabarimala trip. He
consumes a local whisky - 'Mysore Lancer' ONLY. No SIB but definite paranoia.
Patient is oriented. His psychotic symptoms are definitely fluctuating. He seemed fine
while I was leaving for lunch today.Cannabis last use was yesterday.No suicidal ideas but
possible homicidal ideas secondary to psychosis. He may act on these delusions in high
likelihood based on his behavior.Wife had a depressive episode. He had a fall from his chair
26 days ago and hit his head. But no loss of consciousness or nausea/vomiting or ENT
bleed. No substance use other than alcohol.

Possibilities in this case are:

1. Cannabis induced Psychotic disorder or

2. a primary psychotic disorder (ATPD»>Schiz) unmasked by Cannabis or

3. A manic episode (needs clarification on mood and any grandiose ideation/delusion)

Advise:

1. STOP cannabis,

2. In view of new telemedicine guidelines approved during COVID-19 prescription of


psychotropics and benzodiazepines is made possible. Even at 1 st tele psychiatry
consultation pt can be prescribed BZD.

Topic: Treatment resistant OCD


Case Scenario:

Mr. A, a 22-year old man, graduate, unmarried, came to a mental health clinic with the
complaints of being bothered by sudden images of hitting someone, fears that he would
say things that might be offensive or wrong, and concerns about upsetting his
neighbors. To ease the anxiety caused by these thoughts, he often replayed prior
conversations in his mind, kept diaries to record what he said and often apologized for
fear he might have sounded offensive. When he showered, he made sure the water in
the tub only reached a certain level. He was afraid that if he was not careful, he would
flood his neighbors.

He reports other frequent thoughts about being ill and keeps arranging things as he
thinks they are out of symmetry. He says that he used gloves at work and performed
well. He spent most of his free time at home. Although he enjoyed the company of
others, the fear of having to touch something if he was invited to a meal or to another
person’s home wastoo much for him to handle.

He says that he keeps checking repeatedly if he closed the door while leaving home and
keeps repeating and clarifying the things he said. He also says he keeps washing hand
frequently to be sure that there are no germs.
These caused him to spend so much time on his routines and to avoid leaving his
apartment,engaging in social relationships and performing basic errands. He knew that
his fears and urges were “crazy,” but he felt they were out of his control. His daily
routine was disturbed by his complaints

What is the diagnosis? How do you assess and manage him?

• Diagnosis – OCD
• Assess by Y-BOCS
• Management – Serotonin Receptor Inhibitors (SRI) & Exposure and response
therapy (ERP)
MCQs

1. Treatment response in OCD is defined as % reduction of Y-BOCS from baseline?

- 20-25 %
- >25 %
- >35 %

None of the above

2. Up to % of patients with OCD exhibit inadequate response to serotonin reuptake


inhibitors (SRI)?
- 2
- 3
- 5
- 6

3. Non-response or Treatment resistant OCD is defined as

- <40% reduction in Y-BOCS score


- <25% reduction in Y-BOCS score
- <35% reduction in Y-BOCS score
- <45% reduction in Y-BOCS score
- Intrinsic risk factors for poor response to conventional OCD treatment
-
- Comorbid Tics
- Hoarding
- Sexual content
- Poor insight
- Somatic obsessions
- All of the above

4. OCD with which personality disorder is not associated with poor treatment response?
- Anankastic
- Paranoid
- Schizotypal
- Anxious
- Borderline

5. Lifetime prevalence of sexual obsessions in patients with OCD

- 20.7
- 13.3
- 25
- 9.7

6. Sexual obsessions are associated with

- Comorbid Tics
- More common in men
- Poor insight
- Poor treatment response
- Poor sexual satisfaction
- All of the above

Topic: Treatment resistant depression

Case Scenario 1 - A 45 year old man presented with low mood, easy fatiguability, decreased
interest and anhedonia. He is having suicidal thoughts and says this is worst phase of his life and
worst depression he ever felt. Current Duration of symptoms is 8 months. Age of onset is 35
years and this is his fourth episode of depression. He recieved multiple treatments such as
Sertraline, Escitalopram, Bupropion and Venlafaxine. Mother has history of depression. He has
strong suicidal ideation but no active plan as of now. Stressor - Marital conflicts which recently
triggered the episode. Reasons for using different medication was initially he was started on
Escitalopram in 1st and Sertraline in the second episode, there was full remission of symptoms
but he had discontinued the medicine after 6 months as he had full recovery from symptoms
and also developed sexual dysfunction with long term use of medicine. In the later episodes he
was tried on Bupropion due to his earlier side effect profile but there was remission was not
acheived and reported increased suicidal ideation. So finally he was shifted to Venlafaxine for
remission of the symptoms, which showed good response.

⚫ Recurrence of Depression rather than resistance (RDD)


⚫ Here there is need to consider the treatment adherence response and remission in the
patient along with the side effect profile. Major depression can be recurrent and
progress with shorter periods of recovery in between.
⚫ As the patient repsonding to antidepressant medications can be treated with long term
maintenance of antidepressants.
⚫ Mood stabilizers like Lithium can be given to decrease the suicidal intent and also
preventing the recurrence of depression. Can be augmented with psychotherapy
techniques such as CBT to improve depression, adherence and address possible
stressors

Case scenario 2 - Mrs X, 45 years of age with onset of depression in early 20's with unremitting
episode and less than 3 month symptom free episode periods since last 10 years. She has severe
depression with psychomotor slowing.. With Personal history and family history being nil
significant.
Inadequate response to Fluoxetine 60 mg, Sertraline 200 mg and Venlafaxine 225mg individually
with adequate trials in the recent years. Initial years she also recieved clomipramine but dosing
details not available. Even though compliance is doubtful in initial years. She is well compliant
todrugs in past 5 years. No medical comorbidities except for GERD and irritable bowel syndrome.
Ans - Need for consider treatment resistance in this case (2 classes of antidepressants failure
withadequate trial)

Polls/Questions

1. Approximately how many patients fail to respond to antidepressant treatment in first


episode]
- One tenth or 10%

- One third or 33% (Ans)

- Half or 50%

- 2/3rd or 67%

2. Treatment resistance in depression can be defined as


- Failure to respond to multiple drug trials

- Failure to respond to a trial equivalent to 300 mg of imipramine for 6 weeks

- Poor response to two different classes of antidepressants


- Inadequate response to adequate course of treatment in depression

- All of the above (ANS - Different definitions by different authors)

3. Least common of following residual symptoms of depression is


- Fatigue

- Pain
- Insomnia

- Low mood (Answer)

- Decreased concentration

-
4. Pseudo resistance in depression means
- Response to inadequate treatment

- Non response to inadequate treatment (ANS)

- Inadequate response to adequate treatment

- Failure to respond to maximum dose of antidepressant


-

5. All of the following are true regarding course of depression except


- Onset usually in adulthood

- Mean duration is 6-9 months

- Residual symptoms in 50% patients

- Recurrence in 50% patients

- Best response to antidepressants is for age 15-24 years (ANS, Best Response in Adulthood -
25 to 45 years)

6. Augmentation strategy can include all except


- Combining two antidepressants (Ans, Combination strategy)

- Adding CBT or IPT

- Adding Lithium

- Adding Pindolol

- Adding Thyroxine

7. Poor predictors of antidepressant response can be (ALL of the below)


- Frontostriatal atrophy and decreased CBF to PFC

- Subcortical hyperintensities

- Female gender

- Low SES and poor social support

- High theta activity and q EEG abnormalities in PFC/ACC

- CYP2D6 and 5HTT polymorphisms (short 5HTLLPR allele - poor response)


8. STAR-D is funded by
- WHO

- NIMH (Ans)
- NIMHANS

- ICMR

9. In STAR-D the following treatments are used in different levels except ]


- Citalopram

- Sertraline

- Escitalopram (was not used)


- Venlafaxine
- Tranylcypromine

10. True regarding CBT in STAR-D


- CBT showed similar response rates to citalopram when given alone

- CBT with citalopram showed similar remission rates when compared to CBT alone

- Augmentation or switching to other antidepressants showed better response than CBT

- All of the above

- None of the above

11. Level 2 of STAR -D includes following drugs except


- Bupropion
- Buspirone

- Venlafaxine

- Mirtazapine (TCA, Mirtazapine in step 3)

Multiple choice questions (MCQs)


Topic: Somatoform disorders
1. In the DSM 5 diagnosis of Somatic Symptom Disorder, the following diagnoses are covered
except
a. Psychological factors affecting another medical condition
b. Body dysmorphic disorder
c. Factitious disorder
d. Illness anxiety disorder

Answer D.
According to the DSM-5 the key characteristic of somatic symptom disorders is the presence of
one or more persistent somatic symptoms that are associated with excessive thoughts, feelings,
and behaviors related to the symptoms.
In DSM-IV, disorders such as factitious disorder and psychological factors affecting other
medical conditions were grouped elsewhere. This led to some confusion in differential diagnosis.
As a result, such disorders are now included within the rubric of the somatic symptom disorders
chapter.
Due to its strong linkage and similarity to obsessive-compulsive disorders, body dysmorphic
disorder has now been moved out of the somatic symptom disorders and is now included in the
obsessive-compulsive disorders group.
Also, largely because of its pejorative connotation, the classical term “hypochondriasis” has been
removed and the disorder renamed as Illness Anxiety Disorder in DSM-5

2. The term “somatisation” is attributed to


a. Wilhelm Stekel
b. Pierre Briquet
c. Karl Jaspers
d. Juan Jose Lopez Ibor

Answer A.
The psychoanalyst Wilhelm Stekel used this term for the first time in 1911, defining it as the
bodily expression of a deep somatic neurosis or the process of how neurotic conflicts are
presented as physical symptoms.
Although the term “somatization” is credited to Stekel, its formal entry into medical discourse is
due to the work and influence of Zbigniew J. Lipowski, a Polish-born psychiatrist practicing in
Canada. In the 1960s, Lipowski provided a broader view of somatization, defining it as the
tendency to experience, conceptualize, and/or communicate psychological states or contents as
body sensations, functional changes, or somatic metaphors
The description in 1859 of a syndrome with multiple motor and sensory symptoms by the French
neurologist Pierre Briquet allowed an initial separation of somatization from conversion
phenomena.
Karl Jaspers mentioned “hypochondria,” “hysteria,” and the “somatopsychic”. According to him,
the great majority of physical suffering is due to psychological reflection and not to manifested
physical disease.

3. The following are the DSM 5 diagnostic criteria for somatic symptom disorder except
a. Disproportionate and persistent thoughts about the seriousness of one’s symptoms
b. Persistently high level of anxiety about health or symptoms
c. Excessive time and energy devoted to these symptoms or health concerns
d. The symptoms must be medically unexplained
Answer D
The stipulation that these symptoms be medically unexplained has been removed, because of
the recognition that such a distinction in itself is unreliable and because psychiatrists commonly
treat patients with medically established diagnoses who are troubled by or preoccupied with
their physical symptoms, disproportionately. Furthermore, DSM-5 acknowledges that frequently,
somatic symptom disorders co-occur with other psychiatric disorders. The diagnosis of a somatic
symptom disorder is only made when there is significant distress or impairment associated with
these chronic somatic symptoms.

4. What is Smith’s consultation letter?


This consists of a brief “consultation letter” intended for primary care physicians providing them
with “do’s and don’ts” regarding their encounters with patients with multiple medically
unexplained physical symptoms and briefly instructing them on the use of a few key management
techniques.
● Notification that the patient meets criteria for somatization
● Reassuring information regarding the nonlethal course of somatization
● Recommendations that the patient be regularly scheduled for brief appointments with the
primary care physician and that urgent appointments be avoided as much as possible
● A recommendation that the physician look closely for signs of disease rather than taking the
patient’s symptoms at face value
● A suggestion that hospitalization, surgery, or diagnostic procedures be avoided unless indicated
by physical abnormalities
● A recommendation that the physician view the symptoms as part of an unconscious process
rather than telling the patient that the problem is “all in your head”
While it doesn’t improve symptoms in patients, it leads to improved functional capacity

5. Recommendations for general measures like yoga, meditation etc are better received than
recommendations for psychological interventions.
True or false?

Answer: True

6. An exhaustive work up and specialist referral is required to reassure patients with multiple
somatic complaints.
True of False?

Answer: False
Usually, by the time a patient with multiple somatic complaints is referred to a psychiatrist, they
would have already undergone extensive testing by multiple doctors.

7. What is the CARE MD approach?


The CARE MD treatment approach encourages patients to be active participants in their care and
serves as a guide to help PCPs work effectively with people who have somatoform disorders.

8. What are the subtypes of somatisation?


9. What is the BATHE approach?
When evaluating a patient with multiple physical complaints, when doctors focus primarily on
physical symptom relief, they can miss clues about the reason for the visit and miss the
opportunity to explore the patient’s perspective of the symptom and associated psychosocial
stressors. The BATHE technique is an excellent tool for engaging a patient while simultaneously
assessing psychosocial factors. This technique consists of asking 4 specific questions, and
empathetic responses.
• B: Background: “What is going on in your life?” and “What brings you in today?”
• A: Affect: “How do you feel the most about this situation?”
• T: Trouble: “What bothers you the most about this situation?”
• H: Handling: “How are you handling that?”
• E: Empathy: “That must be very difficult for you”

10. The following is a risk factor for developing somatoform disorders


a. Concurrent medical illness
b. Substance use
c. More education
d. Male sex

Answer C
11. The following are the personality types which affect psychological response to illness (Kahana
& Bibring, 1964) (multiple answers)
a. Borderline
b. Masochistic
c. Obsessional
d. Narcissistic
e. Schizoid
f. Dissocial
g. Schizotypal
h. Dependant

Answer: B, C, D, E, H

TYPE CHARACTERISTICS MEANING OF COUNTERTRANSFER TIPS ON


ILLNESS ENCE RESPONSES MANAGEMENT
DEPENDANT Needy, Threat of Positive: doctor feels Reassure within
demanding, clingy abandonment powerful and limits
Unable to reassure needed Schedule visits
self Negative: doctor Mobilize other
Seeks reassurance feels supports
from others overwhelmed and Reward efforts
annoyed; may try to toward
avoid patient independence
Avoid tendency to
withdraw from
patient

OBSESSIONAL Meticulous, Loss of control May admire Try to set routine


orderly over When extreme: Give patient choices
Likes to feel in body/emotions/ anger—a “battle of to increase sense of
control impulses wills control
Very concerned Provide detailed
with right/ information and
wrong “homework”
Foster collaborative
approach/avoid
battle of wills
MASOCHISTIC “Perpetual victim” Ego-syntonic Anger, hate, Avoid excessive
Self-sacrificing Conscious or frustration encouragement
martyr unconscious Helplessness, self- Share patient’s
punishment doubt pessimism
Deemphasize
connection between
symptoms and
frequent visits
Suggest that patient
consider treatment
as another burden to
endure, or
emphasize
treatment’s positive
effect on loved ones
HISTRIONIC Entertaining Loss of love or Anxiety, impatience, Strike a balance
Melodramatic loss of off-putting between warmth
Seductive, attractiveness Erotic; finds patient and formality
flirtatious attractive Maintain clear
boundaries
PARANOID Guarded, Proof that world Anger, feeling Avoid defensive
distrustful is against patient attacked or accused stance

Quick to blame or Medical care is May become Acknowledge


invasive and defensive patient’s feelings
counterattack without disputing
exploitative them
Sensitive to slights
Maintain
interpersonal
distance; avoid
excessive warmth
Do not confront
irrational fears

NARCISSISTIC Arrogant, Threat to self- Anger, desire to Resist the desire to


devaluing concept of counterattack challenge patient’s
perfection entitlement
Vain, demanding Activation of feelings
and of Reframe entitlement
invulnerability to foster treatment
inferiority, or adherence
Shame evoking enjoyment of
Take a humble
feeling of status of stance, provide
working opportunities for
with an important patient
patient to show off, offer
consultations if
appropriate

SCHIZOID Aloof, remote Fear of intrusion Little connection to Respect patient’s


patient privacy
Socially awkward
Difficult to engage Prevent patient from
Inhibited completely
withdrawing

Maintain gentle,
quiet interest in
patient

Encourage routine
and regularity

DSM-5 ICD-10
Somatisation disorder, undifferentiated somatoform
Somatic symptom disorder
disorder, persistent somatoform pain disorder
Conversion disorder Dissociative
Illness anxiety disorder Hypochondriacal disorder
Somatoform autonomic dysfunction, other somatoform
Factitious disorder
disorders, somatoform disorders unspecified, neurasthenia
Somatoform disorder NEC
Psychological factors affecting
another medical condition

Topic: Somatoform disorders

1) Which of the following is not required to diagnose of somatization disorder in ICD-10?


a) 2 years duration of unexplained symptoms
b) History of pain related to 4 different sites or functions
c) Refusal to accept advice regarding lack of a physical explanation
d) Socio-occupational impairment
Ans: History of pain related to 4 different sites or functions is not an ICD-10 criterion for diagnosis
of somatization disorder. It was a criterion in DSM IV, which has been removed subsequently in
DSM-5. All the other 3 options are required for a definitive diagnosis as per ICD-10. Of interest is
the requirement for some degree of socio-occupational impairment, which is not usually
highlighted in most diagnoses in ICD-10, but is a predominant part of DSM-5 diagnoses

2) As per ICD-10, which of the following does not help in contrasting hypochondriacal disorders
from somatization disorder
a) Emphasis on disorder(s) > emphasis on individual symptoms
b) No marked sex differential rate
c) No special familial characteristics
d) Persistent refusal to accept the advice and assurance of several doctors

Ans: Persistent refusal to accept advice and assurance of several different doctors is a feature
shared by all somatoform disorders. All the other options are characteristics of hypochondriasis
as per ICD-10.

3) Body dysmorphic disorder is included under which of the following in ICD-10


a) Somatization disorder
b) Hypochondriacal disorders
c) Obsessive compulsive and related disorders
d) Delusional disorders
e) Undifferentiated somatoform disorders

4) Body dysmorphic disorder is included under which of the following in DSM-5


a) Somatization disorder
b) Hypochondriacal disorders
c) Obsessive compulsive and related disorders
d) Delusional disorders
e) Undifferentiated somatoform disorders

Ans: Body dysmorphic disorder is included under Hypochondriacal disorders in ICD-10 and OCRDs
in DSM-5.

5) Da costa’s syndrome, neurocirculatory asthenia, diarrhoea gas syndrome are diagnoses


included under which category in ICD-10
a) Somatization disorder
b) Undifferentiated somatoform disorders
c) Somatoform autonomic dysfunction
d) Hypochondriacal disorders
e)
Ans: All the above are classified under the category of somatoform autonomic dysfunction in ICD-
10. Diagnostic guidelines for somatoform autonomic dysfunction in ICD-10 are as follows:
(a) Symptoms of autonomic arousal, such as palpitations, sweating, tremor, flushing, which are
persistent and troublesome

(b) Additional subjective symptoms referred to a specific organ or system; (c)preoccupation with
and distress about the possibility of a serious (but often unspecified) disorder of the stated organ
or system, which does not respond to repeated explanation and reassurance by doctors

(d) No evidence of a significant disturbance of structure or function of the stated system or organ.

Topic: Landmark studies in Psychiatry


Q: Can any one suggest few Indian studies for bipolar disorder ?

Q: Which blood group is co related to manic as per study by Gurmeet Singh et al ?

a) A blood group
b) B blood group
c) Bombay Blood group
d) O blood group
e) Mixed type blood group
Ans : O Blood group

Q: Mood stabilizer propagated by most of the studies in bipolar disorder of special


population (Children, adolescents , pregnancy and old age ) is ?

a) Lithium
b) Valproate
c) Carbamazepine
d) Oxcarbazepine
Ans : Lithium

Q: Which of the following are not risk factors associated with suicidality in Bipolar disorder ?

a) Depression as the first episode,


b) Severity of illness, latency,
c) No. Of hospitalizations,
d) No. Of total mood episodes,
e) No. Of depressive episodes,
f) No. Of mixed episodes
g) Positive familial psychiatric disorder history
h) Cooperativenesss

Ans : Cooperativeness

The term Bipolar disorder was coined by :

A) Soranius
B) Kraeplin
C) Kleist
Ans : Kleist

Q Mean age for onset of bipolar disorder is :

a) 21 years
b) 32 years
c) 45 years
d) 52 years
Ans : 21 years

Q : Which of the following statements are true regarding the impact of medicines on
course and outcome of bipolar disorder ?

a) With medication it has been possible to retard the progression of illness by


reducing number and duration of episodes.
b) Florid symptoms are also less likely to be seen with proper medication.
c) The higher the number of episodes greater is the risk of future episodes and
poorer the prognosis.
d) A, b and c are true

Answer : a,b,c are true

Q : Which of the following are true regarding indian studies regarding bipolar disorder
a ) In India greater frequency of manic than depressive relapses is noted
b) In India greater frequency of depressive than manic relapses is noted

c) Patients with first episode of mania and having a recurrence tended to have highernumber
of manic episodes.

d) Both a and c
Q : Which of the following is true regarding course and outcome of bipolar disorder asper evidence ?

A ) It has been seen that median duration and time taken for recovery from manic
/hypo manic episodes to be shorter then depressive episodes.
b) It has been seen that median duration and time taken for recovery from depressive
/dysthymic manic episodes to be shorter then manic episode .

c) Severity for depressive episodes was more in bipolar I than II disorder.

Answer ) It has been seen that median duration and time taken for recovery from manic
/hypo manic episodes to be shorter then depressive episodes.

Q: Which of the following are likely to present with index depressive episode ?
A ) Males
B ) Females
Answer : Females

Q: Medical illnesses associated with bipolar disorder are :


A ) Migraine
B )Cluster headache
C ) Malnutrition
D ) None

Ans : Migraine ( 6 times )

Q: Recovery of Bipolar disorder in children is faster in

a) Indian population
b) Western population

Answer : Indian Children

Q: Bipolar affective disorders in ICD 10 are known as the following in ICD 11 ?

A) Bipolar disorders
B) Bipolar and related disorders
C) Bipolar or related disorders
D) Manic depressive illness
Ans Bipolar or related disorders

Q : Which of the following according to ICD 11 are not required to make a diagnosis ofmanic
episode ?

a) Elevated mood
b) Increased energy levels
c) Subjective experience of increased energy
d) Euphroria
Ans : Elevated mood

Q Which of the following is minimal requirement by ICD 11 to code Bipolar 1 disorder Answer :
A ) Single manic episode lasting for a week
B ) Single depressive episode lasting for 2 weeks
C ) Any one between a and b
D ) Both should be present in complete duration to make a diagnosis

Ans : Single manic episode lasting for a week

Q : Females tend to experience :


A ) More depressive episode
B ) More manic episodes

Answer : a)

Q: Marriage alters course of illnesses in :

a) Males
b) Females
Answer)Female

Q: According to evidence, the manic episodes in females


A ) Euphoric
B ) Dysphoric

Answer : Dysphoric

1. Important epidemiological studies from india are all except


a) IPSS
b) DOSMed
c) SOFACOS
d) CATIe

2. Why do we need to know about mental health surveys ?

a. Mental health survey help to plan, develop, implement, monitor, evaluate and
strengthen services
b. Understand the burden of disease
c. Might come in exams and viva
d. Not imortant to know
3. Epidemiological studies sponsored by WHO are all except

a. DOSMed
b. IPSS
c. ISos
d. Long term cohort of Agra
4. Prevalence of any type of mental disorders in India is

a. 10.6%
b. 8
c. 12
d. 2
5. Prevalence of mental morbidity is high in Indian

a. Urban metros
b. Sub urban
c. Rural
6. Prevalence of common mental disorders and severe mental disorders are ( percentage

a. 10 and 0.8
b. 12 and 2
c. 20 and 10
7. 1%population reported high suicidal risk. The characteristics of the population was all
except

a. Urban living
b. Widow or separated
c. Low income
d. Primary education
e. 40-49
f. Males
8. Efficacy studies DO NOT evaluate performance of an intervention

a. Under controlled situations


b. Under ideal situations
c. Under real world situations
d. Under experimental situations
9. Catie trial was done in

a. UK
b. USA
c. UK and USA
10. Catie n Cutlass were

a. Efficiency trials
b. Efficacy trials
c. Effectiveness trials
11. These two trials definitely prove that

a. First generation is superior second generation antipsychotics


b. Second generation is superior to first generation antipsychotics
c. Could not comment on definite superiority of either
12.A landmark study conducted by Faris and Dunham related to social class and mental illness
is called as

a. Illinois study
b. Chicago study
c. Texas study
d. Washington study
13. Extrapyramidal symptoms are lowest in

a. Olanzapine
b. Ziprasidone
c. Quetiapine

14. QTc prolongation is highest in

a. Risperidone
b. Quetiapine
c. Ziprasidone
d. Haloperidol
15. Positive remark is a statement which expresses

a. Praise of the behaviour or personality of the patient


b. Approval of the behaviour or personality of the patient
c. Appreciation of the behaviour or personality of the patient
d. All of the above
16. What were the inclusion criteria for STAR D trial?

a. Age 18-75
b. HDRS17 score>14 (Moderate depression)
c. Outpatients only
d. Patients must have sought treatment (referred through physician not advertisement
17. For how long must 50 % improvement (as measured on HDRS) last for us to say that a
person suffering form depression has responded to treatment

a. 1 week
b. 2 weeks
c. 3 weeks
d. 4 weeks
18. Which of the following was identified as the most common neurological complication of
COVID19?

a. Altered mental status


b. Cerebrovascular event
c. Peripheral neurology
d. All of the above
19. Which of the following practises of psychiatry have been significantly affected by the
COVID-19 pandemic?

a. Increased use of Telepsychiatry


b. Preparedness for a mental health pandemic
c. Conducting research in Psychiatry (such as adapting digital technologies)
d. Increase in consultation liaison psychiatry referrals
e. All of the above
f. None of the above
20. Which are the most common psychological reactions to COVID-19 pandemic?

a. Mania
b. Psychosis
c. Anxiety and Depression
d. All of the above

1. Most common substance of abuse


a. Tobacco
b. Alcohol
c. Cannabis
d. Others
2. What proportion of people in India suffer from depression

a. 1 in 100
b. 1 in 10
c. 1 in 50
d. 1 in 20
3. Prevalence of common mental disorders and severe mental disorders are

a. 10 and 0.8
b. 12 and 2
c. 20 and 10
4. In Catie, which drug showed lowest discontinuation rates

a. Olanzapine
b. Risperidone
c. Perphenazine
d. Quetiapine
e. Ziprasidone
5.In Catie, which drug showed the greatest reduction in psychopathology

a. Olanzapine
b. Risperidone
c. Perphenazine
d. Quetiapine
e. Ziprasidone
6. Metabolic syndrome in schizophrenia is mostly associated with

a. Olanzapine
b. Risperidone
c. Perphenazine
d. Quetiapine
e. Ziprasidone
7.Drift hypothesis suggests that those with schizophrenia

a. Will move down the socio-economic ladder


b. Will move up the socio-economic ladder
c. Will remain at the same place
8. Incidence of schizophrenia varies by gender

a. True
b. False
9. In age group of less than 40 years , risk of schizophrenia is more in

a. Males
b. Females
10. Lifetime risk in schizophrenia is

a. More in males
b. More in females
c. Equal
11. Which group has more neuroleptic induced agranulocytosis

a. Younger men
b. Elderly men
c. Younger women
d. Elderly women
12. Extrapyramidal symptoms are highest in

a. High potency first generation antipsychotics


b. Middle potency first generation antipsychotics
c. Low potency first generation antipsychotics
13. Prolactin elevation AND sexual side effects are highest in

a. Risperidone
b. First generation antipsychotics
c. Ziprasidone
d. Quetiapine
e. Olanzapine
14. In Expressed Emotions (EE), Hostility is defined as either EXCEPT

a. Generalisation of criticism
b. Rejection of the patient as a person
c. Combination of generalisation and rejection
d. Aggressive arguments with threat to bodily harm
15. All of the following were factors found to be associated with illness burden, except

a. Early age of onset


b. Cannabis use
c. Smoking
d. Anxiety
16. Full form of STEP BD is

a. Systematic Treatment Engagement Program for Bipolar Disorder


b. Systematic Treatment Enhancement Program for Bipolar Disorder
c. Systematic Treatment Engagement Program for Bipolar Depression
d. Systematic Treatment Enhancement Program for Bipolar Depression
17. STEP BD was conducted from

a. 1990 to 2002
b. 1998 to 2005
c. 2001 to 2010
d. 2005 to 2013
18. The term 'Bipolar' was coined by ?

a. Soranius
b. Kraeplin
c. Kliest
19.Which of the following are not risk factors associated with suicidality in Bipolar disorder ?

a. Depression as the first episode


b. Severity of illness,
c. Latency
d. No. Of depressive episodes
e. No of mixed episodes
f. Cooperativeness
20.Which of the following are likely to present with index depressive episode ?

a. Males
b. Females

Topic: Long Acting Injectable antipsychotics


1. All of the following are drawbacks of FGA LAI as compared to SGA LAI, except
a. More propensity to cause EPS
b. Possible hypersensitivity to base oil
c. No test dose is required
d. Optimal dose range is not known

(For FGAs, a test dose consisting of a small dose of active drug in a small volume of oil serves a
dual purpose: it is a test both of the patient’s sensitivity to EPS and of any sensitivity to the base
oil. For SGAs, test doses may not be required less propensity to cause EPS and aqueous base not
known to be allergenic)

2. Steady state plasma levels with LAI are generally achieved after atleast _____ weeks
a. 2
b. 2-4
c. 4-6
d. 6-8
e. 8-10

(without increment in dose. To reach steady state it takes at least 6–8 weeks.)

3. One of these LAI is more effective than others in preventing relapses


a. Flupentixol
b. Zuclopenthixol
c. Haloperidol
d. Fluphenazine

(As per Cochrane systematic review, Zuclopenthixol is more effective than other FGA LAI)

4. Which one of the following SGA LAI is associated with post-injection syndrome
a. Risperidone
b. Olanzapine
c. Paliperidone
d. Aripiprazole

(Post-injection syndrome is caused due to accidental intravasation, and manifests when plasma
levels reach 600 μg/L and leads to delirium and somnolence)

5. Administration of intramuscular procyclidine is recommended along with LAI


a. True
b. False

(The administration of IM procyclidine routinely is illogical as the effects of the anticholinergic


drug will wear off before plasma antipsychotic levels rise or peak)

6. Consistent RCT evidence shows that LAI have better efficacy and tolerability as compared to
oral preparations
a. True
b. False

(There is evidence, but it is not robust and consistent that LAI has better efficacy and tolerability)

7. Which one of the following LAI can has maximum dosing interval
a. Aripiprazole
b. Olanzapine
c. Risperidone
d. Paliperidone

(We have Paliperidone palmitate 3 monthly preparation to which patients receiving Paliperidone
palmitate monthly, can be shifted for further ease of administration)

8. For haloperidol decanoate, peak plasma level is achieved within ____ days
a. 5
b. 7
c. 10
d. 14
e. 28

(Peak plasma levels for Olanzapine pamoate is achieved in 2-3 days, and has shortest duration to
reach the peak, but at the max it can be reached within 7 days, which is same duration as that of
Aripiprazole, flupentixol and haloperidol)

9. Plasma half life is least for which of the following LAI


a. Haloperidol
b. Fluphenazine
c. Risperidone
d. Olanzapine
e. Aripiprazole

(Plasma half life for Risperidone is least which is only 4 days as compared to others: Haloperidol
– 21 days, fluphenazine – 10 days, olanzapine – 30 days and aripiprazole 30 – 46 days)

10. To reach maintenance dose, it is preferable to reduce dose of LAI every ____ months
a. 1
b. 3
c. 4
d. 6

(It is preferable to reduce dose every 6 months and no more frequently than 3 months because
slower the rate of withdrawal, longer the time for relapse)

11. Which of the following LAI has pro-drug formulation?


a. Risperidone
b. Olanzapine
c. Paliperidone
d. Aripiprazole

(Aripiprazole lauroxil is a pro‐drug formulated to be administered at monthly, 6 weekly or 2‐


monthly intervals by IM injection into the deltoid or gluteal muscle depending on the dose)

12. Almost all reactions due to post-injection syndrome occur within ___ hour/s
a. ½
b. 1
c. 2
d. 3

(most of the reactions occur in first hour and observation is mandatory for 3 hours)

13. Which one of the following statements is false pertaining to Aripiprazole LAI
a. Oral form is administered 14 days before administering injectable form
b. Oral form is continued for 14 days after the first injection
c. Peak plasma levels are 50% higher during the first dose compared to steady state
d. Peak plasma levels are 50% higher during steady state as compared to first dose

14. Which one of the following statements is not true


a. Oral 4mg/d risperidone is equivalent to injectable risperidone 50mg/2 weeks
b. Oral 4mg/d paliperidone is equivalent to injectable paliperidone 50mg/month
c. Oral 6mg/d paliperidone is equivalent to injectable paliperidone 150 mg/month
d. Oral 6 mg/d risperidone is equivalent to injectable paliperidone 150 mg/month

15. All of the following are aimed while using rapid tranquilisation, except
a. Reduce self harm to patient(psychological + physical)
b. Reduce harm to others
c. Doing no harm by the treatment options itself
d. Considering long term benefit of the drug used

16. Comment on the effectiveness of intramuscular administration of antipsychotics for rapid


tranquilization
a. Aripiprazole>Olanzapine>Haloperidol
b. Olanzapine>Haloperidol>Aripiprazole
c. Haloperidol>Aripiprazole>Olanzapine
d. Haloperidol>Olanzapine>Aripiprazole

17. All of the following statements are true regarding rapid tranquilization, except
a. Olanzapine 10 mg is as effective as haloperidol 10mg and promethazine 25-50mg
b. Combination of haloperidol 5-10mg and promethazine 50mg is more effective and better
tolerated than only haloperidol 5-10mg
c. Combination of haloperidol 5-10 mg and promethazine 25-50mg is more effective than
lorazepam 4mg
d. Olanzapine is more effective than aripiprazole in first 24 hours

18. In acute psychiatry settings high doses sedation does not seem to be more effective than
lower doses
a. True
b. False

19. Zuclopenthixol Acetate has better than Olanzapine for Rapid Tranquilization
a. True
b. False

20. Flumazenil is administered after respiratory rate falls below ____/minute after use of
benzodiazepines
a. 6
b. 8
c. 10
d. 12

21. Maximum dose of flumazenil that can be given over 24 hours


a. 1 mg
b. 2 mg
c. 3 mg
d. 4 mg

22. There is some evidence that shows that Risperidone LAI is associated with stable white matter
volume as compared to oral risperidone
a. True
b. False
c. Cannot comment

23. LAI are recommended in involuntary admissions for First episode Schizophrenia
a. True
b. False
c. Cannot comment

24. As per Kane & Garcia Ribera


a. Any patient for whom long-term treatment is recommended should be considered
for LAI
b. LAI should be considered only for the patients with recurrent relapses related to partial or full
nonadherence
c. LAI can be considered for patients who are inadequately adherent at any stage
d. LAI can be considered for patients who would prefer such treatment

25. Select statements that point towards the indication to start LAI early
a. LAI has lower relapse rate
b. LAI help to prevent disease progression associated with poor adherence in the early stage of
Schizophrenia
c. Patient prefers a depot preparation
d. All of the above

26. All of the following statements are correct regardin LAI as compared to oral medications,
except
a. To prevent daily administration of drug and hence improves compliance
b. LAI provide more stable steady state concentration
c. LAI may increase possibility of side effects
d. It has a better bioavailability
e. Time to relapse is longer
27. Injection site reaction is minimum with which of the following LAI
a. Risperidone
b. Olanzapine
c. Paliperidone
d. Aripiprazole

28. Which of the following requires minimum time to reach steady state concentration
a. Haloperidol Decanoate
b. Fluphenazine Decanoate
c. Zuclopenthixol Decanoate
d. Olanzapine Pamoate

29. Switching to LAI is appropriate during all stated below instances, except
a. Comorbid substance use
b. Persistent symptoms
c. Preserved insight
d. Lack of routines
e. Lack of family/social support

30. Currently LAI seems to be more cost effective as compared to oral medications due to larger
reduction in indirect costs
a. True
b. False
c. Cannot comment

Topic: Medically Unexplained Symptoms

1. MUS- proposed alternative nomenclature includes all except


a. Persistent physical symptoms (PSS)
b. Functional symptoms/syndromes (FS)
c. Physiologically explained symptoms (PES)
d. Symptom defined illness

ANS: D

Although the term MUS is commonly used in healthcare practice and academic articles it is
unsatisfactory for a number of reasons.

➢ It fails to engage patients in treatments as patients feel it invalidates their symptom


experience and infers that their symptoms are 'all in the
➢ It reinforces dualistic thinking and the idea that illness is either biological or
psychological.
➢ The term defines the illness by what it is not: i.e. it implies no organic cause which is not
necessarily accurate and affords no treatment utility.
➢ Research has shown that most patients want a positive description of symptoms i.e. an
explanation of what it is rather than what is isn’t
➢ The term may appear dismissive and provides the message that nothing can be done.

Other terms in use which appear more acceptable to patients include persistent physical
symptoms or functional syndromes/symptoms (FS). The term “functional” here is used because
it is assumed that the disorder is one of function, which may be physical and/or psychosocial
function, rather than anatomical structure. More recently, physiologically explained symptoms
have also proposed as an alternative

2. What percentage of patients presenting to primary care can have MUS?


a. Up to 50%
b. <10%
c. 20-30%
d. >50%
ANS A

3. MUS is different from somatization.


a. True
b. False

ANS: False

ICD-10 and DSM-IV have offered alternative labels, such as ‘somatisation’, ‘unexplained somatic
complaints’, ‘somatoform disorders’ and ‘somatisation disorder’. DSM-5 has recently replaced
the diagnosis of ‘MUS’ and now refers to ‘Somatic Symptom Disorder’ (SSD)

4. DSM 5 ‘Somatic Symptom and Related Disorders’ includes all except


a. SSD
b. Illness anxiety disorder
c. Dissociative disorders
d. Conversion disorders

ANS: C

The disorders under the rubric of Somatic Symptom and Related Disorders in DSM 5 includes:
SSD, Illness Anxiety Disorder, Conversion (Functional Neurological Symptom Disorder),
Psychological Factors Affecting Other Medical Conditions, Factitious Disorder, Other Somatic
Symptom and Related Disorders and Somatic Symptom and Related Disorders NOS

5. The following are included in Neurotic and Stress Related Disorders (F40) chapter in
ICD 10 except
a. Dissociative disorders
b. Hypochondriasis
c. Conversion
d. Somatization
ANS: C

Conversion disorders are not separately classified by the ICD, as in DSM, and are instead
covered under dissociative disorders (F44).

6. Conversion disorder
a. usually has a chronic course
b. is associated with antisocial personality disorder
c. is commonly comorbid with a schizoid personality disorder
d. responds well to a confrontation of the “false nature” of the symptoms
e. is associated with symptoms that conform to known anatomical pathways

ANS: B

7. Which of the following statements regarding conversion disorder and gender


differences is true?
a. Women are often involved in occupational accidents.
b. Symptoms are more common on the right side than the left side of the body in
women.
c. The ratio of women to men among adult patients is as high as 10:1.
d. In children, there is a higher predominance in boys.
e. There is an association with borderline personality disorder in men.
ANS: C

8. Conversion disorder differs from somatization disorder in that:


a. Conversion disorder includes symptoms in many organ systems
b. somatization disorder begins early in life
c. complaints are limited to pain in conversion disorder
d. Complaints are not limited to neurological symptoms in conversion disorder
e. conversion disorder involves a particular disease rather than a symptom

ANS: B

9. All of the following mental disorders are frequently seen in patients with somatization
disorder (relative to the general population) except
a. bipolar I disorder
b. generalized anxiety disorder
c. major depressive disorder
d. obsessive-compulsive personality disorder
e. schizophrenia

ANS: A

10. Which of the following is not a recommended treatment strategy for a patient with
somatization disorder?
a. Increasing the patient’s awareness that psychological factors may be involved
b. Having several different clinicians involved in caring for the patient
c. Avoiding additional laboratory and diagnostic procedures
d. Seeing patients during regularly scheduled visits at regular intervals
e. Listening to somatic complaints as emotional expressions rather than medical
complaints
ANS: B

11. The most accurate statement regarding pain disorder is


a. It is diagnosed equally among men and women.
b. Peak ages of onset are in the second and third decades.
c. It is least common in persons with blue-collar occupations.
d. First-degree relatives of patients have an increased likelihood of having the same
disorder.
e. Depressive disorders are no more common in patients with pain disorder than in
the general public
ANS: D

12. People with hypochondriasis


a. are usually women
b. are often thanatophobic
c. do not respond to reassurance
d. seek treatment more than explanations
e. in postmortem examinations, have a greater degree of upper gastrointestinal
(GI) inflammation and congestion than normal control subjects
ANS: B

13. True statements about hypochondriasis include all of the following except
a. Depression accounts for a major part of the total picture hypochondriasis.
b. Hypochondriasis symptoms can be part of dysthymic disorders, generalized
anxiety disorder, or adjustment disorder.
c. Hypochondriasis is a chronic and somewhat disabling disorder.
d. Recent estimates are that 4 to 6 percent of the general medical population
meets the specific criteria for the disorder.
e. Significant numbers of patients with hypochondriasis report traumatic sexual
contacts, physical violence, and major parental upheaval before the age of 17
years.
ANS: A

14. In body dysmorphic disorder


a. A. plastic surgery is usually beneficial
b. a comorbid diagnosis is unusual
c. anorexia nervosa may also be diagnosed
d. about 50 percent of patients may attempt suicide
e. serotonin-specific drugs are effective in reducing the symptoms
ANS: E

Topic: Psychocutaneous Disorders

1. Koo and Lee’s classification of PCD (2003) includes all the except
a. Psychophysiologic disorders
b. Psychiatric disorders with dermatological symptoms
c. Dermatological disorders with psychiatric symptoms
d. Mixed psychodermatological disorders
ANS: D
Broadly, PCD can be classified as follows, based either on the nature of the lesion, or the
underlying psychopathology.

One of the most widely used classificatory systems for PCD is Koo and Lee’s classification, given
below

CATEGORY NATURE OF DISORDER EXAMPLES


PSYCHOPHYSIOLOGIC Here, psychiatric disorders are Psoriasis
DISORDERS instrumental in the aetiology and Atopic dermatitis
course of skin conditions. Acne excoriee
Patients have bona fide organic Hyperhidrosis
dermatological conditions that are Urticaria
frequently exacerbated by emotional HSV infection
stress Seborrheic dermatitis
Aphthosis
Rosacea
Pruritus
PSYCHIATRIC DISORDERSPatients present with skin lesions that Dermatitis artefacta
WITH DERMATOLOGIC are self-induced in the absence of Delusional parasitosis
SYMPTOMS or PRIMARYorganic etiology. Trichotillomania
PSYCHIATRIC These patients have normal, non- OCD
DOSOSRDERS diseased skin, hair, and nails. Phobic states
Although their symptoms are the Dysmorphophobia
manifestation of the patient’s Eating disorders
underlying Neurotic excoriation
psychiatric condition, patients are less Psychogenic pruritus
likely to accept referral to a
psychiatrist or to willfully take
psychiatric medications.
DERMATOLOGIC This category includes patients who Alopecia areata
DISORDERS WITH have emotional problems secondary Vitiligo
PSYCHIATRIC to skin disease Chronic eczema
SYMPTOMS or Ichthyosiform disorders
SECONDRAY Rhinophyma
PSYCHIATRIC DISORDERS Neurofibroma
Albinism
CUTANEOUS SENSORY Cutaneous sensory disorders are Chronic idiopathic pruritus
DISORDERS conditions in which the patient Idiopathic pruritus ani,
experiences various abnormal vulvae, scroti
sensations on the skin. Idiopathic glossodynia
Dysesthetic (essential)
vulvodynia
2. What percentage of patients seeking treatment for skin disorders have an underlying
psychiatric or a psychological problem that either causes or exacerbates a skin
complaint?
a. 30-40%
b. 25-30%
c. 8-10%
d. Around 50%
ANS: A

3. CNS and skin share a common embryological origin form the endoderm
a. True
b. False

ANS: False.

They share a common origin from neural ectoderm plate

4. Allergic Object Relationship is:


a. Unusual attachment of individuals with skin disease to an object which is
perceived to alleviate symptoms of skin disease
b. In infants with skin disease, separation from the mother results in exacerbation
of symptoms of skin disease
c. Tendency to associate particular objects with exacerbation of symptoms of skin
disease
d. None of the above

ANS: B

Childhood skin diseases like atopic dermatitis can influence a child’s psychological development
and personality. An infant with unhealthy skin may experience two contrary emotional stimuli
from the skin: the nurturing attention gained from stroking and massaging with skin ointments,
and at the same time the sensations of pain, itching, and burning from skin lesions (caused by
too much pressure on skin lesions and other environmental discomforts). The presence of the
mother and her applying ointment is experienced as the only source of relief and comfort,
leading to fixation on the maternal object, so that separation from the mother is experienced as
a threat and leads to exacerbation in perceptions of pain, itching, and discomfort. This
psychodynamic model is termed as allergic object relationship.
5. The following is false about attachment and skin disease
a. Dermatological Shame influences attachment in intimate relationships
b. Individuals with avoidant attachment style have exaggerated displays of
emotional distress to cope with their shame
c. Individuals with secure attachment have disease specific shame
d. Individuals with anxious ambivalent attachment have self schemas of disgust and
social stigma
ANS: B

The attachment framework elucidates why some individuals with a skin disorder use an
intimate relationship as a support to deal with disease-related stress, like feelings of shame and
sexual problems, while other individuals struggle to communicate or avoid sharing their
distress. S. Kellet described patient responses to disfigurements as a specific form of body
shame and termed it “dermatological shame.” The focus is on the appearance of skin and
relative attractiveness. In individuals with secure attachment the dermatological shame may be
disease-specific, mostly confined to the disease itself, and not generalized to self-schema (belief
about self). Such individuals may be more accepting of the disease and accept their physical
and psychosocial issues in a healthy way. The anxious ambivalent person may experience more
pervasive generalized shame, which merges with self-schema to create an inner experience of
self-disgust, negative beliefs and social stigma. Avoidant persons are more likely to minimize
display of emotional distress in order to hide their shame.

6. The following cells are found in the skin


a. Langerhans cells
b. Macrophages
c. Histiocytes
d. Keratinocytes
e. Merkel cells
f. Glial cells
g. Dendritic cells
h. Dendrocytes
i. Melanocytes
ANS: A, B, D, E, G, I

7. Following neuropeptides are expressed in skin


a. Substance P (SP)
b. Neurokinin A (NKA)
c. Bradykinin
d. Vasoactive Intestinal Peptide (VIP)
e. Somatostatin (SOM)
f. Calcitonin Gene Related Peptide (CGRP)
g. Neuropeptide Y (NPY)
h. Nitric Oxide (NO)
i. Atrial Natruiretic Peptide (ANP)
j. Brain Natriuretic Peptide (BNP)
k. Neurotensin
ANS: A, B, D, E, F, G, H, I, K

(List incomplete, several more are known, some are putative)

8. Following hormones are synthesized/released by skin cells


a. Vitamin D
b. Parathyroid hormone (PTH)
c. ACTH
d. Oxytocin
e. ADH
f. MSH

ANS: A, C, F.

(List incomplete, several more are known, some are putative)

9. Neuropeptides are secreted from


a. C fibres
b. A fibres
ANS: A

Neuropeptides are secreted by unmyelinated C fibres

10. Relation between skin and sleep:


a. Chronic poor sleep quality is associated with increased signs of intrinsic skin
ageing
b. Diurnal pattern of pruritus, with worsening at night
c. Diurnal pattern of pruritus, with worsening in the morning
d. Impaired thermoregulation due to skin disorder leads to increased sedation
e. Transepidermal water loss (TEWL) affects circadian rhythm
f. There is cessation of pruritus and scratching during sleep
g. Sleep scratching can occur during certain stages of sleep
h. Sleep scratching occurs during all stages of sleep
ANS: A, B, E, G

Pruritus can have a diurnal pattern due to:

➢ Higher TEWL in the evening with a compromised epidermal barrier function is


associated with greater itch intensity, which is typically higher in the evening before
bedtime.
➢ Complex circadian-mediated factors including lower cortisol levels, increased skin
temperature, and decline in epidermal barrier function
If thermoregulatory function is impaired as a result of a skin disorder and heat is not dissipated
normally, sleep onset may be prolonged and the patient may experience a decline in restorative
sleep and poor health outcomes

Scratching during sleep, which may vary during different stages of sleep depending upon
sympathetic nervous system activity, usually occurs more frequently during non-rapid eye
movement (NREM) stages 1 and 2 and REM sleep, and least in deep sleep (stage N3) with
lowest sympathetic tone.

11. Highest incidence of Atopic Dermatitis (AD) is during


a. First year of life
b. Second year of life
c. Puberty
d. Third decade of life
ANS: A

The AD manifests in first year of life in about 60 percent of cases, and only 30 percent in the
subsequent 1 year, and is rare after puberty

12. Following is true about genetic factors in AD


a. Mendelian inheritance
b. Polygenic inheritance
c. 30% of patients have positive family history
d. Defective humoral immunity and inceased IgE production
e. Chromosomes 5q and 11q implicated.
ANS: B, E

Patients with AD have a genetic predisposition, and about two-thirds of them have a positive
family history of AD. The concordance rates for monozygotic twins (0.75) is higher than dizygotic
(0.25) twins. The inheritance is considered to be polygenic and multifactorial.

Chromosomes 5q331-33 have genes for several important cytokines (IL-3, IL-4, IL-5, IL-13). There
is some evidence for existence of an atopy gene in chromosomal region 11q13

The chromosomal inheritable traits in AD have been implicated for HPA axis abnormalities and
imbalance of immunoregulatory T-lymphocyte system, leading to defective cell-mediated
immunity and increased IgE production.

13. Stages of AD are all except:


a. Acute
b. Chronic
c. Uninvolved
d. Prodromal

ANS: D

In uninvolved skin the cutaneous microenvironment favors a humoral immunity with


predominance of mast cells, eosinophils, IgE, and circulating TH2 lymphocytes.

In the acute immune response, serum IgE plays a major role in AD pathogenesis and binds to mast
cells, basophils, and Langerhans cells. An acute response is triggered by IgE-mediated inciting
antigen

A persistence of irritative mechanical (scratching) and inflammatory stimuli can result in the
chronic phase. The chronic CMI response is mediated by TH1 cells, which produce cytokine IFN-
γ.

14. Following are true:


a. Individuals with AD have been found to have increased anxiety and depression
b. Individuals with AD have been found to have increased impulse control disorders
c. Serum IgE level in AD patients affects coping
d. NK-cell activity and IL-4 levels are higher in AD patients with depression or
anxiety
e. Higher anxiety levels in AD patients are correlated with ease of conditioning to
develop itch-scratch response
ANS: A, C, E
AD patients with serum IgE levels greater than 100 IU/Ml have increased traits of excitability
and inadequate coping compared to other AD patients

NK-cell activity and IL-4 levels are lower in AD patients with depression or anxiety, and IFN-γ
levels are higher in AD patients than in controls. Decreased NK-cell activity is more strongly
correlated with higher anxiety levels.

15. The following are features of psoriasis


a. Erythematous, scaly patches and plaques
b. Flexor surfaces involved
c. Koebner phenomenon
d. Mouth is often involved
e. Salmon spots
f. Auspitz sign
g. Asymmetrical oligoarthritis
h. PIP > DIP joints involved
i. Sero negative arthritis
j. Negative family history in most cases
ANS: A, C, E, F, G,

Psoriasis commonly involves the extensor surfaces, arising preferentially on the elbows, knees,
and scalp.

Mouth involvement is very rare in Psoriasis, as opposed to Lichen Planus

Psoriatic arthritis is seen in about 30% of patients with psoriasis, and the most common
presentation is an asymmetrical oligoarthritis (<3 joints) (70%). DIP joints are characteristically
affected.

16. The following is false about HIV and psoriasis


a. Higher incidence of psoriasis in HIV positive patients
b. More severe course of psoriasis in HIV positive patients
c. HIV positive psoriasis patients are more prone to develop arthritis
d. Psoriasis in HIV positive patients may become resistant to conventional therapy
ANS: A

17. Psoriasis has a bidirectional relationship with depression


a. True
b. False
ANS: True

18. All have evidence for significant clinical improvement in psoriasis except
a. Pharmacotherapy
b. CBT
c. Habit reversal training (HRT)
d. Arousal reducing techniques
ANS: C

The psychotherapeutic modalities with evidence for significant clinical improvement include
arousal reduction techniques like hypnosis, mindfulness meditation and guided imagery
training.

Cognitive–behavioral therapy may be of specific value in improving individual coping strategies


by addressing illness perceptions, beliefs about control and curability of illness and
interpersonal issues.

TNF-α antagonists are safe and effective in improving both the physical severity of psoriasis and
quality of life.

19. Psychopharmacological treatment is highly recommended in psoriasis


a. True
b. False
ANS: False

There is limited data on controlled trials of psychopharmacological treatments of anxiety and


depression in psoriasis patients and its effect on the course of psoriasis. There is need for caution
about use of psychotropic medications in patients with psoriasis because of several conflicting
case reports of induction or exacerbation of psoriasis with use of fluoxetine and bupropion. Use
of mood stabilizer lithium is commonly associated with precipitation or aggravation of psoriasis.

20. Lithium can cause de novo psoriasis


a. True
b. False
ANS: True

21. Adalimumab can improve depressive symptoms in psoriasis


a. True
b. False
ANS: True
Patients with depression also have increased levels of TNF-α and it is speculated that TNF is
related to fatigue and sleepiness and may account for coexistence of depression with fatigue.
TNF-α antagonists (Adalimumab) may reverse depressive symptoms and fatigue in patients with
psoriasis. Reduction in anxiety and depression scores have also been r eported with use of
ustekinumab (anti-IL-12/23 agent used to treat psoriasis).

22. Severity of depressive symptoms is proportionate to the severity of symptoms/illness


in Acne
a. True
b. False
ANS: False

The severity of acne does not necessarily correlate with severity of depression, and even mild
to- moderate acne has been associated with depression, suicidal ideation, and completed
suicide.

23. Isotretinoin: all true except


a. Black label warning for psychiatric side effects-depression and suicidal ideation
b. Linked to psychosis
c. Interaction with St. John’s wort
d. No interaction with alcohol
ANS: D

Disulfiram like reaction may be seen

24. Delusional parasitosis (DP) is known by the following names except


a. Acarophobia
b. Hypochondriacal psychosis
c. Willis- Ekbom disease
d. Entomophobia
e. Parasitophobia
f. Morgellons
ANS: C

Restless leg syndrome (RLS) is known as Willis- Ekbom disease (WED). DP is known as Ekbom
syndrome

Morgellons is a believed to be a variant of DP where patients believe they have skin lesions/sores
which contain fibres. It has not been scientifically validated as a disease, even after the CDC
undertook extensive research into it. It also believed to be a mass hysteria phenomenon
25. DP in DSM 5 comes under:
a. Delusional disorders
b. OCD
c. Impulse control disorders (ICD)
d. Psychotic disorders
ANS: A

26. Matchbox sign is:


a. Shared delusion seen with DP patients
b. Recurrent impulsive excoriation of skin
c. Formication
d. Bringing the ‘evidence’ of infestation to doctors
ANS: D

27. The subgroups of DP include:


a. Patients with predominantly hypochondriacal traits
b. Patients with paranoid delusions and without hypochondriacal traits
c. Patients with signs of both hypochondriacal as well as paranoid delusions.
d. Patients with DP superimposed on pre-existing psychosis
e. Patients with DP and comorbid ICD
ANS: A, B, C

Patients with predominantly hypochondriacal traits, who are convinced that they have an
incurable physical illness. This is referred to as hypochondriacal delusion of parasitosis or simply
DP. In some cases, those not suffering from organic psychosis, schizophrenia, or affective
disorder may be considered in the group of monosymptomatic hypochondriacal psychosis.

Patients with paranoid delusions and without hypochondriacal traits. They tend to fight against
parasites which impair their existence. This is referred as delusions of infestation. Such patients
consult parasitologists, entomologists, and even pest control companies but seldom see
dermatologists.

Patients with signs of both hypochondriacal as well as paranoid delusions. They are referred as
hypochondriacal delusions of infestation. Such patients usually get repeated dermatological
consultation, like those in the first group, hopelessly trapped in their beliefs

28. The FGA which has greater efficacy in treating DP is


a. Haloperidol
b. Pimozide
c. Trifluoperazine
d. Thioridazine
ANS: B

29. All of the following are true about Pimozide except


a. Increased risk of QT prolongation
b. Less potent than haloperidol
c. Potent opiate antagonist
d. Has anti-pruritic action
ANS: B

Pimozide is more potent than HPL

30. The following are true except:


a. The sensation of an itch to the CNS is mediated by A fibers
b. Itch sensation is modulated by itch-specific receptors
c. There is a separate itch pathway, with ‘second-order histamine-specific itch
neurons’ in the dorsal horn of the spinal cord
d. Cingulate cortex and insula are important in the processing of itch sensation
ANS: A

The sensation of an itch to the CNS is mediated by the nociceptive unmyelinated C-fibers, but
they are functionally different, and modulated by itch-specific C-receptors.

A separate itch pathway has been reported, with ‘second-order histamine-specific itch neurons’
in the dorsal horn of the spinal cord, implicated in neuropathic itch syndromes. From the dorsal
horn, spinothalamic neurons project to the thalamus and anterior cingulate cortex and insula.

31. The following can induce/worsen itch


a. Opioids
b. Lithium
c. Cannabis
d. Histamine
e. Bile salts
f. TCAs
g. Stress
h. Looking at or thinking about itching
ANS: A, D, E, G, H
32. Arepitant is a
a. Substance P agonist
b. VIP antagonist
c. Neurokinin Receptor 1 antagonist
d. GABA agonist
ANS: C

Aprepitant, a neurokinin receptor (NKR)-1 antagonist that has been used as an antiemetic, was
recently reported to be effective for chronic refractory pruritus. NKR-1 is a receptor for substance
P, an important itch mediator.

33. The following is true


a. There is no change in taste sensation in glossodynia
b. Dysaesthetic vulvodynia has less dyspareunia and point tenderness
c. Glossodynia is commoner in men
d. Vulvodynia is not considered a chronic pain disorder
ANS: B

Patients with glossodynia present with chronic altered sensations of pain or burning affecting the
tip and sides of the tongue (or other areas inside the oral cavity). Other associated sensory
changes may include paresthesia, changes in taste and smell, and mouth dryness. Glossodynia
typically affects women in their 50s

The term vulvodynia is defined as “chronic vulvar discomfort with abnormal sensations” in the
absence of skin disease or infection. Affected women with vulvodynia are usually sensitive to
touch or light pressure in the vestibule of the vagina, have pelvic floor musculature with altered
contractile characteristics, and may also have heightened pain sensitivity in theirs arm or legs.
There is growing consensus that vulvodynia is a chronic pain disorder. Vulvodynia patients
experience more sexual dysfunction as a result of their primary symptoms. The term dysesthetic
vulvodynia is used to describe increased pain perception in the vulvar area without a known
cause and may be of central or nerve root origin. This condition often occurs in perimenopausal
or postmenopausal women, and they have fewer symptoms of dyspareunia and less point
tenderness on examination.

34. Effective, recommended treatment for CSS include all except


a. TCAs
b. Venlafaxine
c. NSAIDs
d. CBT
ANS: C
35. In DSM 5, Obsessive Compulsive and Related Disorders include all except
a. Skin picking disorder (SPD)
b. Body dysmorphic disorder (BDD)
c. Onychotillomania
d. Trichotillomania (TTM)
ANS: C

36. The following is true


a. More than one body-focused repetitive disorder is usually not encountered in the
same patient
b. Face and scalp are rarely targeted in SPD
c. Dissociation is a key factor in self-induced dermatoses
d. Onychotillomania is not a valid diagnosis
ANS: C

Dissociation (disruption in integrated functions of consciousness, memory, identity, and


perception) is a symptom of emotional dysregulation, which occurs in the context of severe stress
coping and extremes of emotional tolerance (window of emotional tolerance). Dissociation is
associated with increased threshold for pain perception and numbness of the skin and is often a
key factor in self-induced dermatoses

Onychotillomania is classified as an example of a “Body-Focused Repetitive Behavior Disorder”


under Unspecified OCRD

37. Skin response to stress include all except


a. Increased sweat production
b. Decreased conductance
c. Poor skin barrier function
d. Increased histamine levels from mast cell degranulation
ANS: B

There is increased conductance, which is a direct measure of autonomic arousal

38. SPD what is true


a. Was recognized in previous versions of DSM
b. Patients make repeated attempts to decrease or stop skin picking
c. ‘Cocaine bugs’ is included under SPD
d. Lesions on back are common
ANS: B

SPD is one of the new disorders to be included in DSM 5

SPD is diagnosed only if the skin picking is not attributable to the physiological effects of a
substance (e.g., cocaine) or another medical condition (e.g., scabies)

The distribution of excoriations is mostly confined to accessible areas like face, arms (mostly
extensor side), and anterior thigh, with sparing of middle and upper lateral back (butterfly sign).

39. SPD what is true?


a. Lifetime prevalence in general population is around 5%
b. Commoner in males
c. Onset in childhood is common
d. Nearly 10% prevalence in dermatology outpatients
ANS: D

Lifetime prevalence in general population is estimated to be around 1.4%

Nearly ¾ of patients are female

Mean age of onset is 30-45 years, with some conditions like acne excoriee beginning id
adolescence

40. SPD clinical features include:


a. Skin picking may be preceded or accompanied by various emotional states
b. It may be triggered by feelings of anxiety or boredom
c. There is a sense of tension preceding the picking, followed by gratification or
pleasure once the skin or scab has been picked
d. Picking does not occur without preceding tension
e. Picking preceded by tension is associated with higher emotional dysregulation
f. High level of dissociation responds better to conventional treatment
g. OCD can be comorbid with SPD
h. BDD cannot be comorbid with SPD
i. Both compulsive and impulsive behaviours are characteristic of SPD
ANS: A, B, C, G, I

Skin picking may be preceded or accompanied by various emotional states. It may be triggered
by feelings of anxiety or boredom, may be preceded by an increasing sense of tension (either
immediately before picking the skin or when attempting to resist the urge to pick), and may lead
to gratification, pleasure, or a sense of relief when skin or scab has been picked.
In SPD some individuals engage in skin picking that is more focused, i.e., with preceding tension
and subsequent relief, features that are consistent with obsessive-compulsive symptoms.

Some individuals may engage in more automatic picking, with the picking seeming to occur
without full awareness and without preceding tension. Some authors postulate that patients
engaging in skin picking without preceding tension or full awareness are likely to have higher
levels of emotional dysregulation and dissociation without necessarily meeting the criteria for a
dissociative disorder. Recognition of the dissociative component may be of clinical importance,
as the standard approaches to treating obsessive-compulsive skin picking (with habit reversal
therapy and SSRI antidepressants) are not sufficient when high levels of dissociation are present
(validated by scores on a dissociative rating scale).

Topic: Abnormal involuntary movements

1. Following are the various ways of classifying movement disorders:


a. Based on Phenomenology
b. Based on Neuropathologic findings
c. Based on Etiology
d. A & B
e. B & C
f. A & C
g. All of the above
Based on Phenomenology, there are 3 categories.
1) Parkinsonian disorders are characterized by reduced amplitude of movements (hypokinesia),
slowness of movements (bradykinesia), and
difficulty initiating movements (akinesia).
2) Dyskinetic disorders are characterized by increased motor activity and excessive (hyperkinetic)
involuntary movements such as tremor, chorea, ballism, tics, dystonia, or myoclonus.
3) Ataxias involve gait disturbances, dysmetria, and dysdiadokinesis.

Based on neuropathologic findings generally distinguish


1) extrapyramidal diseases, in which the primary pathology is in the basal ganglia system.
Diseases that primarily affect the cerebellum, cerebral cortex, or motor neurons.
An etiologic classification has the advantage of linking a syndrome directly to its cause and
directing attention at specific modes of pathogenesis, treatment, and prevention. However, the
cause of many diseases is unknown

2. A series of interrelated and interdependent circuits involving the basal ganglia and frontal -
subcortical circuits controls
a. Emotion and Movement
b. Congnition and Movement
c. Emotion, Cognition and Movement
d. None of the above
Therefore patients with motor abnormalities often have emotional or cognitive disorders and
patients with disorders of emotion or cognition frequently have motor abnormalities.

3. All of the following are true except


a. Abnormal movements are side effects of many psychiatric drugs
b. Abnormal movements are absent before exposure to psychiatric drugs
c. Powerful emotions can lead to abnormal movements
d. Primary Psychiatric disorders may present with movement abnormalities

Evaluating movement abnormalities disorders in psychiatric patients should be considered an


important aspect of general psychiatric practice.

First, side effects of many psychiatric medications include abnormal movements, some of which
may be long lasting or even permanent.

Second, some patients have evidence of abnormal movements before exposure to psychiatric
drugs, indicating the presence of gross congenital disorders (such as cerebral malformations),
developmental disorders without gross brain malformation (such as the “soft signs” seen in
patients with schizophrenia or autism), neurodegenerative disorders (such as HD or PD), other
cerebral insults (including strokes and tumors), or even normal aging.

Third, powerful emotions can lead to abnormal movements (e.g., tremulousness with anxiety).

Fourth, primary psychiatric disorders may present with movement abnormalities, as illustrated
by motor slowing seen with depression or the motor hyperactivity with mania.

4. Pyramidal & Extrapyramidal systems control voluntary & involuntary movements


independently
a. True
b. False

The pyramidal and extrapyramidal systems are not independent; the neural circuits of these
systems are interconnected.

5. Caudate nucleus is involved in which of the following condition?


a. Autism
b. Obsessive-Compulsive Disorder
c. Phobia
d. Major Depressive Disorder

6. Which of the following is not a type of tremor?


a. Kinetic Tremor
b. Task-specific Tremor
c. Position-Specific Tremor
d. Isometric Tremor
e. All of the above
f. None of the above

Types of tremor:
1. Resting tremor - muscles of the affected body part are not voluntarily activated; the tremor
usually ceases when a voluntary movement is initiated or performed.

2. Action tremor - produced by voluntary contraction of muscles and covers five types:
(a) Postural tremor - while voluntarily maintaining a position.
(b) Kinetic tremor - during voluntary movement and is further subdivided into:
• Simple kinetic tremor (during purposeless voluntary movements)
• Goal-directed tremor (during target reaching; historically labelled as ‘intention tremor’)
(c) Task-specific tremor - during the performance of specific highly skilled, goal-oriented tasks
such as handwriting or speaking.
(d) Position-specific tremor - during the maintenance of certain postures.
(e) Isometric tremor - during a voluntary muscle contraction that is not accompanied by any
movement.

7. Most common location of tremors:


a. Face
b. Upper Limb
c. Lower Limb
d. No specific area

Tremors can occur in any joint or muscle that is free to oscillate . The upper limb is the most
common location for a tremor to occur, but it can be spared or accompanied by a tremor in other
regions

8. Medium frequency range for tremors that can be visually distinguished is


a. 2-4 Hz
b. 4-7 Hz
c. 7-10 Hz
d. 10 – 13 Hz

9. Enumerate scales used to evaluate tremors?

The Tremor Rating Scale proposed by Fahn, Tolosa and Marin in 1988 is the most used scale and
comprises of three sections.

Section A takes into account the combination of activation tasks (rest, posture and kinetic
tremor) and the different topography according to a 0–4 scale (total score is 84).

Section B evaluates specifi c goal-directed tremors (handwriting for the dominant side, drawing
spirals and lines and pouring water for both hands) according to a 0–4 scale (total score is 36).
Section C is a questionnaire addressing the functional disabilities resulting from the tremor (e.g.
speaking, eating, drinking, etc.) according to a 0–4 scale (total score is 32).
A general assessment of the functional limitations is provided by the examiner and the patient at
the end of the scale (additional 8 points), thus resulting in a maximum score of 152, with higher
scores representing worse tremors.

More recently, the Tremor Research Group Essential Tremor Rating Assessment Scale (TETRAS)
has been designed specifically for the clinical measurement of ET severity; it comprises of a 12-
item activities of daily living subscale and a 9-item performance subscale that quantifies the
tremor in the head, face, voice, limbs and trunk

10. What is Quinquad’s sign?

The patient holds his fingers flexed at the metacarpophalangeal joints and extended at the
interphalangeal joints, while the examiner presses his palm against the finger tips. A worm-like
transmission is transmitted from the proximal joints

11. Frequency of tremors produced in acute alcohol withdrawal is


a. < 8 Hz
b. > 8 Hz
c. Both
d. None

Tremors on acute withdrawal are of two types:


< 8Hz- similar to essential tremors
> 8Hz
People having chronic use of alcohol (not in withdrawal) have postural tremor between 6 – 10.5
Hz

12. Drug/s that may cause tremors:


a. Tricyclics
b. Flunarizine
c. Tetrabenazine
d. All of the above
e. None of the above

You are amidst heavy OPD post lockdown and hence have asked the interns to help you out with
follow up patients. One of the interns come to you that there is a 38 year old male with tremors
in hands and has not brought any old prescription with him…. How will you guide the intern, as
to what should be assessed? and what are the possibilities?
Clinically you need to establish the type of tremors so that history probing can be guided
accordingly

Most commonly encountered tremors in Psychiatry OPD are:


1) Nervousness – rapid, varying from fine to coarse, affecting mainly the fingers, present at rest,
increased by voluntary movement, not increased during automatic movements, frequency 10-15
Hz

2) Anxiety state – similar to nervousness, more marked, coarser & more persistent, greatly
influenced by emotions accompanied by other autonomic features

3) Thyrotoxicosis – fine, rapid, persistent, accompanied by sweating, tachycardia and warm


hands, possible exophthalmos or lid retraction

4) Chronic patients of alcohol use disorders might have coarse postural tremors 6-10.5 Hz
Acute alcohol withdrawal may present with 2 varieties of tremors, one <8 Hz & other >8Hz

5) Withdrawal tremors (alcohol/opioid) are enhanced physiological tremors. Similar tremors are
seen with lithium, nicotine and L-dopa

6) Essential tremors – coarse, postural-action or purely intentional. Frequency about 8 Hz

7) Parkinsonian tremors – rhythmical 4-8 Hz

13. Akathisia is diagnosed by


a. Inner restlessness (urge to move)
b. Irresistible coordinated movement
c. Inner restlessness(urge to move) or Irresistible coordinated movement
d. Inner restlessness(urge to move) & Irresistible coordinated movement

Two core features need to be present to diagnose akathisia:

1)Inner restlessness or ‘ urge to move ’, usually reported by patients in association with


discomforting somatic sensory experience.

2)Irresistible and coordinated movements of the whole body or several body parts ,
followed by a sense of relief of the preceding sensory symptoms .

14. Patients with akathisia can suppress the movements voluntarily


a. True
b. False
c. Equivocal

Patients feel these movements as irresistible but are nevertheless able to suppress them when
asked. Movements also seem distractible by engaging in other attention-demanding tasks.

15. All of the following drugs can cause akathisia, except


a. Antipsychotics
b. SSRI
c. Carbamazepine
d. Topiramate
e. Pregabalin

Drugs that cause akathisia:


Dopamine receptor blockers (acute/subacute; tardive)
Tetrabenazine, reserpine
Selective serotonin reuptake inhibitors
noradrenaline/serotonin reuptake inhibitors
noradrenaline reuptake inhibitors
Calcium channel blockers
Lithium
Carbamazepine
Pregabalin

16. All of the following are biochemical changes associated with akathisia, except:
a. Decreased Dopamine function
b. Increased Noradrenaline activity
c. Decreased Serotonin activity
d. Reduced GABA activity

Particular drugs causing akathisia suggests an underlying hypoactivity of dopaminergic systems


(mesocortical, mesolimbic). And also nondopaminergic systems may also be involved, e.g.
increased noradrenergic and/or decreased serotonergic tone.

17. All of the following statements regarding akathisia are true, except
a. Acute akathisia is a dose related phenomenon
b. Akathisia is considered tardive if the patient is treated with antipsychotics for more than 1
month
c. 7 – 35% people receiving antipsychotics suffer from akathisia
d. Antipsychotic induced akathisia is more common in mood disorders than in Schizophrenia &
First episode psychosis
e. Akathisia increases suicidality in first episode psychosis

Akathisia is considered tardive if the patient has been treated for more than 3 months with
antipsychotics and if the phenomenon has persisted after 1 month from withdrawal, which may
also worsen akathisia

18. Antidepressant showing some evidence to benefit patients with akathisia


a. Bupropion
b. Reboxetine
c. Mirtazapine
d. Tianeptine
Mirtazapine at a dose of 15 mg/day showed significant superiority to placebo and no significant
difference in effect from propranolol in 1-week double-blind, randomized placebocontrolled trial
with an active comparator (propranolol 80 mg/day) with 30 patients per arm

19. All of the following are true regarding restless leg syndrome(RLS), except
a. Urge to move legs, always accompanied by unpleasant sensation
b. Unpleasant sensations begin or worsen during period of rest or inactivity
c. Unpleasant sensations are partially or totally relieved by movement
d. Unpleasant sensations occur exclusively or are worse during evening or night

An urge to move the legs , usually but not always accompanied by or felt to be caused by
uncomfortable and unpleasant sensations in the legs .

20. Periodic Limb Movements during Sleep is seen in _______ cases of RLS
a. 20-40%
b. 40-60%
c. 60-70%
d. 70-80%
e. 80-90%

21. Which of the following is not a characteristic of sleep pattern in RLS


a. Increased sleep latency
b. Increased REM sleep
c. Reduced sleep efficiency
d. Frequent nocturnal awakenings

22. Which of the following is not a predisposing factor for RLS


a. Iron deficiency
b. Pregnancy
c. Portal hypertension
d. Uraemic renal failure

23. Which of the following drug is not used to treat RLS


a. Ropinirole
b. Pramipexole
c. Amitryptiline
d. Pregabalin

24. Dystonic movements are patterned & repetitive in nature


a. True
b. False
c. Cannot comment
Dystonic movements are usually have a patterned and repetitive nature (as they involve the same
group of muscles, in contrast to chorea, where movements are random).

25. Which of the following agents does not cause dystonia


a. Antipsychotics
b. SSRI
c. Methanol
d. Disulfiram

26. Antipsychotic induced dystonia respond to reassurance


a. True
b. False
c. Cannot comment

27. Following structures are involved in dystonia, except


a. Sensory-motor circuits
b. Basal Ganglia
c. Prefrontal cortex
d. Cerebellum

28. Pseudodystonia presents with actual dystonic movements of various groups of muscles
a. True
b. False
c. Cannot comment

Pseudodystonias are dystonia which is not as a result of involvement of basal ganglia

Topic: Sexual disorders


1) Sexual dysfunction is one category of disorders of sexuality and sexual functioning. The term
sexual dysfunction refers to?

a) Problems with the normal sexual response cycle

b) Sexual urges or fantasies involving unusual sources of gratification problems

c) An individual is dissatisfied with their own biological sex and have a strong desire to be
member of the opposite sex.

d) Problems with sexual fantasies

Sexual dysfunction : A disturbance in the processes that characterize the sexual response cyc
or by pain associated with intercourse.
2) In disorders of sexuality and sexual functioning, the term paraphilias refers to?

a) Problems with the normal sexual response cycle

b) Sexual urges or fantasies involving unusual sources of gratification problems

c) An individual is dissatisfied with their own biological sex and have a strong desire to be
member of the opposite sex.

d) Problems with sexual fantasies

Paraphilias: Problematic, high frequency sexual behaviours or unusual sexual urges and activities
that are often directed at inappropriate targets

3) In disorders of sexuality and sexual functioning, the term gender identity disorder refers to:

a) Problems with the normal sexual response cycle

b) Sexual urges or fantasies involving unusual sources of gratification problems

c) Problems with sexual fantasies

d) An individual is dissatisfied with their own biological sex and have a strong desire to be
member of the opposite sex

Gender identity disorder: A sexual disorder where an individual is dissatisfied with their own
biological sex and has a strong desire to be a member of the opposite sex.

4) Sexual aversion disorder is associated with which of the following?

a) Anxiety

b Disgust

c) Fear

d) All of the above

Sexual Aversion Disorder : A sexual disorder in which there is an active avoidance of genital sexual
contact with a sexual partner.

5) Female Sexual Arousal Disorder is defined primarily in terms of a deficiency in a physical or


physiological response, and as a result may be caused by a range of physical or physiological
factors, including:
a) Hormone imbalances

b) Diabetes

c) Medications being taken for other disorders

d) All of the above

Female Sexual Arousal Disorder: A sexual disorder in which there is an inability to attain an
adequate lubrication-swelling response of sexual excitement and to maintain this state until
completion of sexual activity..

6) Which of the following is characteristic of Male Erectile Disorder?

a) A failure to attain an erection from the outset of sexual activity,

b) First experiencing an erection but then losing tumescence prior to penetration

c) Losing tumescence during penetration but prior to orgasm

d) All of the above

Male Erectile Disorder: A sexual disorder in which there is the inability to maintain an adequate
erection during sexual activity. Around 10% of males report erection problems, but this increases
to 20% in the over 50s.

7) The term performance anxiety refers to:

a) Fear of failing to achieve a sustained erection

b) Fear of having sex in a public place

c) Fear of criticism from the sexual partner

d) Fear of not achieving orgasm

Performance Anxiety: The fear of failing to achieve an acceptable level of sexual performance
causing an individual to become distanced from the sexual act and fail to become aroused.

8) Which of the following is the term for the experience of persistent or recurrent delay in or
absence of orgasm following normal sexual excitement which causes the individual marked
distress or interpersonal difficulty?

a) Dysfunctional Orgasmic Disorder


b) Aclimactic Disorder

c) Female Orgasmic Disorder

d) Female Climactic Disorder

Female Orgasmic Disorder: A sexual disorder which is characterized by a delay or absence of


orgasm during sexual activity, and around 10% of adult women may never have experienced
an orgasm.

9) Genital pains that can occur before, during or after sexual intercourse, and can occur in both
males and females are known as

a) Dyspareunia

b) Dysmenhorea

c) Dyskinesia

d) Dyspraxia

Dyspareunia: A genital pain that can occur during, before or after sexual intercourse. Some
clinicians believe this is a pain disorder rather than a sexual dysfunction..

10) The involuntary contraction of the perineal muscles surrounding the outer third of the vagina
when vaginal penetration is attempted is termed:

a) Perinealitis

b) Perivaginitis

c) Vaginitis

d) Vaginismus

Vaginismus: The involuntary contraction of the muscles surrounding the vagina when vaginal
penetration is attempted. Of all women that seek treatment for sexual dysfunctions, around
15-17% are suffering from vaginismus.

11) In men, erectile dysfunction is associated with high levels of which of the following?

a) Testosterone

b) Prolactin
c) Estrogen

d) Prostaglandin

Prolactin: A hormone from the pituitary gland stimulating milk production after childbirth

12) Which of the following is a direct treatment method which deals with symptoms of erectile
dysfunction or male and female orgasmic disorder?

a) Squeeze technique

b) Tickle technique

c) Tease technique.

d) Stroke technique

Tease technique: A direct treatment method designed to deal with symptoms of erectile
dysfunction or male and female orgasmic disorder which involves the partner caressing the
client's genitals, but stopping when the client becomes aroused (e.g. achieves an erection) or
approaches orgasm..

13) A common drug treatment for sexual dysfunction is Viagra. What is the generic term for this?

a) Fluoxitine

b) Metronydasol

c) Diclofenac

d) Sildenafil citrate

Viagra (sildenafil citrate): A drug treatment for sexual dysfunction which is used primarily to
treat erectile dysfunction in men.

14) Yohimbine is a drug used to treat erectile dysfunctions, it works by:

a) Facilitating dopamine production brain

b) Facilitating norepinephrine excretion in the brain

c) Facilitating serotonin excretion in the brain

d) Facilitating testosterone production


Yohimbine: A drug treatment for sexual dysfunction which is used primarily to treat erectile
dysfunction in men by facilitating norepinephrine excretion in the brain.

15) A diagnosis of fetishism involves which of the following?

a) Intense sexually arousing fantasies that occur while observing an unsuspectin


person who is naked, in the process of undressing, or engaging in a sexual activity

b) Sexual arousal and satisfaction from the psychological or physical suffering of other

c) Intense, recurrent sexual urges to touch and rub up against non-consenting people

d) Intense sexually arousing fantasies and urges involving non-animate objects, an


this causes them personal distress or affects social and occupational functioning

Fetishism: A paraphilia which involves sexually arousing fantasies and urges directed at non-
animate objects.

Topic: Paraphilias

1) A diagnosis of fetishism involves which of the following?

a) Intense sexually arousing fantasies that occur while observing an unsuspecting person who
naked, in the process of undressing, or engaging in a sexual activity

b) Sexual arousal and satisfaction from the psychological or physical suffering of others

c) Intense, recurrent sexual urges to touch and rub up against non-consenting people

d) Intense sexually arousing fantasies and urges involving non-animate objects, and th
causes them personal distress or affects social and occupational functioning

Fetishism: A paraphilia which involves sexually arousing fantasies and urges directed at non-
animate objects.

2) The term Frotteurism means which of the following?

a) Observing an unsuspecting person who is naked, in the process of undressing, or engagin


in a sexual activity

b) Sexual fantasies about exposing the penis to a stranger


c) Intense, recurrent sexual urges to touch and rub up against non-consenting people

d) Sexual arousal and satisfaction from the psychological and physical suffering of others

Frotteurism: A paraphila which involves recurrent sexual urges to touch and rub up against other
non-consenting people - usually in crowded places.

3) Which of the following is a paraphilia involving sexual fantasies about exposing the penis to a
stranger, which are usually strong and recurrent to the point where the individual feels a
compulsion to expose himself?

a) Voyeurism

b) Expositionism

c) Exhibitionism

d) Frotteurism

Correct! Exhibitionism: A paraphilia which involves sexual fantasies about exposing the penis to
a stranger, usually either a women or a child.I

4) ) Paedophiles become sexually aroused by sexually immature children, an alternative term for
this type of behaviour is:

a) Non consent molesters

b) Minor molester

c) Preference molesters

d) Pree-teen molester

Preference molesters: Non-incestuous paedophiles who will normally only become sexually
aroused by sexually immature children.

5) The term hypersexuality refers to high rates of sexual activity, and has been linked with which
of the following?

a) Paraphilias

b) Anxiety

c) Depression
d) All of the above

Hypersexuality: The occurrence of high rates of sexual activity

6. Psychodynamic explanations of sexual sadism propose that the sadist is searching relief from
castration anxiety by:

a) Dressing in womens clothing

b) Taking on the role of castrator rather than castrated

c) Seeking out partners who have been castrated

d) Performing castration on a consenting partner

Sexual Sadism: A paraphilia in which a person gains sexual arousal and satisfaction from the
psychological or physical suffering of others.

7) ) Which of the following terms were used by Polaschek& Ward (2002) to describe the concept
that, sex offenders may have developed integrated cognitive schemata that guide the offender's
interactions with their victims and justify their behaviour?

a) Justified actions

b) Cognitive dissonance

c) Implicit Theories

d) Responsibility bias

Implicit Theories: In sexual offending, an integrated cognitive schemata that guide a


sexual offender's interactions with their victims and justify their behaviour

8) A form of aversion therapy where rather than actually experience the pairing of sexual stimuli
with aversive outcomes, the client imagines these associations during controlled treatment
sessions is known as:

a) Classical conditioning

b) Operant conditioning

c) Covert conditioning

d) Overt conditioning
Covert conditioning: An approach to mental health treatment that uses the principles of behavior
modification on the person's capacity to use imagery for purposes such as mental rehearsal

9) The aim of Orgasmic Reorientation treatment for paraphilias is to:

a) Pair inappropriate or distressing sexual activities with an aversive stimulus

b) Suppress inappropriate or distressing sexual activities through drug

c) Suppress inappropriate or distressing sexual activities through castration

d) Suppress inappropriate or distressing sexual activities and replace them with acceptabl
sexual practices

Orgasmic reorientation: A treatment method to replace inappropriate or distressing sexual


activities that aims to make the client sexually aroused by more conventional or acceptable
stimuli.

10) When some individuals develop feelings that they have a sense of gender that is opposite to
the biological sex they were born with, this is known as:

a) Cross gender disorder

b) Gender dysphoria

c) Gender incompatibility disorder

d) Bi-gender disorder

Gender dysphoria: A gender identity disorder in which an individual has a sense of gender that is
opposite to their biological sex

11. In the treatment of sexual dysfunctions, it is important that the client is able to guide their
own behaviour and reduce anxiety. Which of the following is a means of achieving this?

a) Meditation

b) Partner-instructional training

c) Self-instructional training

d) Group therapy
Self-instructional training: A technique that has been used across a range of psychopathologies
in order to establish adaptive behaviour patterns where the client is taught to provide
appropriate self-instructions for behaviour.

12. Sexual masochists can often cause their own suffering, when an individual uses a noose or
plastic bag to induce oxygen deprivation during masturbation, this is known as:

a) Necrophilia

b) Hypoxyphilia

c) Corpophilia

d) Paedophilia

Hypoxyphilia:An act performed by sexual masochists which involves the individual using a noose
or plastic bag to induce oxygen deprivation during masturbation.

13. Sexual dysfunction may be closely associated with relationship problems. One way that this
can be explored is through:

a) Psychoanalysis

b) Self instructional training

c) Couples therapy

d) Orgasmic reorientation training

Couples Therapy: A treatment intervention for sexual dysfunction that involves both partners in
the relationship.

14. Which of the following is NOT considered to be a risk factor for the development of
paraphilias?

a) Early drug abuse

b) Hypersexuality

c) Childhood abuse

d) Childhood neglect

Risk factors: Certain childhood or developmental experiences which may increase the risk
of developing psychopathology later in life
PARAPHILIA

1. Which of the following is NOT an approach to defining abnormal sexual practices?

o unacceptable according to customs of a particular time in history


o statistically rare
o deviations from social norms
o persistent preference for no genital outlets
o highly creative and exotic practices

2. Greg, a paraphile, has exposed his genitals to young children on several occasions. How
is he likely to describe the emotions that precipitated these acts?

o "I don't remember what happened."


o "Everybody does these kinds of things. It's no big deal."
o "My urge just kept getting stronger. I couldn't help it."
o "I don't need to do it to become aroused, I just really enjoy doing this."
o "I didn't know what I was doing. I just felt such love for the children."

3. Which of the following is true of paraphilias?

o the prevalence of paraphilias in the general population remains unknown


o by definition, paraphilias are harmful behaviours toward others
o all paraphilias are harmful and dangerous
o paraphilias are usually extreme and debilitating
o paraphilias occur with the same frequency among women and men

4.All of the following are true regarding fetishism, EXCEPT it

o may involve masturbation.


o is practiced in private.
o is very harmful to self and others.
o may involve materials such as leather or silk.
o may involve articles of clothing such as shoes or bras
5.Most male transvestites are

o exceptionally feminine in nature.


o men with erectile problems.
o gay.
o men who want to be women.
o married and masculine in nature.
6.Peter really enjoys exposing his genitals to strangers. Peter may be defined as a(n)

o exhibitionist.
o masochist.
o voyeur.
o fetishist.
o sadist.

7.Some Canadian psychologists argue that certain behaviours labelled as paraphilias should not
be classified as sexual disorders because

o they are generally victimless.


o they are generally harmless.
o the labels are strongly influenced by cultural values.
o all of the above
o none of the above

8. Bobby really enjoys observing unsuspecting strangers who are naked, disrobing, or engaged in
sexual relations. Bobby may be defined as a(n)

o voyeur.
o fetishist.
o masochist.
o sadist.
o exhibitionist.

9.Which of the following is true of voyeurs?


o numerous instances of men using small video cameras and lenses to aid their
voyeurism have been reported across Canada
o most voyeurs also commit violent crimes such as assault and rape
o voyeurs will often seek out sexual relationships with the people they observe
o voyeurs will often break into and enter into homes or buildings
o anyone who becomes aroused by the site of someone undressing could be
considered a voyeur

10.Marcia associates the receipt of pain or humiliation with sexual arousal. She enjoys being
bound during her sexual activities. Marcia may be defined as a(n)

o voyeur.
o masochist.
o exhibitionist.
o fetishist.
o sadist.
11.Jan also associates pain with sexual arousal, but Jan enjoys inflicting pain on others to gain
arousal. Jan may be defined as a(n)

o masochist.
o exhibitionist.
o voyeur.
o sadist.
o fetishist.
12. Frotteurists experience arousal when

o their victims become fearful or shocked.


o they rub or touch non-consenting people unknown to them.
o they can entice children to submit to rubbing or touching.
o a non-consenting person touches or rubs against a particular fetish object.
o rubbing is followed by intercourse

13. Which statement is accurate regarding biological factors and paraphiles?

o most pedophiles could be considered to have hypoactive sexual desire disorder


o testosterone levels are elevated in all paraphiles
o estrogen levels are elevated in the majority of paraphiles
o abnormal brain structures in the hypothalamus have been found
o paraphiles may have higher than average sex drives

14. Classical psychoanalytic theory views the paraphilias as

o unresolved issues related to the genital stage of development.


o originating in castration fears during the Oedipal period.
o a normal expression of fixated libidinal energy.
o expressions of unconscious rage against an indifferent father.
o a way to vent anger and hostility against the mother.
15. Which theory combines biological, psychological, and sociocultural factors to explain the
development of paraphilias?

o learning theory
o psychobiological theory
o sociological theory
o "lovemap" theory
o theory of addictions

16. To increase sexual arousal to appropriate stimuli, which method would a therapist use?

o psychoanalysis
o systematic desensitization
o orgasmic reconditioning
o covert sensitization
o aversion therapy

17.John Bradford has proposed a six-level schema for treating sexual deviation. The first level
involves the use of

o cognitive-behavioural therapy.
o hormonal treatment.
o psychoanalysis.
o Prozac.
o anti-androgens

18.All of the following are true of a person with a sexual addiction, EXCEPT

o the person engages in illicit sexual behaviour that endangers his or her own well-
being.
o the person lacks control over his or her sexual impulses.
o the person can't resist sexual opportunities.
o the person uses sexual behaviour as a means of reducing anxiety.
o sexual contacts are highly satisfactory experiences for the person.

19.The persistent urge to fondle nonconsenting strangers is called

o toucherism.
o voyeurism.
o frotteurism.
o zoophilia.
o exhibitionism.

20.All of the following are effective ways for a woman to respond if she is exposed to an
exhibitionist, EXCEPT

o report the incident to the police.


o insult the exhibitionist.
o tell the person "you should seek professional help."
o remain calm.
o show no reaction if possible.

Topic: Anorexia Nervosa


1)Which of the following is a diagnostic criterion for anorexia nervosain DSM-5?

• Restriction of energy intake relative requirements leading to significant low body weight
• Intense fear of becoming fat or gaining weightor persistent behavior that interferes with
weight gain
• Disturbance in the way in which the one’s body weight or shape
• All the above

2) According to the DSM , one of the diagnostic criteria for anorexia nervosa is that individual
body weight below …… of their expected weight for their age and height.

• Below 90%
• Below 85%
• Below 70%
• Below 65%

3) what are the sub types of anorexia nervosa ?

• Restrictive type
• Binge eating/purging type
• Dysmorphophobia
• Night eating syndrome

4) Most common age of onset - anorexia nervosa?

• Between 14 and 18 years


• >20 years
• < 12 years
• Between 20-23 years

5) what is the percentage of woman meet the criteria for either anorexia nervosa.bulimia
nervosa, or eating disorders not otherwise specified ?

• 30%
• 10%
• 40%
• 20%
6) Most common co morbidity associated with anorexia nervosa?

• Depression – 65%
• Social phobia- 35%
• OCD- 25%

7) Neurophysiological evidence suggests that those with anorexia nervosa have problems with
executive functions which stem from frontal lobe dysfunction?

• True
• False

8) which is FALSE regarding ECG changes in anorexia nervosa?

• Flat / inverted T wave


• ST segment depression
• Lengthening of QT interval
• QT prolongation

9) A socio-cultural explanation for eating disorders emphasizes ?

• The role of distorded believes in the development of the disorder


• The influence of media which promotes thinness as a desirable characterstic
• The influence of childhood factors in the development of the disorder
• A dysfunction of dopaminergic transmission

10) one of the primary goal of any treatment for anorexia nervosa is to …..

• Reduce weight to normal levels


• Replace binge eating with other behaviours
• Stop the individual exercising
• Increase weight to normal levels

11) The MAUDSELY method is …..?

• A technique of systematic desensitization for bulimia nervosa


• CBT for binge eating disorders
• A technique of family therapy for treating eating disorders
• The compulsory detainment in hospital of those at risk of death from anorexia nervosa
12) In restricted type anorexia nervosa, self starvation is NOT associated with which of the
following ?

• Concurrent purging
• Socializing
• Body dysmorphic issues
• Eating only certain food types

13) In binge -eating type AN ,self starvation is associated with :

• Not eating to help control weight gain


• Not being bothered about weight gain
• Regularly engaging in purging activities to help control weight gain
• Eating only certain food types

14)high rates of co morbidity exist between AN and other Axis I &II disorders. What percentage
of anorexia sufferers who also have a life long diagnosis of major depression?

• 50-60%
• 30-40%
• 20-30%
• 70-80%

15) In BULIMIA NERVOSA , the purging sub-type , vomiting is the most common form of purging.
What percentage of suffers present with this type of purging?

• 50-60%
• 80-90%
• 15-25%
• 50-60%
16) In BULIMIA NERVOSA, the non purging sub-type a behavior which is used to compensate for
binging is

• Exercise
• Controlling intake of certain food types
• Withdrawing from social interaction
• Controlling carbohydrate intake

17) individuals with bulimia have a perceived lack of control over their eating behavior, and often
report which of the following?

• High levels of self-disgust


• Low self esteem
• High level of depression
• All the above

18) Evidence suggests a link between bulimia and Axis II borderline personality disorder. What
percentage of woman with bulimia meet the criteria for a personality disorder?

• 45-55%
• 33-61%
• 20-30%
• 60-65%
Main features are instability in personal relationships, lack of well -defined and stable self
image,regular and unpredictable changes in moods and impulsive behavior.

19) in the 17 thcentury ,which of the following terms was used to describe a disorder
characterized by large food intake followed by vomiting?

• Vomitoria
• Fames canina
• Bulimia nervosa
• Nuxcanina

20) pick one of the following familial factors that plays a role in the development of eating
disorders?

• Parental attitude to sex


• Parental obesity
• Parental attitudes to the media
• Parental attitudes to education

21) In animal research ,lesion to which part of the brain have been shown to cause appetite loss,
resulting in a self-starvation syndrome?

• Lateral hypothalamus
• Cerebellum
• Amygdala
• Basal ganglia

22)Biological accounts of AN and bulimia suggests that maintaining a lowbody weight and self
starvation may be reinforced by

• Endogenous opioids
• Serotonin
• Endorphins
• Dopamine

23) research on bulimia nervosa has found low levels of

• Alpha-dopamine
• Alpha- amphetamine
• Beta-serotonin
• Beta-endorphin

24) which of the following characterstics has regularly been implicated in the aetiology of eating
disorders?

• Perfectionism
• Narcissism
• Extraversion
• Introversion
25) which of the following is a lab procedure developed to provide an objective behavioural
measure of the tendency to binge eat?

• Palatability test
• Food pre-load test – objective behavioural measure of the tendency to binge eat.
• Full capacity test
• Fully loaded test

Anorexia Nervosa Bulimia Nervosa


ABOUT Eating disorder wherein sufferers Eating disorder wherein sufferers go
fear weight gain and avoid eating through a cycle of binging (overeating)
as a result. Mainly affects young followed by purging, due to a fear of
women. weight gain. Mainly affects young
women.
Behavioral andObsession with food, weight, and Obsession with food, weight, and a
Psychological a "thin" body image; extreme fear "thin" body image; extreme fear of
Symptoms of weight gain; compulsive weight gain; compulsive exercise;
exercise; depression and anxiety; depression and anxiety; low self-
low self-esteem; body esteem; body dysmorphic disorder.
dysmorphic disorder.
Physical Usually extremely underweight Many within "normal" weight range
Symptoms and unhealthy figure; physical for height/age, but can be
weakness, deterioration, and underweight; physical weakness,
organ dysfunction; absent deterioration, and organ dysfunction;
menstruation; memory loss, absent menstruation; memory loss,
feeling faint, etc. feeling faint, etc. Noticeable
oral/dental deterioration.
Relationship toAvoids eating, frequently goes on Goes through periods of binging —
Food fasts or restrictive diets, tendency overeating — and purging, usually by
to be secretive about eating vomiting or heavy use of laxatives,
habits and rituals. diuretics, etc.

Topic: Genetic syndromes

1) Syndrome associated with mental retardation except ?

a. Down’s syndrome

b. Crouzon’s syndrome

c. De lange syndrome

d. Kahbaum syndrome

Ans: kahbaum syndrome – catatonia

2) To diagnosis metabolic syndrome what are the required parameters……


a. Abdominal obesity

b. Elevated blood pressure

c. High serum triglyserides

d. Lft

e. Rft

Ans: a) abdominal obesity b) elevated blood pressure c) high serum triglyserides

3) Turners syndrome -----?

a. 45 xo

b. 5p deletion

c. 47,xxy

d. Trisomy 21

Ans: a) 45 xo. Condition where female is partly or completely missing an ’x’


chromosome.

4. Branchial arch syndrome is also known as ……..?

a. Alpert’s syndrome

b. De lange syndrome

c. Bushy syndrome

d. crouzon’s syndrome

Ans: crouzon’s syndrome – autosomal dominant genetic disorder . Named after octave crouzon
.

5. Sterility is the primary feature of ….?

a. Kleinfelters syndrome

b. Cri du syndrome

c. Sturge weber syndrome

d. Lesch nyhan syndrome


Ans: kleinfelters syndrome- 47,xxy- two or more x chromosomes in male.

6. Deficiency of hypoxanthine –guanine phosphoribosyltranse is seen in………

a. Encephalotrigeminal angiomatosis

b. Lesch nyhan syndrome

c. Lejeune’s syndrome

d. Amsterdam dwarfism

Ans: b) lesch nyhan syndrome- caused by deficiency of hypoxanthine –guanine


phosphoribosyltranse – causing buildup of uric uric acid in all body fluids. Commonly seen in
males.

Topic: PERINATAL PSYCHIATRY

Q 1 Which of the following is an established risk factor for experiencing depression during
pregnancy?
Select one:
Smoking cigarettes
Early age at menarche
Past history of depression
First pregnancy
Working mother

• Women with a previous history of a mood disorder severe enough to require treatment
including counselling or medication are at greater risk of developing depression during pregnancy
than women with no prior history of a mood disorder.
• Depression during pregnancy continues/recurs as postnatal depression in 40% of women
(Retrieved from
http://www.blackdoginstitute.org.au/public/depression/inpregnancypostnatal/cause).

2. Which drug is widely used by perinatal services in the for the treatment of schizop hrenia in
pregnancy?
Select one:
Olanzapine
Amisulpride
Quitiapine
Risperidone
Aripiprazole
Risk associated with untreated schizophrenia is high. The current consensus is to use an
antipsychotic agent at every stage of pregnancy. Patients with a history of psychosis, who are
maintained on antipsychotic, particularly if they have frequent relapses, are best maintained on
antipsychotic during and after pregnancy. Most experience is with chlorpromazine,
trifluoperazine, haloperidol, olanzapine and clozapine. Olanzapine is widely used by perinatal
services in the UK.

Q 2.Which of the following is not an effective medication in the treatment of premenstrual


dysphoric disorder ?
Select one:
Clomipramine
Venlafaxine
Sertraline
Fluoxetine
Reboxetine

• Treatment of PMS: Mild symptoms- lifestyle changes, CBT, exercise or dietary regulation.
Severe PMS requires SRIs.
• Response rate 60-90% compared to 30-40% for placebo. Effective medications are serotonergic
tricyclic antidepressant like clomipramine, SSRIs such as citalopram, escitalopram, fluoxetine,
paroxetine, sertraline and SNRIs like venlafaxine.
• The medications are helpful in reducing both mood and somatic symptoms, improves the
quality of life and social functioning.
• Two most studied SSRIs are fluoxetine and sertraline. The most effective agent appears to be
fluoxetine.

Q 3 To screen for Ebstein's anomaly, pregnant mothers should undergo level 2 ultrasound and
echocardiography of the foetus at :
Select one:
2 and 10 weeks gestation
2 and 6 weeks gestation
24 and 32 weeks gestation
6 and 18 weeks gestation
20 and 24 weeks gestation

• To screen for Ebstein's anomaly, mothers should undergo level 2 ultrasound and
echocardiography of the foetus at 6 and 18 week's gestation. (Maudsley Guidelines 2019).
• For women on maintenance treatment, the serum lithium level should be monitored every 4
weeks throughout the pregnancy.
• Lithium dosage should be adjusted to match the lower end of the therapeutic range.
• Lower doses and frequent blood monitoring should be the norm in pregnant women starting
lithium in the first trimester of pregnancy.
• Lithium commenced in second and third trimester of pregnancy or perinatal period can help
reduce the risk of puerperal psychosis.

Q4 : What is the chance of having a baby with congenital malformation if lithium is continued
through the first trimester?
Select one:
1 in 10
1 in 1000
1 in 2
1 in 100
1 in 5

• There is 1 in 10 chance of having a malformation if lithium is continued through the first


trimester.
• The UK National Teratology Information Service have concluded that lithium increases the risk
of all types of malformation of approximately three-fold and with a weighting towards cardiac
malformations of around eight-fold (Williams and Oke, 2000).
• Maximum risk is at 2-6 weeks after conception when many pregnancies are still undiscovered.

Q 5 : The most sensitive period for foetal malformations on exposure to teratogenic drugs in
pregnancy is :
Select one:
32-36 weeks
At labour
14-28 weeks
2-14 weeks
28-32 weeks

The risk of teratogenicity is highest in the first 12 weeks of gestation (2 to 14 weeks).

Q 5 Mrs. Sadhama has a history of postpartum psychosis, which occurred two years ago. She is
pregnant again and would like to know the risk of recurrence rate in subsequent pregnancies.
The correct answer is
Select one:
0.1-0.25%
5%
10%
50-90%
25-40%

• The risk of postpartum psychosis is 0.1-0.25% in general population,


• 50% in bipolar disorder and 50-90% in a patient with a history of postpartum psychosis.
Puerperal psychosis can take the form of mania, schizophreniform / acute polymorphic psychosis,
or with confusion and disorientation.
• The incidence of puerperal psychosis is about one per 1000 births. It is strongly linked with
bipolar disorder.

Q 6 The rate of depression during pregnancy is


Select one:
30%
20%
10%
25%
5%

• The risk of depression during pregnancy is estimated to be 7 to 15% (around 10%). This may
be higher in developing nations.
• The rate of depression in a woman outside the perinatal period is only 7%.
• The relapse rate of depression in those who had a past history is around 50% when they are
pregnant.
• The high prevalence of antenatal depression may be partly due to relapse/recurrences and
partly due to freshly increased the incidence of depression. (O'Keane and Marsh 2007)

Q 7 : Postpartum psychosis is most commonly associated with


Select one:
hysteria
delusional disorder
delusional jealousy
schizophrenia
bipolar disorder

• The prevalence of postpartum psychosis in the general population is 1-2 per 1,000 childbirths,
and the rate is 100 times higher in women with bipolar disorder or a previous history of
postpartum psychosis.
• Bipolar disorder and, to a lesser extent, schizophrenia have elevated prevalence in postpartum
psychosis
• (Excerpt from http://ajp.psychiatryonline.org/doi/full/10.1176/appi.ajp.2008.08121899 ).

Q8 : Which of the following effects on the fetus is NOT associated with a mother being treated
with SSRIs during pregnancy?
Select one:
Neonatal irritability
Spontaneous abortion
Premature birth
Neural tube defects
Reduced birth weight

• SSRI: No increase in major malformation (exception- paroxetine); the neonatal withdrawal are
reversible complications.
• However there is 13.3% increase in spontaneous abortion (also with mirtazepine and amp;
bupropion), decreased gestational age (mean 1 week) and low birth weight (mean 175 gms).
• Paroxetine, particularly high dose first trimesters exposure, is clearly linked to cardiac
malformation VSD and ASD.
• Third trimester use can give rise to neonatal complication due to abrupt withdrawals.

Q 9 Mrs. S has a history of bipolar disorder and remained on mood stabilizers whilst pregnant.
Following delivery, the breastfed infant suffered from thrombocytopenia and anaemia which
resolved after stopping breastfeeding. The most likely offending drug is
Select one:
Gabapentin
Lamotrigine
Lithium
Sodium valproate
Carbamazepine

• Sodium valproate achieves a serum range in breastfed infants that varies from being
undetectable to up to 40%.
• Adverse effects reported include thrombocytopenia and anaemia. These features resolve
after stopping breast feeding.

Q 10 : If a patient continues to take Sodium Valproate throughout her pregnancy what is the
risk of the baby having any birth defect?
Select one: 1 in 1000
1 in 10
1 in 10000
1 in 100 000
1 in 100
• Sodium Valproate is the most teratogenic of all psychotropics.
• Risk for any birth defect quoted 7.2% in Maudsley Guidelines; NICE and the UK epilepsy and
pregnancy registry give comparable values; North American registry quotes a 10% risk.

Q11 If a patient continues to take Sodium Valproate throughout her pregnancy what is the risk
of the baby having neural tube defect?
Select one:
1 in 10
1 in 1000
1 in 10 000
1 in 100 000
1 in 100
• Sodium valproate is associated with a risk of neural tube defect ( 1-2 %), a risk of spina bifida (
10 fold increase),digital and limb defects, heart defects ( VSD, Pulmonary stenosis etc. 4 fold
increase),
• Urogenital malformations, low birth weight and psychomotor slowness.

Q 12 With respect to postpartum mental illnesses, a significant increase in new psychiatric


episodes occur in
Select one:
First 1 week of postpartum period.
First 12 months of postpartum period.
First 6 months of postpartum period.
First 3 months of postpartum period

First 1 day of postpartum period.


Significant increase in new psychiatric episodes in first 3 months of postpartum period. 80% are
mood disorder (mainly depression).
Q 13 Which of the following has the most evidence base for use in premenstrual tension?
Select one:
Sertaline
escitaloptram
Citalopram
Fluoxetine
paroxetine

• Selective serotonin reuptake inhibitors such as fluoxetine and sertraline are the most effective
pharmacologic agents against PMT.
• Prostaglandin inhibitors and diuretics may provide some relief of symptoms.
• Only weak evidence supports the effectiveness of gonadotropin-releasing hormone agonists,
androgenic agents, estrogen, progesterone, or other psychotropics, and side effects limit their
use.

Q 14 What is the relative risk of developing Ebstein's anomaly for a child when the mother is
on lithium throughout her pregnancy?
Select one:
3.5
3.8
7.4
9.5

• Relative risk of Ebstein's anomaly in mothers taking lithium compared to general population is
10-20times higher, but the absolute risk is relatively low at 1:1000.
• (Absolute spontaneous risk of Ebsteins is 1 in 20,000. Cohen et al., JAMA 1994;271: 146 -150).
Maximum risk is seen at 2-6 weeks after conception when many pregnancies are still
undiscovered. The closest answer from the list is - 9.5 times.

Q 15 What percentage of new mothers develop postpartum depression?


Select one:
25-30%
65-70%
45-50%
1-2%
10-15%

Postpartum depression affects 10%--15% of new mothers within the first year after giving birth.
Younger mothers and those experiencing partner-related stress or physical abuse,and those with
lower educational attainment might be more likely to develop postpartum depressive symptoms.

Q 16 The prevalence of postpartum blues is reported to be as high as around


Select one:
20% of all pregnancies
10% of all pregnancies
90% of all pregnancies Incorrect
2% of all pregnancies
50% of all pregnancies

Postpartum blues is seen in around 50-60% of all pregnancies in first few days of delivery.

Q 17 The least likely risk factor for postnatal depression among the following is
Select one:
Previous history of postnatal depression
Having a first degree relative with bipolar disorder
Having a first degree relative with postnatal depression
Having postpartum blues after delivery
Diagnosis of bipolar affective disorder

Postpartum blues are said to occur in around 70% of childbirths. They are very non-specific and
do not have a causal association with postnatal depression.

Q 18 The most severe and early symptoms seen in premenstrual dysphoric disorder are
Select one:
Anxiety and Irritability
Depressed mood and anhedonia
Breast tenderness and bloating
Insomnia and poor appetite
Concentration difficulties and memory loss

• Symptoms of PMS peak 2 days before start of menses and last for few days to 2 weeks. The
most severe symptoms are anger and irritability, which start earlier that other symptoms.
• Women with PMS tend to have have the same symptoms from one cycle to the next. Severe
PMS affects 3-8% women in reproductive age.
• Comorbidity with mood disorder is seen in 30-70%. The risk of developing premenstrual
depression and postnatal depression are higher in women with PMS.

Q 19 What proportion of women with history of depression relapse when they discontinue
antidepressants during pregnancy?
Select one:
90%
43%
68%
10%
25%

• In a naturalistic study investigating this issue, 43% women experienced a relapse of major
depression during pregnancy.
• The relapse rate was 26% for women who maintained their medication throughout their
pregnancy, compared to 68% in women who discontinued medication.
• Women who discontinued medication relapsed significantly more frequently over the course
of their pregnancy compared with women who maintained their medication (hazard ratio, 5.0;
95% confidence interval, 2.8-9.1;P<0.001). Cohen et al., JAMA 2006;295(5):499-
Q 20 Which of the following is not a feature of premenstrual dysphoric disorder according to
DSM-5 diagnostic criteria?
Select one:
Irritability
Guilt feelings
Depressed mood
Anxiety
Decreased interest in usual activities

• Premenstrual dysphoric disorder is thought to affect 2% -5% of premenopausal women.


• It was orignally included in the Appendix B of DSM-IV, but with mounting evidence for its
construct valitiy, it has been moved to the main text as a diagnosis in DSM-5.
• While many of its symptoms overlap with depressive episode, the physical symptoms of breast
pain and bloating are not seen in major depressive disorder.
• Similarly, guilt is associated with depression, not premenstrual dysphoria.

Q 21 A 32 year old patient, since delivering a baby boy, has been very tearful, low and avoiding
food. She spends most nights awake in a fearful state and for the last 2 weeks has fou nd it
difficult to breastfeed her baby and believes that her breasts are involuting. The most likely
diagnosis is
Select one:
Postpartum depression
Adjustment disorder
Conversion disorder
Maternity blues
Postpartum psychosis

Given the immediacy of onset and psychopathology suggestive of delusions, this lady is most
likely having postpartum psychosis.

Q 22 : The antidepressant with favourable evidence for use in postpartum breastfeeding


mother is
Select one:
Citalopram
Desipramine
Sertraline
Trazadone
Duloxetine

• Fluoxetine and its active metabolite norfluoxetine were detected in plasma and breast milk but
not in infant's plasma.
• Single case reports have indicated adverse effects such irritability, cyanosis, somnolence and
unresponsiveness with higher doses.
• It does not effect the development; does not cause cognitive dysfunction or neurological
abnormality. the Sertraline is first line of treatment in USA.
• The samples for Sertraline studies are large.
• Paroxetine has a lower milk/plasma ratio than fluoxetine and Sertraline. No studies of MAOI's
or Bupropion use in breast-feeding are available. MAOI's should be stopped in mothers planning
to breast-feed.

Q 23 Which of the following neurotransmitters has a role in the pathogenesis of premenstrual


syndrome?
Select one:
Nor Adrenaline
Dopamine
Adrenaline
Serotonin
Acetyl choline
• Etiology of PMS is unclear. The most accepted hypothesis is an increased sensitivity to normal
fluctuation of gonadal hormones.
• PMS is not caused by a drop in progesterone concentration. Serotonin appears to have a role
in the pathogenesis.
• This is supported by the observation that serotonin enhancing treatments reduce PMS
symptoms.
• Imaging studies suggest a role for GABA role in PMS; there is a physiological interaction
between progesterone metabolites and GABA-A receptors, which might have gone awry in PMS.

Q 24 : A 31 year old mother of a 3 weeks old baby scores high (>16) on the Edinburgh depression
rating scale. This increases the risk of
Select one:
delirium
spousal violence
suicidal ideations
harm to baby
psychotic symptoms

• Edinburgh Postnatal Depression Scale (EPDS) is an important screening instrument that is used
routinely with mothers during the postpartum period for early identification of postnatal
depression.
• High score suggests elevated self harm risk and clinical depression. The scale has no direct
questions regarding the child's safety.

Q 25 What drug should be immediately given to the mother and the neonate after delivery
when valproate or carbamazepine is used during pregnancy?
Select one:
Prophylactic Vitamin A
Prophylactic Vitamin D
Prophylactic thyroid supplements
Prophylactic Vitamin K
Prophylactic thiamine

• During pregnancy, if valproate or carbamazepine is prescribed; Prophylactic folic acid (5 mg


daily, from at least a month before conception) should be used.
• Prophylactic Vitamin K should be given mother and neonate after delivery when valproate or
carbamazepine is used.

Q 26 What is the relapse rate of depression in those who had a past history of depression when
they are pregnant?
Select one:
15%
50%
7%
1%
25%

• The relapse rate of depression in those who had a past history is around 50% when they are
pregnant.
• The high prevalence of antenatal depression may be partly due to relapse/recurrences and
partly due to freshly increased incidence of depression. (O'Keane and Marsh 2007)

Q 27 A 33-year-old pregnant depressed woman refuses pharmacotherapy for her depression.


She has severe depressive features and suicidal risk. Which of the following factors is most
relevant if she is considered for ECT?
Select one:
Increased protein binding in pregnancy
Decreased protein binding of drugs during pregnancy
Increased renal clearance during pregnancy
Delayed gastric emptying during pregnancy
Increased seizure threshold in pregnancy

• Complications of ECT during pregnancy are uncommon and transient.


• Barbiturates and atropine used for inducing anaesthetic state can reduce beat-to-beat
variability in the foetal heart rate, and atropine can cause foetal tachycardia.
• Uterine smooth muscle does not routinely contract during a seizure so premature labour is
almost unheard of.
• During pregnancy, prolonged gastric emptying time increases the risk of gastric regurgitation
and aspiration pneumonitis (Yonkers. et al 2004).

Q 28 What is the risk of developing spina bifida in the children of women treated with
carbamazepine during pregnancy?
Select one:
1-2%
0.5-1%
0.2
0.1
2-5%

• Carbamazepine is associated with a 0.5 -1% risk of spina bifida, craniofacial anomalies, growth
retardation and decreased average head circumference.
• There is some similarity between the teratogenic effects of carbamazepine and fetal hydantoin
syndrome (phenytoin exposure) probably related to the common arene oxide pathway through
which both drugs are metabolised.
• Hence the epoxide intermediate rather than carbamazepine itself may be the principal
teratogenic agent.

Q 29 Mrs. X suffers from severe insomnia following childbirth. She has no features of clinical
depression. Which sedative would be safe during breast feeding?
Select one:
Zovirax
Zorax
Zaleplon
Zopiclone
Combination therapy with thyroxine can be suggested
She must have clinical depression to have response to sertraline
Sertraline must be used at a higher than antidepressant dose

Advantages of intermittent SSRI dosing in PMT include a reduced withdrawal rate due to side
effects and being more effective than continuous dosing ( Wikander et al.1998).

Q 30 The disadvantages of intermittent dosing in the treatment of PMS is;


Select one:
Increased frequency and severity of side effects
Less effective than continuous dosing
Higher withdrawal rate due to side effects
Expensive

Less effective for somatic than for mood symptoms


• Advantages of intermittent dosing: - A. More effective than continues or semi-continues dosing
(Wikander et al.1998) B. Cheaper C. Fewer withdrawal rate due to side effects D. Less frequency
and severity of side effects
• Disadvantages of intermittent dosing: Less effective for somatic than for mood symptoms, Less
effective for somatic symptoms compared to continuous treatment

Q 31 The absolute spontaneous risk of developing Ebstein's anomaly is


Select one:
1 in 20000
1 in 1000
1 in 100000
1 in 2000
1 in 10000

The absolute spontaneous risk of Ebstein's is 1 in 20,000. Cohen et al., JAMA 1994;271:146-150

Topic: Recent advances


Q Which of the following is true regarding Metabolomics ?
a ) Metabolome is an accurate biochemical profile of the organism in health and disease
b)Metabolomics detects disequilibrium in the metabolome
c )It reflects genomic, transcriptomic and proteomic fluctuations
d ) It helps in.Identification of biomarkers for early AD diagnosis
e ) None of the above

Answer e ) None of the above

Q ) Which of the following is not a “ proposed”mono clonal antibodies for treatment of


alzheimer’s disease ?:
a )Bapineuzumab
b) Solanezumab
c) IVIG
d)Tramiprosate

Answer d) Tramiprosate

Q : Which of the following are not predictors of antidepressant response ?


A ) Genetic variation of FKBP5 gene
B ) Genetic variation of TREK 1 Gene
C ) Genetic variation of COMT gene

Answer b) Genetic variation of TRK 1 gene

Predictors of antidepressant response:


1.Genetic variation in FKBP5:
A protein that regulates cortisol binding to glucocorticoid receptor
2.Genetic variation in the (COMT) gene
Predictors of antidepressant non-response:
Genetic variants in TREK1:
A potassium channel mediating SSRI mechanism of action

Q : Which of the following is a triple reuptake inhibitor (NE,DA, 5HT ) ?


A) Lamotrigine
B) Vortioxetine
C ) Tesofensine
D ) Pramipexole

Answer )c ) Tesofensine

Q : Which of the following is non monoaminergic agent which has potential antidepressant
property ?
A ) Aprepitant
B ) Sibutramine
C )Tesofensine
D ) Sertraline
Answer A ) Aprepitant

Q : Nemifitide with regards to depression is :


A ) omega 3 fatty acid analogue
B ) Selective antagonist of NR2B subunit of NMDA receptor
C ) Melanocyte inhibiting factor with antidepressant properties
D ) Melanocyte stimulating factor with antide Melanocyte inhibiting factor with antidepressant
properties pressant properties

Answer : C ) Melanocyte inhibiting factor with antidepressant properties

Q : Which of the following is NOT a target for Deep Brain Stimulation of brain in depression :
a.Subgenual cingulate white matter
b.The anterior limb of the internal capsule
c.The habenula
d.Nucleus accumbens
e.Thalamic peduncle
f . None of the above

Ans : None of the above

RECENT ADVANCES IN TREATMENT OF DEPRESSION Focal Brain Stimulation

1. Vagus Nerve Stimulation: (VNS)


VNS is FDA-approved for long-term adjunctive treatment for recurrent or chronic depression not
responding to four or more medications

2. tDCS: Trans Cranial Direct Current Stimulation


Lateral PFC tDCS demonstrated greater antidepressant efficacy compared to occipital tDCS
(active control) and sham tDCS in a single double-blind, randomized, controlled study

3. rTMS: Repeated Trans cranial Magnetic Stimulation


High-frequency rTMS over the left (DLPFC) at adequate doses for a minimum of 10 sessions has
statistically significant antidepressant effects in treatment resistant depression

4. Magnetic seizure therapy: (MST)


MST involves using an rTMS device to create a generalized seizure
Antidepressant effects similar or equivalent to high-dose right unilateral ECT

5. Deep brain stimulation:


Produced by a subclavian subcutaneous pulse generator that connects to neuro-surgically
implanted electrodes that stimulate a focused region in the brain
DBS targets in depression:
1.Subgenual cingulate white matter
2.The anterior limb of the internal capsule
3.The habenula
4.Nucleus accumbens
5.Thalamic peduncle

Q : Which of the following is not true regarding “prodrome’’ of psychosis ?


A ) 35% of the individuals with prodromal symptoms convert to schizophrenia
B ) Generalized cognitive impairments marked by some degree of selective impairment in
olfaction and verbal memory
C ) accelerated fronto-temporal cortical GM volume reduction
D) decreased cortisol secretion

Ans : D ) decreased cortisol secretion

Q : Which of the following is D2 partial agonist and D3 antagonist :


A ) Tolcapone
B ) Lurasidone
C ) Cariprazine
D ) Adrogolide

Ans : c ) Cariprazine

Q ) Which of the following is true regarding change in classification of OCD in DSM 5 ?


A. No longer under anxiety disorders
B. Poor insight as specifier
C. Hoarding disorder as separate disorder
D. All of the above

Ans : All of the above

Q : Which of the following is not a deficit found in neuropsychological testing ( endophenotype


) in OCD ?
a) •Set-shifting ability (attention deficit)
b) •Response inhibition
c) •Non-verbal memory
d) •Cognitive flexibility
e) •Motor impulsivity
f) None of the above

Answer : f ) None of the above


Neuropsychological deficits ( endophenotypes )
Studies done on unaffected relatives of OCD patients commonly reveal
impairments in the following areas:
•Set-shifting ability (attention deficit)
•Response inhibition
•Non-verbal memory
•Cognitive flexibility
•Motor impulsivity
•These findings point towards involvement of dorso-lateral PFC and OFC
Q : Which of the following represents the “marked pathway” in the given diagram ?
A ) Direct pathway
B ) indirect pathway

Ans a ) Direct Pathway


Recent research ( Milad & Rauch 2012) suggests that additional pathways that are more spirally
related “cascading from one loop to the next”
•Medial and lateral orbitofrontal cortices – Reward processing and affect regulation
•dACC – Error processing and fear expression
•Amygdalo-cortical circuitry – Expression and regulation of fear

Q :Specific features of pediatric OCD include:


A )More compulsions with poor insight.
B )Shift in particular symptoms over time though often maintaining same symptom dimension.
C )Comorbidities like tic disorders and specific developmental disorders noted in pre-pubertal
patients.
D ) Commonly noticed with pts. with ADHD.
E ) All of the above
Ans : All of the above

Topic: Boderline and Antisocial Personality disorders


1. Borderline personality disorder shares phenotypic similarity with which major
clinicalsyndrome?
a. Schizophrenia
b. Depression
c. Cyclothymia
d. Bipolar disorder
2. According to tripartite etiopathogenetic model of borderline PD, which of the
following arenot included?
a. Childhood trauma
b. Vulnerable temperament
c. Childhood sexual abuse
d. Series of triggering events
3. Abnormality not seen in patients with borderline PD
a. Shortened REM latency
b. Abnormal DST results
c. Abnormal Thyrotropin releasing hormone
d. Abnormal gamma wave arousal
4. Which of these is not the characteristic of micro psychotic episode?
a. Derogatory
b. Circumscribed
c. Fleeting
d. Doubtful
5. Which of the following are not the clinical features seen in borderline disorder?
a. Franatic efforts to avoid real/ imagined abandonment
b. Unstable and intense interpersonal relationships
c. Feeling secure with human connections than non human objects
d. Transient paranoid ideation/ dissociative symptoms
6. Which of the following is not true about borderline PD?
a. Primary etiological factors are: early traumatic events and biological
vulnerabilities.
b. Premature death can happen due to suicide
c. PTSD can be one of the co-morbid disorder
d. Cyclothymia cannot be considered as differential diagnosis
7. Not true about prevalence rates of Borderline PD
a. 2% in general population
b. 5% among patients with PDs
c. 10% in psychiatric out patients
d. 20% in psychiatric in patients
8. State true or false: Diagnosis of Borderline PD is more common in elderly than younger
individuals?
a. True
b. False
9. Defense mechanism not seen in borderline PD?
a. Splitting
b. Projection
c. Identification
d. Suppression
10. Which of the following is not the future prognosis of Borderline PD
a. Schizophrenia
b. Frequent job loss
c. Interrupted education
d. Marital disharmony
11. Psychotherapy not used in borderline PD
a. Mentalisation based treatment
b. Affective regulation therapy
c. Dialectical behaviour therapy
d. Transference focused psychotherapy
12. Which of the following therapy is superior to others in treatment of borderline PD?
a. Dialectical behaviour therapy
b. Transference focussed therapy
c. Psychodynamic psychotherapy
d. None is superior to other
13. Which of the following are not part of DBT used for Borderline PD
a. Behaviourism
b. Existential approach
c. Zen principles
d. Dialectical philosophy
14. True about Antisocial PD is?
a. Highest prevalence is found among women with alcohol use disorder
b. Among prison population it is 50% prevalent
c. Boys with this disorder come from larger families than girls with this disorder
d. Onset is after 20 years
15. Answer true or false: familial pattern of inheritance is seen in Antisocial PD
a. True
b. False
16. What neurological findings can be encountered in patients with Antisocial PD?
a. Abnormal EEG and soft neurological signs
b. Absence of corpus callosum
c. Aberrant middle cerebral artery supply
d. Erratic pattern of beta activity
17. Following are the clinical features of antisocial PD, except?
a. Impulsivity
b. Promiscuity
c. Inflated self esteem
d. Ability to maintain monogamous relationship
18. Not true about prevalence rates of antisocial PD
a. 3:1 for females:males
b. 3 to 30% in clinical setting
c. Higher frequency in low socio economic status
d. Under-diagnosed in females
19. Not a differential diagnosis for antisocial PD
a. Narcissistic PD
b. Paranoid PD
c. Borderline PD
d. Histrionic PD
20. Antisocial personality disorder shares phenotypic similarity with which major
clinicalsyndrome?
a. Delusional disorder
b. Mania
c. Cyclothymia
d. OCD
21. Points to be taken care while doing psychotherapy for Antisocial PD?
a. They are amenable for treatment when immobilised (IP based setting)
b. When among peers, their motivation for change appears
c. Self help groups are more useful then jails in alleviating the disorder
d. All of the above are true
22. Which of the following are true about pharmacotherapy in Antisocial PD?
a. Medications must be used judiciously
b. Methylphenidate can be used if there are symptoms of ADHD
c. Antiepileptics can be used to control impulsive behaviour
d. All the above are true
Topic: TRANSSEXUALISM
1. An individual who was assigned female at birth but who currently identifies as a man is termed
a

A) cisgender man.

B) cisgender woman.

C) transgender man or transman.

D) transgender woman or transwoman.

DEFINITIONS

Transgender man (transman): A transgender individual who, assigned female at birth, currently
identifies as a man. In this course, the terms transgender man, female-to-male transgender
person, and FTM are used interchangeably.

2. In the DSM-5, individuals who experience distress based on the incongruence between their
experienced or expressed gender and their assigned gender fall under the diagnosis of

A) transsexualism.

B) transgenderism.

C) gender dysphoria.

D) gender identity disorder.

HISTORICAL PERSPECTIVE

The DSM-IV abandoned the term "transsexualism" and instead used the term "gender identity
disorder" [10]. The DSM-5 now uses the term "gender dysphoria." This is defined as "the distress
that may accompany the incongruence between one's experienced or expressed gender and
one's assigned gender". The critical element of this diagnosis is the presence of clinically
significant distress associated with the condition, as the American Psychiatric Association (APA)
points out that gender nonconformity is not a mental disorder. The intent of this change was to
better characterize the experiences of affected children, adolescents, and adults and to avoid
stigma and ensure clinical care for individuals who see and feel themselves to be a different
gender than their assigned gender.

3 . While the definitive etiology for gender dysphoria has not been found, evidence seems to
indicate

A) physiologic causes.

B) psychologic causes.

C) hormonal imbalance.

D) chromosomal abnormalities.

ETIOLOGY OF GENDER DYSPHORIA

At present, scientific investigation has neither established the true incidence nor clarified the
etiology of non-conforming gender identity formation. Genetic, physiologic, and psychologic
causes have been investigated, and available evidence seems to support physiologic causes.
Gender dysphoria cannot be explained by variations in chromosomal patterns or identifiable
hormonal abnormalities, nor is there convincing evidence that psychologic factors (e.g., being
exposed to certain family dynamics or being raised as a member of the opposite sex) cause this
condition .

4 . Difficulties in trying to estimate the prevalence of transgender individuals are the result of

A) the variety of terminology used.

B) the differences in presenting symptoms.

C) a reluctance to disclose this because of the social stigmatization related to this


condition.

D) All of the above

PREVALENCE OF TRANSGENDER INDIVIDUALS

Because of variability in terminology and definitions, differing modes of presentation, and


reluctance to disclose for fear of social stigmatization, it is difficult to obtain accurate data about
the prevalence of transgender individuals in the general public . However, as society becomes
more accepting and familiar with transgender individuals, more transgender individuals are
willing to "come out." The result is that this condition is more prevalent than previously thought.

5 . Transgender individuals have a higher incidence of HIV for all the following reasons, EXCEPT:

A) Unprotected sex
B) Intravenous drug use

C) Sharing needles for illicit hormone injections

D) Blood transfusions associated with gender-confirmation surgery (GCS)

UNIQUE HEALTHCARE ISSUES

Transgender individuals are at risk for HIV as a result of engaging in unprotected sex and from
sharing needles used for hormone injections. A significant percentage of transgender individuals
engage in sex work; 11% of transgender respondents in a large national survey reported engaging
in sex work for income, compared with 1% of women in the United States. This survey has been
the largest to date of the transgender population, involving 6,450 transgender and gender non-
conforming individuals across all 50 states, as well as the District of Columbia, Puerto Rico, Guam,
and the U.S. Virgin Islands. The study was conducted on behalf of the National Gay and Lesbian
Task Force and the National Center for Transgender Equality.

6 . The Veterans Administration does NOT provide transgender veterans with

A) GCS.

B) postsurgical care.

C) hormone therapy.

D) mental health care.

ACCESS AND BARRIERS TO CARE

Before 2014, the Centers for Medicare and Medicaid Services (CMS) did not cover the cost of GCS
based on a 1981 evaluation that described the surgery as experimental and cited "the lack of
well-controlled, long-term studies of the safety and effectiveness" and "a high rate of serious
complications". However, long-term studies done in Europe have found that GCS is effective, with
low complication rates. This policy banning GCS was overturned in May 2014, but because policy
guidelines are issued from Medicare contractors by region, widespread implementation has been
slow . While the Veterans Administration (VA) has policies in place to provide comprehensive
care to transgender veterans, including ongoing hormone therapy, mental health care, and long-
term care following GCS, the VA specifically does not cover GCS, on the basis of a VA regulation
excluding gender alterations from the medical benefits package.

7. The MAIN REASON transgender individuals seek psychologic therapy is to

A) come out to self, work, and family.

B) obtain a referral for hormone therapy.

C) understand the meaning of their feelings.


D) follow up on information found on the Internet.

PSYCHOLOGIC MANAGEMENT

While obtaining a referral for hormone therapy is the main reason that transgender individuals
seek psychologic therapy, other reasons include understanding the meaning of their feelings;
whether to externally express those feelings; coming out to self, work, and family; seeking to
network following a negative experience; or following up on information obtained on the
Internet. In some cases, an individual may have been "caught" expressing his or her gender
variance by a spouse or significant other or when there has been a program on transgender issues
in the popular media . Coming out trans is a time of heightened vulnerability . Not everyone
transitions. Some learn to live comfortably in their role consistent with their assigned birth sex,
and others may transition partially or totally.

8. Which of the following is a predictor of suicide among transgender individuals?

A) Parental rejection

B) Past psychiatric hospitalizations

C) Previous suicide attempts or near attempts

D) All of the above

PSYCHOLOGIC MANAGEMENT

Research on the incidence of suicide in transgender people is scarce, but transgender people are
believed to have similar suicide risks as other people who experience major life changes,
relationship difficulties, chronic medical conditions, or discrimination on the basis of minority
status. The incidence of suicide ideation is as high as 64% and suicide attempts as high as 38% in
the adult transgender population . These rates are significantly higher than in the general
population . Predictors of suicide among transgender individuals are similar to those of the
general population, most notably previous suicide attempts or near attempts, past psychiatric
hospitalizations, and past psychiatric treatment. Parental rejection is one of the highest risk
factors for suicide among transgender youth .

9 . In the MTF individual, estrogen therapy

A) increases libido.

B) increases muscle mass.

C) induces breast development.

D) causes an increase in vocal pitch.


HORMONAL AND NONSURGICAL MANAGEMENT

There are several choices of hormone therapy for the MTF transition and maintenance. The goals
of hormone therapy prescribed for MTF transsexuals are to induce breast formation and a more
female distribution of fat and to reduce male-pattern hair growth while neutralizing the biologic
effects of endogenous androgens. Additional effects include decreased muscle mass and
strength, softening of skin, decreased libido, decreased sperm production and testicular volume,
and less frequent and less firm erections. Commonly, hormonal treatment of MTF individuals will
include an anti-androgen along with an estrogen.

10. Side effects of the anti-androgen medication spironolactone include all of the following,
EXCEPT:

A) Hypotension

B) Hypokalemia

C) Gastrointestinal upset

D) Increased urinary output

HORMONAL AND NONSURGICAL MANAGEMENT

Anti-androgens reduce endogenous testosterone levels, allowing the full effect of estrogen
therapy. The anti-androgen of choice in the United States is usually spironolactone (Aldactone),
a potassium-sparing diuretic that directly inhibits testosterone secretion and inhibits androgen
binding to the androgen receptor. It is usually given orally at a dose of 50 mg twice daily . Side
effects of spironolactone include gastrointestinal upset, hyperkalemia, increased urinary output,
and hypotension. It is contraindicated in patients with renal insufficiency or with serum
potassium levels greater than 5.5 mEq/L. Spironolactone should not be given with digoxin,
angiotensin-converting enzyme inhibitors, other potassium-sparing diuretics, and angiotensin
receptor blockers . Patients taking spironolactone should have baseline levels of electrolytes,
blood urea nitrogen (BUN), and creatinine, and repeat levels in two months or at every dose
change, and then every six months when the dose is established .

11. Estrogen administered sublingually, transdermally, or parenterally

A) is easy to titrate or stop.

B) has a higher incidence of side effects.

C) avoids first pass metabolism and has less effect on liver enzymes.

D) has increased vascular complications in patients older than 40 years of age.

HORMONAL AND NONSURGICAL MANAGEMENT


Estrogen can be given orally, transdermally, or parenterally, and each type of delivery has
advantages and disadvantages. Estrogen given sublingually, transdermally, and parenterally
avoids first-pass metabolism (with less effect on liver enzymes) and is associated with fewer
vascular complications in patients older than 40 years of age . Oral forms of estrogen have the
additional advantage of being easy to titrate or stop. One study of long-term cross-sex hormone
usage revealed that the use of ethinyl estradiol in MTF individuals was associated with an
increased risk of death from thrombotic events. This is one reason that the Waddell protocols
and WPATH Standards of Care no longer recommend ethinyl estradiol as a safe medication for
feminizing hormone therapy.

12 . The goal of voice therapy for MTF individuals is

A) changing the choice of language.

B) changing intonation (the rhythm of speech).

C) raising or "feminizing" the resonance of the voice.

D) All of the above

HORMONAL AND NONSURGICAL MANAGEMENT

Voice therapy involves the conscious manipulation of the vocal mechanism to produce a more
feminine-sounding voice . The goal of therapy is not merely raising the pitch of the voice, but also
"feminizing" the resonance, intonation (rhythm of speech), rate of speech, volume, intensity,
choice of language, articulation, and social rules of communication. The process includes an
assessment of the individual's current voice, resonance, articulation, spoken language, and non-
verbal communication. The patient is also assessed for vocal health and current practices that
may be damaging to the voice.

13 . In order to be perceived as female, the fundamental frequency of the voice should be

A) 120 Hz (110 Hz–130 Hz).

B) 140 Hz (130 Hz–150 Hz).

C) 172 Hz (165 Hz–180 Hz).

D) 190 Hz (180 Hz–200 Hz).

HORMONAL AND NONSURGICAL MANAGEMENT

Feminine voices have a higher pitch and breathier vocal quality, and they resonate at higher
frequencies and have different speech rates, inflections, and intonations. The fundamental
frequency (pitch) of the adult male voice is about 100 Hz, with a range of 77–482 Hz; the adult
female voice has an average frequency of 195 Hz, with a range of 137–634 Hz. There is
considerable overlap between the two ranges, but in order to be perceived as female, the
fundamental frequency should be around 172 Hz (165–180 Hz). According to a study by King et
al., a mean speaking pitch above 180 Hz and maintaining a speaking pitch range of approximately
140–300 Hz appear to be the most powerful acoustic features or markers in the perception of a
female voice in MTF individuals . Raising the fundamental frequency near this range is one of the
goals of voice therapy.

14. In the FTM individual, testosterone except

A) decreases libido.

B) decreases skin oiliness.

C) increases muscle mass.

D) results in significant breast atrophy.

HORMONAL AND NONSURGICAL MANAGEMENT

The goal of hormone therapy for FTM individuals is masculinization of the body through the use
of testosterone . Testosterone therapy in FTM individuals results in cessation of menses within
three to five months due to suppression of the hypothalamic-pituitary axis. However, ovulation
may continue, and pregnancies have been reported in FTM persons even after prolonged
testosterone treatment . As such, it is important to discuss contraception with FTM individuals
who have sex with men. Other effects of testosterone therapy include increased libido, increased
facial and body hair, increased skin oiliness, increased muscle, mild breast atrophy, and
redistribution of fat mass, usually within three months of the initiation of testosterone therapy.

15. Which of the following physiologic changes due to testosterone therapy is reversible?

A) Clitoromegaly

B) Increased libido

C) Deepened voice

D) Male pattern hair loss

HORMONAL AND NONSURGICAL MANAGEMENT

Within one year, the voice usually deepens, clitoromegaly occurs, and male pattern hair loss may
be apparent . After 13 years of testosterone use, androgenic alopecia occurs in 50% of FTM
individuals . On average, the clitoris enlarges to 3–5 cm, and in some cases, testosterone may be
applied topically to the clitoris to stimulate growth . These changes in voice range, hair follicles,
and clitoral size are permanent. Other effects, such as increased muscle mass, acne, increased
libido and energy level, and amenorrhea are reversible if testosterone is discontinued . FTM
individuals report a better quality of life after receiving male hormones regardless of the duration
of treatment .
16. Testosterone should be used with caution in individuals with

A) pregnancy.

B) hyperlipidemia.

C) endometrial cancer.

D) estrogen receptor-sensitive breast cancer.

HORMONAL AND NONSURGICAL MANAGEMENT

Contraindications to testosterone include estrogen receptor-sensitive breast cancer; uterine,


endometrial, and/or ovarian cancer; pregnancy; and hypersensitivity to sesame or cottonseed
oil, if injectable testosterone is used. Testosterone should be used with caution in individuals
with uncontrolled coronary heart disease or any unstable heart disease, hyperlipidemia,
diabetes, liver disease, cigarette smoking, extreme obesity, hypertension, kidney failure,
prolactinoma, or active thyroid disease .

17 . Spontaneous vaginal bleeding in an FTM individual on testosterone is commonly caused by

A) pregnancy.

B) weight loss.

C) vaginal atrophy.

D) excessive testosterone.

HORMONAL AND NONSURGICAL MANAGEMENT

Spontaneous vaginal bleeding may be caused by missed testosterone doses, excessive


testosterone, weight increase, or thyroid disorders. Atrophic vaginitis may occur, resulting in
bacterial vaginitis or candidiasis. If bacterial vaginitis is suspected, the vagina should be cultured
for atypical bacteria . Pelvic cramping has been noted in some people on testosterone for longer
than three to six months, in some cases associated with orgasm. The pain typically lasts 10 to 15
minutes and can be alleviated by pre-medicating with nonsteroidal anti-inflammatory drugs .

18. The criteria for metoidioplasty or phalloplasty in FTM individuals and for vaginoplasty in MTF
individuals include all of the following, EXCEPT:

A) Two referrals from mental health professionals

B) Orchiectomy in MTF individuals and salpingo-oophorectomy in FTM individuals

C) 12 continuous months of hormone therapy as appropriate to the patient's gender


goals
D) 12 continuous months of living in a gender role that is congruent with the patient's
gender identity

SURGICAL INTERVENTIONS

At some point in transition, transgender individuals typically seek surgical interventions to meet
their goals to masculinize/feminize the body. Although obtaining a true estimate of the number
of individuals undergoing surgical procedures are part of gender transition is difficult, available
research indicates that more transgender patients are having surgery . The American Society of
Plastic Surgery’s 2016 annual report included 1,759 MTF and 1,497 FTM patients undergoing GCS,
an increase of 27% and 10%, respectively, compared with 2015 . In addition, many surgeries are
performed by other surgical subspecialties (e.g., urologists, obstetricians/gynecologists,
maxillofacial surgeons). A large-scale survey found that among FTM individuals, 21% had chest
reconstruction, 8% hysterectomy, 1% metoidioplasty, and 1% phalloplasty; among MTF
individuals, 10% had vaginoplasty or labiaplasty, 9% orchiectomy, 8% augmentation
mammoplasty, and 6% facial surgery .

• The WPATH guideline suggests separate criteria for non-genital and genital surgery for
transgender individuals. The criteria for non-genital surgery include [68]:

➢ One referral from a mental health professional


➢ Persistent, well-documented gender dysphoria
➢ Capacity to provide informed consent
➢ Age of majority
➢ Reasonable control of significant medical or mental health concerns, if present

• Criteria for genital surgery (except for metoidioplasty or phalloplasty in FTM individuals
and vaginoplasty in MTF individuals) include those outlined for non-genital surgery as well
as:

➢ Two referrals from mental health professionals


➢ 12 continuous months of hormone therapy as appropriate to the patient's gender goals
(unless the patient has a medical contraindication or is otherwise unable or unwilling to
take hormones)

The criteria for metoidioplasty or phalloplasty in FTM patients and for vaginoplasty in MTF
patients include all of these criteria plus 12 continuous months of living in a gender role that is
congruent with his/her gender identity. This criterion is based on expert clinical consensus that
"real-life" experience allows the patient to experience and socially adjust to his/her desired
gender role before undergoing irreversible surgery [68]. Occasionally, patients may have
unrealistic expectations about what it means to be a member of the opposite sex; it is important
that these are brought to light before surgery is done. The real-life experience has been shown
to reduce gender dysphoria and improve social and sexual functioning .

19. Of the following, the most common complication of breast augmentation surgery is

A) hematoma.

B) serious infection.

C) capsular contraction.

D) malpositioned implants

SURGICAL INTERVENTIONS

The most common complications of breast augmentation surgery include the general surgical
complications of bleeding, or hematoma. Complications specific to augmentation mammoplasty
are capsular contraction, asymmetry, malpositioned implants, and altered nipple sensation.
Infection is rare.

20. An MTF individual is preparing for a chondrolaryngoplasty (tracheal shave) to reduce the size
of the laryngeal prominence. The provider realizes she needs more education when she states,

A) "I might have a scar following the procedure."

B) "I might have a sore throat following the procedure."

C) "This procedure will also raise the pitch of my voice."

D) "The surgeon will place the incision in my neck crease."

SURGICAL INTERVENTIONS

A chondrolaryngoplasty (tracheal shave) may be requested to remove or reduce the laryngeal


prominence (Adam's apple). This procedure is not designed to feminize the voice—only to alter
the appearance. A small incision is made on the upper crease of the neck or in a wrinkle in the
skin. Using a laryngoscope, the vocal cords are visualized and this location is marked externally.
The surgeon then exposes the thyroid cartilage and removes all of the prominent cartilage and
its borders above the vocal cord marking. The incision is then closed and cleaned and a bandage
is placed over the incision. During the first 24 to 48 hours after the surgery, it is common to
experience some bruising and swelling as well as a sore throat. Discomfort is typically minor;
however, pain medication is often given in order to keep the patient as comfortable as possible.
Complications may include a scar, changes in vocal quality, pain, and difficulty swallowing .
21 . Metoidioplasty

A) allows penetrative intercourse.

B) always allows urination while standing.

C) has fewer complications than phalloplasty.

D) results in less erogenous sensation than phalloplasty.

SURGICAL INTERVENTIONS

In some cases, the clitoris becomes sufficiently hypertrophied after testosterone exposure to
serve as a microphallus. Otherwise, a phalloplasty or metoidioplasty may be performed (Table
6). The choice of technique may be restricted by anatomical or surgical considerations or the
patient's financial considerations. If a patient wishes to have a phallus of good appearance, the
ability to urinate while standing, sexual sensation, and/or coital ability, there are several separate
stages of surgery and frequent technical difficulties that may require additional operations. Even
metoidioplasty, which in theory is a one-stage procedure for construction of a microphallus,
often requires more than one surgical procedure, and the goal of standing micturition with this
technique cannot always be ensured .

Metoidioplasty involves elongation and reconstruction of the hormonally enlarged clitoris as a


small neophallus with erectile function, analogous to penile tissue. During this procedure, the
clitoris is released from its surrounding tissue and a flap of skin from the labia minora is wrapped
around the clitoris to create a small phallus. The resulting neophallus is sensate. With ring
metoidioplasty, a variation of the simple metoidioplasty, the urethra is lengthened using a flap
of tissue from the anterior vaginal wall and labia minora to create the urethral extension. This
carries urine to the distal end, similar to a natural penis. This procedure is less complex than a
phalloplasty, has fewer complications, and has the benefit of providing greater erogenous
sensation. However, the resulting neophallus is often not large enough to use for sexual
penetration. Using this technique results in an average phallus length of 5.7 cm, with a range of
4–10 cm [124]. Testicular implants may be placed in the labia majora as part of the procedure.

22. An MTF individual, 51 years of age, has been on estrogen for 30 years and had GCS and
augmentation mammoplasty with saline implants 20 years ago. What breast cancer screening is
recommended for this patient?

A) She should have an MRI instead of a mammogram.

B) She should have an annual breast exam, including mammogram.

C) As breast cancer risk is very low, a mammogram is not indicated.

D) She should have a breast ultrasound instead of a mammogram.


LIFELONG PREVENTIVE AND PRIMARY CARE

Annual breast exam, including mammogram, is indicated in MTF women who are older than 50
years of age who have been taking estrogen for five or more years. While this is recommended
as a screening protocol, the actual risk is likely no different than in natal males. In a series of
2,200 MTF individuals studied between 1975 and 2005, there were no cases of breast cancer
reported.

23. Which of the following should be included in the annual exam of an FTM individual who is 42
years of age, has used testosterone for 10 years, and has had a bilateral mastectomy but no
"bottom" surgery?

A) Prostate exam

B) Visual field exam

C) Pelvic exam and Pap test

D) Dual-energy x-ray absorptiometry screening

LIFELONG PREVENTIVE AND PRIMARY CARE

For patients using androgen therapy who have not had a complete hysterectomy, there may be
an increased risk of endometrial and ovarian cancer . Therefore, a regular Pap test is indicated. It
is important to be sensitive when performing a pelvic exam on a transsexual man. As noted, FTM
individuals often experience emotional and psychologic distress with pap testing due to gender
dissonance. This may be given as a reason for avoiding gynecologic examinations altogether . It
may be difficult to get a good sample, as the cervix will be atrophic; intravaginal estrogen cream
can be used a few days prior to the Pap test to improve results.

24. Which of the following models of therapy for children who display gender dysphoria
encourages the child's exploration of gender identity?

A) Affirming approaches

B) Supportive approaches

C) Corrective approaches

D) Psychoanalytic approaches

MANAGEMENT OF TRANSGENDER CHILDREN AND ADOLESCENTS

In general, psychologic therapy and support can assist transgender children/adolescents to


complete developmental tasks on schedule, achieve self-acceptance and understanding, and
manage and cope with social problems (e.g., peer and/or family conflict) and the stress of the
change process [141]. Three models of therapy for children who display gender dysphoria have
been described. The first of these is affirming approaches, based on the concept that being
transgender is not a mental illness [27]. This approach encourages the child's exploration of
gender identity and assists the child and his or her family to explore interventions such as social
transitioning and hormone therapy. Dreger defines this approach as the "accommodation" mode
[143]. The second treatment approach involves supportive therapies—a "wait and see"
approach to determine how the child's gender identity unfolds. With this approach, there are no
gender-related interventions. The third approach is a "corrective" approach that seeks to align
the child's gender identity with his or her biological sex. This approach is also described by Dreger
as the "therapeutic" model that views the child's gender dysphoria in terms of familial
dysfunction and seeks to guide the child into a less stressful, more sustainable family
environment and gender identity. This last approach has been generally dismissed or condemned
by organizations like the APA .

Topic: IQ TESTING

1. Who designed the first IQ test? What was the basis for it?
Alfred Binet and Theodore Simon, in response to a change in education laws. They wanted to
identify children with lower intelligence (1905)

Lewis Terman adapted it for use in the US, named it the Stanford-Binet test (1916)

Indian adaptation by Dr. V. V. Kamat, 1958. Three languages: Hindi, Kannada, Marathi. Used for
ages 3-22. Basal age, terminal age

2. Fundamental difference between WAIS and SBT


SBT- Based on the concept of ‘g’, a single factor of intelligence. “Mental age”- looks at how a
specific individual, at a specific age, performs intellectually, compared to average intellectual
performance for that individual's actual chronological age. Earliest form was a ratio IQ

Weschler scales: replaci the ratio IQ with the deviation score, a method that calculated IQ by
converting the sum of subtest scores into a standard score, using the mean and standard
deviation at each age level.

MCQS
1. The following are tests of IQ except (c,e,g)
a. WISC
b. WAIS
c. AVLT
d. Stanford Binet test
e. California Verbal Learning test
f. Woodcock Johnson Psychoeducational Battery
g. VSMS

➢ Rey’s AVLT and California VLT are tests of memory


➢ VSMS is a test of social and adaptive functions or social competency. It gives a
Social Quotient, or SQ. The SQ correlates highly with IQ (0.8-0.9), but is not a
measure of intelligence
➢ It has 2 Indian adaptations, Malin (1965) and Bharat Raj (1992), which are both
slightly different.
➢ Can be used in the age group 0-15 years
2. What are hold tests
➢ Hold tests are neuropsychological tests which tap abilities which are thought to
be largely resistant to cognitive declines following neurological damage.
➢ Used to test premorbid intelligence following neurological damage
➢ Magnitude of difference between current and premorbid intelligence is an
important prognostic factor for cognitive functioning and degree of rehab that can
be attempted
➢ Hold tests measure crystallised intelligence, i.e; stored knowledge and skills.
Examples of crystallised intelligence include vocabulary, general information,
abstract word analogies, and mechanics of language.
➢ Examples of Hold tests:
▪ National Adult Reading Test (NART)
▪ North American Adult Reading Test (NAART)
▪ Picture Completion subtest of Wechsler Adult Intelligence Scale
▪ Similarities subtest of Wechsler Adult Intelligence Scale
▪ Wechsler Test of Adult Reading (WTAR)
▪ Wide Range Achievement Test

3. Is there a standardized Indian adaptation of WAIS-IV? (a)


a. Yes
b. No

4. The WAIS can be used in patients with neurological damage. (a)


a. Yes
b. No

5. What is the age range of the WAIS-IV (a)


a. 16-90
b. 18-90
c. 16-75
d. 18-75
For individuals under 16 years, the Wechsler Intelligence Scale for Children (WISC, 6 –16 years)
and the Wechsler Preschool and Primary Scale of Intelligence (WPPSI, 2½–7 years, 7 months) are
used.

6. The following two subscales of WISC/WAIS are used in the Weschler Abbreviated Scale of
Intelligence (a,b)
a. VCI
b. PRI
c. PSI
d. WMI

➢ The Wechsler Abbreviated Scale of Intelligence (WASI-II; 2011) is an abbreviated measure


of intelligence for individuals 6 to 90 years of age.
➢ It can be used in clinical, psychoeducational, and research settings when administration
of a full battery is not feasible or necessary
➢ The WASI-II consists of four subtests
o Vocabulary and Similarities form the Verbal Comprehension Index (VCI)
o Block Design and Matrix Reasoning form the Perceptual Reasoning Index (PRI)
➢ All four subtests are combined to form the Full-Scale IQ-4 Subtests (FSIQ-4)
➢ Vocabulary and Matrix Reasoning are combined to form the Full-Scale IQ-2 Subtests
(FSIQ-2).

7. Which of the following are important in determining intelligence? (b,d,e,h)


a. Degree of connectivity between the 2 hemispheres
b. Increased cortical gyrification
c. Global cortical thickness
d. Regional cortical thickness
e. Cortical functioning
f. Cortical surface area
g. WM tracts of midbrain
h. Parietofrontal integration

Neural correlates of intelligence:

➢ Brain volume: larger brains associated with higher intelligence


➢ Brain areas: the structure of frontal Brodmann areas 10, 45–47, parietal areas 39 and 40,
and temporal area 21, frontal eye field, orbitofrontal area, inferior and middle temporal
gyrus, parahippocampal cortex and auditory association cortex positively contribute to IQ
scores
➢ Developmental trajectory: Children with higher IQ demonstrate a particularly plastic
cortex, with an initial accelerated and prolonged phase of cortical increase and equally
vigorous cortical thinning by early adolescence
➢ Functional activation of structures:
o Lateral frontal cortex, with its role in reasoning, attention and working memory,
supports fluid intelligence, but the parietal lobe is also implicated
o Fluid intelligence is strongly linked to both function and structure of frontal lobe
regions
o Crystallized intelligence depends more on the cortical structure and cortical
thickness in lateral areas of temporal lobes and temporal pole
➢ White Matter:
o IQ scores significantly correlate with the integrity of multiple white matter tracts,
especially in right uncinate fasciculus that connects parts of temporal lobe with
the frontal lobe areas
o Specific patterns of white matter dysconnectivity are associated with heritable
general cognitive and psychopathology factors
o Changes in white matter integrity through the life span also affect intelligence
through reduced information processing speed
➢ Genes:
o Intelligence is a highly heritable trait
o It is polygenic, with genetic influences attributed to miniscule effects by a large
number of genes
o The majority of associated genes are implicated in early, most probably prenatal
development,
o Some of the associated genes are essential for synaptic function and plasticity
throughout lifespan
o Genes associated with educational attainment and intelligence preferentially
express together in nervous tissue. Specifically, hippocampal, midbrain and
generally cortical and frontal cortical regions show the highest enrichment of
expression of these genes

8. A “superior” IQ on Weschler scale indicates a score of (c)


a. >120
b. >130
c. 120-129
d. >110
9. The concept of fluid and crystallised intelligence was given by (d)
a. Spearman
b. Binet
c. Piaget
d. Cattell

10. The indices on WAIS-IV include all except (b, f)


a. Verbal Comprehension Index
b. Verbal Fluency Index
c. Perceptual Reasoning Index
d. Working Memory Index
e. Processing Speed Index
f. Memory Index

11. The final WAIS-IV score is


a. Full scale IQ (FSIQ)
b. General Ability Index (GAI)
c. General Intellectual Ability (GIA)
d. Adaptive Intelligence (IA)

The final result of the WAIS-IV is reported as General Intellectual Ability, and is derived from:

• Full Scale IQ (FSIQ), based on the total combined performance of the VCI, PRI, WMI, and
PSI
• General Ability Index (GAI), based only on the six subtests that the VCI and PRI comprise.

12. The following is true about neural efficiency (b)


a. More intelligent individuals show more cerebral activation during tasks
b. More intelligent individuals show less cerebral activation during tasks
c. Neural efficiency is not dependent on task difficulty
d. Less intelligent individuals show greater activation of PFC during difficult tasks

Topic: COGNITIVE DISTORTIONS

1) Aspecific event is seen as being characteristic of life in general rather than as being one event
among many:

A. Fortune Telling

B. Mind Reading

C. Dichotomous Thinking

D. Overgeneralization

2.Positive characteristics or experiences are treated as real but insignificant:

A. Disqualifying the Positive

B. Minimization

C. Mind Reading

D. Fortune Telling

3) Attaching a global label to oneself rather than referring to specific events or actions:
A. Minimization

B. Selective Abstraction

C. Labeling

D. Mind Reading

4)The individual reacts as though his/her negative expectations about future events are
established facts:

A. Selective Abstraction

B. Dichotomous Thinking

C. Mind Reading

D.Fortune Telling

5) The use of "should" and "have to" statements to provide motivation or control behavior:

A. Dichotomous Thinking

B. Overgeneralization

C. Personalization

D. Should Statements

6)One aspect of a complex situation is the focus of attention, and other r elevant aspects of the
situation are ignored:

A. Catastrophizing

B. Selective Abstraction

C. Minimization

D. Labeling

7) Positive experiences that would conflict with the individual's negative views are discounted by
declaring that they "don't count":

A. Disqualifying the Positive

B. Minimization
C. Personalization

D. Emotional Reasoning

8)Negative events that might occur are treated as intolerable catastophes rather than being seen
in perspective:

A. Dichotomous Thinking

B. Catastrophizing

C. Disqualifying the Positive

D. Minimization

9) Things are seen in terms of two mutually exclusive categories with no "shades of gray" in
between:

A. Minimization

B. Overgeneralization

C. Dichotomous Thinking

D. Emotional Reasoning

10) Assuming that emotional reactions necessarily reflect the true situation:

A. Dichotomous Thinking

B. Minimization

C. Personalization

D. Emotional Reasoning

11) Assuming that one is the cause of a particular external event when, in fact, other factors are
responsible:

A. Personalization

B. Selective Abstraction

C. Minimization

D. Disqualifying the Positive


12) The individual assumes that others are reacting negatively without evidence that this is the
case:

A. Selective Abstraction

B. Mind Reading

C. Catastrophizing

D. Overgeneralization

Topic: DBT

1) What is the full form of DBT Dialectical Behavioural Therapy

2) DBT was introduced to treat which condition? Suicidal Behaviour in Borderline


Personality Disorders

3) DBT was designed by Marsha Linehan

4) DBT is a type of
a. Behaviour Therapy
b. Cognitive Therapy
c. Mindfulness Therapy
d. Cognitive Behaviour Therapy

5) DBT involves treatment in which of the following modes?


a. Individual
b. Group
c. Family
d. A & B
e. A & C
f. All of the above

6) % of patients from outpatient mental health services have clinical presentationof


Borderline Personality Disorder
a. 5-6%
b. 8-10%
c. 11-20%
d. 21-30%
7) DBT is based on Dialectical and Biosocial Theory of Psychological Disorders that
emphasises regulation of emotions in which direction?
a. Under control
b. Over control
c. Both
d. None

8) How many modes of treatment are there in DBT?


a. 2
b. 3
c. 4
d. 5

9) Enumerate the modes of treatment used in DBT Individual Therapy


Group Skills Training Phone Coaching
Therapist Consultation Team

10) What does notion of dialectics refer to?


Reality is not static, but comprises of internal and opposing forces out of which
integration occurs. This change is continuous and integral to life.

11) Sets of dialectical dilemma that patients exhibit


a. Emotional vunerability vs self-invalidation b.
c.
d. Active passivity vs apparent competence
e. Unrelenting crises vs inhibited grieving
f. A & B
g. A & C
h. All of the above
Emotional vulnerability vs self-invalidation: Tendency to vacillate between intense,
uncontrollable emotional suffering and dismissal, judgment, and invalidation of
suffering.

Active passivity vs apparent competence: Passivity in solving one’s problems while


actively engaging others to solve problems, coupled with the tendency of others to
overestimate the capabilities of the individual.
Unrelenting crises vs inhibited grieving: A tendency to experience life as a series of
extreme problems contrasted with an inability to experience emotions associated with
trauma or loss.

12) What are the indications of DBT?


depression, anxiety disorders, personality disorders, PTSD and addictions that shows
symptoms related to (instability in emotion regulation, impulse control, interpersonal
relationships, and self-image)

13) Which are the 4 primary DBT skills training modules? Mindfulnes skills
Interpersonal Effectiveness SkillsEmotion Regulation Skills Distress Tolerance Skills
Mindfulness involves finding the synthesis between extremes by orienting to the truth in each
position. These skills also include focusing attention by observing, describing, and participating
in the present moment without trying to change one’s present experience andwhile assuming a
non-judgmental stance, focusing awareness on one thing at a time, and developing
effectiveness. Mindfulness skills are central to DBT and thus are woven throughout the other
skills modules.
Distress tolerance teaches impulse control, distracting, and self-soothing strategies for
tolerating aversive contexts, surviving crises, and radically accepting situations that cannot be
changed without resorting to dysfunctional behaviour.
Emotion regulation teaches methods for identifying and describing emotions, determining
whether an emotion is justified by current circumstances, modulating emotions via acting
opposite to the emotion or problem solving, reducing vulnerability to unwanted negative
emotions, and increasing experience of positive emotions.
Interpersonal effectiveness teaches assertiveness skills aimed to help clients achieve their
objectives in interpersonal interactions while also maintaining positive relationships and their
self-respect.

14) The skills taught to patients reflecting key dialectic can be categorised as:
a. Acceptance skills & change skills
b. Interaction skills & control skills
c. Mindfulness skills & Tolerance skills
d. Emotion regulation skills & Interpersonal skills

15) Usual duration of DBT is


a. 3 months
b. 6 months
c. 9 months
d. 12 months

16) There are how many stages of DBT


a. 3
b. 4
c. 5
d. 6

17) Which of the following are core strategies of DBT


a. Acceptance of self
b. Validation
c. Distractions during high emotional intensity
d. Problem Solving
e. A & B
f. A, B & CA & D
g. B&C
h. B&D
i. C&D

Topic: Family therapy

1) Which of the following statements is false?


a. First generation family therapy involves Bowenian Family Therapy
b. Second generation family therapy includes Structural Family Therapy
c. Second generation family therapy belongs to the era between 1970 & 2000
d. Third generation family therapy emphasises on special population

2) Which of the following is not a model of Family Therapy


a. Dynamic
b. Flexible
c. Structural
d. System’s Approach

3) Structural Model of Family Therapy was given by


a. Minuchin
b. Bowen
c. Freud
d. Haley

4) Domino effect refers to


a. A family member who dominates in the family can be key person to addressall the
problems
b. If the therapist has the final say in the conversation then the therapy is bound to be
effective
c. If one problem is properly addressed it leads to reduction of other problems
d. Solution to minor problems ultimately leads to resolution of the major problem

5) Theory of emotional triangle holds true for which kind of Family Therapy?
a. Dynamic
b. Structural
c. Strategic
d. System’s Approach

6) Which of the following models of family therapy aims to find positives in a system & builds
on them?
a. Dynamic
b. Structural
c. Strategic
d. System’s Approach
e. All of the following are elements in which families are assessed,
exceptCommunication
f. Problem-Solving
g. Self-concept reinforcement
h. Family interactional pattern
i. Family Climate

7) Which of the following is a contraindication for Family Therapy


a. Problems in the relationship within the family
b. Development of stress in one family member when other family member improves
c. Families in which tense, dysfunctional equilibrium is present
d. Failure of individual therapy where family tensions have not been handled

8) Which of the following is not a part of the six layers of iceberg?


a. Behaviour
b. Coping
c. Feelings
d. Perceptions
e. Cognition
f. Expectations

9) All of the following are techniques used in structural family therapy, except
a. Reframing
b. Enactment
c. Paradoxical tasks
d. Accommodation & Joining

10) Solution focused Family therapy is belong to which generation?


a. First
b. Second
c. Third
d. Fourth

Miscellaneous topics
1) Ebstein’s Anomaly is sometimes associated with drugs other than Lithium. Choose one such
drug.
a) Diazepam
b) Paroxetine
c) Duloxetine
d) Carbamazapine

2) Drug to be avoided in an ADHD child with liver dysfunction:


a) Methylphenidate
b) Atomoxetine
c) Clonidine
d) Dexamphetamine

3) 50 year old man with bipolar disorder developed mania post renal transplant. Mood stabiliser
of choice?

a) Lithium
b) Sodium Valproate
c) Lamotrigine
d) Topiramate

4) A male patient on clozapine has gained 20 kgs in 3 months, though psychotic symptoms are
well controlled. The next step is to :

a) Reduce dose of clozapine


b) Switch to high dose haloperidol
c) Switch to Risperidone
d) Lifestyle modification

5) Characteristic feature of simple schizophrenia:

a) Memory loss
b) Motor incoordination
c) Social withdrawal
d) Delusions

Extra material to read (available as journal articles in the internet)

1. Reed GM, First MB, Kogan CS, Hyman SE, Gureje O, Gaebel W et al. Innovations and changes
in the ICD 11 classification of mental, behavioural and neurodevelopmental disorders. World
psychiatry 2019; 18(1): 3-19. (for changes in ICD 11)

Few TIPPS for recently asked University questions

Q What are the recent advances in Bipolar disorder ? (you may take help from
undermentioned points and may add your own points as per your revision )
• Genetic overlap of bipolar disorder ( write about genetic studies, GWAS : key terms ,
CACNA1C,ODZ4 etc )
• Write briefly about psychiatric genetic consortium , overlap between bipolar disorder
and schizophrenia
• Gene environmental interaction ( write about methylation studies in BDNF and other
epigenetic interactions )
• Write about CANMAT 2018 : Placing, lithium quetiapine , lamotrigine , introduction of
Cariprazine as per evidence
• Recent studies about lithium : start from BALANCE ( lithium and valproate ), lithium
being the endo phenotype of bipolar disorder , write about Consortium of lithium
genetic ( ConLiGen ) , Lithium’s role in pleuripotent stem cells in hippocampus ,lithium
affecting phosphorylation of Collapsing response mediator protein 2 ( CRMP2 ) , long
term effect of lithium on very small embryonic like cells (VSELs), Lithium and suicde
prevention , recent studies , explain neuroprotective effects of lithium .

Q Sigma receptors and their role in emotional regulation


• IP3 , DAG receptors
• Role in cardiovascular, schizophrenia , cocaine abuse and cancer
• Structure has 223 amino acids
• Function to release calcium , myocyte contractilitu , explain about G protein coupled
receptors.
• Sigma receptors have antidepressant like properties explained through behavioral
models
• Serotonergic, glutaminergic transmission by sigma receptors
• Mention about sigma 1 and sigma 2 receptors , their role in movement disorders
• Steroids binding to sigma receptors, role in psychosis and depression
• Dehydroepiandrosterone and Estrogen
• SSRIs and Second generation antipsychotics
• Mutation associated with distal spinal muscle atrophy gene 2
• Sigma 1 agonists are : donepezil, fluvoxamine, citalopram, amitryptilline, opipramole
• Sigma 1 antagonists are sertraline
• Positive allosteric modulators : methylphenyl piracetam
• Multifunctional agonists : haloperidol .

Q Emission imaging in psychiatric disorders ?


Write about PET and SPECT in psychiatric disorders ( refer comprehensive textbook of
psychiatry , 10 th edition )
• The principles of each , the diagrams of principles
• PET quantification in neuropsychiatry
• Role of P glycoprotein
• Neuroimaging in seasonal and winter depression,ptsd,anxity depression, personality
disorders ( WRITE ABOUT THEIR FINDINGS )
• PET and SPECT in neuroimaging in schizophrenia
• Diagnostic feature for dementia
• PET and SPECT in non pharmacological interventions

Q : Post schizophrenic depression


• Explain the term post schizophrenic depression , differentiate from schizo depression
and schizoaffective disorders .
• Duration of schizophrenia followed by depression for 2 weeks
• Theories of formation of post schizophrenic depression : stigma and isolation ,
substance abuse ,psychotic depression , negative symptoms and drug induced EPS ( see
the table above )
• Suicide and post schizophrenic depression, strong correlation with suicide ( ICD supports
it too )
• Antipsychotics and explain treatment for each of the differential

Q Mental health literacy movement


• Mental health literacy means knowledge and beliefs about mental disorders which aid
their recognition , management and prevention . mental health literacy includes ability
to recognize mental disorders , knowing how to seek health information , knowing of
risk factors and causes of self treatment , knowledge of risk factors, , professional help
available and attitudes that promote recognition and appropriate help seeking .

• Explain the differences between literacy and mental health literacy

• Framework of mental health literacy :

• Recognition

• Knowledge ( information , risk factors , causes of mental disorders , self help ,


professional help )

• Attitudes

• Public outlook : stereotype, stigma

Limitations :
• Low literacy
• Diverse population
• Recognition of Uncommon mental diorders

Approaches :
• Community campaigns
• School based interventions
• Individual training programs
• Websites and books aimed at public
• Rapport between mental health professionals and clients

You might also like